Sunteți pe pagina 1din 304

CARDIOLOGÍA

ANATOMÍA Y FISIOLOGIA CARDIOCIRCULATORIA.

¿De que capa germinal proceden el corazón, los vasos y las células
sanguíneas?
Del mesodermo.

¿Cuáles son las estructuras cardiacas primitivas en el embrión?


Porcion craneal: Da origen a los arcos aórticos
Porción caudal: Origina el ventrículo embrionico.
Bulbo cardiaco: Se divide en tercios. La porción inicial origina el cuerpo
del ventrículo derecho o ventrículo primitivo. La porción media origina el
conos cordis que constituirá los tractos de salida del ventrículo derecho y
el ventrículo izquierdo. La porcion distal da origen al truncus arteriosus.

¿De que estructuras embrionarias proceden el nodo sinoauricular (NSA


o de Keith y Flack) y el nodo auriculoventricular (NAV o de Aschoff-
Tawara)?
El nodo SA se desarrolla a partir de estructuras del lado derecho y el no-
do AV de estructuras del lado izquierdo, esta es la razon por la que el
nodo SA se encuentra inervado por el vago derecho y del nodo AV por el
vago izquierdo.

¿Cuál es el potencial de membrana en reposo de las células miocárdi-


cas?
-90mv.

¿Cuáles son las fases de la despolarización cardiaca?


Fase 0: Despolarización rápida: Apertura de canales rapidos de Na+.

43
Fase 1: Repolarización parcial: Inactivación de los canales de Na+, aper-
tura de canales de K+, con salida de K+.
Fase 2: Meseta: Apertura de canales de Ca++. Con entrada del mismo.
Fase 3: Repolarización: Cierre de canales de Ca++. Persiste discreta sali-
da de K+.
Fase 4: Despolarización progresiva por entrada de Na+.

¿Dónde se encuentran ubicados el nodo SA y el nodo AV?


El nodo SA se encuentra situado en la unión de la vena cava superior
con la aurícula derecha y el nodo AV se encuentra situado en la porción
posterior derecha del tabique interauricular.

¿Cómo se encuentran conectados el nodo SA y el nodo AV?


Se conectan gracias a tres fascículos de fibras de Purkinge, llamados de
Bachman (anterior), de Wenckebach (medio) y de Thorel (posterior).

¿A que se llama ciclo cardiaco?


Al conjunto de movimientos del corazón en el que se encuentra una fase
diastólica de llenado y una sistólica de expulsión. En cada ciclo cardiaco
se expulsan 60 a 70ml (algunos marcan de 70 a 90ml o bien el 65%)
promedio de sangre. Al final de la diástole el volúmen ventricular en pro-
medio alcanza 130 ml por tanto un promedio de 50ml se considera el vo-
lúmen residual después de la sístole.

44
¿Cuál es la correlación entre los eventos mecánicos, las presiones y los
fenómenos estetoacústicos del ciclo cardíaco?

Sistole Contracción isovolu- Duración 0.5 Cierre de válvulas Primer Ruido o “lub” o S1
isovolumétrica métricahasta alcanzar seg AV que se abom-
80mmhg en aorta y ban hacia las au-
10mmhg en la pul mo- rículas
nar
Fase de Elevación de la presión Al inicio es rá- Válvulas AV se
expulsión hasta 120 en al VI y pida enlentece traccionan hacia
ventricular hasta 25 en el VD. I segun progresa abajo porlas
la síatole. cuerdas tendino-
sas
Protodiástole Descenso inicial de la Duración Termina con el Segundo ruido o “dub” o S2
presión ventricular 0.04seg cierre de las vál-
vulas aórtica y
pulmonar
Relajación Finaliza cuando la pre- Apertura ventricu- Tercer ruido o S3 por llenado ven-
ventricular sión ventricular es me- lar de valvulas AV tricular rápido.
isovolumétrica nor que en las permitiendo el Cuarto ruido o S4 se ausculta en
aurículas llemado ventriculos rigidos o si hay presión
auricular alta, justo antes del S1.

¿En que consiste el pronunciamiento de Frank Starling?


La energia de contracción es proporcional a la longitud inicial de la fibra
muscular, dicho de otra forma mientras mas se distienda el miocardio (se
precargue) mayor será el volúmen sistólico ventricular, dando lugar a la
curva de FranK Starling. Es gracias a esta regulación heterométrica que
un corazón transplantado (no inervado) puede aumentar el gasto si las
demandas aumentan.

¿Qué relación se expresa mediante la fórmula de Poiseuille-Hagen?


Esta fórmula expresa la relación entre el flujo en un tubo largo estrecho,
la viscosidad del líquido y el radio del tubo. Su expresion matemática es
la siguiente: F= (PA-PB)x(π/8)x(1/n)x(r4/L).

F=flujo n= viscosidad r= radio del tubo L= longitud del tubo.


PA-PB= diferencia de presión en los extremos del tubo.

45
¿Qué relación se expresa en la ley de Laplace?
La relación entre la presión de distensión y la tensión. Se dice que la pre-
sión de distensión (P) es igual a la tensión (T) dividida por el radio del tu-
bo. P=T/r.

¿Qué parámetro hemodinámico se ve reflejado en la presión central pul-


monar?
La precarga. La presión central pulmonar mayor de 15 indica insuficien-
cia cardiaca izquierda y la menor de 15 indica hipovolemia.

Aqui introducir conceptos de hemodinamica

¿Cuales son los principales mecanismos de control de la presión sanguí-


nea?
La presión sanguínea es resultado directo de multiplicar el gasto cardíaco
por las resistencias periféricas. Entonces la TA se regula principalmente
mediante dos mecanismos imbrincados: 1) Los baroreflejos mediados
por el sistema nervioso simpático y 2) el sistema renina angiotensina al-
dosterona.
¿Quién se encarga de la regulación de la TA a corto plazo?
Los baroreceptores en el arco aórtico y el seno carotídeo, disminuyen
sus estímulos aferentes hacia la médula espinal cuando la presión cae;
esto provoca aumento de la actividad simpática y disminución de la pa-
rasimpática. Como resultado hay vasoconstricción y aumento en el gasto
cardíaco.

46
¿Cuáles son las acciones que genera la activación de estos receptores?
 α1: Se encuentran en los vasos, su efecto es vasoconstrictor co-
ronario, renal, piel, mucosas, utero, vejiga, midriasis. Inhibe la li-
beración de insulina
 α2: Se encuentran en las membranas prey post sinápticas del sis-
tema nervioso central y periférico, vasos sanguíneos, plaquetas y
leucocitos. A nivel del centro vasomotor provocan disminución de
la presión arterial. Aumenta la secreción de glándulas saliváceas,
de insulina por los islotes de Langerhans. Disminuye la secrecion
de renina.
 β1: taquicardia, inotrópico positivo, aumento de la lipólisis
 β2: causa relajación del músculo ciliar del ojo, músculo esqueléti-
co, bronquial, vesical, uterina. Vasodilatación esplácnica y renal.
Incrementa la secreción de renina en el parato yuxtaglomerular.
 β3 aumenta la liberación de glucagon.
 Los alfa tienen mayor afinidad a la noradrenalina y los beta a la
adrenalina.
 β1 y β2 a nivel cardiaco son inotropicos y cronotropicos positivos
¿Qué es el sistema renina angiotensina aldosterona?
Es el encargado de regular la presión sanguínea a largo plazo. Los baro-
receptores renales conformados por el aparato yuxtaglomerular, regulan
la secracion de renina, esta transforma el angiotensinógeno en angioten-
sina I y II provocando aumento de la presión sanguínea.
¿Dónde se forma cada componente del sistema renina angiotensina al-
dosterona?
Angiotensinógeno Se forma en hígado y circula en el plasma en la fracción α-2-
globulina.
Renina Se forma en las células del aparato yuxtaglomerular, localizadas
en la arteriola aferente del glomérulo
Angiotensina I Este decapéptido se desdobla del extremo Terminal N del angio-
tensinógeno. En el plasma en presencia de renina.
Enzima convertidora de Está situada en las células endoteliales de todo el organismo
angiotensina
Angiotensina II La mayor parte de la transformación es anivel pulmonar pero
también se realiza en otras partes del cuerpo.

47
¿Cuales son las acciones de la angiotensina II?
 Produce vasoconstricción arteriolar
 Actúa directamente sobre la corteza suprarrenal aumentando la
secreción de aldosterona con retención final de sodio.
 Actúa sobre las neuronas simpáticas posganglionares facilitando
la liberación de noradrenalina
 Produce contracción de las células mesangiales renales. Pero
principalmente causan constricción en la arteriola EFERENTE,
manteniendo la TFG renal.
 A nivel central aumenta la TA, aumenta la ingesta de agua e in-
crementa la secreción de vasopresina, endotelina y ACTH.
 Estimula la fibrosis, el PAI 1 y la formación de superóxidos

HIPERTENSIÓN

¿Cómo se realiza la exploración de la TA (tensión arterial)?


Debe hacerse en reposo, evitando estados de tensión o nerviosismo, es
recomendable no haber ingerido en los 30 a 45 minutos previos, café o
alcohol, ni haber fumado. Se coloca el manguito en el brazo a la altura
del corazón, y se aplica presión sobre la arteria humeral, hasta que se
impide la circulación. La auscultación se realiza sobre la flexura del codo.

¿A quienes se considera pre-hipertensos y como se manejan?


Toda persona que tiene PAS (presion arterial sistólica) de 120-139 o
PAD (presión arterial diastólica) de 80-89. Se les debe exhortar a modifi-
car su estilo de vida únicamente.

¿Cuáles son los estadíos 1 y 2 de la hipertensión?


 Estadío 1 es 140-159 de PAS y 90-99 de PAD
 Estadío 2 es >160 de PAS y >100 de PAD

¿Cuáles son las modificaciones en el estilo de vida recomendadas por el


JNC7?:

48
Reducción de peso IMC 18,5-24.9 Kg/m2
Dieta DASH Rica en frutas y vegetales, baja en
grasas saturadas.
Reducción de sodio en la dieta Na+ no mas de 100mmol/día (2.4g
de Na o bien 6g de cloruro de Na)
Actividad física Ejercicio aerobico al menos 30
min/día
Moderación en el consumo de al- No más de 30ml de etanol/día (dos
cohol copas) en varones, 1 en mujeres.

¿Cuáles son causas de hipertensión seundaria?


 Apnea del sueño
 Causas inducidas o relacionadas a fármacos
 Enfermedad renal crónica
 Aldosteromismo primario
 Enfermedad renovascular
 Corticoterapia crónica y Síndrome de Cushing
 Feocromocitoma
 Coartación de aorta
 Enfermedad tiroidea o paratifoidea

¿A que se le llama hipertensión resistente?


Es el fracaso de conseguir los objetivos en pacientes que tienen adhe-
rencia a decuada a una pauta de 3 antihipertensivos a dosis tope que in-
cluya diurético. Deben descartarse primero las causas de hipertensión
secundaria y luego revisarse las causas de hipertensión resistente.

¿Cuáles son las causas de hipertensión resistente?


1) Medida inadecuada de la presión arterial
2) Volumen plasmático elevado y pseudotolerancia
 Escaso volumen de sodio
 Retención de volumen por enfermedad renal
 Terapia diurética inadecuada

49
3) Inducida por fármacos
 No adherencia o dosis inadecuadas
 AINES, Inhibidores COX
 Contraceptivos, corticoides, regaliz
4) Condiciones asociadas
 Obesidad y excesivo consumo de alcohol.

¿Según el JNC-7 que fármaco es la base del tratamiento en HAS (hiper-


tensión arterial sistémica) no complicada?
Debe usarse hidroclorotiazida sola o combinada.

¿Cuál es el objetivo del tratamiento de la TA?


Mantener al paciente con 140/90. En los diabéticos y pacientes renales
es de 130/80.

¿Cuál es el manejo de las diferentes variantes clínicas de la hipertensión


arterial sistémica (HAS)?
 La HAS grado I monoterapia y II deben recibir farmacoa asocia-
dos.
 HAS con síndrome hipercinético: - bloqueadores.
 HAS con síntomas de expansión del espacio extracelular: diuréti-
cos.
 HAS de predominio sistólico y edad avanzada: calcioantagonis-
tas.

¿Cuáles son las fármacos de elección en hipertensos con cardiopatía is-


quémica?
En los pacientes con angina estable se usan betabloqueadores o cal-
cioantagonistas de acción larga. En pacientes con síndromes coronarios
agudos o posinfartados se unasn IECAS,, betabloqueadores (BB) y anta-
gonistas de la aldosterona (AA).

50
¿Cuáles son los beneficios de los antihipertensivos en los diabéticos?
↓ ECV e ictus ↓Nefropatía ↓macroalbuminuria
ARA II + + +
IECA + + -
BB + - -
AA + - -
TIAZÍDICOS + - -
BBC + - -

¿Cuál es la definición del ATP III del síndrome metabólico?


Se define como la presencia de 3 o mas de las siguientes:
 Obesidad abdominal mayor de 102 en hombres y 89 en mujeres
 Intolerancia a la glucosa (glucemia basal 110 mg/dl)
 TA 130/85 mmHg
 Triglicéridos elevados 150mg/dl
 HDL< 40mg/dl en hombres y <50mg/dl en mujeres

¿Cuáles son los únicos fármacos antihipertensivos que no modifican fa-


vorablemente la hipertrofia ventricular izquierda?
Los vasodilatadores directos hidralacina y minoxidil.

¿Cuáles son las consideraciones para la hipertensión en mujeres?


 La TA y el riesgo de hipertensión aumentan con el consumo de
anticonceptivos orales
 Por el contrario la terapia hormonal sustitutiva no causa hiperten-
sión
 Las mujeres hipertensas embarazadas deben controlarse con me-
tildopa, betabloqueadores y vasodilatadores por la seguridad del
feto.
 IECA y ARAII están contraindicados en el embarazo.

¿Cuáles son ls principales familias de antihipertensivos?

51
Diuréticos, betabloqueadores, inhibidores de la enzima convertidota de
angiotensina (IECA), antagonistas de receptores de angiotensina II
(ARAII), calcioantagonistas, bloqueadores alfa, antihipertensivos de ac-
ción central (agonistas α2).

¿Cuáles son las acciones de los betabloqueadores?


 Inhiben competitivamente los efectos de las catecolaminas en los
receptores beta, disminuyendo la frecuencia cardíaca y el gasto
cardíaco.
 Disminuyen los niveles de renina
 Regulan a las barorreceptores para aceptar niveles más bajos de
TA.
 Generan liberación de prostaglandinas vasodilatadores
 Disminuyen el volumen plasmático.
 Disminuyen el flujo de salida simpático del SNC

¿Según su mecanismo de accion como se dividen los betabloqueadores?


Cardioselectivos y no cardioselectivos.

¿Cuáles son los betabloqueadores con actividad simpaticomimetica in-


trinseca?
Son fármacos con actividad β agonista parcial que además de bloquear
el acceso de las catecolaminas, estimulan levemente al receptor adre-
nérgico produciendo menor enlentecimiento de la frecuencia cardiaca
con menor depresion de la conducción aurículo-ventricular.Los farmacos
con estas propiedades son: pindolol, oxprenolol, celiprolol y acebutolol

¿Cuál es el modelo de los bloqueadores beta generales?


El propanolol.

¿Cuáles son los bloquadores beta-1 selectivos?


El metoprolol, el esmolol y el atenolol.

¿Cuáles son los efectos adversos de los betabloqueadores?

52
Alteran el perfil lipídico aumentando los triglicéridos, disminuyendo HDL.
Provocan aumento en las cifras de glucosa plsmática. Pueden provocar
bloqueo AV, fenómeno de Raynaud, impotencia y depresión.

¿Cuáles son las dos acciones de la enzima convertidora de la angio-


tensina (ECA)?
 La conversión de la angiotensina I en angitoensina II.
 La degradación de la bradicinina (un potente vasodilatador que
causa liberación óxido nítrico y prostaciclinas del endotelio vascu-
lar)
 Disminuye la morbimortalidad en pacientes posinfarto.
 Disminuye la remodelación cardíaca.

¿Cuáles son algunas indicaciones de los inhibidores de la ECA?


Posinfarto, como antihipertensivos en pacientes con diabetes mellitus e
hiperlipidemias y en la falla cardiaca crónica.
¿Cuáles son los efectos adversos de los inhibidores de la ECA?
Hipotensión, tos, fiebre, exantema, hiperkalemia y leucopenia.

¿Cuáles son algunas propiedades beneficosas de los IECA?


 Incrementan la sensiblidad a la insulina por lo que son utiles en
diabeticos
 disminuye la resistencia vascular periferica y la poscarga por lo
qus se utilizan en la insuficiencia cardíaca.
 Incremento del flujo renal y coronario
 Disminuye la presión intraglomerular.

¿Por qué puede producir hiperkalemia el captopril?


Porque al disminuir la angiotensina II disminuye la aldosterona y se pro-
duce aumento de potasio, disminución de sodio y pérdida de agua.
¿Cuáles son las tres clases de bloqueadores de canales de calcio?
 Fenilalkilaminas: verapamil.
 Benzotiacepinas: diltiazem.
 Dihidropiridinas: nifedipina, amlodipina, felodipino.
¿Cuáles son los canales de calcio mas conocidos?

53
Los L,T, N, P y R. Los que se encuentran a nivel cardiaco son los L.
¿Qué ventajas tienen la nicardipina y la amlodipina?
Tienen una menor interacción con la digoxina y la warfarina.

¿Cuáles son las indicaciones de los bloqueadores de los canales de cal-


cio?
 Las taquiarritmias supraventriculares,
 Síndromes coronarios agudos.
 Enfermedad vascular periférica.
 Su falta de efectos en el metabolismo de la glucosa, los lípidos los ha-
ce utiles en hipertensión con dislipidemia y diabetes
 Enfermedad renal crónica.
 Espasmo esofágico

¿Cuáles son los beneficios de los calcioantagonistas en la cardiopatía hi-


pertensiva?
Reducen la hipertrofia ventricular izquierda, reducen las arritmias, los de-
fectos de llenado y de contractilidad. Los no dihidropiridínicos reducen el
riesgo de reinfarto.

¿Cuáles son los efectos adversos de los bloqueadores de los canales de calcio?
Mareo, constipación, cefalea e hipotensión.

¿Cuales farmacos son inhibidores de vasopeptidasa?


Son farmacos capaces de inhibir la endopeptidasa neutra y la ECA lo que
incrementa los péptidos natriuréticos e incrementa la vida media de otros
péptidos incluyendo la bradicinana y la adrenomodulina. El omapatrilat es
el primer farmaco de este grupo y se utiliza en hipertensión e insuficien-
cia cardiaca.

¿Como funcionan los farmacos antagonistas de receptores de angioten-


sina II (ARAII)?
Existen dos tipos de receptores de angiotensina, los AT1 y los AT2. A la
acción sobre los AT1 se deben la mayor parte de efectos conocidos de la
angiotensina. Se indican estos fármacos ante tolerancia a los IECA o

54
bien cuando los efectos adversos por acúmulo de bradicinina (tos y an-
gioedema) son importantes.

¿Qué fármacos son vasodilatadores arteriales?


La hidralazina, el diazóxido y el minoxidil.
¿Qué efectos adversos provoca la hidralazina?
Cefalea y síndrome lupus-like (síndrome similar al lupus)
¿Cuáles son los bloqueadores alfa generales?
La fenoxibenzamina y la fentolamina.
¿Qué son la clonidina y la metildopa??
Son agonistas alfa-2 que actúa en el centro presor del tallo y por retroali-
mentación negativa disminuyen el flujo simpático de noradrenalina cau-
sando disminución en la TA. Secundariamente desplazan a la
noradrenalina de los receptores alfa periféricos impidiendo su acción en
el músculo liso vascular.
¿Qué es la prazocina?
Es un bloqueador alfa-1 de acción directa en arteriolas.
¿Cuáles son los antihipertensivos centrales (bloqueadores alfa centrales)?
La metildopa, la clonidina, el pentolinium, la prazocina y la guanabencina.
¿Cuáles son los antihipertensivos periféricos (bloqueadores alfa periféricos)?
La reserpina (depleta las vesículas de noradrenalina) y la guanetidina (in-
hibe la liberación de noradrenalina).
¿Cuál es la acción principal de los bloqueadores alfa?
Inhibir la función de la noradrenalina.
¿Cuáles son algunas indicaciones de los bloqueadores alfa?
Los síndromes coronarios agudos, el periodo posinfarto, las taquiarrit-
mias supraventriculares, el glaucoma y en la migraña.
¿Cuál es el efecto de la prazocina?
La prazocina es un bloqueador alfa-1 selectivo, cuyo efecto consiste en
disminuir la tensión arterial sin afectar el gasto cardiaco.
¿Cuáles son los efectos adversos de los bloqueadores alfa?
 Pérdida de la libido.

55
 Rebote cuando cesa su administración debido a que provocan una
sobreexpresión de los receptores beta.
 Anormalidades en los lípidos.
 Incontinencia urinaria. Disminuyen el tono de los esfínteres y por eso
tienen utilidad en el prostatismo (terazosina)

¿Qué es una crisis hipertensiva?


Es una elevación abrupta y sustancial de la presión arterial que amenaza
la vida o ciertas funciones. En general una TAD mayor de 120 se consi-
dera crisis hipertensiva. Las crisis pueden ser urgencias o emergencias.
Tambien algunos consideran hipertensión acelerada cuando aparecen
cefalea, visión borrosa y focalizaciones. Asimismo se considera hiperten-
sión maligna cuando hay edema de papila.

¿Qué es una emergencia hipertensiva y cuales son los objetivos de tra-


tamiento?
Es una crisis hipertensiva en la que se demuestra daño a organo blanco
progresivo principalmente en el sistema nervioso central, cardiovascular,
renal y en la retina.
Se busca reducir la TA 25% o bien alcanzar TAD 100-110 en una hora.

¿Qué familias de fármacos antihipertensivos intravenosos se utilizan para


tratar las crisis hipertensivas?
Vasodilatadores: nitroprusiato, nitroglicerina, diazóxido, hidralazina, ni-
cardipina y enalaprilat.
Betabloqueadores: Esmolol, labetalol.
Alfabloqueadores: Fentolamina.
Bloqueadores ganglionares: camsilato de trimetafán
Agonistas dopaminérgicos: Fenoldopam.

¿Qué es el nitroprusiato de sodio y en que tipo de crisis está indicado?

56
Es un vasodilatador arterial y venoso de acción directa y corta (3-5min)
de elección para algunas emergencias hipertensivas. Un producto final
de su metabolismo es el cianuro. La intoxicación por este se trata con
tiosulfato de sodio para producir un metabolito menos tóxico que es el
tiocianato. Se usa en dosis de 0.5 a 10 gammas.

¿Qué es la nitroglicerina y cuando se usa?


Es un nitrato orgánico, tiene efectos a nivel periférico en los vasos veno-
sos y a nivel de las coronarias. En este caso, causa vasodilatación veno-
sa reduciendo la precarga, el gasto cardiaco y secundariamente la TA.
Su tiempo de acción es de 3-5 minutos y la dosis es de 5-100µg/min. Sus
principales efectos adversos son cefalea náusea y vómito. Tiene su prin-
cipal indicación en casos de hipertensión y SICA del tipo IAM.

¿Cuáles son los fármacos de elección en las crisis hipertensivas por feo-
cromocitoma?
Se usan fentolamina y betabloqueadores en combinación.
¿Según el JNC 7 cuales son las dosis terapéuticas de los siguientges
fármacos?
Fármaco Dosis (frecuencia) Fármaco Dosis (frecuencia)
Hidroclorotiazida 12.5-50mg/día (1) Valsartan 80-320mg/día (1)
Clortalidona 12,5-25 (1) Diltiazem 180-420 (1)
Furosemide 20-80 (2) Verapamil 80-320 (2)
Espironolactona 25-50 (1-2) Amlodipino 2.5-10 (1)
Metoprolol 50-100 (1-2) Felodipino 2.5-20 (1)
Propranolol 40-160 (2) Nifedipino ret 10-40 (1)
Captopril 25-100 (2) Prazocina 2-20 (2-3)
Enalapril 2.5-40 (1-2) Metildopa 250-1000 (2)
Losartan 25-100 (1-2) Hidralacina 25-100 (2)
Telmisartan 20-80 (1) Minoxidil 25-80 (1-2)

INSUFICIENCIA CARDIACA

¿Cuál es la definición de insuficiencia cardíaca (IC)?

57
Es un síndrome clínico que resulta de cualqier desorden cardiaco funcio-
nal o estructural que impida el adecuado llenado y vaciado del corazón.
Las manifestaciones cardinales de la falla cardiaca son disnea y fatiga
que pueden limitar la tolerancia al ejercicio y retención de líquidos que
puede condicionar congestión pulmonar o edema periférico.

¿Como se clasifica la IC según su evolución?


 Etapa A: En riesgo de insuficiencia cardiaca pero sin desorden
estructural
 Etapa B: Con desorden estructural pero sin síntomas de IC.
 Etapa C: Historia de síntomas de IC con daño estructural
 Etapa D: IC terminal que requieremanejo especializado, como
inotrópicos, apoyo mecánico externo y finalmente transplante.

¿Cual es la clasificación funcional de la IC de la NYHA?


 Clase I: Sin limitación de actividades, sin síntomas
 Clase II: Con limitación ligera, están cómodos en reposo o con ac-
tividad ligera
 Clase III: Marcada limitación funcional, están cómodos solo en re-
poso.
 Clase IV: Confinado a la cama o silla, cualquier actividad física des-
encadena síntomas

¿Cuáles son algunas causas identificables de IC?


Muchas veces no puede identificarse un factor desencadenante pero
tambien pueden encontrarse desordenes sistemicos (hipertiroidismi, he-
mocromatosis, hypocalcemia) o antecedente de exposición a cardiotóxi-
cos (alcohol, cocaína, antraciclinas o trastuzumab). Debe buscarse
también datos de inflamación o infiltración cardíaca (sarcoidosis, ami-
loidosis).

¿Cual es la causa mas frecuente de IC?


La cardiopatía isquémica es responsable hasta del 70% de la IC.

¿Cual es el marcador clinico más confiable para diagnóstico de IC?

58
La disnea de esfuerzo es un síntoma con especificidad del 17% pero
sensibilidad del 100%

¿Cuales son datos clínicos de IC en la exploración física?


Reflujo hepatoyugular: una presión en hipocondrio derecho por 30 se-
gundos con elevación de 4cm en la presion venosa.
Desviación del choque de la punta: En paciente en decíbito lateral iz-
quierdo. Es positivo cuando a la espiración el choque rebasa la línea me-
dio clavicular.
Ritmo de galope: En decubito lateral izquierdo, ausculte con la campana
del estetoscopio.
Ingurguitación yugular. Medir en decubito supino a 45 grados con la
cabeza hacia la derecha. Medir desde el angulo de Louis.

¿Cómo se dividen las causas de la IC?


Se dividen en causas subyacentes y causas precipitantes. La causa sub-
yacente mas común es la cardiopatía isquémica, seguida de las cardio-
miopatías, cardiopatías congenitas, enfermedades valvulares y
finalmente la hipertensión.

¿Cuales son las formas clínicas principales de IC?


 Sistólica y diastólica
 De alto gasto y bajo gasto
 Aguda y crónica
 Derecha e izquierda
 Anterógrada y retróigrada

¿Cuales son los criterios de Framingham para la falla cardiaca?


Mayores:
 3 pulmonares: edema pulmonar, disnea paroxística nocturna y es-
tertores
 3 de cuello: reflujo hepatoyugular, presion venosa>16cmH2O, dis-
tensión yugular
 2 cardiacos: cardiomegalia y galope S3.
Menores

59
 4 Respiratorios: capacidad vital a 1/3 de lo normal, disnea de
ejercicio, derrame pleural y tos nocturna.
 Edema de extremidades
 Hepatomegalia
 Taquicardia mayor de 120lpm
Mayor o menor
 Perdida de mas de 4.5kg después de 5 días de tratamiento
El diagnóstico requiere al menos un mayor y dos menores.

¿Qué tipo de IC causan las miocardiopatías y como se dividen?


Causan IC de tipo diastólico. Se estudian como primarias y secundarias.
Primarias son la miocardiopatía restrictiva idiopática, la fibrosis endo-
miocárdica (enfermedad de Davies) y la miocardiopatía eosinofílica (en-
fermedad de Löffler).
Secundarias son la amiloidosis, la hemocromatosis, las enfermedades
por atesoramiento y la sarcoidosis entre otras.

¿Cuales son los criterios ecocardiograficos de IC diastólica?


Tiempo de relajación isovolumétrica (IVRT) mayor de 100.
La relación entre el flujo transmitral precoz (ondaE) y el tardío (onda A,
sistole auricular). La relacion normal en jóvenes es 2:1 y en mayores de
50 1:1. Una relación E/A baja con IVRT prolongado corresponde a IC
diastólica
¿Cuáles son causas de IC de alto gasto?
Anemia, Fístulas arteriovenosas (síndrome de Osler-Weber-Rendu)
hipertiroidismo, beri-beri (deficiencia de tiamina), la enfermedad de Pa-
get, el síndrome de Albright, el mieloma múltiple y el embarazo.

¿Qué es la enfermedad de Paget?


Es una enfermedad en la que la resorción y la formación ósea están au-
mentadas provocando cortocircuitos intraóseos, su causa es probable-
mente genética o viral

¿Qué es el síndrome de Osler Weber Rendú?


Es un desorden autosómico dominante también conocido como, telan-
giectasia hemorrágica hereditaria en la que hay epistaxis y sangrados in-
testinales frecuentes por malformaciones capilares y en el que puede
haber grandes fístulas arteriovenosas pulmonares.

¿Qué es el síndrome de McCune-Albright?

60
También llamado simplemente síndrome de Albright, se caracteriza por la
triada de manchas café con leche, displasia fibrosa polióstica y disfunción
endócrina que puede incuir tirotoxicosis, acromegalia., hirsutismo o galacto-
rrea. Hay que dirferenciarlo de la neurofibromatosis, eslcerosis tuberosa,
ataxia telangiectasia, síndrome de Fanconi e hiperplasia adrenal congénita
así como de tumores adrenales o gonadales entre otros.

¿Cuáles son las características clínicas de la IC de alto gasto?


 Taquicardia con aumento de la presión del pulso
 Latido hiperdinámico
 Calor en manos y piés
 En ocasiones soplo discreto en el ápex y el borde esternal iz-
quierdo.

¿Cuáles son los fármacos que deben evitarse en IC?


Antiarrítmicos: Pueden ser cardiodepresores o proarrítmicos. Solo se
aprueba la amiodarona.
Calcioantagonistas: Aumentan el riesgo de eventos cardiovaaculares,
excepto el amplodipino.
AINES: pueden causar vasoconstricción periférica y retención de sodio

¿Cuales son los 4 farmacos de uso recomendado en IC?


Diuréticos, IECA, betabloqueadores y digitálicos. Su uso depende de las
caracteristicas clinicas predominantes.

¿En quienes pueden ser perjudiciales los IECA?


En aquellos pacientes que dada su patología de base tienen muy dismi-
nuido el flujo renal y por tanto dependen de la regulación del sistema re-
nina angiotensina aldosterona, en especial de la acción de la
angiotensina en la arteriola eferente. Al bloquear su producción los pa-
cientes pueden caer en insuficiencia renal. Los pacientes con alto riesgo
de esta complicacion son los que presenta IC clase IV de la NYHA, los
pacientes hiponatrémicos, los que tienen estenosis de la arteria renal bi-
lateral o los consumidores de AINES (ya que prostaciclina, prostaglandi-
nas y tromboxanos también autoregulan favorablemente el flujo renal)

¿Cuales son las ventajas de los diureticos en IC?

61
Su efecto aparece en días u horas, a diferencia de la digital, betablo-
queadores o IECA. Cuyo efecto se establece eficazmente en semanas.
Deben usarse asociados

¿Deben usarse betabloqueadores en pacientes con IC clínicamente es-


tables?
Aunque hubiere una buena respuesta al uso de IECA, diuréticos y digitá-
licos, los betabloqueadores deben prescribirse (si no tienen contraindica-
ción) ya que disminuyen el riesgo de la progresión de la enfermedad, de
muerte súbita y de deterioro clínico.

¿Que valores hemodinamicos nos guian en el tratamiento de la IC agu-


da?
El Indice Cardiaco: se considera disminuido cuando es <2.2L/min/m2
La presión capilar pulmonar en cuña (se encuentra como PCWP en in-
glés): se considera baja si es <14mmHg y alta si es > 18-20mmHg.
¿Qué medicamentos se utilizan para aliviar los síntomas de la sobrecarga
(edema, ortopnea)?
Los diuréticos.
¿Cuál es el tratamiento ideal de la insuficiencia cardiaca con falla sistólica?
Digoxina, más un inhibidor de la ECA (enzima de conversión de la an-
giotensina).
¿Cuál es el tratamiento de la falla cardiaca diastólica?
Diuréticos, más -bloqueadores (que disminuyen la frecuencia cardiaca).
¿Cuál es el tratamiento del edema pulmonar agudo?
Oxígeno, diuréticos, vasodilatadores (nitroprusiato o nitroglicerina), sul-
fato de morfina, intubación y presión positiva.
¿Cuáles son los medicamentos que se utilizan en la falla cardiaca congestiva?
 Vasodilatadores: inhibidores de la ECA, hidralazina, isosorbide,
minoxidil y nitroprusiato de sodio.
 Diuréticos: diuréticos de asa, tiazidas.
 Inotrópicos: glucósidos cardiacos (digoxina y digitoxina), agonistas
-adrenérgicos (dobutamina) e inhibidores de la fosfodiesterasa
(amrinona y milrinona).
¿Cuándo están indicados los digitálicos?

62
Cuando existe insuficiencia cardiaca izquierda (no así en la derecha).
¿Cuáles son los beneficios del uso de digitálicos?
El tratamiento por 1-3 meses mejora la calidad de vida, la tolerancia al
ejercicio y reduce los síntomas, por tanto, una pauta para su uso es ini-
ciarlos ante una mediana respuesta al uso IECAs, diuréticos o betablo-
queadores con persistencia de la sintomatología.

¿Que medicamentos se utilizan para la fibrilación auricular (FA) con IC?


Se utiliza rutinariamente digoxina si además existe fibrilación auricular.
Sin embargo los betabloqueadores controlan mejor la respuesta ventricu-
lar ante el estímulo auricular sobre todo durante el ejercicio.

¿Cómo funcionan los digitálicos?


Inhibiendo la actividad de la Na-K ATPasa. Esta acción a nivel car-
díaco provoca aumento del inotropismo mediante el aumento de Na y
Ca intracelulares. A nivel de las fibras vagales sensibiliza a los baro-
receptores cardiacos que secundriamente disminuyen el flujo simpáti-
co eferente del sistema nervioso central. A nivel renal disminuye la
resorción tubular de sodio y el aumento consecuente de sodio a nivel
del túbulo distal causa inhibición de la secreción de renina.

¿A que dosis se usa comunmente la digoxina?


Se inicia y se mantiene a dosis de 0.125 a 0.25 mg diarios. En pacinetes
mayores de 70 años se usan dosis bajas de 0.125, también en los pa-
cientes con insuficiencia renal y aquellos con poca masa muscular

¿Cuales son los efectos adversos de los digitálicos?


Arritmias cardiacas: ritmos ectópicos, de reentrada y bloqueos cardia-
cos. El mas importante es la taquicardia ventricular.
Sintomas gastrointestinales: anorexia nausea y vómito
Síntomas neurológicos: anormalidades visuales, desorientación y con-
fusión.
La toxicidad aparece en niveles séricos por arriba de 2 mg pero ocurre
con niveles menores si existen hipocalemia, hipomagnesemia e hipotiroi-
dismo.

63
.
¿Qué medicamentos propician la toxicidad digitálica?
El uso concomitante de quinidina, verapamilo, espironolctona, fecainida,
propafenona o amiodarona pueden incrementar la digoxina sérica y favo-
recer su toxicidad. Si se usan estos fármacos solo hay que disminuir la
dosis de digoxina.
¿Cómo se maneja la intoxicación por digital?
Prehospitalario: oxígeno, monitoreo cardiaco.
Hospitalario:
 Guiar la terapeutica con niveles de digoxina.
 Si la intoxicacion es leve solo suspender el medicamento
 A las bradiarritmias inestables se les aplica atropina de 0.5 a 1
mg/dosis o marcapasos
 En cao de contracciones ventriculares prematuras, bigemninismo
o trigeminismo inestables utilizar lidocaína.
 La taquicardia ventricular responde a:
 Anticuerpos FAB contra digoxina (digibind)
 Lidocaína en bolos de 100mg o en infusión de 1-4mg/mg.
 Fenitoína sódica IV en bolos de 100mg c/5-10 minutos
hasta alcanzar una dosis de carga de 15mg/kg.
 Magnesio en bolo de 2g y luego en infusión de 1 a 2g
también es útil.
 Evitar el uso de procainamida y bretilio.
¿Cuáles son los efectos cardiovasculares de la adrenalina?
Aumenta la frecuencia cardiaca y hace más amplia la tensión arterial
(aumenta la sistólica y disminuye la diastólica).
¿Cuáles son los efectos cardiovasculares de la noradrenalina?
Mantiene la frecuencia cardiaca normal y tiene un efecto presor neto
pues aumenta igualmente la tensión arterial sistólica y la diastólica.

CARDIOPATÍA ISQUEMICA
¿Cuales son y como se dividen los factores de riesgo coronario?
No modificables
 Sexo masculino
 Edad (55 en hombres y 65 en mujeres)

64
 Menopausia
 Historia familiar de enfermedades cardiovasculares prematuras
(antes de 55 en hombres y antes de 65 en mujeres)
Modificables
 Tabaquismo
 Consumo de alcohol
 Obesidad (IMC>30kg/m2)
 Sedentarismo
 Dislipidemia
 Fibrinógeno
 Anticonceptivos orales
 Hipertensión
 Diabetes Mellitus
 Microalbuminuria o TFG<60ml/min

¿Cuál es el perfil lipídico deseable en los pacientes diabéticos?


 LDL < 100mg/dl
 TG < 150mg/dl
 HDL > 45mg/dl

¿Cuál es el papel de las lipoproteínas de alta densidad en la aterosclerosis?


Extraen el colesterol de la íntima de los vasos por lo que tienen un efecto
protector.
¿Cuál es el índice de la aterosclerosis entre el sexo masculino y el femenino?
4:1
¿Cuál es la personalidad que se asocia a la aterosclerosis?
La personalidad tipo A.
¿Cuáles son los llamados nuevos factores de riesgo cardiovascu-
lar?
La homocisteína, el fibrinógeno, el aumento de la reactividad de las pla-
quetas, la lipoproteína (a) y la hipercoagulabilidad se mencionan como
algunos nuevos factores de riesgo, de estos el mas importante es la ho-
mocisteína.

65
¿Cuáles son los síndromes coronarios agudos?
Angina inestable (AI), infarto no Q y el infarto agudo del miocardio.

¿Qué otros nombres recibe el infarto no Q?


No transmural, no ST, sin elevación ST (IAMNST).

¿Cuál es la diferencia fisiopatológica entre el síndrome AI/IAMNST y el


IAMST?
La angina inestable y el IAM NST son extensiones de un mismo sídrome
clínico en el que la base de la fisiopatología es un imbalance entre el
aporte y la demanda de oxígeno, en estas la causa es la inestabilidad de
la placa que provoca un trombo no oclusivo. En el IAM ST el trombo
ocluye completamente la luz del vaso.

¿A qué se le llama angina?


Se llama angina al síndrome clínico en el cual hay “discomfort” o dolor de
pecho, mandíbula, espalda, hombro o brazo. Esta molestia puede ser
igualmente de origen cardiaco, esofágico, de la pared torácica o de los
pulmones. Para caracterizarse correctamente deben interrogarse, cali-
dad, localización, duración, factores precipitantes y atenuantes. Se divide
en angina estable e inestable.

¿Cuál es la clasificación de la angina de la Sociedad Cardiovascular Ca-


nadiense (CCSC)?
Clase I: Actividad física ordinaria sin angina. La angina solo ocurre en
actividad física extenuante.
Clase II: Angina en las primeras horas de la mañana, al subir escaleras
mas de un piso, caminar 2 cuadras, después de las comidas, en ambien-
te frío o bajo estrés.
Clase III: Limitaciones marcadas de la actividad física. Angina al caminar
una cuadra, o subir escaleras solo un piso.
Clase IV: Inhabilidad de cualquier actividad física por presencia de angi-
na

¿Cuál es el cuadro clínico típico de la angina estable?


Se presenta en pacientes mayores de 50 años, con historia de males-
tar precordial descrito como opresivo y el puño cerrado sobre el pe-
cho. El dolor se irradia al hombro izquierdo y a ambos brazos. Es
desencadenado por el ejercicio y emociones excesivas y aliviado por
el descanso.

66
¿A qué se llama angina inestable?
Operacionalmente la angina inestable es aquella que se presenta de una
de estas tres maneras:
Angina en reposo: angina de mas de 20 minutos que haya sucedido du-
rante la semana previa a la valoración.
Angina de reciente inicio: Angina de grado III de la CCSC que haya ini-
ciado al menos dos meses antes de la valoración.
Angina en aumento: Angina ya conocida con patrón cambiante es de-
cir: mas frecuente, con mayor duración o un menor umbral de presenta-
ción.
Algunos textos también consideran inestable la angina posinfarto.

¿Cuál es la diferencia clínica y electrocardiográfica entre angina inestable


y un infarto sin elevación ST?
No existe diferencia clínica o electrocardiográfica. Ambas pueden no te-
ner alteraciones en el ST o bien alteraciones inespecíficas como un in-
fradesnivel. La diferencia es que el infarto sin elevación ST sí tiene
cambios enzimáticos.

¿Cuáles son los cinco parámetros para realizar la estratificación del ries-
go de la angina inestable?
La historia clínica, las características del dolor, los hallazgos clínicos, los
hallazgos electrocardiográficos y los marcadores bioquímicos cardiacos.

¿Cuál es el perfil de la angina inestable de alto riesgo?


 Síntomas en aumento durante un periodo de 48hr
 Angina de reposo en evolución con duración mayor a 20 minutos
 Cambios clínicos probablemente secundarios a isquemia (edema
pulmonar, nuevo soplo, hipotensión, taquicardia o bradicardia) y
edad mayor a 75 años
 Angina de reposo con: cambios en el ST mayores a 0.05mv, nue-
vo bloqueo de rama o taquicardia ventricular sostenida.
 Marcadores cardiacos francamente elevados.
 Se considera de alto riesgo con la presencia de cualquiera de los
anteriores.

¿Cuál es el perfil de la angina inestable de riesgo intermedio?


 Historia previa de enfermedad arteriooclusiva como: IAM, EVC,
revascularización o uso de aspirina.

67
 Angina de reposo mayor de 20 minutos que ha cedido per se o
angina <20min que cede con nitratos.
 Inversión de la onda T mayor a 0.2 mv
 Ondas Q patológicas
 Marcadores cardiacos medianamente elevados.
 No debe existir ningun indicador de riesgo alto y al menos uno de
los anteriores

¿Cuál es el perfil de la angina inestable de riesgo bajo?


 Angina clase III o IV en ls dos semanas previas.
 EKG sin cambis durante el episodio anginoso.
 Marcadores normales
 Sin indicadores de riesgo alto o intermedio

¿Qué pacientes deben manejarse con medidas de isquemia aguda?


Pacientes que al ingreso presenten:
 Cambios en el ST o en la onda T
 Angina en evolución
 Cambios hemodinámicos
 Marcadores bioquímicos positivos
 Prueba de esfuerzo positiva
Pacientes que se presentan sin cambios pero que los desarrollan al ser
observados de 4 a 8 hr.

¿Cuáles son las medidas para la isquemia aguda?


 Antiagregantes plaquetarios: Acido acetil salicílico, tienopiridinas
(ticlopidina y clopidogrel) o antagonistas de la glicoproteina IIb/IIIa
 Medidas antiisquemia: oxígeno, betabloqueadores, calcioantago-
nistas, IECA.
 Anticoagulación: heparina no fraccionada, heparina de alto peso
molecular.

¿Cuál es la principal toxicidad de la ticlopidina?


La toxicidad hematológica es la mas frecuente haciendo que este total-
mente contraindicada en pacientes con antecedentes de agranulocitosis
trombocitopenia o leucopenia.

¿Cómo es la farmacocinética del clopidogrel?

68
Se puede encontrar en el plasma después de una hora de su administra-
ción, tiene vida media de 8 hr. Tiene un efecto antiagragante significativo
a los 2-3 días de uso pero su efecto máximo aparece hasta los 4 a 7 días
de uso y el efecto persiste hasta por 7 a 10 días después del cese del
tratamiento.

¿Cuándo están recomendados los inhibidores de la GP Iib/IIIa?


 Cuando persiste la sintomatología después de que se ha iniciado
terapia con ASA y tienopiridinas.
 Cuando hay un perfil de alto riesgo
 Cuando se planea realizar intervención percutánea posteriormen-
te.

¿Cuál es el mecanismo de acción de los antagonistas GPIIb/IIIa?


Eptifibatide y tirofiban: son antagonistas sintéticos que imitan la secuen-
cia de aminoácidos del fibrinógeno y tienen afinidad específica por el re-
ceptor con vida media de 2 a 3 hr.
Abciximab: Es un fragmento FAB murino humanizado que actúa contra
GPIIB/IIIA

¿En qué casos no se indican las pruebas de esfuerzo para el estudio de la


angina?
En la angina de Prinzmetal y en la angina inestable.

¿Cuándo se considera una prueba de esfuerzo positiva en los casos de an-


gina estable?
Cuando presenta una depresión del intervalo ST mayor a 0.1 mV bajo la
línea de base y con una duración de 80 milisegundos a partir del punto J,
el segmento ST debe ser horizontal o descendente. Con menos frecuencia
se observa positividad del ST con criterios similares a los del infranivel
(0.5 a 10% de los casos).

¿Cuáles son las indicaciones de la arteriografía coronaria?


Pacientes con angina crónica estable.
Pacientes con angina inestable severamente sintomáticos a pesar de la te-
rapéutica farmacológica y que son prospectos para revascularización.
Pacientes sintomáticos que no han sido diagnosticados propiamente.

69
¿Cuál es la angina de Heberden?
La angina estable con características clásicas asociada a esfuerzo físico.

¿Qué es la angina de Prinzmetal?


La angina variante de Prinzmetal e una forma de angina inestable
que se caraceriza por una elevación cíclica y transitoria de el seg-
mento ST que se resuelve comunmente sin progresar a IAM, es cau-
sada por espasmo de las coronarias y presenta respuesta a
nitroglicerina, nitratos de acción prolongada y a calcioantagonistas en
dosis altas (nifedipino 60 a 120 mg/d, verapamilo 240 a 480 mg/d y
diltiazem 120 a 360 mg/d)

¿Qué es el síndrome X coronario?


Es un síndrome de vasodilatación coronaria insuficiente causada por
un tono vascular aumentado. En el que los pacientes presentan angi-
na o dolor de tipo anginoso con el ejercicio y presentan infradesnive-
les en el segmento ST en la prueba de esfuerzo pero coronarias
normales con flujo lento en la angiografía.

¿Cuáles son los dos signos clínicos de la angina?


El ritmo de galope y el soplo apical transitorio.
¿Cómo se evidencia el espasmo coronario en el episodio anginoso?
Por una elevación concomitante del segmento S-T.

¿Cómo se clasifican los fármacos antianginosos?


Se clasifican en nitratos, beta-bloqueadores y bloqueadores de canales de
calcio.
¿Cómo actúan los nitratos?
Los nitratos, como el dinitrato de isosorbide y la nitroglicerina, relajan el
músculo liso vascular por su conversión intracelular a nitritos y poste-
riormente a óxido nítrico que activa a la guanilatociclasa aumentando los
niveles intracelulares de GMPc. El GMPc defosforila a las cadenas lige-
ras de miosina, lo que resulta en la relajación del músculo liso vascular.

70
¿Cuál es el cuadro clínico del infarto del miocardio?
 20 a 60 % de los pacientes presenta agravamiento de una angina pre-
existente.
 El dolor dura más de 30 minutos; es retroesternal y opresivo.
 El dolor se irradia al cuello, el tórax anterior y posterior, la mandíbu-
la, el epigastrio, los brazos y los hombros.
 La tercera parte de los pacientes presenta disnea como síntoma aso-
ciado.
 Aparece náusea y vómito en 40% de los casos y diaforesis en 50%.
 También pueden coexistir palpitaciones, debilidad generalizada, sín-
cope, déficit neurológico, embolias periféricas y confusión mental.
¿Cuáles son las enzimas de valor diagnóstico en el infarto agudo del mio-
cardio?
 La fracción MB de la creatincinasa (CKMB): cuando es mayor 16UI,
más de 4% de la CK total, a las 6 horas. Tiene sensibilidad y especifi-
cidad mayores a 97%. Las determinaciones seriadas de CKMB son el
estándar de oro para diagnóstico de infarto agudo del miocardio.
 La creatincinasa (CK) total: es positiva en las primeras 4 a 8 horas y
alcanza su valor pico a las 24 horas en promedio.
 La deshidrogenasa láctica (DHL): sobrepasa su valor normal en las
primeras 24 a 48 horas y alcanza su valor pico de 3 a 6 días
 Trop T, I: tienen una sensibilidad de 50% en las primeras 2 a 4 horas
y de 75% después de 4 horas. Son marcadores inequívocos de necro-
sis miocárdica aún con un electrocardiograma normal.

71
¿Cuándo es mas baja la sensibilidad de la de la CKMB?
La sensibilidad de la CKMB para detectar un IAM es muy baja antes
de 6hr y despues de 36hr de ocurrido el evento isquémico.

¿Cuál es el marcador cardiaco mas util para el IAM temprano?


Las isoformas de la CKMB son el marcador mas util para IAM de 3 a
6 hr despues del inicio de los sítomas. Las troponinas y la CKMB total
tienen baja sensibilidad en este periodo.

¿Con que entidades se hace diagnóstico diferencial del IAM?


Hay que diferenciar el IAM de enfermedades como pericarditis, miocardi-
tis, disección aórtica aguda, embolismo pulmonar, y colecisttis aguda en-
tre otros.

¿Cómo se realiza el diagnóstico definitivo electrocardiográfico de IAM?


Al tomar el EKG debe existir un supradesnivel del segmento ST de mas
de 1mm, en dos derivaciones contiguas, a menudo con cambios en espe-
jo recíprocos en las derivaciones contralaterales.

¿Cómo se clasifican los infartos según el grupo de derivaciones afectadas?


 V1, V2 Septal
 V1, V2, V3 Anteroseptal
 V2, V3 Apical
 V1, V2, V3, V4 Anterior
 V1, V2, V3, V4, V5, V6 Anterior extenso
 D1, AVL,V5, V6 Lateral
 D1, AVL Lateral alto
 V5, V6 Lateral bajo
 DII, DIII, AVF Inferior
 V1, V2, V3, V4, V5, V6, DI, AVL Anterior extenso en extensión lateral.

¿Cuáles son los cinco subgrupos de infartos según la arteria responsa-


ble?
 Anterior extenso con trastornos de conducción: territorio de la
descendente anterior (DA)
 Anterior extenso sin trastorno de la coducción: DA media
 Lateral: DA distal o bien la primera diagonal o la circunfleja.

72
 Inferior extenso con cambios en espejo: territorio de la cronaria
derecha (CD) proxmal.
 Inferior pequeño: CD distal.

¿Cuáles son los criterios para diagnosticar IAM en presencia de bloqueo


de rama izquierda?
 Elevación del ST>1mm concordante con el QRS en DII (5puntos)
 Depresión del ST>1mm en V1, V2 y V3 (3 puntos)
 Elevación del ST mayor de 5 mm discordante con el QRS en DIII
y AVF (2 puntos)
 Si se suman mas de 3 puntos la especificidad es mayor al 90%

¿Cuál es la estratificación del riesgo del IAM según Killip y Kimball?

Clase Características Mortalidad (%)


I Sin evidenca de ICC 5.1
II Crepitantes, ingurgitación yuguglar y 13.6
3er ruido
III Edema pulmonar 32.2
IV Choque cardiogénico 57.8

¿Cuáles son las medidas terapeuticas iniciales para el IAM?


Aspirina y teprapia antiplaquetaria: Aspirina (325mg vo seguidos de
150mg/d), Ticlopidina (500mg vo seguidos de 250mg c/12hr), clopidogrel
(300mg vo seguidos de 75mg/d).
Oxígeno
Nitroglicerina: 0.4mg sublingual, 10 a 20 mcg/min IV con aumentos de 5
a 10 mcg cada 5 a 10 min
Reperfusión.

¿Cuáles son las terapias de reperfusión aceptadas?


Fibrinolisis y la reperfusión por vía percutánea.

73
¿Cuáles son indicaciones precisas de fibrinolisis?
Clase I: En ausencia de contraindicaciones, pacientes dentro de las pri-
meras 12 horas desde el inicio de los síntomas con elevación de más de
0.1mv del segmento ST de dos derivaciones contiguas o bien con blo-
queo de rama izquierda nuevo .
Clase II: En presencia de un infarto posterior verdadero o si aun se en-
cuentra dentro de 24 hr del inicio de los síntomas.
Clase III: Nunca después de 24 hr, ni cuando existan solo infradesniveles
ST sin comprobar que se trata de un infarto posterior verdadero.

¿En que situaciones se prefiere utilizar fibrinolisis?


 Cuando el tiemo de evolucion es menor a 3 hrs
 Cuando el tiempo desde la admisión hasta la angioplastía (door-
to-balloon) es mayor de 90 minutos.
 Cuando no se dispone de equipo para angioplastía o el personal
no está suficientemente capacitado.
Si estas condiciones no se cumpen es preferible la angioplastía primaria.

¿Cómo actúa la estreptocinasa?


 Produciendo plasmina a partir de plasminógeno

¿Cuáles son las dos contraindicaciones absolutas de la trombolisis?


Hemorragia interna activa y evento vascular reciente.

¿Cuáles son las indicaciones precisas de angioplastía primaria?


Clase I:
 En pacientes que cumplan los criterios electrocardiográficos que
estén dentro de las primeras 12 hr de los síntomas, a menos de
90 minutos desde admisión hasta angioplastía (door-to-balloon).
 Excepto aquellos que como ya se mencionó tengan menos de 3hr
de evolución y que no tenan angioplastía disponible en la hora si-
guiente.
 En pacientes menores de 75 años.
 En aquellos inelegibles para fibrinolisis.
Clase III: Nunca en pacientes que se encuentran estables hemodinámica
y elécticamente a mas de 12 horas de evolución.

¿En quienes se encuentra indicada la cirugía de puenteo de arterias co-


ronarias (Coronary artery bypass grafting-CABG) de urgencia?

74
Clase I:
 En aquellos pacientes con angioplastia fallida que continuan ines-
tables
 En pacientes con contraindicaciones para angioplastía o tromboli-
sis
 En pacientes menores de 75 años con las condiciones previas
 Cuando la anatomía coronaria es adecuada para el procedimien-
to.

¿En quienes debe sospecharse infarto del ventrículo dercho (VD)?


Debe buscarse evidencia de infarto del VD en todos aquellos pacientes
que tengan infarto inferior y en los qe se presenta el síndrome clínico de
hipotensión, aumento de la presión venosa yugular y auscultación pul-
monar normal.

¿Cuales son las manifestaciones elecrocardiograficas de infarto del VD?


Se debe realizar la toma de derivaciones derechas en especial la deriva-
ción V4R. Puede acompañarse de supradesniveles en el ST mayores a 1
mm en v1 a v3. Los signos electrocardiográficos se resuelven en las 10
primeras horas de evolución.

¿Cuáles son las medidas de tratamiento del infarto del VD?


Mantenimiento de la precarga, dismiución de la poscarga del VD, soporte
inotrópico del VD y reporfusión temprana.

¿Cuáles son los dos principales tipos de complicaciones de un IAM?


Electricas: Taquicardia ventricular, taquicardia supraventricular, extrasís-
toles ventriculares, firilación ventricular, ritmos acelerados de la unión y
disfunción sinusal entre otros.
Mecánicas: Ruptura de cuerdas tendinosas con disfunción valvular mi-
tral, ruptura septal, ruptura de la pared libre y ruptura de aneurisma ven-
tricular.

¿Cuáles son los objetivos de la prevención secundaria de un IAM?

75
¿Por cuál mecanismo el captopril es un protector posinfarto contra insufu-
ciencia cardiaca secundaria?
Disminuye la dilatación ventricular y la falla de bomba, ya que también
disminuye la remodelación muscular patológica; además atenúa la acti-
vidad neurohumoral.

ALTERACIONES DE LA CONDUCCIÓN
¿Cuánto dura la demora aurículo-ventricular normal?
100 milisegundos.
¿Cuáles son las principales perturbaciones de la conducción?
El bloqueo sinoauricular, el bloqueo aurículo-venticular y el bloqueo in-
traventricular.
¿Cuáles son las principales manifestaciones de la disfunción del seno?
 Bradicardia sinusual extrema.
 Paros sinisuales.
 Bloqueos sinoauriculares.
 Síndrome de bradicardia-taquicardia.
 Fibrilación auricular de respuesta ventricular lenta.
 Ritmos de escape de la unión (nodal).
¿Cuáles son los tipos de bloqueo sinoauricular?
 Bloqueo de salida sinoauricular de primer grado: el tiempo de conducción
sinoauricular está aumentado, no se detecta por electrocardiografía
convencional; requiere registros intracardiacos.
 Bloqueo de salida sinoauricular de segundo grado: es una falla intermitente
en la conducción del impulso; se manifiesta por la ausencia intermi-
tente de ondas P.
 Bloqueo de salida sinoaricular de tercer grado o completo: se caracteriza por
la falta de actividad auricular, o bien por la presencia de un marcapa-
so ectópico; sólo se puede distinguir del paro sinusal mediante regis-
tros intracardiacos de la actividad sinusal.
¿Qué es el bloqueo aurículo-ventricular de primer grado?
Es un retraso en la conducción evidenciado por un intervalo P-R aumen-
tado más de 0.21 seg. Clínicamente se observa en los sujetos con tono va-

76
gal aumentado (atletas), pacientes que utilizan fármacos que causan bra-
dicardia sinusal como la digital, los bloqueadores de canales de calcio o
los simpaticolíticos (beta-bloqueadores), en las miocardiopatías, en los in-
fartos de la pared inferior.
¿Qué es el bloqueo aurículo-ventricular de segundo grado, tipo Mobitz I?
También se conoce como bloqueo de Wenckebach, ya que en este blo-
queo el intervalo P-R se prolonga progresivamente hasta lograr el blo-
queo de un impulso. Ocurre secundariamente a una afección nodal. La
pausa siguiente no se considera pausa compensatoria, pues es menor que
dos intervalos sinusales normales. Se reconoce fácilmente si el intervalo
P-R del nuevo impulso es claramente menor del que ocurrió en el último
impulso antes de la pausa. Se puede observar en la intoxicación por cual-
quier fármaco que cause bradicardia sinusal, en los infartos de la pared
inferior y en los individuos con tono vagal aumentado. No requiere tera-
pia agresiva.
¿Qué es el bloqueo aurículo-ventricular de segundo grado, tipo Mobitz II?
Es un bloqueo secundario a un daño dentro del sistema His-Purkinge;
por tanto, se ubica infranodalmente. Se puede observar un P-R prolonga-
do constante, ya sea en relación 2:1, 3:1, etc. Ocurre generalmente tras
los infartos ánteroseptales o en los desórdenes esclerodegenerativos del
esqueleto fibroso del corazón. Es de gran importancia reconocerlo, ya
que evoluciona frecuentemente a bloqueo aurícilo-ventricular completo,
por lo que requiere implantación de marcapaso.
¿Qué es el bloqueo aurículo-ventricular completo?
Es aquel en que observamos disociación aurículo-ventricular y ritmo de
escape.
¿Cuáles son los tipos de bloqueo aurículo-ventricular completo?
 De QRS normal: cuando el escape es de morfología nodal, la frecuencia
es de 40 a 55 latidos por minuto; y se incrementa por el ejercicio o los
atropínicos; la lesión probablemente se ubica en el nodo aurículo-
ventricular.
 De QRS ancho: cuando el escape es de morfología ventricular y las fre-
cuencias menores a 40 latidos por minuto la lesión se encuentra en el
Haz de His o distal a éste.

77
¿Qué son las crisis de Stokes-Adams?
Son episodios de isquemia cerebral que se manifiestan como síncope se-
cundario a bloqueo A-V avanzado.
¿Cómo se trata la disociación A-V?
Descontinuando cualquier agente que cause bradicardia sinusal, acele-
rando el ritmo con vagolíticos (anticolinérgicos) o mediante la inserción
de un marcapaso si el ritmo de escape es insuficiente.
¿Qué es un escape nodal?
Es un QRS angosto, no precedido de onda P y que ocurre posteriormente
a una pausa diastólica mayor que la normal (0.64 segundos en lugar de
0.54); por tanto, se puede observar en los diferentes bloqueos, pausas y
bradicardias.
¿Cuáles son las características de un escape ventricular?
Son complejos QRS posteriores a una pausa anormal de morfología se-
mejante a la que se observa en el bloqueo de rama avanzado y no prece-
dido de una onda P.
¿Qué es un bloqueo de rama?
Es un daño intrínseco de la conducción del sistema intraventricular, ya
sea izquierdo o derecho, que se manifiesta por un QRS prolongado: más
de 120 milisegundos, si es completo (avanzado), y de 100 a 120 milise-
gundos si es incompleto. El vector del QRS se orienta en dirección de la
región miocárdica donde la despolarización está retardada.
¿Cuáles son las características del bloqueo de la rama derecha del Haz de
His?
Presenta complejos predominantemente positivos en V-1 tipo rSR y nega-
tivos en V-6 del tipo qRS; el vector del QRS se orienta hacia la izquierda.
Podemos observarlo en la enfermedad cardiaca congénita (septal) o ad-
quirida (valvular).
¿Cuáles son las características del bloqueo de la rama derecha del Haz de
His?
Presenta complejos predominantemente negativos en V-1 de tipo QS y
positivos en V-6 de tipo R; el vector del QRS se orienta hacia la derecha.

78
Se observa en la cardiopatía isquémica, la hipertensión crónica, la car-
diomiopatía o la enfermedad valvular aórtica severa.
¿Cuáles son las características del bloqueo parcial?
Es un bloqueo que sólo afecta a uno de los fascículos de la rama izquier-
da del Haz de His, ya sea el anterior o el posterior (hemibloqueo), que
generalmente no prolonga la duración del QRS pero sí altera la dirección
del vector QRS.
¿Cuáles son los tipos de bloqueo bifascicular?
 Bloqueo de la rama derecha del Haz de His y bloqueo fascicular iz-
quierdo posterior.
 Bloqueo de la rama derecha del Haz de His y bloqueo fascicular iz-
quierdo anterior.
 Bloqueo de la rama derecha del Haz de His completo.
 La alternancia entre BRDHH y BRIHH es un signo de enfermedad tri-
fascicular.
¿Qué es el síndrome de Wolff- Parkinson-White?
Es un síndrome caracterizado por un intervalo P-R corto debido a que
existe empastamiento de la onda P en la rama ascendente del QRS, lo
que se conoce como onda delta. También hay onda T invertida.

ALTERACIONES DEL RITMO.


¿Cómo se clasifican las arritmias?
En taquiarritmias y bradiarritmias.
¿Cuáles son la bradiarritmias con conducción aurículo-ventricular normal?
La bradicardia sinusal y la pausa sinusal.
¿Cuáles son los ritmos pasivos?
Los escapes, la migración del marcapaso, y los ritmos nodal e idioventri-
cular.
¿Cuáles son los focos miocárdicos que determinan el automatismo cardiaco?
 Nodo sinusal o de Keith y Flack: determina el ritmo sinusal, su rango
de frecuencia normal es de 60 a 100 latidos por minuto.
 Nodo aurículo-ventricular o de Aschoff-Tawara: determina el ritmo
nodal; su frecuencia va de 50 a 60 latidos por minuto.

79
 Haz de His y fibras de Purkinge: cuando estos centros toman la fun-
ción de marcapasos se llama ritmo idioventricular. El ritmo es de 40 a
50 y menor de 40 latidos por minuto respectivamente.
¿Cuáles son las características del ritmo nodal?
La frecuencia cardiaca es de 50 a 60 latidos por minuto; la morfología del
QRS es normal, o bien, semejante a la del bloqueo de rama y la onda P
es negativa en D-II, D-III y AVF y puede preceder o seguir al QRS
¿Cuáles son las características del ritmo idioventricular?
La frecuencia cardiaca es menor de 40 latidos por minuto, el complejo
QRS es ancho y mellado y hay ausencia de onda P.
¿Qué es el ritmo idioventricular acelerado?
También se conoce como taquicardia ventricular lenta, ya que tiene fre-
cuencias de 60 a 120 pero con morfología idioventricular. Ocurre en el
infarto del miocardio o durante la reperfusión.
¿Cómo se clasifican las extrasístoles supraventriculares?
En auriculares y complejos aurículo-ventricular de la unión.
¿De dónde provienen los complejos aurículo-ventricular de la unión?
Provienen de la parte superior del Haz de His, anterior a su bifurcación.
Es mucho menos frecuente ver este tipo de extrasístoles que las auricula-
res o las ventriculares.
¿Cuáles son las características de las extrasístoles auriculares?
La onda P que se adelanta y tiene una morfología diferente a la del com-
plejo sinusal y que puede presentarse positiva, negativa o difásica, sea o
no seguida del complejo ventricular, con un intervalo P-R más corto y
que se observa mejor en D-I sugiere una extrasístole auricular.
¿Cuáles son las características de los complejos aurículo-ventricular de la
unión?
Son complejos QRS de morfología normal, no precedidos de onda P. La
onda P aparece negativa y posterior al QRS en D-II, D-III y AVF (onda
P retrógrada). Se puede observar en cualquier enfermedad cardiaca o en
la intoxicación por digitálicos.
¿Cuál es el significado clínico de las extrasístoles supraventriculares?
Se observan en sujetos simpaticotónicos, hipertiroideos; en la insuficien-
cia cardiaca, en la fiebre reumática activa, en la dilatación auricular se-

80
cundaria a valvulopatía mitral o tricuspídea, en la isquemia miocárdica,
en la intoxicación por cafeína, nicotina o alcohol y otras.
¿Cuáles son las características de las extrasístoles ventriculares?
Se puede observar un complejo QRS ancho (>0.14 segundos) no precedido
de onda P. El segmento ST y la T tienen una dirección opuesta al QRS. Las
extrasístoles ventriculares y los complejos sinusales pueden asociarse de dos
maneras: la unifocal exhibe complejos QRS de morfología consistente y aco-
plamiento fijo al complejo sinusal; cuando el acoplamiento a dichos complejos
no es fijo y la periodicidad entre extrasístoles es independiente y constante se
sospecha un foco ectópico, lo que convierte al fenómeno en multifocal.
¿Cómo se clasifican los mecanismos de las taquiarritmias?
En desórdenes de la propagación del impulso y desórdenes de la forma-
ción del impulso.
¿Cuáles son los desórdenes de la propagación del impulso (reentrada)?
Cuando hay un bloqueo unidireccional en alguna parte del sistema eléc-
trico cardiaco se puede producir la circulación repetitiva de un impulso
causando una taquicardia sostenida. Las arritmias por reentrada pueden
iniciar y terminar por complejos prematuros y estimulación rápida. La
respuesta de estas arritmias a la estimulación puede ayudar a distinguirlas
de las ocasionadas por actividad desencadenada.
¿Cuáles son los desórdenes de la formación del impulso?
Desórdenes por actividad incrementada y desórdenes por actividad des-
encadenada.
¿Cuáles son las características de los desórdenes por actividad incrementada?
Cuando alguno de los diferentes focos del miocardio especializado sufre
un incremento en su excitabilidad puede ganar actividad como marcapa-
so automático. Esto puede ser a consecuencia de la acción de catecola-
minas endógenas, desórdenes electrolíticos (hiperkalemia), hipoxia,
isquemia, efectos mecánicos y drogas (digital).
¿Cuáles son las características de los desórdenes por actividad desencadenada?
Requieren un cambio en la frecuencia cardiaca como factor desencade-
nante. Pueden ser causados por posdespolarizaciones tempranas que ocurren
en las fases 2 y 3 del potencial de acción. Se observan en la bradicardia,

81
la hipokalemia y, en general, en cualquier situación que prolongue la du-
ración del potencial de acción. Las posdespolarizaciones tardías ocurren al
completarse la fase 3 del potencial de acción y se observan en los casos de
intoxicación digitálica, hiperkalemia, hipercalcemia o de concentraciones
locales aumentadas de catecolaminas.
¿En qué se diferencian las taquiarritmias comunes?
Taquicardia Aleteo au-
Taquicar- Fibrilación Taquicardia
Variables paroxística ricular
dia sinusal auricular ventricular
auricular (flutter)
Frecuencia 100 a 200 169 a 190 160-90 140 a 160 100 a 230
Ritmo Regular Regular irregular regular Irregular
QRS Normal normal normal normal anormal
Actividad auri- Ausente o no Ondas de
P sinusal ausente P sinusal
cular sinusal aleteo
Relación Encubierta Encubierta
Si no No
P-QRS con RVR* con RVR
Sin respuesta Incremento
Masaje carotí- Se torna Sin res- Sin respues-
o se torna si- del blo-
deo lento puesta ta
nusal queo
El QRS es normal siempre que no exista una alteración concomitante en la conducción
*RVR= Respuesta ventricular rápida
¿Qué efecto tienen las maniobras sobre el seno carotídeo?
Las maniobras vagales tienen una doble función terapéutica y diagnósti-
ca. El masaje al seno carotídeo y la maniobra de Valsalva aumentan el
tono vagal disminuyendo la descarga del nodo sinusal y prolongando el
tiempo de conducción del nodo A-V así como su refractariedad.
¿Cuál es la respuesta de la arritmia por reentrada intranodal y de la taqui-
cardia auricular bloqueada a las maniobras vagales?
La primera muestran una disminución abrupta de la frecuencia ventricu-
lar media, en tanto que en la segunda también disminuye la frecuencia
ventricular media y retorna al ritmo de base aunque la arritmia persiste.
¿Cuál es el tratamiento para la fibrilación auricular?
Depende de que se acompañe o no de insuficiencia cardiaca. Si hay insu-
ficiencia cardiaca el tratamiento de elección es la digoxina (0.5mg IV ini-
cialmente): si no hay insuficiencia cardiaca se utilizan beta-bloqueadores.

82
¿Cuál es el tratamiento de la taquicardia paroxística supraventricular?
La adenosina; otras arritmias no reaccionan a este fármaco.
¿Cómo se trata la taquicardia venticular aguda?
El tratamiento se encuentra determinado por la presencia de deterioro
hemodinámico y la duración de la arritmia. Si hay hipotensión, insufi-
ciencia cardiaca o angina debe realizarse cardioversión eléctrica con 100
a 360 J. Si el paciente tolera la arritmia entonces se utiliza lidocaína
(1mg/kg en bolo IV).
¿Cuál es el tratamiento de la taquicardia ventricular sostenida?
La taquicardia ventricular sostenida es una variante crónica en la que hay
descargas de tres o más latidos con duración mayor a 30 segundos. El tra-
tamiento se lleva a cabo con antiarrítmicos o un desfibrilador-
cardiovertor implantable en los casos refractarios a fármacos.
¿Cuáles son los antiarrítmicos clase I (bloqueadores de canales de sodio)?
 IA: hacen más lenta la despolarización en la fase 0 del potencial de
acción: quinidina, procainamida, disopiramida y moricizina.
 IB: acortan la repolarización de la fase 3: lidocaína, mexiletina y feni-
toína.
 IC: hacen marcadamente lenta la despolarización de fase 0: flecainida
y propafenona.
¿Cuáles son los antiarrítmicos clase II?
Actúan como bloqueadores del adrenorreceptor beta: esmolol, metopro-
lol y propranolol.
¿Cuáles son los antiarrítmicos clase III?
Actúan bloqueando los canales de potasio y son la amiodarona, el breti-
lio y el sotalol.
¿Cuáles son los antiarrítmicos clase IV?
Son los que actúan bloqueando a los canales de calcio: verapamilo y dil-
tiazem.

VALVULOPATÍAS Y CARDIOPATÍAS CONGÉNITAS.

¿Cómo se llama el pulso de la insuficiencia aórtica?

83
Pulso colapsante, de corrigan o de martillo hidráulico.
¿A qué cardiopatía se asocia un ritmo de cinco tiempos?
A la anomalía de Ebstein.
¿A qué cardiopatía se asocia el P2 fijo?
A la comunicación interauricular.
¿A qué cardiopatías se asocia el S-3?
Se asocia al galope ventricular, insuficiencia mitral y la insuficiencia ven-
tricular significativa.
¿Cuáles son dos causas de chasquido de eyección?
La válvula semilunar estenótica y la arteria pulmonar hipertensa disten-
dida.
¿Cuáles son causas de chasquido mesosistólico o telesistólico?
Prolapso de la válvula mitral y prolapso de válvula tricúspide.
¿Cuáles son causas de S-2 permanente?
Todas las que involucran al ventrículo derecho como la estenosis pulmo-
nar, la comunicación interauricular y el bloqueo de la rama derecha del
Haz de His.
¿Cuándo son normales el soplo carotídeo y el soplo de Still?
En niños de 3 a 8 años.
¿Cuándo es normal un soplo pulmonar por flujo?
De los 6 a los 18 años.
¿Cuándo encontramos pulsus alternans?
Cuando la contractilidad cardiaca está disminuida o existen grandes de-
rrames pleurales.
¿Cuándo aparece el pulso paradójico de Kussmaul?
La presión sistólica disminuye en 10 mm Hg durante la inspiración. Se
encuentra en el taponamiento cardiaco.
¿Cuáles son los tres signos de la hipotensión ortostática?
 Disminución de 20 mm Hg en la presión diastólica.
 Disminución de 10 mm Hg en la presión sistólica.
 Aumento de 20 latidos por minuto en la frecuencia cardiaca.
¿Como se relaciona la pérdida de volúmen con la clínica?

84
 20% (1000ml):taquicardia.
 30%(1500ml):hipotensión ortostática.
 40%(2000ml):hipotensión supina y taquicardia.
 50%(3000ml):choque.
¿Cuáles son las indicaciones de reparación valvular?
Endocarditis fúngica, falla cardiaca crónica y falla de la terapia antibiótica.
¿Cuáles son las valvulopatías más importantes y como se identifican clíni-
camente?
 Estenosis mitral: el soplo se localiza en el ápex.
 Regurgitación mitral: el soplo se transmite hacia la axila izquierda y el
área infraescapular izquierda.
 Estenosis aórtica: el soplo se escucha en el segundo espacio intercostal
derecho, en las carótidas y en el área interescapular superior.
 Regurgitación aórtica: el soplo se escucha mejor en el tercer o cuarto
espacio intercostal izquierdo sobre el borde esternal, y se relaciona
con un soplo apical mesodiastólico (soplo de Austin-Flint).
 Estenosis y regurgitación tricuspídeas: el soplo se percibe en el tercero o
cuarto espacio intercostal, a lo largo del borde esternal izquierdo.
 Estenosis pulmonar: el soplo se irradia a la axila y espalda y se escucha
difusamente en el tórax en los lactantes.
¿Cuáles son la 1ª y 2ª causas de regurgitación tricuspídea?
Falla cardiaca izquierda y endocarditis por drogas administradas por las
vía intravenosa.
¿Cuáles hallazgos clínicos acompañan a la estenosis aórtica?
Un soplo en la base, y electrocardiograma con depresión del segmento S-
T e inversión de la onda T.
¿Cuál es la válvula más afectada por la cardiopatía reumática?
La mitral.
¿Qué soplos son mesosistólicos?
Los de la estenosis aórtica y de la estenosis pulmonar.
¿Qué soplos son holosistólicos?
Los de regurgitación aurículo-ventricular y los de cortocircuito izquierda-
derecha a nivel ventricular.
¿Qué soplos son diastólicos tempranos?

85
Los de la insuficiencia aórtica o insuficiencia pulmonar.
¿Cómo afectan a los soplos la maniobra de Valsalva y la bipedestación?
Hacen que la mayoría de los soplos disminuyan en duración e intensidad.
Los soplos de la cardiomiopatía hipertrófica y del prolapso de la válvula
mitral aumentan con la bipedestación.
¿Cómo afecta a los soplos la posición de cuclillas?
La mayoría de los soplos aumenta en esta posición, con la excepción de
los soplos de la cardiomiopatía hipertrófica y del prolapso de la válvula
mitral que se tornan más suaves.
¿Cómo afectan a los soplos los latidos ventriculares prematuros y la fibri-
lación auricular?
Los soplos sistólicos de semilunares estenóticas aumentan. Por el contra-
rio, los soplos sistólicos de la regurgitación aurículo-ventricular disminu-
yen, se hacen más cortos o no cambian.
¿Cómo afecta la respiración a los soplos?
Todos los soplos correspondientes a estructuras de la circulación derecha
aumentan con la inspiración; también aumentan el S-3 y S-4. Todos los
soplos de la circulación izquierda aumentan con la espiración.
¿Qué soplos son mesodiastólicos?
 El de la estenosis mitral.
 El de Carey-Coombs (mesodiastólico apical que aparece en la fiebre
reumática).
 Los de flujo mitral aumentado, como sucede en la insuficiencia mi-
tral, los defectos ventriculares septales o la persistencia del conducto
arterioso.
 El de la estenosis tricuspídea.
 Los de flujo tricuspídeo aumentado como en los defectos septales au-
riculares, la insuficiencia tricuspídea y el drenaje venoso anómalo.
¿Qué soplos son continuos?
El de la persistencia del conducto arteriorso y el de la ruptura de un
aneurisma del seno de Valsalva.
¿Cuál es el síndrome de Holt-Oram?

86
Es un defecto de tipo ostium secundum acompañado de deformidades
óseas en los miembros superiores como la agenesia de pulgares.
¿Cuál es la tríada del taponamiento cardiaco?
Aumento de la presión venosa, disminución de la presión arterial y pre-
cordio apagado (triada de Beck).
¿Cuáles son los factores predisponentes para la endocarditis?
Los procedimientos quirúrgicos dentales, urológicos y gastrointestinales;
el uso de drogas IV y lesiones preexistentes.
¿Cuál es la endocarditis de Liebman-Sacks?
Es la endocarditis no bacteriana que se presenta en el lupus eritematoso
sistémico.
¿Cuáles son los signos clínicos de miocarditis?
Anormalidades de la conducción, arritmias y falla cardiaca.
¿Cuáles son las manifestaciones inmunológicas de la endocarditis bacte-
riana subaguda?
 Manchas de Roth: citioides en la retina.
 Nódulos de Osler: nódulos hipersensibles en la falange terminal (dife-
renciar de los de Heberden).
 Lesiones de Janeway: manchas indoloras en las palmas y plantas.
 Petequias y púrpuras.
 Glomerulonefritis proliferativa.
¿Cuáles son los criterios mayores de Duke para el diagnóstico de endocar-
ditis?
 Dos cultivos positivos para S. viridans, S. aureus o microorganismos
del grupo HACEK.
 Evidencia de endocardio involucrado por ecocardiografía o la pre-
sencia de un nuevo soplo.
¿Cuáles son los criterios menores de Duke?
 Predisposición por daño valvular o por ser usuario de drogas IV.
 Fiebre >38º
 Fenómenos vasculares, embolismo, aneurisma micótico.

87
 Fenómenos inmunológicos: manchas de Roth, nódulos de Osler y
glomerulonefritis.
 Ecocardiografía que no es concluyente de un criterio mayor.
 Evidencia menor de infección.
¿Cómo se interpretan los criterios de Duke?
Se considera diagnóstico de endocarditis cuando hay dos criterios mayo-
res, tres menores y un mayor o cinco menores.
¿Cuál es el tratamiento de la endocarditis de válvula nativa?
El causante principal es S. viridans. El tratamiento se realiza con penicili-
na (12 a 18 millones) más gentamicina (1g), o bien penicilina (12-18 mi-
llones) más ceftriaxona (2 mg) durante 4 semanas.
¿Cuál es el tratamiento de la endocarditis de válvula artificial?
Vancomicina (15 mg/kg) más gentamicina (1 mg) durante 14 días.

88
89
CIRUGÍA

¿Qué arteria irriga el polo superior de la tiroides?


La tiroidea superior, rama de la carótida externa.
¿Qué arteria irriga el polo inferior de la tiroides?
La tiroidea inferior, rama del tronco tirocervical, rama de la subclavia.
¿Qué arteria irriga el istmo de la tiroides?
La tiroidea imma, rama del arco aórtico.
¿Qué síntomas presenta la tiroides lingual?
Obstrucción y dificultad para hablar.
¿Cuál es la infección de tejidos cervicales más frecuente?
La angina de Ludwing.
¿Qué es la angina de Ludwing?
Es una infección de los espacios sublingual y submaxilar secundaria a in-
fección de las raíces dentales.
¿Cuál es el tratamiento quirúrgico de la angina de Ludwing (AL)?
Se trata mediante drenaje por incisiones submentonianas bilaterales. De-
be hacerse da manera urgente cuando la vía aérea está comprometida.
¿Cuáles son los agentes causantes de la AL y su tratamiento?
Estreprococos, estafilococos, bacteroides y fusobacterium. Se trata con
tres regímenes farmacológicos: penicilina-metronidazol, ampicilina-
sulbactam o clindamicina-cefalosporina.
¿Cuáles son las zonas anatómicas en que se divide el cuello?
 Zona I: comprende la base del cuello y es la entrada al tórax.
 Zona II: se encuentra por arriba de la base del cuello y abajo del ma-
xilar inferior. Las estructuras que se encuentran en esta zona son la
carótida, la yugular, la laringe, la tráquea y el esófago. Una lesión

90
penetrante en esta zona requiere exploración quirúrgica sin importar
los datos clínicos.
 Zona III: se encuentra por arriba del ángulo del maxilar inferior.
¿Cuál es el tumor parotídeo más común?
 El adenoma pleomórfico.
¿Cuál es la tumoración del cuello que sigue a los movimientos de deglu-
ción?
Las neoplasias tiroideas siguen generalmente este comportamiento.
¿Qué estructura se ocluye más frecuentemente por sialolitiasis?
El conducto de Wharton.
¿Cuál es la lesión esofágica benigna más frecuente?
El leiomioma es la lesión más frecuente y se presenta con dolor y disfagia.
El tratamiento es la resección extramucosa con abordaje por toracoto-
mía.
¿Qué es el divertículo de Zenker?
Es un divertículo que se produce por tracción en la parte alta del esófago
y se trata mediante miotomía del cricofaríngeo.
¿Cuál es el tratamiento profiláctico antibiótico para pacientes programa-
dos para cirugía dental?
Amoxicilina: 50 mg/kg, en niños; 2 g, en adultos.
¿Cuáles son los síndromes posgrastrectomía?
 Síndrome de reflujo alcalino: es el trastorno más frecuente, que cursa
con dolor epigástrico posprandial, náusea, vómito y pérdida de peso.
 Síndrome de asa aferente: es una obstrucción intermitente de la anas-
tomosis que causa síntomas gastrointestinales que se alivian única-
mente con el vómito.
 Síndrome dumping (de vaciamiento rápido): cursa con síntomas de
respuesta simpática por la llegada directa de líquido hiperosmolar al
intestino delgado causando distensión yeyunal.
 Diarrea posgastrectomía.
¿Cuál es el tratamiento para el síndrome de reflujo alcalino y el de asa afe-
rente?
Es quirúrgico. Se debe realizar una Y de Roux.

91
¿Cuál es el tratamiento del síndrome dumping?
Hay que administrar ocreotide y evitar un alto contenido de carbohidratos
en la dieta.
¿Cuál es el cuadro clínico de una perforación intestinal?
Dolor agudo, rigidez abdominal, rebote positivo y ruidos intestinales
apagados.
¿Cuándo se debe sospechar una intususcepción?
En pacientes de 2 meses a 2 años que representan cólicos que se alternan
con ondas peristálticas, una masa con forma de “salchicha” en la palpa-
ción abdominal, evacuaciones en “jalea de grosella” y generalmente se-
cundaria a una infección viral (verificar el antecedente de fiebre).
¿Cuándo se debe sospechar un vólvulo en los pacientes pediátricos?
Cuando hay vómito agudo de características biliares en pacientes que se
encuentran dentro del primer año de vida.
¿Qué se observa en los pacientes con obstrucción a nivel gástrico?
Alcalosis, hipocloremia (por el vómito repetido) e hipokalemia.
¿Cómo se maneja la hemorragia del tubo digestivo alto secundaria a vári-
ces esofágicas?
Si no es aguda, se ligan las várices; si es aguda se esclerosa.
¿Cuál es el tratamiento de emergencia de la hemorragia del tubo digestivo
alto secundaria a várices esofágicas?
El shunt o derivación porto-cava.
¿Cuál es el riesgo operatorio de la derivación porto-cava según la clasifica-
ción de Child?
A= 2%, B=10%, C=50%.
¿Qué es el procedimiento de Warren?
Es un shunt portosistémico (esplenorrenal).
¿Qué es el procedimiento de Sujiura?
Es la devascularización esofágica.
¿Cuál es la mortalidad en casos de trauma hepático?
10-20% en lesiones parenquimatosas; 50% en lesiones vasculares.
¿Cómo se logra la hemostasia hepática?
Con empaquetamiento o compresión portal (Pringle).

92
¿De qué se componen los cálculos biliares?
75% de colesterol y 10 a 25% de bilirrubinato de calcio.
¿De qué se componen las sales biliares?
Ácido quenodesoxicólico, ácido desoxicólico, taurina y glicina.
¿Cuál es el método de elección para detectar cálculos biliares?
El ultrasonido.
¿Cuál es el método de elección para diagnosticar cálculos biliares?
La colecistografía oral.
¿Para qué se utiliza el paneo hepatobiliar con ácido inmunodiacético (HIDA)?
Para diagnosticar colecistitis, quistes del colédoco y derrame biliar. Si se
usa asociadamente con colecistocinina (CCK) se puede estudiar la disci-
nesia biliar.
¿Dónde se ubica más frecuentemente la obstrucción íleo biliar?
En el ileon terminal (en la válvula ileocecal).
¿Cómo son los cálculos biliares de colesterol?
Grandes y suaves.
¿Cómo son los cálculos biliares de pigmento?
Son suaves y de color verde o negro.
¿Con qué se asocian los cálculos biliares de calcio?
Con inflamación, infección o aumento de la beta-glucoronidasa.
¿Cuáles son causas frecuentes de colecistitis acalculosa?
Salmonelosis, poliarteritis nodosa, sepsis y trauma.
¿Cuál es la triada de la colangitis?
Fiebre, ictericia y dolor. En ocasiones la coledocolitiasis puede presentar-
se con el mismo cuadro clínico, por lo que hay que hacer diagnóstico di-
ferencial entre éstas.
¿Cuál es el tratamiento profiláctico antibiótico para pacientes programa-
dos para cirugía gastro-biliar y de colon?
Cefazolina (1-2 mg IV) más metronidazol (0.5mg IV).
¿Cuáles son los dos abscesos abdominales más comunes?
El subfrénico y el subhepático.
¿Cuáles son las características clínicas de los abscesos?
 Fiebre en agujas después de 2 semanas de posoperatorio.
 Dolor y masa abdominal.

93
¿Cuál es la antibioterapia empírica para el traumatismo penetrante abdo-
minal?
Se aplica un doble esquema de metronidazol-amikacina.
¿Cuál es la causa más frecuente de hemorragia retroperitoneal en un trau-
matismo?
La fractura pélvica.
¿Cuáles son los principales ligamentos pélvicos ginecológicos?
 Ligamento cardinal: fija el cuello a la pared pelviana y la parte supe-
rior vaginal. Las arterias uterinas discurren a través el ligamento car-
dinal.
 Ligamento úterosacro: se extiende del cuello al sacro.
 Ligamento ovárico: es un ligamento avascular que se extiende del
ovario al mesosálpinx.
 Ligamento redondo: une al útero con la trompa y discurre por el ca-
nal inguinal para terminar en los labios mayores.
¿Cuáles son las paredes del conducto inguinal?
 Anterior: oblicuo externo.
 Superior: el ligamento formado por el oblicuo interno y el transverso.
 Posterior: fascia transversalis.
 Inferior: Ligamento inguinal.
¿Cuándo se presenta una hernia directa?
Cuando existe una debilidad del piso del canal inguinal.
¿Qué estructura se encuentra dañada en las hernias femorales?
El ligamento de Cooper.
¿Cuáles son la primera y segunda causas de obstrucción intestinal en ni-
ños?
Adherencias y hernias.
¿A los cuántos meses se presenta la hernia inguinal?
A los 6 meses.
¿Cuál es la cirugía más frecuente en pacientes pediátricos?
La reparación de la hernia inguinal.
¿De qué lado se localizan más frecuentemente las hernias?
60% del lado derecho, 30% del izquierdo, 10-15 % bilateral.

94
¿Cuál es el procedimiento quirúrgico más utilizado para reparar las hernias
directas?
El de Bassini, que sutura al mismo tiempo la fascia transversalis y el li-
gamento inguinal al tendón conjunto.
¿Con cuál procedimiento se reparan las hernias femorales?
Con el de McVay, que sutura el tendón conjunto y la fascia transversalis
al ligamento de Cooper.
¿Cuál es el tratamiento de las hernias encarceladas?
Es quirúrgico y se debe realizar dentro de las primeras 24 horas por el
riesgo de isquemia.
¿Cuál es el diagnóstico diferencial de la fiebre posoperatoria (recordar las
3 “w”)?
 Wind: pulmonares de 1-3 días.
 Water: vías urinarias de 3-5 días.
 Wound: herida infectada de 5-8 días.
¿Cuál es el tratamiento empírico de la celulitis?
Dicloxacilina o cefalexina IV; cefalexina oral a altas dosis (750-1,000
mg).
¿Cuál es el cuadro clínico de la fascitis necrosante?
 Bulas hemorrágicas.
 Toxicidad sistémica progresiva.
 Descarga fétida.
 Aire en los tejidos.
¿Cuál es el tratamiento antibiótico en los casos de fascitis necrosante, gan-
grena gaseosa y choque tóxico esptreptocócico?
Penicilina sódica cristalina (24,000,000 U) más clindamicina (900 mg).
¿Cuál es el cuadro clínico de la fascitis necrosante (miositis por clostri-
dios)?
 Ocurre en heridas profundas y contaminadas.
 A las 48 horas se encuentra dolor y crepitación.
 En un extendido se observan bacilos esporulados grampositivos.
¿Cuál es el sitio más frecuente para el desgarro de la aorta?
Distal, a la salida de la subclavia izquierda.

95
¿Cuáles son los hallazgos clínicos en la claudicación intermitente?
Dolor en la pantorrilla que ocurre mientras el paciente camina y que cesa
cuando el paciente se mantiene en reposo.
¿Cuáles son los hallazgos clínicos en el dolor de origen isquémico del
miembro pélvico?
Dolor en el pie, incluyendo la cara plantar; se exacerba al elevar el pie,
por lo que el paciente duerme con el pie en el piso. Puede presentarse ne-
crosis y requerirse la amputación.
En caso de claudicación intermitente, ¿con qué padecimientos debe hacerse
el diagnóstico diferencial?
El dolor lumbar o la claudicación neurógena.
¿Cuál es el diagnóstico diferencial del dolor isquémico del miembro pélvico?
La neuropatía diabética y la artritis.
Ante la sospecha de gangrena, ¿con qué padecimiento se debe hacer el
diagnóstico diferencial?
Las úlceras del pie diabético o la osteomielitis subyacente.
¿Cuáles son los índices brazo-tobillo y qué nos indican?
 1: normal.
 0.5: sugiere claudicación intermitente.
 0.2: sugiere origen isquémico del dolor.
 <0.2: inminencia de amputación.
¿Cuáles son las tres complicaciones más frecuentes de la arteriografía?
 Debidas al contraste: nefrotoxicidad y alergia.
 De origen arterial: trombosis y embolismo.
 En el sitio de punción: hematoma y pseudoaneurisma.
¿Cuál es el método diagnóstico de elección para un aneurisma disecante de
la aorta?
La ecografía transesofágica.
¿Cuál es la triada del síndrome de Leriche?
 Claudicación en muslo y en el glúteo.
 Pulso femoral ausente.
 Impotencia sexual.

96
¿Cuál es la diferencia entre la isquemia mesentérica aguda y la crónica?
 Aguda: aparece como un dolor exquisito y fuera de proporción con
los hallazgos físicos (no hay abdomen agudo, rigidez, etc.)
 Crónica: la angina intestinal provoca dolor posprandial que provoca
en el paciente “miedo a comer” con la consecuente pérdida de peso.
¿Cómo se conoce el síndrome de la mesentérica superior?
Como síndrome del corsé.
¿Cuándo está indicado el tratamiento quirúrgico de una obstrucción caro-
tídea?
Cuando es de 60 a 90%, sin importar si existe o no sintomatología.
¿Dónde se observa el signo del cascarón (eggshell)?
En el aneurisma de la aorta abdominal .
¿Cuándo está indicado el tratamiento quirúrgico del aneurisma de la aorta
abdominal?
Cuando es mayor de 5 cm, aún si es asintomático.
¿Cuál es la triada de Virchow?
 Aumento de la coagulabilidad.
 Disminución de la velocidad del flujo de la sangre.
 Trauma.
¿Cuál es el cuadro clínico de la trombosis venosa superficial y su tratamiento?
 Presencia de vena indurada, roja y dolorosa.
 El tratamiento es conservador (medias, elevación de miembros). La
cirugía se indica únicamente por razones estéticas, dolor recurrente o
episodios de sangrado.
¿Cuál es el cuadro clínico de la trombosis venosa profunda y su tratamiento?
Ocurre después de procedimientos de ortopedia (50-70%) y cirugía ab-
dominal (30%). Se presenta con edema, signos de Homans y Orlow posi-
tivos. Puede acompañarse de embolismo pulmonar. El tratamiento se
lleva a cabo con heparina, warfarina y filtro intracaval.
¿Cuál es la diferencia entre phlegmasia alba dolens y phlegmasia cerúlea dolens?
 Phlegmasia alba: se obstruyen las venas ilíaca y femoral. Por contigüi-
dad se afecta la arteria femoral y causa su obstrucción, por lo que se
presenta extremidad fría, pálida y sin pulso.

97
 Phlegmasia cerúlea: se obstruye exclusivamente el retorno venoso. Se
observa una extremidad edematizada, dolorosa, morada y caliente.
¿Cuál es la complicación más frecuente de la cirugía laparoscópica?
El neumotórax. Se identifica por la aparición de hipotensión súbita, desa-
turación y aumento en la presión ventilatoria. Puede ser secundario a de-
fectos en el diafragma, en el espacio retropleural o en el espacio
retroperitoneal.

98
99
DERMATOLOGÍA

¿Cuáles son las capas de la piel?


 Epidermis: córnea, granulosa, espinosa y basal.
 Dermis: papilar, reticular y profunda.
 Tejido celular subcutáneo.
¿Cuál es la causa de aparición del acné?
El aumento en la producción de andrógenos, que aumentan la produc-
ción sebácea.
¿Cuál es el tratamiento del acné?
Comedolíticos, antiqueratolíticos, inhibidores de las glándulas sebáceas
por vía sistémica (isotretinoína) y antiandrogénicos sistémicos (dexame-
tasona o anticonceptivos hormonales).
¿Cuál es la diferencia entre leucoplaquia y candidiosis?
La leucoplaquia es una lesión blanca que no puede ser removida frotán-
dola (a diferencia de la candidiosis) y que tiene de 2 a 6% de probabilida-
des de desarrollar cáncer.
¿Cuál es la incidencia del cáncer en la eritroplaquia?
90% de los casos evolucionan a cáncer.
¿Cuáles son la distribución y antecedentes de la dermatitis atópica?
 En los adultos aparece en las áreas flexoras de los miembros.
 En los lactantes, en las áreas facial y extensoras de los miembros.
 Existe el antecedente de asma o alergia en el sujeto o en la familia.
 Es común la presencia de obscurecimiento infraorbitario.
¿Cuál es el tratamiento de la dermatitis atópica?
Medidas generales y corticoides tópicos.
¿Qué es el eritema polimorfo?
Es una enfermedad inflamatoria de etiología infecciosa o secundaria a
fármacos que presenta lesiones cutáneas en diana (en blanco de tiro, en
iris), que son lesiones con anillos concéntricos y una púrpura central,
principalmente en las porciones distales de los miembros, aunque tam-
bién se pueden ver en las palmas y en la cara.

100
¿Qué es el síndrome de Stevens-Johnson?
Es una forma grave del eritema polimorfo en la que se involucran la mu-
cosa oral, la faringe y la región anogenital.
¿Qué es el eritema nodoso?
Es una enfermedad inflamatoria de la piel y del tejido subcutáneo, carac-
terizada por la presencia nódulos rojos dolorosos principalmente en la re-
gión pretibial. Las causas más frecuentes son las infecciones
estreptocócicas y la sarcoidosis.
¿Cómo es la lesión del líquen simple?
En placas y con huellas de rascado; se trata con esteriodes tópicos.
¿Cuál es el cuadro clínico de la psoriasis?
Se observan escamas plateadas sobre placas rojas en los codos, las rodi-
llas y el cuero cabelludo; pueden coexistir artritis y onicolisis.
¿Cuál es el tratamiento de la psoriasis?
Para la variedad localizada se utilizan queratolíticos y para la variedad gene-
ralizada se emplean metotrexate y rayos ultravioleta.
¿Cuál es el cuadro clínico de la pitiriasis rosada?
Se observa la placa heráldica (lesión confluente de gran tamaño localizada
en el pecho) y existe descamación periférica. El agente etiológico es el vi-
rus HHV7.
¿Cuál es el diagnóstico diferencial de la pitiriasis rosada?
La lesión es similar a la de la sífilis secundaria, por lo que hay que hacer
prueba de reagina (RPR) y también debe diferenciarse de la tinea corporis
mediante una prueba con KOH.
¿Cuál es la diferencia entre psoriasis, dermatitis seborreica y caspa?
 Psoriasis: placas eritematosas delimitadas, no confluentes, con des-
camación.
 Dermatitis seborreica: hay eritema generalizado o confluente con
descamación.
 Caspa: no hay eritema; la descamación está presente.
¿Cuál es el tratamiento de la dermatitis seborreica?
 Shampoo con alquitrán o pitirona de zinc diariamente.

101
 Ketoconazol en solución tópica cada tercer día. No se usa por su ac-
ción antimicótica sino más bien por otra no especificada).
¿Cómo es la lesión de la queratosis seborreica?
Se observan nódulos grandes como “empotrados”.
¿Cuál el el tratamiento de la rosácea?
Deben evitar cualquier alimento que cause vasodilatación como el al-
cohol o los alimentos condimentados.
¿Cuándo se presenta el eritroderma macular?
Es el signo dermatológico que acompaña al síndrome de choque tóxico.
¿Cuál es el diagnóstico diferencial de las lesiones hipopigmentadas?
Vitiligo, piebaldismo, tinea versicolor, albinismo oculocutáneo, psoriasis y
sífilis secundaria, entre otras.
¿Qué es la tinea versicolor?
Máculas hipo o hiperpigmentadas en la parte superior del tronco, causa-
das por Malasezia furfur. Debido al tipo de lesión, hay que diferenciarla
del vitiligo.
¿Qué es el vitiligo?
Es la pérdida de melanocitos que se presenta en 1% de la población y
puede relacionarse con hipertiroidismo, hipotiroidismo, anemia pernicio-
sa, diabetes mellitus y enfermedad de Addison.
¿Qué es el mal del pinto?
Es una treponematosis no transmitida sexualmente causada por T. Cara-
teum; es endémica de las áreas rurales de América Latina. En la lesión
inicial en la piel provoca una coloración lila o negra seguida de atrofia de
la zona e hipopigmentación.
¿Cuál es la diferencia entre la enfermedad de Bowen y la queratosis actínica?
Las dos presentan una placa con vesículas color rojo. La queratosis actí-
nica miden de 0.2 a 1 mm y en la enfermedad de Bowen alcanzan de 1 a
3 cm.

102
ENDOCRINOLOGÍA

¿A qué se llama retroalimentación positiva?


Es el mecanismo por el cual una hormona actúa estimulando a la misma
glándula que la secreta a aumentar la secreción de la propia hormona.
¿Qué es la retroalimentación negativa?
Es un mecanismo por el cual la hormona secretada estimula a la propia
glándula para que disminuya la secreción de dicha hormona.
¿Cuál es la diferencia entre el diagnóstico de diabetes mellitus y el de into-
lerancia a la glucosa?
En la prueba con 75 g de glucosa, el diagnóstico de diabetes se hace con
un valor de 200 de las 0 a las 2 horas. La intolerancia a la glucosa se
diagnostica cuando el valor de 200 se presenta más allá de las 2 horas.
¿Para qué sirve la determinación de hemoglobina glicosilada (HbA1C)?
El nivel de HbA1C indica cuál ha sido el promedio de glucemia en las úl-
timas 8 a 12 semanas. Es útil para valorar si hay un buen control de la
glucemia.
¿Para qué se utiliza la determinación de fructosamina sérica?
Para evaluar la glucemia de las últimas 2 semanas.
¿Cuál es el riesgo de padecer diabetes mellitus cuando los progenitores son
diabéticos?
5-10%
¿Cuál es el riesgo de padecer diabetes mellitus cuando el padre es el afectado?
El riesgo es cinco veces mayor y está relacionado a HLADR4
¿Qué es la diabetes juvenil de inicio tardío (tipo MODY, por sus siglas en
inglés)?
Es una hiperglucemia leve con resistencia a la cetosis causada por los ge-
nes MODY 1, 2, 3. Su nombre proviene de las siglas en inglés para Matu-
re Onset Diabetes of the Young, que quiere decir, diabetes juvenil de inicio
tardío.

103
¿Qué es el fenómeno de Somogy?
Es una hiperglucemia de rebote cuando se sobreutiliza la insulina. Al
causar hipoglucemia, ésta activa los mecanismos contrarreguladores co-
mo el glucagon, la hormona de crecimiento, el cortisol y las catecolami-
nas, que causan hiperglucemia.
¿Por qué la hiperinsulinemia causa resistencia a la insulina?
Porque aumenta la síntesis de grasas y el transporte de las mismas por
VLDL haciendo de éstas el principal sustrato energético en lugar de la
glucosa, lo que disminuye la necesidad de utilizar insulina en la célula.
¿Cuáles son los tipos de insulina disponibles?
Duración
Tipo Inicio Pico Vía de
Ejemplo de la
de insulina de la acción de acción administración
acción
Acción ultracorta Insulina lispro 0-15 minutos 1-2 hr 3-5 horas SC, IV
Insulina aspart 10-12 minutos 40-50 min 4-6 horas SC, IV
Acción rápida Insulina regular 30-60 minutos 2-4 hr 6-8 horas SC, IV
Acción Insulina NPH 1-2 horas 6-12 hr 18-24 horas SC
Intermedia Insulina lenta 1-3 horas 6-12 her 18-24 horas SC
Acción Insulina ultralenta 3-4 horas 4-12 hr 12-18 horas SC
Prolongada Insulina ultralarga 4-6 horas Ninguno 18-30 horas SC

¿Cómo se lleva a cabo la terapia convencional con insulina?


15 unidades de lenta o NPH, divididas en dos dosis.
¿Cuáles son los fármacos hipoglucemiantes de elección?
Las sulfonilureas (glibenclamida) que actúan estimulando la liberación de
insulina desde las células beta. Todavía son los fármacos de elección, se-
guidos por las biguanidas (metformina).
¿Cómo actúa la metformina?
Inhibiendo la gluconeogénesis hepática.
¿Qué es la clorpropamida?
Es una sulfonilurea de primera generación con efecto hipoglucemiante que
también tiene un uso terapéutico en la diabetes insípida, ya que potencia
el efecto de la hormona antidiurética.
¿Qué es la troglitazona?

104
Es un hipoglucemiante del grupo tiazolidinediona, de primera generación,
que actúa disminuyendo el estado de resistencia a la insulina en los dia-
béticos.
¿Qué se requiere para desarrollar cetoacidosis?
Un estado de hipoinsulinemia, ya que éste favorece la movilización de
los depósitos de grasa; también hiperglucagonemia, que favorece la oxi-
dación de los ácidos grasos en el hígado.
¿Cómo se valora la respuesta al tratamiento de la cetoacidosis?
Sólo con el PH y el anion gap calculado.
¿Cómo se lleva a cabo el tratamiento con insulina en la cetoacidosis?
Primero se debe usar de manera intravenosa continua a través de una
bomba de infusión, a dosis de 0.1 U/kg, después de una dosis de carga en
bolo de 0.1U/kg y según las necesidades del paciente. Posteriormente,
cuando se haya administrado 75% de la dosis por bomba de infusión, se
procede a iniciar dosis intermitentes IV o subcutáneas.
¿Cuáles son las hormonas que producen los diferentes lóbulos de la hipófisis?
 Lóbulo anterior: hormona estimulante del tiroides (TSH, tirotropina),
hormona adenocorticotrópica (ACTH, corticotropina), hormona de
crecimiento (GH, somatotropina), hormona foliculoestimulante
(FSH) y hormona luteinizante (LH).
 Lóbulo intermedio: hormona estimulante de los melanocitos (MSH,
melanotropina); gamma-lipotropina (gamma-LPH), péptido similar a
la corticotropina (CLIP) y otros derivados de la proopiomelanocorti-
cotropina (POMC).
 Lóbulo posterior: vasopresina (antidiurética, ADH) y oxitocina.
¿Qué es un macroadenoma hipofisario?
Es un tumor de la hipófisis, que mide más de 10 mm.
¿Qué proteínas son subproductos de la proopiomelanocorticotropina
(POMC)?
La beta-endorfina, ACTH, MSH y la metencefalina, entre otras.
¿Cuáles son los principales antagonistas de la hormona liberada de gona-
datropina (GnRH)?

105
La prolactina, la dopamina, la ACTH y el cortisol.
¿Cómo actúa la dopamina?
Actúa sobre la eminencia media para inhibir la secreción de hormona li-
berada de gonadatropina (GnRH).
¿Cómo afecta la noradrenalia a la GnRH?
Tiene un efecto estimulante sobre el área preóptica y favorece la secre-
ción de GnRH.
¿Cuál es el mecanismo por el cual los análogos de GnRH inhiben a la FSH
y la LH?
Proveen un estímulo potente y continuo diferente al estímulo fisiológico
(por pulsos) de la GnRH humana.
¿Qué hormonas son inhibidas por el cortisol?
La ACTH y el factor liberador de cortisol (CRF).
¿Cuáles son los cambios metabólicos en la acromegalia?
 Intolerancia a la glucosa: por las propiedades antiinsulínicas de la
hormona de crecimiento.
 Hiperfosfatemia: por su acción directa sobre la reabsorción de fosfato.
 Más de 10 ng/ml de GH.
 Aumento del factor insulínico de crecimiento -1 (IGF-1) en el hígado.
¿Cuál es el síndrome clínico de la enfermedad de Graves?
Hipermetabolismo, signos oculares anormales y edema pretibial inusual.
¿Cuál es la fisiopatología de la enfermedad de Graves?
Es un trastorno autoinmunitario en que los linfocitos T producen anti-
cuerpos a algunos antígenos tiroideos. La inmunoglobulina estimulante
de tiroides, sustancia estimulador tiroideo de larga acción (LATS o TSI),
es un anticuerpo que se fija al receptor de la hormona estimulante de ti-
roides (TSH) e incrementa su crecimiento y funcionamiento.
¿Cómo se obtiene el índice de tiroxina libre?
Es el producto del valor de la proporción de captación de hormona tiroi-
dea (THBR) multiplicado por la concentración sérica total de tiroxina.
Constituye una estimulación de la concentración de T4 libre.
¿Cómo se encuentran los índices tiroideos en la enfermedad de Graves?

106
 Aumento de T4.
 Aumento de T3RU.
 Aumento de T3.
¿Cuál es la diferencia entre enfermedad de Graves y la tirotoxicosis sin hi-
pertiroidismo?
Sólo en la primera están amentados el índice de toroxina libre (ITL) y la
captación de yodo.
¿Qué es la tormenta tiroidea?
Es una exacerbación intensa de la tirotoxicosis en la que se encuentra hi-
permetabolismo, exceso de la respuesta suprarrenal, sudación y fiebre.
De no tratarse, puede ocasionar la muerte por colapso cardiovascular. Se
trata con: propranolol para evitar las complicaciones cardiacas, con una
solución intravenosa saturada de yoduro potásico, para evitar la liberación
de hormona tiroidea, y con propiltiouracilo, para bloquear la producción
de dicha hormona.
¿Cuál es la diferencia de acción entre el propiltiouracilo (PTU) y el metimazol?
El PTU inhibe la conversión periférica de T-4 a T-3 e inhibe la yodación
de los grupos tirosil y la agrupación de yodotirosinas para formar T3 y T4
en la tiroides. El metimazol sólo tiene acción en la glándula y carece de
acción periférica.
¿Cuáles son los tipos de bocio difuso?
 Por deficiencia de yodo.
 Tiroiditis aguda supurativa y tiroiditis subaguda
 Tiroiditis crónica (de Hashimoto)
¿Cuál es la forma más frecuente de bocio?
La nodular.
¿Cuál es la diferencia entre tiroiditis de Hashimoto y tiroiditis nodular?
Las dos tienen T-4 y T-3 (T3RU) normales y son asintomáticas, pero la
primera es un bocio difuso.
¿Qué cambio produce el hipertiroidismo en la citología hemática?
Granulocitopenia.
¿Cuáles son los bocios tóxicos que requieren tratamiento quirúrgico?
 El multinodular o enfermedad de Plummer. Es más benigno en muje-
res mayores de 50 años.

107
 El bocio difuso de Graves.
¿Cuáles son los bocios no quirúrgicos?
 Coloide: la causa es la falta de yodo; los pacientes se encuentran euti-
roideos.
 Tiroiditis aguda (bacteriana): sólo hay dolor y se observa eutiroidis-
mo.
 Tiroiditis subaguda de Quervain (viral): los pacientes se encuentran
hipertiroideos.
 Tiroiditis crónica de Hashimoto (autoinmune): existe eutiroidismo.
 Tiroiditis fibrosa de Riedel: puede presentar síntomas obstructivos.
¿A qué se le llama síndrome del eutiroideo enfermo (euthyriod sick syndro-
me)?
Es una variedad de padecimientos agudos o crónicos no tiroideos que se
asocian con pruebas de función tiroidea anormales en pacientes eutiroi-
deos, especialmente en los de edad avanzada. Las variedades principales
son: el síndrome de T4 aumentada, síndrome de T4 disminuida y el síndrome de
T3 baja.
¿Para qué sirve determinar la relación cloruro/fosfato?
Es un indicador más confiable que la hipofosfatemia para apoyar el diag-
nóstico de hiperparatiroidismo cuando es mayor de 33.
¿Por qué mecanismo elevan la calcemia los tumores no óseos?
Producen un péptido similar a la paratohormona.
¿Por qué mecanismo aumenta la calcemia la sarcoidosis?
Produce 1, 25 – dehidroxocolecalciferol (1,25-DHC).
¿Por qué mecanismo se produce aumento del calcio en la hipercalcemia hi-
pocalciúrica familiar?
Por un defecto genético no existe el sensor de calcio en las paratiroides ni
en el túbulo.
¿Cuáles son las funciones de la paratohormona?
Remueve el calcio del hueso, aumenta la resorción renal de calcio, au-
menta la producción de 1, 25 DHC y aumenta la fosfaturia.
¿Cuáles son los efectos generales del hiperparatiroidismo?
Hipercalcemia, hipofosfatemia, hiperfosfaturia.

108
¿Cuáles son las principales causas de hiperparatiroidismo?
80% de los casos se debe a un adenoma solitario y 10-15%, a hiperplasia
de las cuatro glándulas.
¿Dónde se secreta la calcitonina?
En las células parafoliculares (también llamadas células claras o células
C) de la tiroides.
¿En dónde actúa la calcitonina?
Actúa en dos niveles: a nivel óseo impide la resorción, y a nivel renal
aumenta la calciuria. El efecto neto es una disminución del calcio sérico.
¿Cómo se calcula la carga filtrada de calcio?
Es el 40% del calcio sérico multiplicado por la TFG.
¿Cuál es el tratamiento para la hipercalciuria?
Diuréticos tiazídicos.
¿En qué enfermedad se encuentran niveles elevados de calcitonina?
En el carcinoma medular tiroideo.
¿Cuáles son tres estímulos importantes para la secreción de hormona re-
ductora del calcio (calcitonina)?
Los beta-adrenérgicos, los niveles elevados de gastrina (como en el sín-
drome de Zollinger-Ellison) y los niveles altos de calcio.
¿Qué es el síndrome de hueso hambriento?
Tras la paratiroidectomía se presenta hipocalcemia (e hipofosfatemia)
debido a la entrada excesiva de calcio a los huesos que se encontraban
descalcificados secundariamente al exceso de paratohormona.
¿Cuál es el nivel normal de fósforo sérico y en qué proporción se une a pro-
teínas?
El nivel normal va de 2.8 a 4 mg/dl y el 12 % a proteínas (en compara-
ción al calcio, que se une en 50%)
¿Cuál es la diferencia entre osteomalacia y osteoporosis?
En la primera no se encuentra reducida la matriz ósea y en la segunda sí.
¿Cómo se hace el diagnóstico de osteopenia?
Sólo se hace por densitometría, que debe evidenciar un cambio >1 y <2.5
desviaciones estándar en la medición de la densidad ósea.
¿Cómo se hace diagnóstico de osteoporosis?

109
Cuando la medición de la densidad ósea muestra una disminución de la
densidad >2.5 desviaciones estándar.
¿Qué fármacos son inhibidores de la actividad osteoclástica?
Bifosfonatos, nitrato de galio, plicamicina, calcitonina.
¿Cuál es la histología de la corteza de las glándulas suprarrenales?
 Capa glomerulosa externa, que produce mineralocorticoides.
 Capa fasciculata intermedia, que produce glucocorticoides.
 Capa reticularis interna, que produce andrógenos y estrógenos.
¿Cuáles son las horas pico y valle de la producción del cortisol?
6:00 am y 8-12 pm
¿Qué produce la médula suprarrenal?
Catecolaminas.
¿Cómo se clasifican los esteroides?
 Esteroides C19 (17 KS) o DEHA o andrógenos
 Esteroides C21 (17 OHCS), que son mineralocorticoides o análogos
de la aldosterona.
 Esteroides C21 17- hidroxicorticosteroides (17OHKS), que son glu-
cocorticoides o análogos del cortisol.
¿Cómo se llaman las vías que producen mineralocorticoides y glucocorti-
coides y dónde se encuentra el defecto para la síntesis de las enzimas invo-
lucradas?
Son las vías “17-desoxy” y la “17-hidroxy”, respectivamente. El defecto
se encuentra en el brazo corto del cromosoma 6.
¿Cuáles son las diferentes acciones del cortisol?
El cortisol es principalmente catabólico en la mayoría de tejidos, pero
anabólico en el hígado.
¿Qué características conforman la insuficiencia suprarrenal?
 Insuficiencia de cortisol (glucocoriticoide): ocasiona pobre tolerancia
al estrés, anorexia, pérdida de peso, hiperpigmentación de la piel, hi-
poglicemia e hipotensión.
 Insuficiencia de aldosterona (mineralocorticoide): causa hiponatre-
mia, hiperkalemia, hipovolemia, azoemia y acidosis.
¿Qué es la prueba de cosintropina y para qué se usa?

110
Es una prueba de estimulación de ACTH, que se usa para hacer el diag-
nóstico diferencial de la insuficiencia adrenal. El resultado positivo
(normal) es un aumento de 2 veces sobre el nivel base del cortisol plasmá-
tico (>20 g/dl) y un aumento de los 17 OHKS urinarios, después de la
administración del medicamento. Este resultado descarta enfermedad de
Addison (insuficiencia suprarrenal primaria).
¿Qué es la prueba de metirapona?
La inhibición de la secreción pituitaria de ACTH por la producción
adrenal de cortisol se bloquea con la metirapona, fármaco que bloquea la
11-beta hidroxilación (paso último en la producción de esteroides). En los
individuos con deficiencia de ACTH esta no se eleva. En los individuos
normales o con Cushing pituitario se eleva la ACTH, se eleva el 11-
desoxicortisol, disminuye el cortisol, y los 17-OHKS urinarios se elevan.
En la hiperplasia adrenal no hay cambio en los 17-OHKS urinarios, ni en
la producción de cortisol.
¿Cuál es la diferencia entre la enfermedad y el síndrome de Cushing?
La enfermedad es de causa pituitaria; la causas del síndrome son varias,
de origen extrapituitario.
¿Cuál es la incidencia de las etiologías del síndrome de Cushing?
15% es adrenal, 15% es extraadrenal (incluye el medicamentoso) y 70%
es pituitario.
¿Cuál es el protocolo para estudiar el síndrome de Cushing?
 Demostración de hipercortisolismo.
 Determinación de ACTH para descartar una causa pituitaria.
 Uso de tomografía axial compaturizada (TAC) o resonancia magné-
tica nuclear (RMN) para la localización del adenoma.
¿Cuáles son las pruebas para la detección de hipercortisolismo?
 Niveles de cortisol a las 8:00 am (30 g); (nivel base >5.0g/dl).
 Prueba de supresión con dosis altas de dexametasona.
 Detección de cortisol libre (17OHKS) en orina de 24 horas el cual
debe ser mayor de 100 g.
¿Cómo se interpreta la prueba de supresión con dosis altas de dexametasona?
 Se inyecta 1 mg a las 23:00 horas.

111
 Se mide el cortisol sérico a las 8:00 de la mañana.
 Un resultado normal es <3g. En los estados de hipercortisolismo es
>3g e incluso a veces >10g.
¿Qué otra utilidad tiene el la prueba de supresión con dosis altas de dexa-
metasona?
Es una prueba para diferenciar el síndrome de Cushing (en el que la su-
presión por retroalimentación negativa es mínima) de los tumores adre-
nales o de los estados de producción ectópica de ACTH.
¿Cómo afecta el síndrome de Cushing a la citología hemática?
Produce eosinofilia.
¿Cuáles son los hallazgos clínicos en el coma mixedematoso?
 EKG de bajo voltaje, creatincinasa (CK) elevada, PCO2 elevada,
PO2 baja.
 Hiperglucemia, debido al efecto antiinsulínico anabólico de los este-
riodes.
 El hematocrito menor a 30% y la hiponatremia son reflejo del secues-
tro de líquidos por actividad mineralocorticoide.
¿Cómo se evalúa la prueba de supresión estándar en 48 horas (bajas dosis)?
 Se utilizan 0.5 mg de dexametasona cada 6 horas por 48 horas.
 El resultado normal debe ser un cortisol sérico <2g.
¿Para qué sirve la prueba de dosis bajas?
Se realiza con 8 dosis de 0.5 mg de dexametasona cada 6 horas. (una va-
riante es una dosis única de 1 mg a las 11 de la noche y recolección de
muestra de sangre a las 8 de la mañana). Sirve para descartar síndrome
de Cushing si es positivo (disminución del cortisol sérico a <5g y el uri-
nario de 24 hr a <25g).
¿Cuál es el estudio de elección para diagnosticar el síndrome de Cushing
primario y pituitario?
TAC o RMN.
¿Qué drogas inhiben la síntesis adrenal de los esteroides?
La metirapona, el trilostane y la aminogluteimida.
¿Cuáles son los antagonistas de los esteroides?

112
Prednisona, tamoxifén, estrógenos, leuprolide, flutamida y la aminoglu-
teimida.
¿Cómo actúa el tamoxifén?
Se une a los receptores de estrógenos sin activarlos y los bloquea.
¿Cómo actúan los estrógenos (dietiletilbestrol)?
Actúan inhibiendo la producción de hormona LH. Por consecuencia, és-
ta no actúa sobre las células de Leydig y se detiene la producción de tes-
tosterona.
¿Cómo actúan el leuprolide y la groserelina?
Son análogos hormona liberadora de hormona luteinizante que se unen a
los receptores hipofisarios causando su desensibilización y, por ende,
disminuyendo la FSH y LH y, finalmente, la síntesis de andrógenos y es-
trógenos.
¿Cómo actúa la flutamida?
Se une al receptor de andrógenos y aumenta el efecto inhibitorio de la
testosterona en la secreción de LH (causa aumento de LH y testostero-
na).
¿Cómo actúa la aminogluteimida?
Actúa inhibiendo el paso de conversión adrenal de colesterol a pregneno-
lona y la aromatización periférica (en el tejido graso principalmente) de
androstenediona a estrógeno.
¿Qué drogas actúan inhibiendo a la hipófisis?
La ciproheptadina, que es un agonista de la serotonina y la bromocriptina,
que es un agonista de la dopamina.
¿Qué es el síndrome de Conn?
Es el hiperaldosteronismo primario.
¿Cuál es la diferencia entre el síndrome de Conn y el aldosteronismo se-
cundario?
En el primero, los niveles de renina son bajos.
¿Cómo se toma la muestra para determinar los niveles de aldosterona?
Debe tenerse cuidado de reponer previamente el potasio; si no se hace, la
hipokalemia causa hipoaldosteronismo y un resultado normal.
¿Cuál es el tumor del 10%?

113
El feocromocitoma: 10% bilateral, 10% maligno, 10% múltiple, 10% fa-
miliar.
¿Qué es el síndrome de Nelson?
Es un adenoma pituitario secundario a adrenalectomía total bilateral.
¿Qué pruebas de laboratorio se usan para hacer el diagnóstico de feocromo-
citoma?
 Niveles de metanefrinas y ácido vanililmandélico (VMA) en orina.
 Niveles fraccionados de catecolaminas en el plasma.
¿Cómo se hace la prueba de supresión en caso de feocromocitoma?
Se utilizan bloqueadores ganglionares como la clonidina o el pentoli-
nium. Cuando existe un tumo productor de catecolaminas, éstas no dis-
minuyen.
¿Qué neoplasia se conocen como tumores endocrinos?
A los tumores de los islotes del páncreas. Se pueden dividir en no funcio-
nantes, cuyos síntomas pueden ser principalmente obstructivos y funcio-
nantes que son: insulinoma, vipoma, somatostatinoma, gastrinoma,
glucagonoma, tumor carcinoide, tumor secretor de ACTH. Estos pueden
quedar incluidos en los síndromes de neoplasia endocrina múltiple (NEM).
¿Qué es el síndrome de Zollinger-Ellison?
Es el gastrinoma. Puede estar localizado en el páncreas y en la pared duo-
denal. Se asocia a otras anomalías como los adenomas paratiroideos, ti-
roideos, ováricos e hipofisarios. El cuadro dominante es el de una
diátesis ulcerosa péptica agresiva de localización atípica (distal al bulbo
duodenal) refractaria a tratamiento médico y quirúrgico.
¿Cuáles son las pruebas de provocación para el gastrinoma?
 Calcio 5 mg/kg/hr IV durante 3 horas. Causa incremento notable de
gastrina.
 2 U de secretina/kg/hr IV rápida: causa aumento paradójico.
 Comida de prueba: existe un aumento menor del 50 % (el nivel ya es-
tá elevado).
¿Cuál es el tratamiento del gastrinoma?
 Localizado: se utilizan bloqueadores H2, octreótida o gastrectomía
total.
 Metastásico: estreptozocina más 5-fluoruracilo.

114
¿Cómo se presenta el insulinoma?
Se presenta con episodios de hipoglucemia que deben diferenciarse de
trastornos neurológicos.
¿Qué es la triada de Whipple?
Glusemia <40 mg/dl, síntomas de la hipoglucemia y remisión de los sín-
tomas administrando glucosa.
¿Cómo se realiza el diagnóstico de insulinoma?
Tras una dosis de tolbutamida en ayuno, la hipoglucemia persiste con ni-
veles altos de insulina medida por radioinmunoanálisis de insulina (IRI>
20 U/ml). En casos de diagnóstico difícil, se hace una prueba de supre-
sión del péptido C.
¿Qué es el péptido C?
La cadena de preproinsulina de 23 aminoácidos, que se pliega para for-
mar la proinsulina, se escinde en dos partes: el péptido C y la insulina. Al
liberar insulina, las células beta también liberan una cantidad equimolar
de péptido C.
¿En qué consiste la prueba de supresión de péptido C?
Se utiliza insulina sintética para disminuir el nivel sérico de insulina por
retroalimentación negativa. Como es imposible distinguir entre la insuli-
na externa y la pancreática, se utiliza el nivel de péptido C para ver si la
última ha disminuido.
¿Cuál es el tratamiento del insulinoma?
Es principalmente quirúrgico; como tratamiento médico se utilizan el
diazóxido y la estreptozocina.
¿Qué otros nombres tiene el vipoma?
Síndrome de Verner Morrison, DAHA (diarrea acuosa, hipokalemia
aclorhidria), DAHH (diarrea acuosa, hipokalemia, hipoclorhidria), cóle-
ra pancreático.
¿Cuáles son las funciones del péptido intestinal vasoactivo (VIP)?
Estimula la secreción intestinal de agua y electrolitos; provoca también la
relajación del músculo intestinal, la vasodilatación de vasos sanguíneos
periféricos y la inhibición de la secreción gástrica ácida.
¿Cuál es la triada del somatostatinoma?

115
Diabetes, esteatorrea y litiasis vesicular.
¿Cómo se presenta el glucagonoma?
Es un tumor de células alfa muy poco frecuente. Produce diabetes, hi-
poaminoacidemia, hipolipidemia, anemia normo-normo. Su característi-
ca distintiva es la erupción rojo-parduzca que aparece en las
extremidades, llamada eritema migratorio necrolítico.
¿Cuál es el síndrome de Wermer?
Es la neoplasia endocrina múltiple tipo I (MEN-I) o adenomatosis endo-
crina múltiple tipo I.
¿Cuál es el síndrome de Sipple?
Es la neoplasia endocrina múltiple tipo II (MEN-IIA) o adenomatosis
endocrina múltiple tipo II.
¿En quiénes se debe sospechar la MEN-I?
En quienes tengan síntomas que sugieren gastrinoma o tumor pituitario.
¿En quiénes se sospecha la MEN-IIA?
En personas que son hijas de pacientes con diagnóstico de carcinoma
medular tiroideo, hiperaldosteronismo o feocromocitoma.
¿Cuál es la presentación clínica del somatostatinoma?
Diabetes, esteatorrea, cálculos biliares y aclorhidria.
¿Cuál es la presentación clínica del glucagonoma?
Diabetes mellitus, anemia, pérdida de peso y rash.

116
GASTROENTEROLOGÍA

¿Qué enzimas degradan las proteínas de la dieta?


La pepsina gástrica y las proteasas pancreáticas.
¿Qué nutrientes se absorben en el íleon proximal?
Ácido fólico, hierro y calcio.
¿Qué nutrientes se absorben en el íleon terminal?
Ácidos biliares y vitamina B12.
¿Cuáles son dos causas de íleo paralítico?
Hipokalemia y cirugía abdominal reciente.
¿Dónde se produce la secretina?
Se produce en la porción superior del intestino delgado y su función con-
siste en aumentar la secreción de bicarbonato por el páncreas.
¿Dónde se produce la colecistocinina (CCK)?
Se produce en la porción superior del intestino delgado y provoca la con-
tracción de la vesícula biliar, la secreción de enzimas pancreáticas y po-
tencia la acción de la secretina.
¿Qué producen las células parietales oxínticas?
Acido clorhídrico (HCl) y factor intrínseco.
¿Qué producen las células parietales cimógenas?
Pepsinógeno.
¿Qué producen las células antrales en forma de botella?
Secretan gastrina, que aumenta la producción de ácido clorhídrico.
¿Qué es la gastritis Tipo A?
Es la gastritis del fondo asociada a anticuerpos anticélula parietal, niveles
altos de gastrina y anemia perniciosa.
¿Qué es la gastritis Tipo B?
Es la gastritis del antro y presenta anticuerpos antigastrina; la lesión se debe
a reflujo duodenobiliar asociado a cáncer gástrico.
¿Qué es la enfermedad de Ménétrier?
Es la gastritis hipertrófica.

117
¿Cómo se hace el diagnóstico diferencial de la gastritis?
La gastritis produce dolor más intenso después de la ingesta de alimentos;
en los casos de úlcera, el dolor disminuye tras la ingesta de alimentos pa-
ra recurrir 2-3 horas después.
¿Cómo se realiza el diagnóstico de síndrome de Zollinger-Ellison?
Tras una prueba de estimulación con secretina, la gastrina alcanza valo-
res de 200 U.
¿Cuáles son las causas de la úlcera duodenal?
Células parietales hipersensibles a la gastrina (hiperacidez) y la presencia
de H. pylori, que debilita la mucosa.
¿Dónde aparece la mayoría de las úlceras duodenales?
En la primera porción del duodeno.
¿Cuál es el patrón del dolor en la úlcera duodenal?
El dolor se alivia con la ingesta de alimentos sólo para recurrir algunas
horas más tarde.
¿Cuáles son las causas de la úlcera gástrica?
Antiinflamatorios no esteroidesos, reflujo biliar e hipersecreción gástrica.
¿Dónde aparece la mayoría de las úlceras gástricas?
En el cuerpo y en la unión de las células parietales del cuerpo; las de gas-
trina en el antro.
¿Qué condiciones se asocian al cáncer gástrico?
Anemia perniciosa, gastritis atrófica y la presencia de H. pylori.
¿Cómo se manifiesta el linfoma gástrico?
Se presenta con dolor y su estirpe histológica es de tipo no Hodgkin.
¿Qué enfermedades no infecciosas se presentan con diarrea, sangre y cóli-
cos?
El síndrome hemolítico urémico (SHU) y la colitis ulcerativa crónica
inespecífica (CUCI).
¿Qué enfermedades se presentan con heces formadas, sangrado y sin dolor?
Divertículo de Meckel, poliposis juvenil y colitis eosinofílica.
¿Cuál es la causa más frecuente de hemorragia del tracto digestivo alto
(HTDA)?
 Enfermedad ácido-péptica (EAP): 75%.

118
 Várices esofágicas: (5-15%).
¿Cuáles son las causas más frecuentes de hemorragia del tracto digestivo
bajo (HTDB)?
 Aguda: diverticulosis y angiodisplasia (1er. y 2º lugar en el anciano).
 Crónica: hemorroides y cáncer.
 Jóvenes: divertículo de Meckel, enfermedad inflamatoria.
¿Cómo se corroboran la HTDA y la HTDB?
 HTDA: mediante una sonda nasogástrica o endoscopía superior.
 HTDB: mediante el tacto rectal, anoscopia o rectosigmoidoscopia.
¿Cuál es el diagnóstico diferencial de la hemorragia del tacto digestivo
(HTD)?
 Sangrado con dolor: isquemia, inflamatoria, aneurisma roto.
 Sangrado sin dolor: divertículo de Meckel, diverticulosis, angiodis-
plasia, hemorroides.
 Sangrado con diarrea: infección, causa inflamatoria.
 Sangrado con dolor rectal: fisura hemorroides.
 Sangrado con estreñimiento: neoplasia.
 (Otros datos como la edad o el tiempo de evolución son importantes
para el diagnóstico).
¿Cuál es la diferencia entre la poliposis juvenil y el divertículo de Meckel?
Ambos padecimientos producen sangrado del tubo digestivo, indoloro y
con heces bien formadas. El primero aparece entre los 2 y los 12 años y el
segundo, antes de los dos.
¿Cómo es el sangrado en la poliposis, en la colitis eosinofílica y en el di-
vertículo de Meckel?
En los primeros hay estrías de sangre y en el último, melena abundante.
¿Cuándo está indicado el tratamiento quirúrgico de la hemorragia del trac-
to digestivo alto?
Cuando requiere hasta 5 unidades de sangre en 24 horas.
¿Cómo se tratan las hemorroides de primero y segundo grados?
Con ligadura por arriba de la línea dentada, con fotocoagulación o con
escleroterapia.

119
¿Cómo se tratan las hemorroides de tercero o cuarto grados?
Con hemorroidectomía.
¿Cuál es el tratamiento médico de las fisuras anales?
Sediluvios y nitroglicerina tópica al 0.2%
¿Qué sangrado se detecta por angiografía?
El menor de 0.5ml/min.
¿Qué sangrado se detecta por gammagrafía?
El menor de 0.05ml/min.
¿Con qué dato de laboratorio se sospecha hemorragia del tacto digestivo
(HTD)?
Con un índice BUN/creatinina de 36. (BUN= nitrógeno ureico en sangre).
¿Cuál es el tratamiento médico de la hemorragia activa por várices esofá-
gicas?
 Vasopresina más nitroparche.
 Somatostatina u ocreótida.
¿Cuál es el tratamiento de elección de la hemorragia por varices esofági-
cas?
La escleroterapia.
¿Cuál es la clasificación endoscópica del sangrado por úlcera péptica?
 Úlcera con sangrado visible (recidiva de 85%).
 Úlcera sin sangrado visible (55%).
 Úlcera con coágulo centinela.
¿Cuáles son las indicaciones del tratamiento quirúrgico de la hemorragia
por enfermedad ácido-péptica?
 Inestabilidad hemodinámica.
 Pérdida de más de 30% del volumen sanguíneo en 24 horas.
 Necesidad de 5 paquetes globulares o más en 24 horas.
¿Cuáles son la úlceras de Cushing y de Curling?
 Cushing: úlcera secundaria a daño intracraneal.
 Curling: úlcera secundaria a estrés por quemaduras de más de 35% de
la superficie corporal.
¿Cómo se presentan más frecuentemente las angiodisplasias?
 En mayores de 40 años.

120
 En el colon derecho.
 Como manchas rojo cereza.
¿Qué parte del colon se afecta más en la colitis isquémica?
El ángulo esplénico.
¿Cuál es la diferencia entre rectorragia y hematoquezia?
La primera es sangre color rojo brillante; la segunda, de rojo vinoso.
¿Qué es la acalasia?
Es un trastorno motor en el que disminuye la peristalsis esofágica. Al
mismo tiempo, se encuentra aumentada la presión del esfínter esofágico
inferior.
¿Cuál es el cuadro clínico de la acalasia?
Disfagia para sólidos y líquidos, dolor torácico, regurgitación, tos noc-
turna y pérdida de peso, de muchos años de evolución.
¿Cuál es la imagen radiológica típica de la acalasia?
En “pico de ave”.
¿Cuál es el tratamiento de la acalasia?
 Esofagotomía modificada de Séller.
 Toxina botulínica.
¿Cuál es el tratamiento del espasmo esofágico difuso?
Bloqueadores de los canales del calcio y nitratos.
¿Cuáles son las causas principales de la perforación esofágica?
Iatrogénica en 50% y traumática en 20%
¿Dónde se localiza con más frecuencia la úlcera gástrica?
En el cuerpo del estómago.
¿Cuál es el diagnóstico diferencial de la apendicitis?
Gastroenteritis aguda, adenitis mesentérica, divertículo de Meckel y en-
fermedad de Crohn. En mujeres jóvenes debe considerarse los tumores
ováricos, quistes ováricos rotos y enfermedad inflamatoria pélvica (EIP).
En pacientes mayores se debe descartar la diverticulitis, hernia encarce-
lada, carcinoma cecal y trombosis mesentérica.
¿Dónde se localizan principalmente los tumores carcinoides del intestino?
Apéndice, íleo y recto.
¿Cuáles son los cuatro tipos de pólipo, en orden de frecuencia?
Hiperplásico, hamartomatoso, inflamatorio o edematoso y neoplásico.

121
¿Cuál es el único pólipo con potencial maligno y que debe resecarse?
El neoplásico (más el velloso).
¿Con qué se diagnostican los pólipos?
El primer estudio debe ser un enema baritado contrastado con aire libre.
El de elección es la colonoscopia.
¿Cuál es la diferencia entre el síndrome de Peutz-Jeghers y la poliposis ju-
venil?
Ambos tienen pólipos de tipo hamartomatoso. En el síndrome de Peutz-
Jeghers también hay manchas hipopigmentadas en la boca, y los pólipos
se extienden a lo largo del tubo digestivo; en cambio, la poliposis juvenil
sólo aparecen en el recto.
¿Cuál es la diferencia en el tratamiento del pólipo sésil y el pedunculado?
El primero requiere colectomía parcial; el segundo, sólo polipectomía
endoscópica.
¿Qué es la poliposis adenomatosa familiar (PAF) y cuál es su gen?
Es una enfermedad donde generalmente hay más de 100 pólipos, que
desarrollan cáncer 100% de las veces. La presencia de C5PAF aumenta el
riesgo de cáncer del colon.
¿Qué es el síndrome de Gardner?
Es una variante de la poliposis adenomatosa familiar, asociada con os-
teomatosis, quistes epidermoides y fibromas de la piel; sus genes son el
5q21 y 5q22.
¿Qué es el síndrome de Turcot?
Es la poliposis adenomatosa familiar (PAF) con tumores malignos del
sistema nervioso central.
¿Cuáles síndromes de poliposis son autosómicos dominantes?
 El síndrome de Peutz-Jeghers (SPJ), la poliposis juvenil difusa y la
poliposis adenomatosa familiar (cromosoma 5).
¿Cuáles son las complicaciones de los síndromes de poliposis?
 Todos pueden causar intususcepción y sangrado.
 Los adenomatosos (SPJ, PAF) pueden causar un estado de pérdida
de proteínas.
 La PAF evoluciona a cáncer en 100% de los casos.
¿Cuál es la diferencia entre el tratamiento de la PAF y el SPJ?

122
En la primera se requiere colectomía total y en la segunda sólo se extirpa
un segmento o los pólipos.
¿Qué es el síndrome de Trousseau?
Es la coagulación intravascular diseminada trombótica crónica en pa-
cientes con carcinomatosis. Es más frecuente en el cáncer del páncreas y
del estómago.
¿Cómo se realiza la evaluación inicial de diverticulitis?
Con tomografía y radiografía del abdomen y del tórax, que revelarán aire
subdiafragmático.
¿Cuál es el tratamiento de la diverticulitis?
Dieta baja en residuo y metronidazol o cefalosporinas de segunda o terce-
ras generación.
¿Cuál es el diagnóstico diferencial de la enfermedad de Crohn?
Linfoma, TB, Yersinia, enteritis por radiación.
¿Cuáles son las manifestaciones extraintestinales de la enfermedad de
Crohn?
Fiebre, malestar, eritema nodoso, pioderma gangrenoso, episecleritis,
uveítis, nefrolitiasis y afección hepatobiliar.
¿Cuáles son los hallazgos radiológicos en la enfermedad de Crohn?
Úlceras profundas (collar button), signo de la cuerda y áreas respetadas.
¿Cuáles son los hallazgos anatomopatológicos en la enfermedad de Crohn?
Inflamación transmural, engrosamiento de la pared del intestino, granu-
lomas y una imagen con áreas respetadas y afectadas en forma alterna.
¿Qué anticuerpo se asocia a la enfermedad de Crohn?
El anticuerpo anti-Saccharomices cerevisiae (ASCA).
¿Qué medicamentos se utilizan para tratar le enfermedad de Crohn?
Sulfasalazina a dosis de 3-4 gr/día e infliximab.
¿Qué es el infliximab?
Es un anticuerpo quimérico que se une al factor de necrosis tumoral-alfa.
¿Cuáles son las manifestaciones extraintestinales de la colitis ulcerativa
crónica inespecífica?
Artitis asimétrica, sacroileítis, eritema nodoso, pioderma gangrenoso,
uveítis y colangitis esclerosante.

123
¿Cuáles son los hallazgos anatomopatológicos en colitis ulcerativa crónica
inespecífica (CUCI)?
Microabscesos de las criptas de Liberkun.
¿Cuáles son los anticuerpos encontrados en enfermos de CUCI?
Anticuerpos anticitoplasma de neutrófilo y anticuerpos antisacaromices.
¿Cuándo se indica el ácido 5-aminosalicílico para pacientes con CUCI?
Cuando los pacientes son alérgicos a la sulfasalazina.
¿Cuáles son los cambios anatomopatológicos encontrados en el esprue celíaco?
Atrofia de vellosidades, hipetrofia de las criptas, cambios cuboidales e in-
filtración extensa de la lámina propria.
¿Cuáles son las complicaciones de la enfermedad celíaca?
Linfoma, úlceras, estenosis, dermatitis herpetiforme y esprue colágeno.
¿Cuáles son los antígenos asociados a la enfermedad celíaca?
HLA-B8, HLA-DW3 y el antígeno de linfocito B.
¿Cuál es el cambio anatomopatológico en la enfermedad de Whipple?
Macrófagos PAS positivos (contienen a los bacilos).
¿Cuáles son las pruebas para evaluar la función sintética del hígado?
Colesterol, proteínas séricas, tiempo de protrombina y glucosa.
¿Cuáles pruebas se utilizan para evaluar la función depurativa del hígado?
Bilirrubina indirecta y amonio.
¿Cuáles tests se utilizan para evaluar la función excretora del hígado?
Determinación de bilirrubina directa, aspartato-aminotransferasa (AST) y
gamma-glutamiltranspeptidasa (gamma-GTP).
¿Qué prueba se utilizan para evaluar la extensión del daño hepático?
La determinación de aspartato-aminotransferasa (AST) y alanina-
aminotransferasa (ALT).
¿Cuáles son los patrones en el estudio de las pruebas del funcionamiento
hepático?
 Bilirrubinas: 30-40 (y se mantienen) en enfermedad obstructiva; 50 en
adelante en enfermedad hepatocelular.
 AST y ALT: son indicadores de daño hepatocelular. Mayor que 500
indica daño viral o alcohólico, 100-2000, daño por drogas. Mayor

124
que 4000, indica daño por acetaminofén o por sustancias hepatotóxi-
cas.
 Relación AST/ALT: mayor que 2 indica daño alcohólico; mayor que
3 indica daño por enfermedad hepática.
 ALP: Es un indicador de obstrucción biliar. Obstrucción: fosfatasa al-
calina (FA) 3-10 veces el valor normal; ALT y AST, poco eleva-
das.Enfermedad parenquimatosa: AST, ALT 3 a 10 veces lo normal y
FA poco elevada. Índice GT/ALP mayor que 5 indica hepatopatía
alcohólica.
 Colesterol: Marcadamente disminuido, en hepatitis y cirrosis; marca-
damente aumentado, en procesos obstructivos intra y extrahepáticos
y en la cirrosis biliar primaria.
 -globulina:aumentada en la enfermedad hepática crónica.
¿Cuáles son las alteraciones de laboratorio de la ictericia obstructiva?
Bilirrubinas altas, AST mayor de 300 y ALP aumentada tres veces lo
normal.
¿Cuáles son las alteraciones de laboratorio de la hepatitis alcohólica?
AST/ALT mayor de 3, ALT mayor de 300.
¿Cuáles son las alteraciones de laboratorio en el alcoholismo crónico?
Gamma-GTP/ALP mayor de 5 (también se usa para evaluar la toxicidad
por anticomiciales).
¿Cuándo se utiliza la determinación de gamma-GTP?
Para el diagnóstico de enfermedad hepática cuando coexisten embarazo,
enfermedad ósea, en los niños (porque tienen normalmente elevada la
ALP por el alto recambio óseo) y en el alcoholismo oculto.
¿Cómo se presenta la hepatitis crónica?
Enzimas elevadas durante más de 6 meses, malestar, fatiga y vasculitis.
¿Cómo se hace la distinción entre la hepatitis B crónica activa y el estado
de portador asintomático?
En una biopsia se observa inflamación portal con necrosis progresiva
cuando es hepatitis crónica activa.
¿Qué enfermedades se asocian a la hepatitis B?
La poliarteritis nodosa y la glomerulonefritis.

125
¿Qué enfermedades se asocian a la hepatitis C?
La porfiria cutánea tarda y la crioglobulinemia.
¿Por qué se administra espironolactona a los pacientes con ascitis?
Porque el metabolismo hepático está disminuido y la depuración de la
aldosterona está afectada; por lo tanto, su efecto se prolonga y el paciente
tiende a retener sodio y agua.
¿Por qué la hipokalemia provoca encefalopatía portosistémica?
Porque junto con la alcalosis que produce disminuye la excreción renal
de NH3
¿Qué es el síndrome hepatorrenal?
Es una disfunción renal secundaria a vasoconstricción sostenida de los
vasos renales en pacientes cirróticos en fase avanzada y con ascitis. Apa-
rece en la insuficiencia hepática terminal con azoemia, sodio urinario ba-
jo, sodio sérico bajo e hipotensión arterial.
¿Qué significa un gradiente de albúmina suero-ascitis >1.1?
Que hay mas proteínas en el suero que en el líquido de ascitis; por tanto,
la presión oncótica está conservada y la causa de la fuga de líquido es un
aumento en la presión hidrostática portal.
¿Qué significa un resultado con proteínas totales <1 en líquido de ascitis?
Riesgo aumentado para peritonitis bacteriana espontánea.
¿Cuál es el cuadro clínico de la peritonitis bacteriana espontánea?
El paciente con ascitis presenta fiebre, dolor abdominal y una cuenta de
más de 250 PMN en el citológico del líquido de ascitis.
¿Para qué sirven el USG, la TAC y la angiografía en el estudio de la pato-
logía hepática?
 USG: investigación de lesiones parenquimatosas.
 TAC: estudio de lesiones que involucran el parénquima y estructuras
adyacentes.
 Angiografía: estudio de lesiones parenquimatosas extensas.
¿Cuál es la presentación clínica del quiste y del absceso hepáticos?
El primero, con escasos síntomas focales (masa y dolor). En el segundo,
predominan los síntomas sistémicos de inicio subagudo y por varias se-
manas (fiebre, malestar, anemia, sepsis y malnutrición); puede haber do-

126
lor pleurítico derecho y hepatomegalia, así como dolor en el cuadrante
superior derecho.
¿Cuáles son causas de absceso hepático bacteriano?
Colangitis, apendicitis, diverticulitis. Cuando el foco es abdominal pre-
dominan los gram-negativos y cuando es extraabdominal los gram-
positivos.
¿Cómo se presenta el absceso hepático amibiano?
Con síntomas de absceso; en la imagen se observa solitario y en 90% de
los casos se ubica del lado derecho.
¿Cuáles son las manifestaciones clínicas de la cirrosis?
Encefalopatía, varices esofágicas y hemorragia del tracto digestivo alto,
malnutrición, ascitis, red venosa colateral y esplenomegalia.
¿Qué enfermedades hepatobiliares tienen como síntoma inicial el prurito?
La cirrosis biliar primaria (90% en mujeres) y colangitis esclerosante
(80% en hombres)
¿Cuál es el tratamiento de la cirrosis biliar primaria?
Esencialmente sintomático y con ácido ursodesoxicólico.
¿Cuáles son los cambios anatomopatológicos en la deficiencia de alfa-1-
antitripsina?
Se observan glóbulos PAS positivos en las áreas portales.
¿Por qué aparecen galactorrea, masculinización, feminización y síndrome
de Cushing en la cirrosis?
Porque en el hígado se depuran los esteroides y se produce la globulina fi-
jadora de hormonas esteroideas. Por tanto la actividad esteroide aumenta.
¿Cuál es la causa principal de la hipertensión portal?
La cirrosis hepática.
¿A qué presión portal aparecen las várices esofágicas?
20 mm Hg. La presión normal es de 5-6mmHg.
¿Cuál es el tratamiento farmacológico de la hemorragia del tracto digesti-
vo alto secundaria a várices esofágicas?
 Vasopresina y nitroglicerina: por su efecto vasodilatador sistémico
reducen la presión portal.

127
 Somatostatina y ocreótida: son vasodilatadores esplácnicos.
 Metoclopramida y pentagastrina: aumentan el tono del esfínter eso-
fágico inferior, que disminuye la circulación a ese nivel.
¿De cuánto es el cociente amilasa-creatinina que indica la posibilidad de
pancreatitis?
Mayor de 5.
¿Cuáles son los signos radiológicos de la pancreatitis?
El del colon cortado y el del 3 invertido.
¿Qué es la prueba de Nardi?
Es un espasmo biliar con elevación secundaria de la amilasa, que se pro-
duce con la administración de narcóticos. Se utiliza para demostrar la
existencia de pancreatitis recidivante.
¿Cuál es la etiología principal de la pancreatitis crónica?
El alcoholismo.
¿Qué cambios radiológicos son propios de la pancreatitis crónica?
Calcificaciones e imagen de red de lagos.
¿Cuál es el estándar de oro para diagnosticar insuficiencia pancreática?
Se hace una prueba con colecistocinina y se miden las enzimas y el bicar-
bonato que se producen después de su aplicación.
¿Cuáles son las manifestaciones del síndrome de insuficiencia pancreática?
 Malnutrición.
 Esteatorrea: las grasas no se emulsifican para su absorción.
 Hemorragia: por absorción deficiente de vitamina K.
 Raquitismo: por deficiencia de vitamina D.
 Anormalidades neurológicas: Nictalopía secundaria a deficiencia de
vitamina A.
¿Cuáles son los cuatro procedimientos quirúrgicos para el tratamiento de
la pancreatitis crónica?
Colangiografía pancreática retrógrada endoscópica (ERCP), Puestow,
Duval y Child.
¿Cuál es la presentación clínica del pseudoquiste pancreático?
Ocurre tras un caso de pancreatitis. Cursa con dolor crónico, fiebre poco
elevada, leucocitosis mínima cifras altas de amilasa y masa palpable.
Aparece en 5% de los casos.

128
¿Cuál es la localización más frecuente del cáncer de pancreático?
La cabeza del páncreas; se presenta como ictericia indolora.
¿Qué se encuentra detrás de la primera porción del duodeno y de la cabeza
del páncreas?
La vena porta.

129
130
GINECOLOGÍA Y OBSTETRICIA

¿Cuál es la causa más frecuente de quiste vaginal en la infancia?


El uréter ectópico. El quiste vaginal es dependiente del tejido ureteral que
deriva de las estructuras de Wolff (mesonéfricas) en especial del conducto
de Gartner.
¿Cuál es el orden de aparición de los caracteres sexuales secundarios en la
mujer?
Telarquia, pubarquia, tasa máxima de crecimiento y menarquia.
¿Cuándo se considera pubertad precoz?
Cuando la aparición de los caracteres sexuales secundarios ocurre antes
de los 9 años.
¿Cuándo ocurre el pico de estradiol y cómo se evidencia clínicamente?
El pico de hormona luteinizante ocurre en el día 14 del ciclo; dos días an-
tes (en el día 12), ocurre el pico de estradiol que se evidencia por un au-
mento en la temperatura.
¿A qué niveles ocurren las anomalías que explican la amenorrea de la ado-
lescente?
 Anatómico (Mülleriano): síndrome de Mayer Rokitansky Kunster
Hauser e himen imperforado, aplasia vaginal y uterina y septos vagi-
nales.
 Hipotalámico: síndrome de Kallman [disminución de la hormona li-
beradora de gonadotropinas (GnRH) y anosmia].
 Gonadal: síndrome de Turner (45XO), síndrome de Swyer (disgene-
sia gonadal pura 46xy).
¿Cuándo deben estudiarse la pubertad retrasada y la amenorrea primaria?
 Cuando hay ausencia de caracteres sexuales secundarios después de
los 13 años.
 Cuando hay ausencia de menarca después de los 16 años.
¿Qué fármacos inhiben la dopamina?
Fenotiacinas, butirofenonas, tricíclicos, metildopa, reserpina, metoclo-
pramida, cimetidina y otros.

131
¿Cuál es el efecto de la inhibición dopaminérgica sobre la hipófisis?
Aumenta la secreción de prolactina.
¿Por qué la hiperprolactinemia es causa de amenorrea?
Porque inhibe directamente a la hormona liberadora de gonadotropinas
(GnRH), y tiene acción inhibitoria sobre el ovario.
¿En quién debe sospecharse la presencia de endometriosis?
En cualquier mujer con infertilidad, dispareunia y amenorrea.
¿Cuál de las tres infecciones vaginales más frecuentes presenta mayor sin-
tomatología?
La candidiosis.
¿En qué porcentaje son asintomáticas la vaginosis bacteriana y la tricomo-
niasis?
En 50%.
¿Qué se observa en las laminillas de muestras en estos padecimientos:
 Candidiosis: hifas y pseudohifas (preparada con KOH)
 Vaginosis bacteriana: linfocitos escasos y células clave.
 Tricomoniasis: leucocitos abundantes y parásito flagelado móvil.
¿Qué diferencia hay entre menorragia y metrorragia?
En las dos el sangrado es prolongado y abundante, pero en la menorragia
ocurre a intervalos regulares y en la metrorragia es irregular.
¿Cuáles son tres causas de ciclos anovulatorios?
La inmadurez del eje hipotálamo-hipófisis, la disfunción del eje hipotálamo-
hipófisis y la falla de la retroalimentación por aumento del estradiol.
¿Cuáles son las causas de aumento del estradiol?
El embarazo, la falla del metabolismo y la producción ectópica (adenoma
o mucho tejido adiposo).
¿Qué causas producen disfunción del eje hipotálamo-hipófisis?
 Hiperprolactinemia.
 Stress y ansiedad.
 Pérdida de peso.
 Trastorno limítrofe (borderline).
 Anorexia nervosa.

132
¿Cuál es el tratamiento médico de la hemorragia uterina disfuncional
(HUD) aguda?
Dosis altas de estrógenos ya que estimulan la “cicatrización” de los vasos
sangrantes.
¿Cuál es el tratamiento quirúrgico de la hemorragia uterina disfuncional
aguda?
Dilatación y curetaje.
¿Cuál es el tratamiento de la HUD crónica (hiperplasia endometrial)?
Progestina (10mg/dl), por su acción antimitótica.
¿Cuál es la triada de la endometriosis?
Dismenorrea, dispareunia e infertilidad
¿Cómo se hace el diagnóstico de endometriosis?
En orden de importancia, primero con la historia clínica y la exploración
física. En algunos casos se requiere laparotomía exploradora.
¿Cuáles son los hallazgos a la exploración física en la endometriosis?
 Nodularidad en el ligamento úterosacro, a la palpación rectovaginal.
 Masa anexial en el ligamento redondo, a la palpación vaginal.
¿Qué marcador tumoral se eleva en la endometriosis?
El CA125.
¿Cuál es el tratamiento médico de la endometriosis?
 Progestina y estrógeno para producir la atrofia del tejido ectópico.
 AINES, 2 a 3 días antes de iniciar cada periodo, para aliviar la dis-
menorrea.
 El danazol también causa atrofia del tejido endometrial, pero tiene
efectos antiestrogénicos.
 Las progestinas de depósito, como la medroxiprogesterona, causan
cambios deciduales en el tejido ectópico que alivian los síntomas.
¿En qué padecimientos está indicado el uso de danazol?
Endometriosis y mastopatía fibroquística.
¿Cuál es el tratamiento de la endometriosis severa?
Quirúrgico.
¿Cuáles son los exámenes endocrinológicos utilizables para el estudio de
las amenorreas?

133
 Niveles de prolactina y TSH.
 Niveles matutinos de cortisol.
 Niveles de dehidroepiandrosterona y testosterona.
 T3 y T4.
¿Cuál es el diagnóstico diferencial del dolor pélvico ginecológico?
 Cólico: cuando existe un órgano hueco obstruido; puede originarse
en el intestino, la vesícula, el uréter y el apéndice.
 Súbito: perforación o isquemia.
 Lento: por inflamación u obstrucción (salpingitis, apendicitis, obs-
trucción).
 Localizado: anexos o focal del útero.
 Irritación peritoneal: aumenta con los movimientos abdominales y
generales; aumenta con los movimientos intestinales o vesicales; es
generalizado.
 Masa anexial dolorosa: embarazo ectópico, quiste ovárico y masa in-
flamatoria.
 Vómito precoz: apendicitis, colecistitis, salpingitis, pielonefritis.
 Vómito tardío: obstrucción intestinal.
¿Cuál es el tratamiento de la dismenorrea severa?
Inhibidores de las prostaglandinas y un ciclo de anticonceptivos orales.
¿Cuáles son los pasos en el estudio de la dismenorrea?
 Citología hemática y velocidad de sedimentación globular (VSG) pa-
ra descartar causas infecciosas.
 Exudado vaginal y pruebas específicas para descartar enfermedades
de transmisión sexual (ETS).
 USG pélvico para descartar quistes ováricos.
 Laparotomía exploradora para establecer el diagnóstico de endome-
triosis o para descartar otra patología.
¿Qué son las células clave?
Son células epiteliales vaginales rodeadas de restos de gram-negativos en-
contradas en la secreción vaginal de pacientes con vaginosis bacteriana.
¿Qué serotipos de Chlamydia causan tracoma?
Los serotipos A y C.

134
¿Qué serotipos de Chalmydia causan la conjuntivitis de inclusión?
El D y K.
¿Cuál es el síndrome de Fitz-Hugh-Curtis?
Es una perihepatitis secundaria a enfermedad pélvica infecciosa (EPI) y
causada por Chlamydia o gonococo.
¿Cuáles son las contraindicaciones de contraceptivos hormonales en muje-
res >35años?
Diabetes, tabaquismo, obesidad y migraña.
¿Cuáles son las contraindicaciones generales de los contraceptivos hormo-
nales?
Historia de cáncer endometrial o mamario, hipertensión arterial, antece-
dente de enfermedad vascular cerebral (EVC) y de estados trombóticos.
¿Cuáles son los estímulos estrogénicos que predisponen al cáncer endome-
trial?
Menarca temprana, menopausia tardía, obesidad, síndrome de Stein Le-
venthal, terapia estrogénica sin progestina, tumores secretantes de estró-
geno
¿En qué proporción se desarrolla el cáncer endometrial a partir de la hi-
perplasia endometrial?
1 a 14 % de los casos desarrolla cáncer.
¿Cuáles son los tipos de hiperplasia endometrial?
La hiperplasia simple y la adenomatosa. La adenomatosa puede ser con
atipia o sin ella.
¿Cuál es el tratamiento de la hiperplasia endometrial?
 Se administra una progestina cada 10 a 12 días durante 3 a 6 meses y
se monitorea con biopsias cada 3 meses.
 Si la hiperplasia persiste se somete a la paciente a histerectomía.
¿Cuál es el tratamiento de la hiperplasia endometrial en las adolescentes?
Se administra estrógeno y progestina durante 6 meses.
¿Cuándo se positivizan las deteminaciones de la fracción beta de la gona-
dotropina coriónica humana (HGC) en caso de embarazo?

135
Se puede detectar la fracción beta de la gonadotropina coriónica humana
(HGC) después de 7 días de la concepción y 7 días antes de la fecha espe-
rada de menstruación.
¿Cuándo una prueba sin esfuerzo es negativa?
Cuando no hay aceleraciones en el ritmo cardiaco que duren más de 15
segundos durante un periodo de 20 minutos.
¿Qué es la reactividad?
Es un aumento de 15 a 20 latidos por minuto en la frecuencia cardiaca fe-
tal basal ante un estímulo que dure 15 o 20 segundos.
¿Cuál es la causa más frecuente de hiporreactividad no patológica?
Durante el sueño fetal puede haber periodos de hasta 40 minutos de hi-
porreactividad.
¿Cómo se define el sufrimiento fetal?
Como un aumento en la frecuencia cardiaca del feto acompañado de una
disminución en el pH fetal.
¿Cuáles son los resultados de la hipoxia fetal prolongada?
Acidosis respiratoria que evoluciona a metabólica y disminución de la
frecuencia cardiaca fetal.
¿Cómo se interpreta la variabilidad disminuida de la frecuencia cardiaca
fetal (FCF)?
Como un pobre control autonómico sobre la FCF, lo cual es de mal pro-
nóstico para el producto.
¿Qué exámenes están indicados desde la semana 15 a la 20 del embarzo?
Determinación de alfa-fetoproteína para la detección de defectos del tubo
neural, niveles de gonadotropina coriónica para detección de síndrome de
Down y determinación de estriol no conjugado para detectar trisomía 18.
¿Cuáles son dos signos de embarazo ectópico encontrados en la explora-
ción física?
Masa anexial y dolor a la movilización del cérviz.
¿Por qué las infecciones bacterianas son un factor de riesgo de parto pre-
término?
Las bacterias producen prostaglandinas que estimulan al miometrio.
¿Cuál es el agente etiológico más común de la infección puerperal grave?

136
Clostridium.
¿Cuál es el agente de la infección puerperal sin hemólisis?
Estreptococo grupo B.
¿Cuál es la incidencia de hemorragia del tercer trimestre del embarazo?
Con un episodio previo sucede en 1 de 250. El abruptio placentae ocurre 1
en 120.
¿Cuál es la incidencia del aborto recurrente?
Si es aislado, 2%. Si es recurrente, de 25 a 30%.
¿Cuál es la triada del embarazo ectópico?
Amenorrea, manchado (sangrado) y dolor.
¿Cómo se clasifica la hipertensión durante el embarazo?
En hipertensión crónica complicada con el embarazo e hipertensión in-
ducida por el embarazo.
¿Cuáles son los subtipos clínicos de la hipertensión inducida por embarazo?
 Preeclampsia: cuando existe compromiso renal y se detecta proteinu-
ria.
 Eclampsia: cuando hay compromiso del SNC y ocurren convulsio-
nes.
 Síndrome HELLP: presencia de hemólisis, enzimas hepáticas eleva-
das y plaquetas bajas.
¿Cuándo se diagnostica la hipertensión en el embarazo?
 Cuando existe un aumento de 30 mm Hg en la sistólica y de 15 mm
Hg en la diastólica en relación con valores previos antes de las 20 se-
manas de gestación (SDG).
 Cuando se detecta una cifra aislada de 140/90 después de las 20
SDG
 Cuando hay un aumento de 20 mm Hg en la tensión arterial media
(TAM) en relación a valores previos o un valor total de 150 mm Hg
en la TAM.
¿Cómo se diagnostica la proteinuria?
Normalmente hay una excreción de proteínas de 0.15 g en 24 horas.
Cuando las proteínas en orina alcanzan 3 g se habla de proteinuria.
¿Cómo se diagnostica la presencia de edema?

137
Con el signo de godete o la ganancia de 2,300 g en una semana.
¿Cuáles son los criterios de severidad de la preeclampsia?
Presión sistólica de 160 mm Hg y diastólica de 110 mm Hg.
Proteinuria mayor a 5 g/24horas.
Creatinina sérica mayor de1.4.
Convulsiones.
Edema pulmonar.
Menos de 10,000 plaquetas.
Disfunción hepática, dolor epigástrico o ruptura de la cápsula.
Trastornos vasculares como espasmo retiniano y anemia microangiopática.
Trastornos fetoplacentarios como RCIU o sufrimiento fetal.
¿A qué se llama isoinmunización Rh?
Es la producción de anticuerpos maternos ante la exposición a células fe-
tales que contienen antígenos del grupo Rh como el Cc, Dd y Ee. Estos
anticuerpos atacan a los eritrocitos fetales y prodecen diferentes grados
de hemólisis e incluso hidrops fetalis.
¿Qué es la eritroblastosis fetal?
Es la presencia de hemólisis fetal secundaria a anticuerpos anti-d mater-
nos, anemia secundaria a hemólisis, obstrucción umbilico-portal fetal que
simula hipertensión portal, cese de las funciones hepáticas normales, caí-
da de la presión coloidosmótica fetal, que causa edema.
¿Cómo se trata la isoinmunización Rh?
Con la administración de gamma-globulina anti-d dentro de las primeras
72 horas posparto.
¿Cómo se diagnostica la intolerancia a la glucosa en el embarazo?
A las 24 o 28 semanas debe haber al menos dos prueba de tolerancia a la
glucosa con valores alterados.
¿Por qué la diabetes en el embarazo causa polihidrammios?
Porque la hiperglucemia produce poliuria en el producto.
¿Cuáles son las dosis de insulina para tratar la diabetes gestacional?
 Inicial 15 a 20 U. En la primera mitad del embarazo se calcula 0.5
U/kg. En la segunda mitad del embarazo se calcula a 0.7 U/kg.

138
 Se administran 2/3 de la dosis total en la mañana. La dosis matutina
puede estar compuesta por 2/3 de insulina intermedia y 1/3 de insu-
lina rápida.
 Se administra el 1/3 restante por la noche. Esta dosis se compone de
½ rápida y ½ lenta.
¿Por qué la insulina no afecta al producto?
Porque debido a su peso molecular mayor de 6,000 no cruza la barrera
placentaria.
¿Cuál es la incidencia de la enfermedad trofoblástica?
Benigna: 1 en 1,200; maligna: 1 en 20,000.
¿Cuándo se habla de neoplasia gestacional metastásica de bajo riesgo?
Cuando los niveles de gonadotropina coriónica beta son menor de 40,000
y la evolución es menor a 4 meses.
¿Qué es una mola incompleta?
Es una variedad de enfermedad trofoblástica en la que hay presencia de
un feto y membranas. Las vellosidades coriónicas se encuentran edema-
tizadas y el cariotipo es 69XXX.
¿Cuál es la tétrada de la enfermedad trofoblástica?
 Sangrado en el primer trimestre
 Preeclampsia
 Hipertiroidismo clínico (por reacción cruzada de la -HGC en exceso)
 Náusea vómito, dolor abdominal.
¿Cuándo está indicado utilizar quimioterapia e histerectomía simple en la
enfermedad trofoblástica?
En la neoplasia trofoblástica gestacional no metastásica y en la que cursa
con metástasis de bajo riesgo.

139
140
HEMATOLOGÍA

¿Cuáles son los diferentes sitios donde se lleva a cabo la hematopoyesis


durante la vida fetal?
 0 a 12 semanas en el saco vitelino.
 12 a 24 semanas en el hígado.
 A partir de las 24 semanas en la médula ósea.
¿Cuáles son los índices eritrocitarios?
 Hemoglobina corpuscular media (HCM)= Hemoglobina (Hb)/Hematíes
(RBC).
 Concentración de hemoglobina corpuscular media (CMCH)= Hb/
hematocrito (Hto).
 Volumen corpuscular medio (VCM)= Hto/RBC.
¿Qué pruebas se usan en los estudios hematológicos?
 Ferritina: 15-150 g/dl. Es la principal proteína para el almacenaje
de hierro; se usa para el diagnóstico de deficiencia de fierro o de su
exceso, como en la hemocromatosis.
 Hierro sérico: 50-150 mg/dl. Para el diagnóstico diferencial de las
anemias. Bajo en las ferropénicas, elevado en las hemolíticas.
 Hierro medular: 50-60%. es el estándar de oro para el diagnóstico de
ferropenia.
 Haptoglobina: es un indicador de hemólisis crónica; su nivel dismi-
nuye en las reacciones postransfusionales.
 Fosfatasa alcalina leucocitaria: para el diagnóstico diferencial entre la
leucemia mielógena crónica y la reacción leucemoide.
 Capacidad de fijación de transferrina: 300-360 g/dl.

141
 Saturación de transferrina: 30-50%.
 Citometría de flujo: se utiliza para el diagnóstico de las leucemias,
mielodisplasias y los linfomas; para medir la actividad sintética de
DNA en los tumores y para el conteo de subpoblaciones de linfocitos.
 INR: 2-3. Es otra forma de informar el tiempo de protrombina (TP)
para medir la actividad de los factores dependientes de la vitamina k
(vía extrínseca) y monitorear la acción de los cumarínicos.
¿Cuáles son causas de anemia microcítica?
 Falla en la síntesis del hem como en la falta de hierro, la sideroblasto-
sis o la inflamación crónica.
 Falla en la síntesis de globina, como en las talasemias o en las enfer-
medades por hemoglobinas anormales.
¿Cómo se clasifican las anemias macrocíticas?
 Con médula megaloblástica, como en las deficiencias de vitamina B-
12 y ácido fólico.
 Sin médula megaloblástica como en las hepatopatías, el hipotiroi-
dismo y la enfermedad de médula ósea.
¿Cuáles son las causas de anemia normo-normo?
 Anemia aplásica.
 Hemorragia.
 Hemólisis.
 Insuficiencia renal crónica.
¿Cuáles son las anemias por trastornos del metabolismo eritrocitario?
 Los defectos de la vía de Embden-Meyerhof.
 Los defectos en el shunt hexosa-monofosfato (deficiencia de glucosa-
6-fosfato-deshidrogebasa) (DG6PD).
¿Cuáles son las anemias por trastornos congénitos de la membrana eritro-
citaria?
 Eliptocitosis hereditaria: transtorno de herencia autosómica domi-
nante, eliptocitos, escasa o nula hemólisis, esplenomegalia ocasional.
 Esferocitosis hereditaria: heredada como rasgo dominante, con he-
mólisis franca e ictericia.

142
¿Qué datos se observan al microscopio en las enfermedades siguientes?
 Talasemia: microcitosis y poiquilocitosis.
 Anemia sideroblástica: sideroblastos en anillo (depósitos de fierro mi-
tocondrial vistos con azul de Prusia).
 Anemia perniciosa: macrocitosis, polimorfonucleares hipersegmen-
tados; en la médula ósea se observan células con asincronía citoplas-
ma-núcleo.
 Chédiak Higashi: Inclusiones granulomatosas gigantes en los neutró-
filo.
 Anemias microangiopáticas (coagulación intravascular diseminada
(CID) púrpura trombocitopénica trombótica (PTT), válvula cardia-
ca): esquistocitos.
¿Cuáles son causas de anemia microcítica y cuenta baja de eritrocitos?
Anemia ferropénica y talasemia alfa.
¿Cuáles son causas de anemia microcítica y cuenta de eritrocitos normal o
alta?
La talasemia beta menor.
¿Cuáles son las alteraciones de laboratorio en la ferropenia?
Ferritina menor de 15, hierro sérico menor de 60, capacidad de fijación
de transferrina mayor de 360, saturación menor de 20%.
¿Cuáles son las alteraciones de laboratorio en la anemia microcítica hipo-
crómica?
VCM menor de 80 fl, CMHC menor de 30g/dl, Hb menor de 11, Hto
menor de 38, capacidad de transporte de hierro aumentada, hierro sérico
disminuido.
¿Cuáles son las alteraciones de laboratorio de la anemia de la enfermedad
crónica?
Capacidad de transporte de hierro baja, hierro sérico bajo anemia nor-
mocítica normocrómica.
¿Cuál es el tratamiento de la anemia de la enfermedad crónica?
Eritropoyetina.
¿Qué es la anemia perniciosa?
Es una anemia macrocítica con: polimorfonucleares hipersegmentados,
médula hipercelular, eritropoyesis inefectiva, hemólisis intramedular que

143
causa deshidrogenasa láctica menor de 500 (normal de 100-2,450), glosi-
tis atrófica y síntomas neurológicos. Se debe a un déficit de vitamina B-
12 que puede ser por una dieta insuficiente, o bien, por deficiencia de se-
creción de factor intrínseco por las células parietales gástricas, como se
observa en ciertos tipos de gastritis, como la atrófica tipo A, que tiene an-
ticuerpos anticélula parietal.
¿Qué complicación grave puede acompañar a la anemia perniciosa?
La degeneración de las astas anteriores medulares.
¿Qué es la anemia aplástica?
Es una falta de células madre en la médula ósea.
¿Cuál es la diferencia en el tratamiento de la eliptocitosis y la esferocitosis?
En la esferocitosis es necesaria la esplenectomía; en la eliptocitosis puede
ayudar, pero generalmente no es necesaria si la hemólisis y la anemia son
leves.
¿Qué es la deficiencia de glucosa-6-fosfato-deshidrogenasa (DG6PD)?
Es una alteración en la vía de la pentosa fosfato para producir NADPH que
active la glutatión-reductasa. El glutatión reducido actúa como antioxidan-
te en los eritrocitos eliminando todos los radicales O2.
¿Dónde observamos los cuerpos de Heinz?
En la deficiencia de glucosa-6-fosfato-deshidrogenasa.
¿Por qué se observan “células mordidas” en la DG6PD?
En los individuos con bazo activo, éste extrae de los hematíes los cuerpos
de Heinz (estadios avanzados) y les da esa apariencia.
¿Qué fármacos desencadenan hemólisis en pacientes con DG6PD?
Primaquina, salicilatos, sulfas, nitrofurantoína.
¿Cuáles son las variedades de la DG6PD?
 La tipo A, que se da en Asia y África
 La tipo B, que es el tipo mediterráneo.
¿Qué es la deficiencia de piruvatocinasa (DPK)?
La piruvatocinasa produce ATP en los eritrocitos; si éste falta, se inacti-
van las bombas sodio-potasio.
¿Cuál es la diferencia en el tratamiento de la DG6PD y de la DPK?
En la primera no se indica la esplenectomía; en la segunda sí.

144
¿Cómo está formada la hemoglobina?
La hemoglobina es una proteína con un peso molecular de 64450 forma-
da por cuatro subunidades, cada una de ellas contiene un hem conjugado
con un polipéptido. El hem es un derivado de la porfirina y los polipépti-
dos se llaman cadenas alfa, beta, gamma, etcétera.
¿Cuáles son los pasos en la formación del hem?
Ácido delta aminolevulínico  porfobilinógeno  hidroximetilbilano 
URO III  coproporfirinógeno  protoporfirinógeno  protoporfirina
 hem.
¿Qué es la metahemoglobina?
Es la hemoglobina cuya fracción de hierro se encuentra en estado férrico
en lugar de ferroso.
¿Qué sistema evita la sobreproducción de metahemoglobina?
El sistema reductasa NADH-metahemoglobina.
¿A qué se llama anemias por síntesis defectuosa de hemoglobina?
Son alteraciones genéticas de la molécula de Hb que se demuestran por
cambios en las características químicas, en la movilidad electroforética o
en otras propiedades.
¿Cuáles son las hemoglobinas del adulto?
HbA (cadenas alfa-2-beta-2), HbA2 (alfa-2-delta-2), HbA1c. Todas son
normales. La más abundante es la HbA.
¿Cuáles son las hemoglobinas fetales?
Hemoglobina F (alfa2-gamma2).
¿Cuáles son las hemoglobinas embrionarias?
Gower 1(z22) y Gower 2 (alfa2-épsilon2).
¿Cuál es el error genético que da origen a la alfa-talasemia?
La deleción del gen que codifica la cadena alfa.
¿Cuál es el error genético que da origen a la beta-talasemia?
Una anormalidad en la trascripción para codificar la cadena beta.
¿Cuántos genes existen para la codificación de la cadena alfa?
Cuatro.
¿Cómo se le llama al estado en que existen sólo tres genes para la cadena alfa?
Portador asintomático.

145
¿Cómo se le llama al estado en el que existen sólo dos genes para cadena alfa?
Rasgo de alfa talasemia.
¿Por qué la beta talasemia causa hemólisis?
Porque el exceso de cadenas se precipita en el eritrocito y produce su li-
sis.
¿Qué pasa con el exceso de cadenas beta en la alfa talasemia?
Forman la hemoglobina H también conocida como beta-4.
¿Cómo se llama el procedimiento cuando hay sólo un gen de cadena alfa?
Enfermedad de hemoglobina H.
¿Cuál es la diferencia entre la Hb de Bart y la Hb H?
En la primera hay 4 genes afectados de la cadena alfa y en la segunda
tres. En ambas hay hemoglobina gamma-4F. En la primera hay hidrops fe-
talis y muerte in útero. En la segunda, el producto llega a término.
¿Qué hemoglobinas aumentan en la beta-talasemia?
La HbA2 (alfa2-delta2) y la HbF (alfa2-gamma2).
¿Qué es la anemia de Cooley?
Es la beta-talasemia mayor. Cursa con hepatoesplenomegalia, anemia
hemolítica severa, hiperplasia medular, hiperostosis facial y absorción
aumentada de fierro, que causa hemocromatosis.
¿Cuál es el fenotipo de la anemia de Cooley?
Beta°-beta°. Aparece a los seis meses con hemólisis severa e hiperplasia
medular.
¿Cuál es el tratamiento de las talasemias?
Ácido fólico, transfusiones y transplante de médula ósea.
¿Cuáles son los otros nombres de la anemia de células falciformes?
Anemia drepanocítica, drepanocitosis hereditaria, enfermedad de la he-
moglobina S.
¿Cuál es el cuadro clínico de la anemia de cédulas falciformes?
El cuadro está dominado por dos puntos principales:
 La anemia grave, con ictericia y bilirrubinemia de 2-4 mg/dl, y que
se torna en verdaderas crisis aplásticas tras alguna infección viral.
 Las lesiones vasooclusivas, a causa de que la membrana eritrocitaria
anormal muestra una adherencia aumentada al endotelio. Esto causa

146
dolor a diferentes niveles, como los huesos largos y de manos y pies
(síndrome mano-pie); puede causar necrosis avascular del carpo y
metacarpo, así como autoesplenectomía. Es causa de tórax y abdo-
men agudos. En otros casos pueden ocluirse vasos intracraneales que
causen hemiplejia o una variedad de déficits neurológicos.
¿Qué es la hemoglobina S?
Es la sustitución de valina por ácido glutámico en la sexta posición de la
cadena beta de la globina, que la hace menos soluble en oxígeno.
¿Cuál es el mecanismo fisiopatológico para desarrollar células falciformes?
Al desoxigenarse la hemoglobina, se aglutina y se hace rígida, lo que de-
forma a la célula y la vuelve falciforme (sickle cell).
¿Hasta cuándo está protegido el infante contra la hemoglobina S?
Hasta los seis meses cuando todavía tiene hemoglobina F.
¿Qué medicamento aumenta la hemoglobina F?
La hidroxiurea.
¿Qué infecciones son más frecuentes en los enfermos que padecen esta
anemia?
Las infecciones causadas por encapsulados, como H. influenza o Salmonella.
¿Qué provoca el Parvovirus B19 en los pacientes con anemia de células fal-
ciformes?
Puede provocar crisis aplásticas.
¿Cuál es el tratamiento de la anemia de células falciformes?
La esplenectomía y los hematínicos carecen de valor. Se aplican transfu-
siones en caso de anemia grave y se utilizan medidas sintomáticas.
¿Cuál es el tratamiento profiláctico antibiótico para pacientes con anemia
de células falciformes?
Amoxicilina: 125 mg/día.
¿Qué es la anemia de Fanconi?
Es la anemia aplástica congénita.
¿A qué edad se manifiesta clínicamente la anemia de Fanconi?
Entre los 3 y los 8 años.
¿Cuál es el cuadro clínico de la anemia de Fanconi?
Piel pigmentada, retraso en el crecimiento, anormalidad renal, anormali-
dades óseas como la agenesia del pulgar, radio o metacarpianos, hemo-
globina F y macrocitosis.

147
¿Hasta cuándo está presente la hemoglobina F?
Hasta los 9 o 12 meses; generalmente hacia los 6.
¿Qué factores causan anemia aplástica?
El benceno, el cloramfenicol, las sulfas y la radiación.
¿Qué organismos pueden causar supresión de médula ósea?
El virus del dengue, parvovirus humano B19, virus de Epstein-Barr, virus
de la inmunodeficiencia humana y el citomegalovirus.
¿Cuál es la dosis diaria de fierro para tratar la anemia ferropriva?
6 mg/kg día.
¿Cuál es la dosis en la terapia con ácido fólico?
De 5 a 10 mg/día.
¿Qué refleja el hallazgo de ferritina sérica baja?
Que el hierro medular está bajo.
¿Qué es la anemia sideroblástica?
Es la anemia por la deficiente utilización de Hierro para la formación de Hb.
¿Qué factores causan anemia sideroblástica?
 Deficiencia de piridoxina (vitamina B-6) como en el alcoholismo.
 Intoxicación por plomo.
 Defecto de la delta-aminolevulínicosintetasa.
 Isoniacida.
¿Qué se observa en un frotis de sangre periférica y de médula ósea el caso
de la anemia sideroblástica?
 En la sangre periférica se observan células diana policromatófilas (en
blanco de tiro).
 En la médula ósea se observan hiperplasia eritroide y sideroblastos en
anillo. El “diamante del anillo” es un cúmulo de mitocondrias sobre-
cargadas de fierro. Existe eritropoyesis inefectiva.
¿Cuál es el tratamiento de la anemia sideroblástica?
Se trata con transfusiones y eritropoyetina.
¿Qué porcentaje de las anemias sideroblásticas desarrollan leucemia aguda?
10 %
¿Cuáles son la causas de la sobrecarga de Fe en las talasemias?
 La absorción está aumentada.

148
 Las transfusiones frecuentes causan hemocromatosis (diabetes bron-
ceada) y afectan el hígado, páncreas, corazón, pulmones y piel.
¿Cómo se transporta la vitamina B-12 en el plasma?
Por medio de la transcobalamina.
¿Cómo se administra el tratamiento con vitamina B12?
Una dosis de carga de 1,000 mg y una dosis mensual de 100 mg.
¿Cuál es el síndrome de Diamond Blackfan?
Es la aplasia pura de células rojas.
¿Cuál es el aumento de hemoglobina (Hb) y de hematrocito (Hto) al admi-
nistrar un concentrado eritrocitario?
Aumenta 1 g/dl de Hb y 3% de Hto.
¿Cuándo se usan glóbulos rojos lavados?
Cuando hay reacción anafiláctica, urticaria no prevenible por antihista-
mínicos y disminución de IgA.
¿Cuál es el límite inferior de neutrófilos?
1,500. Se considera neutropenia grave cuando existen menos de 500.
¿Qué infecciones son frecuentes ante una neutropenia de 500 a 100?
Mucocutáneas principalmente.
¿Para qué se usa la prueba de aglutinación a la ristocetina?
Cuando las coagulopatías son debidas a deficiencia del factor vW la
prueba de ristocetina resulta baja.
¿Cuál es la neoplasia infantil más común?
La leucemia linfoblástica que representa 80% de las leucemias aguda
(LLA).
¿Cómo se clasifican las leucemias agudas en niños?
Leucemia linfoblástica aguda (LLA) y leucemia mieloblástica aguda
(LMA).
¿Cómo se clasifican las leucemias crónicas en niños?
Solo existen las no linfocíticas.
¿Cuáles son las LLA?
La leucemia linfoblástica aguda (de célula B, CALLA positivo), el linfo-
ma de Burkitt y la leucemia de células T.
¿Cuáles son las leucemias mieloblásticas agudas (LMA)?
La clasificación comprende 7 tipos celulares, lo que depende de la etapa
de maduración de la célula mieloide.

149
¿Cuáles son las leucemias linfocíticas crónicas (LLC)?
La clasificación comprende cuatro tipos, en los que domina la linfocitosis
de células maduras.
¿Cuál es el tratamiento de la LLC?
Clorambucil (alquilante), bajas dosis de radiación y fludarabina (antime-
tabolito).
¿Cuáles son las leucemias mielocíticas crónicas (LMC)?
Son leucemias que tienen la fosfatasa alcalina baja, neutrófilos maduros
aumentados y cromosoma Filadelfia (Ph) positivo.
¿Cuál es el cuadro clínico dominante en las leucemias?
 Fallo la médula ósea, que afecta todas las líneas celulares.
 Fallo del sistema reticuloendotelial.
 Dolor óseo.
 Sitios “aislados” involucrados como el sistema nervioso central (SNC).
¿De que células se conforman las LLA?
84% de precursores de células B, 1% de células B, 15 % de células T.
¿Cuáles son marcadores de buen pronóstico en los casos de ALL?
 Poliploidia mayor de 56 cromosomas.
 Gen TEL-AML1.
¿Cuáles son marcadores de mal pronóstico en los casos de ALL?
 Pseudodiploidia.
 Hipoploidia menor de 46 cromosomas.
¿Cuáles son marcadores para LLA Pre-B?
El antígeno CALLA positivo y el TdT positivo.
¿Cuáles son marcadores para células B?
El antígeno CALLA negativo y el TdT negativo.
¿Cuáles son las fases de la leucemia mielocítica crónica (LMC)?
Crónica y acelerada.
¿Cómo actúa la prednisona en las leucemias?
Como un citotóxico, gracias a su metabolito la prednisolona.
¿Qué es la enfermedad de Hodgkin?
Es un grupo de cánceres de origen celular incierto, caracterizados por la
presencia de células de Reed-Stemberg (células gigantes binucleadas), que se

150
divide fundamentalmente en cuatro tipos: de predominio linfocítico, con
esclerosis ganglionar, de celularidad mixta y con depleción de linfocitos.
¿Cuál es la incidencia de la enfermedad de Hodgkin?
Presenta una incidencia bimodal según la edad de aparición; tiene un pi-
co en los adultos jóvenes de 10 a 25 años (predomina la esclerosis gan-
glionar y es más benigna), y otro en los adultos mayores de 50 años
(predomina la celularidad mixta y es más agresiva).
¿Cuál es la clasificación de Ann Arbor para la enfermedad de Hodgkin?
 I: se involucra sólo una región de linfáticos (I) o un solo sitio extralin-
fático (IE).
 II: se involucran dos regiones de linfáticos (II) o extralinfáticos (IIE)
de un solo lado del diafragma.
 III: se involucran múltiples regiones de linfáticos (III) o extralinfáti-
cos (IIIE) (a ambos lados del diafragma o el bazo (IIIB).
 IV: hay una enfermedad diseminada linfática o extralinfática.
¿Cómo se subclasifica la enfermedad de Hodgkin?
Según los síntomas que presente, se divide en A para los asintomáticos y
en B para los sintomáticos.
¿Cuáles son los síntomas B?
Fiebre de 38ºC, sudores nocturnos, pérdida inexplicada de 10 % de peso.
¿Cómo se presenta el linfoma de Burkitt?
Como una masa abdominal o masa en el maxilar inferior.
¿Morfológicamente cómo se clasifica el linfoma no hodgkiniano?
 Linfoblástico: parecido a L1 y L2 (de tipos celulares T, derivados del
timo).
 No linfoblástico: parecido a L3 (de tipos celulares B, derivados de la
médula ósea).
¿Cómo se presenta el linfoma de células T?
Como una masa mediastinal anterior y derrame pleural.
¿Cómo se presenta el linfoma de células B?
Como una masa abdominal.
¿Cuál es la forma más frecuente de presentación de la enfermedad de Ho-
dgkin?
La adenopatía cervical.

151
¿Cuáles son los efectos secundarios del tratamiento de la enfermedad de
Hodgkin?
Hipotiroidismo, esterilidad, nausea y vómito.
¿Cuál es el tratamiento del linfoma de Hodgkin diseminado primario
(IIIA, IIIB, IV)?
La quimioterapia combinada: mecloretamina, vincristina, procarbazina y
prednisona (programa MOPP).
¿Cuál es el tratamiento para el linfoma de Hodgkin diseminado recidivante?
La combinación de adriamicina, bleomicina, vinblastina y decarbazma
(ABVD).
¿Cuál es el tratamiento de la enfermedad de Hodgkin en etapas I y II?
La radioterapia.
¿Qué son las gammopatías monoclonales?
Son discrasias de células plasmáticas caracterizadas por la proliferación
desordenada de un clon de célula plasmática (encargado de la producción
de una Ig para un antígeno específico) y por la presencia en la sangre o en
la orina de una Ig electroforética o estructuralmente homogénea, es decir,
monoclonal.
¿Cuál es el orden en que aparecen las diferentes globulinas en una electro-
foresis?
Se evidencian por picos. El primer pico es el que representa la albúmina y
es la fracción más grande. Los siguientes son alfa1, alfa2, beta y gamma.
¿Cuáles son las gammopatías sintomáticas más frecuentes?
El mieloma múltiple, la macroglobulinemia de Waldeström, la amiloido-
sis sistémica primaria no hereditaria y la enfermedad de cadenas pesadas.
¿Cuál es el pronóstico para cada una de las siguientes enfermedades?
 Leucemia linfoblástica aguda: la quimioterapia cura la mayoría de los
casos.
 Leucemia mieloblástica aguda: la quimioterapia sólo cura hasta la
mitad de los casos; 2/3 de los casos curan con transplante de médula
ósea.
 Leucemia mielocítica crónica: el adulto sobrevive 2 años; los más jó-
venes 9 meses en promedio.

152
 Linfoma no-Hodgking: la quimioterapia y la radioterapia son curativas.
 Enfermedad de Hodgkin: a) la etapa I tiene 90% de curación; b) la
etapa II: 60 a 70%.
¿Qué neoplasias dan eritrocitosis?
El hipernefroma, el quiste renal, el hemangioblastoma y el hepatoma.
¿Cuáles son causas de eritrocitosis relativa?
El síndrome de Gaisbock (eritrocitosis por estrés) y por disminución del
volumen plasmático.
¿Cuáles son causas de policitemia con médula autónoma?
La policitemia vera y la mielofibrosis.
¿Cuáles son los criterios mayores para el diagnóstico de policitemia vera?
 El aumento en la masa eritrocitaria.
 La saturación de oxígeno mayor de 92% (para descartar la hipoxia
como causa de la eritrocitosis).
 La esplenomegalia.
¿Cuáles son los criterios menores para el diagnóstico de policitemia vera?
 La leucocitosis.
 La trombocitosis.
 El aumento de la fosfatasa alcalina leucocitaria.
 El aumento de la vitamina B-12 sérica.
¿A qué se le llama agranulocitosis?
A un cuadro constituido por cifras de neutrófilos menores de 500, fiebre,
postración, faringitis necrótica y formas celulares inmaduras (insuficien-
cia medular).
¿Cómo se hace el diagnóstico de leucemia mieloide crónica?
Con la presencia del cromosoma Filadelfia y niveles bajos de fosfatasa
alcalina.
¿Qué es la hemoglobinuria paroxística nocturna?
Es una debilidad de las células madre para la lisis por complemento faci-
litada por la seudoacidosis del sueño.
¿Qué enfermedades hematológicas se pueden tratar con esplenectomía?
 La beta-talasemia mayor de Cooley, cuando las transfusiones son
mayores a 250 ml/k/año.

153
 Los defectos de la membrana eritrocitaria: eliptocitosis, esferocitosis.
 Las anemias autoinmunes, por medicamentos, neoplasia o enferme-
dad de la colágena.
 Deficiencia de piruvatocinasa mas no en la DG6PD.
 La púrpura trombucitopénica idiopática.
¿Cuál es el cuadro clínico de la esferocitosis hereditaria?
Malestar, dolor abdominal, ictericia, anemia y esplenomegalia; existen
úlceras en los miembros pélvicos y cálculos biliares. El tratamiento es la
esplenectomía.
¿Cómo se hace el diagnóstico de esferocitosis?
Se deben observar esferocitos, reticulocitos aumentados y pruebas de fra-
gilidad osmótica positivas.
¿Qué otro nombre reciben los factores K dependientes?
Complejo protrombina.
¿Cómo actúa el factor de VonWillebrand?
Hace un puente entre el colágeno vascular y las plaquetas. También dis-
minuye la pérdida de factor VIII.
¿Cuáles son las reacciones que inician y terminan la cascada de la coagula-
ción?
La vía extrínseca inicia con el factor VII activado; la vía intrínseca, con la ac-
tivación del factor VII. La serie de reacciones que inician en las dos vías
culminan con la activación del factor II (protrombina) para formar trom-
bina, que actúa sobre el fibrinógeno para formar fibrina.
¿Cuáles son los estímulos que activan a las plaquetas?
 La disminución de prostaciclina: en endotelio sano produce PGI2 que
eleva el nivel intraplaquetario de AMPc (que inhibe la liberación de
gránulos plaquetarios). Cuando el endotelio se daña, la PGI2 ya no se
une a los receptores plaquetarios y favorece la activación.
 El colágeno expuesto: al dañarse el subendotelio, las plaquetas auto-
máticamente se adhieren al colágeno mediante receptores específicos,
los cuales median la liberación de gránulos plaquetarios que contie-
nen ADP y serotonina. Estos factores median la activación de la glu-
coproteína IIa/IIIb (receptores plaquetarios de fibrina); mediante esta
activación pueden formarse puentes de fibrina entre las plaquetas.

154
 La síntesis aumentada de tromboxano: la estimulación plaquetaria
provocada por trombina, colágeno y ADP provoca que la enzima ci-
clooxigenasa produzca prostaglandina H2, que en un segundo paso es
transformada en tromboxano A2, esencial para la formación rápida
del tapón hemostático.
¿Cuáles son los tres mecanismos anticoagulantes?
 La antitrombina III: se fija a todas las proteasas del sistema de coagu-
lación e inhibe su acción.
 La trombomodulina: junto con la trombina forma el complejo trom-
bina-trombomodulina, que activa a la proteína C (y a su cofactor, la
proteína S), que inactiva a los factores V y VIII.
 La plasmina (fibrolisina): la trombina y el activador tisular del plas-
minógeno favorecen la formación de plasmina, que es el componente
activo del sistema fibrinolítico cuya función es lisar a la fibrina y al
fibrinógeno.
¿Sobre qué factores actúa la vía antitrombina-heparansulfato?
IX, X, XI y XII.
¿Sobre que factores actúa la proteína CS?
Sobre el V y VII activados.
¿Cuáles son los antiagregantes plaquetarios?
 La aspirina: actúa inhibiendo a la ciclooxigenasa plaquetaria (con la
consecuente falta de tromboxano A2) de manera irreversible; por tan-
to, las plaquetas pierden su capacidad de agregación durante toda su
vida media (7 días).
 La ticlopidina y el clopidogrel: inhiben la vía del ADP que inhibe la
activación del receptor de fibrina.
 El dipiridamol: es un vasodilatador coronario que se usa combinado
con aspirina. Su acción consiste en inhibir a la fosfodiesterasa plaque-
taria con el consecuente aumento de AMPc plaquetario.
 Los inhibidores de la glucoproteína IIa/IIIb: el abciximab, eptifibati-
de y tirofibán conforman este grupo y actúan uniéndose a la gluco-
proteína IIa/IIIb y asi evitan la formación de puentes de fibrina entre
las plaquetas.

155
¿Cuál es el mecanismo de acción de los cumarínicos?
Una vez que ha sido utilizada como cofactor, la vitamina k se transforma
en epóxido y requiere de la enzima vitamina k-epóxido-reductasa para reuti-
lizarse. La acenocumarina, el dicumarol y la warfarina bloquean esta en-
zima, y afectan, por tanto, a todos los factores K-dependientes.
¿Cuál es el mecanismo de acción de la heparina?
Aumenta la afinidad de la antitrombina III por todas las proteasas que
inhibe y potencia su acción.
¿Cómo se clasifican los trastornos de la coagulación?
 Trastornos vasculares.
 Enfermedades de las plaquetas.
 Trastornos hereditarios de la coagulación.
 Trastornos adquiridos de la coagulación.
¿Cuáles son enfermedades hemorrágicas por defecto de los vasos?
Escorbuto, síndrome hemolítico-urémico (SHU), púrpura de Henoch-
Schönlein (PHS), púrpura senil, púrpura por esteroides, hemorragia-
telangiectasia hereditaria.
¿Cuál es el tiempo de tromboplastina parcial (TTP) activado normal?
30-40 segundos.
¿Cuál es el tiempo de protrombina normal?
12-15 segundos.
¿Cuál es la forma de presentación más común de la hemofilia A?
Hemartrosis.
¿Cuál es el tratamiento de la Hemofilia B?
PFC o purificados de complejo protrombina que contienen factores II,
VII, IX y X.
¿Qué enfermedades aumentan el TTP sin historia de sangrado?
Deficiencia de factor XII, deficiencia de prekalicreína y deficiencia de ci-
nógeno de alto peso molecular.
¿En qué síndrome se encuentra tiempo de sangrado prolongado con todo lo
demás normal?
En el síndrome de Glanzman.

156
¿Cómo se presenta la enfermedad de VonWillebrand (EVW)?
Con estigmas de deficiencia plaquetaria (sangrado mucocutáneo) y es-
tigmas de deficiencia de factores (hemartrosis), pues afecta la función de
las plaquetas y del factor VIII.
¿Cómo se presentan las pruebas de laboratorio en la EVW?
 TTP prolongado: lo que indica anormalidad de la vía extrínseca.
 Tiempo de sangrado aumentado: porque no se forma el puente pla-
queta-colágeno.
 Prueba de coagulación con ristocetina alterada.
¿Cuándo se indica el PFC?
Cuando el tiempo de protrombina es mayor de 15 el tiempo de trombo-
plastina parcial es mayor de 40, en casos de sobredosis de anticoagulante
oral y en caso de hemofilia B.
¿Qué contienen los crioprecipitados?
Fibrinógeno, factor V, factor VIII y factor XIII.
¿Cuándo se indican los crioprecipitados?
Cuando el fibrinógeno es menor de 100 g/dl
¿Cuál es la dosis de los crioprecipitados?
7 a 10 mg/kg de peso.
¿Qué porción del factor VIII se encuentra en los concentrados?
Porción VIII procoagulante.
¿Qué porciones del factor VIII se encuentran en el crioprecipitado?
VIII procoagulante y VIII antigénico.
¿Cuál es el tratamiento de la enfermedad deVon Willebrand (EVW)?
Crioprecipitados con ambas partes del factor VIII.
¿Qué medicamento aumenta el factor VIII?
La desmopresina (DDAVP).
¿En qué leucemia se utiliza rutinariamente la heparina?
En la promielocítica.
¿Cómo se monitorean las heparinas de bajo peso molecular?
Con el factor X activado.
¿Cuál es la actividad y vida media del concentrado de factor VIII?
1 U/K aumenta 2% y dura 12 horas.

157
¿Cuál es la actividad y vida media del concentrado de factor IX?
1 U/k aumenta 1% y dura 24 horas.
¿Cuáles son los estados de hipercoagulabilidad?
Se encuentran en este estado los pacientes con medicación anticonceptiva
oral, cáncer, deficiencia de las proteínas C, S y antitrombina III, y en los
que hay presencia del factor V de Leiden.
¿Cómo actúa la heparina?
Potenciando la acción de la antitrombina III.
¿Cómo actúa la warfarina?
Alterando la síntesis de factores K-dependientes.
¿Cómo se evalúa la actividad de la warfarina?
Con el INR y el T.P.
¿Cuál es la dosis de carga de la estreptocinasa?
250,000 U IV durante 30 minutos.
¿Qué es la enfermedad de Rendu-Osler-Weber?
Es la ataxia-telangiectasia hereditaria hemorrágica relacionada con los
genes HHT 1 y 2.
¿Cuál es el mínimo de plaquetas viable en el preoperatorio?
50,000.
¿Cuáles son causas de trombocitopenia?
 La disminución de la producción: vitamina B-12 disminuida, infiltra-
ción medular, congénita.
 El secuestro: hiperesplenismo.
 La destrucción acelerada: uremia, drogas (heparina), púrpura trom-
bótica trombocitopénica y púrpura trombótica idiopática.
¿Por qué la uremia causa disfunción plaquetaria y cuál es su tratamiento?
Porque altera los complejos de factor VIII; se corrige con hemodiálisis y
crioprecipitados.
¿Qué es la púrpura trombótica idiopática (PTI)?
Es una trombocitopenia autoinmune en la que aparece IgG antiplaqueta-
ria que provoca la destrucción de las plaquetas en el bazo, con una forma
aguda (niño) y una crónica (adulto), sin esplenomegalia.
¿Qué es la púrpura trombótica trombocitopénica (PTT)?

158
Es una anemia hemolítica microangiopática de causa autoinmune, rápi-
damente progresiva y fatal; se acompaña de síntomas neurológicos y su
lesión patognomónica son los depósitos hialinos en los vasos.
¿Qué es el síndrome hemolítico-urémico (SHU)?
La PTT y el SHU no son dos entidades distintas; más bien son una pro-
gresión de la misma enfermedad. Mientras en la PTT se destacan los da-
tos neurológicos, en el SHU se agrega insuficiencia renal. Por tanto, en el
SHU se encuentra anemia hemolítica microangiopática, trombocitopenia
e insuficiencia renal y pruebas de coagulación normales.
¿Cuál es el tratamiento de la PTI?
Se usan prednisona o gamma-globulina sólo cuando las plaquetas son
menores de 20,000.
¿Cuáles son los tratamientos para la PTT y la PTI?
 PTT: en la forma complicada se utiliza metilprednisolona o predni-
sona más gamma-globulina. En la forma no complicada, solamente
esteroides.
 PTI: esplenectomía.
¿Cuándo ocurre la PTI aguda?
De los 2 a los 6 años, secundaria a infección viral.
¿Qué es el síndrome de Evans?
Es la anemia hemolítica autoinmune que acompaña a la PTI.
¿Cómo se presenta la púrpura anafilactoide?
La púrpura de Henoch-Schönlein se presenta como un exantema morbili-
forme que afecta la mitad inferior del cuerpo, acompañado de poliartral-
gias y edema que afecta tobillos, rodillas, caderas y codos. Hay afección
renal en 50% de los casos, que causa hematuria y proteinuria.
¿Cuál es el tratamiento de la púrpura anafilactoide?
Esteroides e inmunosupresores (azatioprina y ciclofosfamida).
¿Qué se debe pensar ante la presencia de anemia, trombocitopenia y prue-
bas de coagulación alteradas?
La posibilidad de coagulación intravascular diseminada (CID).
¿Cuándo está indicada la reposición de factores en la CID?
Cuando el fibrinógeno es menor de 75 mg/dl se inicia el uso de criopre-
cipitados.

159
¿Qué se debe pensar ante la presencia de anemia, trombocitopenia y prue-
bas de coagulación normales?
Se debe hacer el diagnóstico diferencial entre PTT y SHU.
¿Cuándo se utiliza la transfusión de plaquetas y cuánto las aumenta?
Cuando la cuenta es de 5,000 a 10,000 y aumentan 10,000 por concen-
trado (en un hombre de 70 kg).
¿Cómo se calcula el aumento plaquetario esperado?
Incremento = (# de concentrados X 5,000)/ (peso/10)
¿A qué se le llama refractariedad a plaquetas?
A un aumento de las plaquetas de sólo 4,000 o 5,000 después de 1 hora
de administrar el concentrado plaquetario.
¿Qué se utiliza en el paciente refractario a plaquetas?
Inmunosupresión, gamma-globulina anti-D e IgG intravenosa.
¿Cuáles son los síndromes mieloproliferativos?
Policitemia vera, eritrocitosis secundaria, mielofibrosis, trombocitemia
primaria.
¿Cuáles son el primero y segundo pasos en el estudio de la eritrocitosis
primaria?
 Buscar signos de hipovolemia para descartar eritrocitosis espúrea.
 Saturación Hb mayor de 92% para descartar que sea secundaria a hi-
poxia.
¿Cuál es la presentación clínica del hiperesplenismo?
Anemia, leucopenia, trombócitopenia, esplenomegalia y reticulocitosis.
¿Cuál es el tratamiento del hiperesplenismo primario?
Quirúrgico; los esteroides no sirven.
¿Qué enfermedad provoca autoesplenectomía?
La anemia de células falciformes.
¿Cuál es el mecanismo principal del hiperesplenismo secundario?
La hipertensión portal; la causa que ocupa el segundo lugar es la trombo-
sis de la vena esplénica.
¿Cómo se manifiesta la incompatibilidad de grupo ABO?
Coagulación intravascular diseminada, necrosis tubular aguda y muerte.

160
¿Cuáles son los tipos de reacciones transfusionales?
 Hemolítica o por incompatibilidad A, B, O, Rh
 No hemolítica: se presenta con una reacción febril causada por anti-
cuerpos del receptor contra leucocitos del donador.
¿Cuál es el tratamiento de la reacción alérgica transfusional?
Difenhidramina (25 mg. I.V.), epinefrina (0.5 ml. I.V)., hidrocortisona
(100 mg. I.V.).

161
162
INFECTOLOGÍA

¿Cuál es el tratamiento del herpes zoster?


Aciclovir: 800 mg 5 veces al día durante cinco días.
¿Cuál es el diagnóstico diferencial de la faringitis por estreptococo beta-
hemolítico del grupo A?
Mononucleosis infecciosa, faringitis viral gorronea, infección por Myco-
plasma pneumoniae o por Clamydia pneumoniae.
¿Hasta que temperatura se considera fiebre de origen viral?
De 40.6º C a 41.1º C.
¿A partir de qué temperatura se considera fiebre de origen bacteriano?
A partir de 41.1º C.
¿Cuál es la conjuntivitis viral más frecuente y cuál su tratamiento?
Es la fiebre faringoconjuntival por adenovirus tipo 3, y se aplican gotas
de sulfas y compresas para prevenir una complicación bacteriana.
¿Qué enfermedades infecciosas provocan síntomas de resfriado y exantema?
El sarampión, el síndrome retroviral y la mononucleosis infecciosa.
¿Qué es la enfermedad de Paget?
Es una enfermedad de los huesos producida por paramixovirus (moquillo
canino), que causa destrucción y reparación excesiva con deformidad re-
sultante.
¿Cuál es el diagnóstico diferencial de la enfermedad de Paget?
El mieloma múltiple, las metástasis óseas y la osteítis fibrosa quística.
¿Qué es la enfermedad de Kawasaki?
Es una vasculitis de etiología incierta, aunque presumiblemente infeccio-
sa, que afecta a pacientes menores de 4 años. Se diagnostica con la pre-
sencia del criterio principal y 4 de 5 adicionales.

163
La presencia de fiebre por 5 días es el criterio principal. Los siguientes 5
criterios son:
 Síndrome extrapiramidal.
 Cambios en las mucosas; incluye xerostomía y lengua en fresa.
 Eritema, edema o descamación de la piel de las extremidades.
 Exantema cutáneo.
 Linfadenopatía cervical.
¿Cuáles son las principales complicaciones de la enfermedad de Kawasaki?
Trombocitosis y aneurismas cardiacos.
¿Cuáles son los ocho tipos principales de herpesvirus?
Virus simple tipo 1 (HSV-1), HSV-2, virus de la varicela-zoster (tipo 3),
virus de Epstein-Barr (VEB), virus de la mononucleosis infecciosa (tipo
4), citomegalovirus (CMV, tipo 5), virus de la roséola (exantema súbito
HHV-6), el virus relacionado a la roséola (HHV-7) y el relacionado al
sarcoma de Kaposi (HHV-8).
¿Cómo se presenta la mononulceosis infecciosa?
Con linfocitosis atípica, retinitis y prueba monospot positiva.
¿Qué signos neurológicos y no neurológicos aparecen en la mononucleosis
infecciosa?
Parálisis de Bell, meningitis aséptica, encefalitis, neumonitis, miocarditis,
faringitis, fiebre, linfadenopatía, esplenomegalia y exantema.
¿Qué alteraciones de laboratorio se presentan en la mononucleosis infec-
ciosa?
En 50% de los casos aparece monocitosis y en 10%, linfocitosis atípica y
transaminasas elevadas.
¿Cuál es el diagnóstico diferencial de la mononucleosis infecciosa?
Faringitis exudativas, infección por citomegalovirus (CMV), rubéola y
toxoplasmosis.
¿Cómo se presenta la infección por CMV?
Con linfocitosis atípica y alteración de las pruebas hepáticas y, clínica-
mente, con linfadenopatía generalizada, fiebre y exantema.
¿Cuál es el estándar de oro para hacer el diagnóstico de infección por
CMV?
El cultivo viral.

164
¿Cuál es el cuidado que deben seguir los contactos de pacientes con varicela?
La varicela es transmisible desde 48 horas antes de que aparezca el exan-
tema y deben vacunarse todos los contactos inmunodeprimidos.
¿Cuál es el antiviral de elección para todas las infecciones comunes por
herpes?
El aciclovir.
¿Cuáles son los antivirales análogos de la guanosina?
El aciclovir, famciclovir y ganciclovir. Actúan compitiendo con la desoxi-
guanosinatrifosfatasa viral (dGTP) como sustrato para la DNA-polimerasa,
lo que causa una terminación prematura de la cadena de DNA viral.
¿Cuándo está indicado utilizar ganciclovir?
Aunque tiene el mismo espectro de acción que el aciclovir, sólo está
aprobado en la terapia de la retinitis citomegálica.
¿Cuál es la principal indicación del famciclovir?
El herpes zoster agudo.
¿Qué es el foscarnet y cuándo está indicado?
Es un fosfonoformato derivado del pirofosfato (no es un análogo de puri-
nas o pirimidinas), que a pesar de su amplia actividad antiviral in vitro,
sólo está aprobado para el tratamiento de las infecciones virales resisten-
tes al aciclovir y al ganciclovir que se presentan en pacientes infectados
con HIV.
¿Cuáles son los principales antivirales utilizados en las infecciones respira-
torias?
 La amantadina y la rimantadina están indicadas como agentes profi-
lácticos en las infecciones por virus de la influenza A y pueden ser
utilizadas junto con la vacuna. Están indicadas en pacientes no vacu-
nados y en contactos o personas con riesgo elevado de infección.
 La rivabirina es un análogo de la guanosina, que tiene un amplio es-
pectro antiviral contra virus RNA y DNA, pero que se utiliza princi-
palmente en infecciones por el virus sincicial respiratorio, influenza A
o B y la fiebre de Lassa.
¿Cuál es la conducta a seguir para contactos de enfermos de hepatitis A?
Todos los contactos domiciliarios deben ser vacunados; el periodo de in-
cubación es de 25 a 30 días.

165
¿Cuáles son las pruebas serológicas para infección por el virus de la hepati-
tis B?
 HBsAg: el antígeno de superficie es el primer marcador serológico
que se hace positivo y también el más confiable; puede aparecer tan
temprano como los primeros 14 días de infección, aunque normal-
mente el promedio es de 27 a 41 días. Se positiviza de 7 a 26 días
después de la aparición de las anormalidades bioquímicas.
 Anti-HBsAg: la presencia del anticuerpo contra el antígeno de superficie
en ausencia de HBsAg indica recuperación, ausencia de infectividad
e inmunidad para futuras infecciones por el virus de la hepatitis B.
 HBeAg: el antígeno e indica un estado de replicación viral en que el
paciente es altamente infeccioso.
 Anti-HBeAg: El anticuerpo contra HBeAg aparece cuando el antí-
geno desaparece y es un marcador de buen pronóstico y resolución de
la infección aguda.
Anti-HBcore: La fracción total de este anticuerpo se positiviza en la in-
fección aguda y persiste durante toda la vida. La fracción IgM es la única
altamente específica; está presente en el periodo en que los antígenos e y
el de superficie han desaparecido, pero todavía no han aparecido los anti-
cuerpos (periodo de ventana del core).
¿Qué es el interferón?
Es una familia de glicoproteínas inducidas naturalmente, que interfiere
con la habilidad de los virus para infectar células. Existen tres tipos sinte-
tizados por tecnología de DNA recombinante: alfa, beta y gamma. Ac-
tualmente se utiliza el interferón alfa-2b para el tratamiento de la
hepatitis B y C, así como para tratar la leucemia de células peludas y el
sarcoma de Kaposi.
¿Qué infecciones causan neutropenia?
Influenza A o B, hepatitis por virus sincicial respirtatorio (VSR), rubéola,
varicela , por virus de Epstein-Barr (VEB), salmonella, brucella, tulare-
mia.
¿Qué microorganismos afectan frecuentemente al paciente neutropénico?
Aspergillus, Cándida, E. coli y Pseudomonas.

166
¿Cuáles son las manifestaciones neuromusculares en la infección por HIV?
 Es más común la meningitis por toxoplasma que por criptococo.
Cuando su aparición es muy temprana, la etiología es el propio HIV.
 Ocurre polirradiculitis tipo Guillain-Barré secundaria a infección por
HIV y citomegalovirus (CMV).
 La miopatía puede ser por HIV (proximal) y por el uso de AZT (so-
bre todo si hay afección renal).
¿Cuál es la manifestación endocrina más frecuente en la infección por
HIV?
La insuficiencia suprarrenal.
¿Cuál es el estándar de oro para hacer el diagnóstico de infección por HIV?
El cultivo viral.
¿Qué tipo de análisis son el ELISA y Western blot?
Son análisis cualitativos para detectar anticuerpos contra el HIV. El ELI-
SA es útil para hacer la detección (debe ser positivo en dos muestras dife-
rentes) y el Western blot se utiliza para hacer la confirmación.
¿Qué prueba se usa para confirmar el Western blot, según el protocolo pa-
ra diagnóstico de infección por HIV?
Un Western blot que informa un resultado dudoso debe confirmarse por
reacción de cadena de la polimerasa (PCR).
¿Qué pruebas se utilizan para hacer diagnóstico en el producto de una ma-
dre infectada por el HIV?
ELISA a las 18 semanas de vida extrauterina, la detección del antígeno
P-24 por PCR y cultivo directo a partir de linfocitos.
¿Cuáles son las cuatro categorías definitorias de inmunodeficiencia (cate-
goría C)?
Infección bacteriana grave recurrente.
Infección por oportunistas.
Síndrome de desgaste
Enfermedad neurológica progresiva.
¿Qué profilaxis con antibiótico se usan en los pacientes con SIDA, según
sus determinaciones de subpoblaciones de linfocitos CD4?
 Menos de 300CD4: profilaxis contra Criptococcus y Cándida con flu-
conazol.

167
 Menos de 200CD4: dar profilaxis contra Pneumocystis carinii con tri-
metoprima-sulfametoxazol o dapsona (100 mg durante tres semanas).
 75-100 CD4: dar profilaxis contra Mycobacterium avium-intracellulare
con claritromicina 500 mg y rifabutina 300 mg (opcional).
 Menos de 50 CD4: dar profilaxis contra Citomegalovirus con ganci-
clovir.
¿Cuál es el agente más común del absceso cerebral en pacientes con SIDA?
Toxoplasma.
¿Cuál es el agente más común de la retinitis en enfermos de SIDA?
Citomegalovirus.
¿Cuál es el agente más común de la leucoplaquia peluda oral en casos de
SIDA?
El virus de Epstein-Barr.
¿Cuál es el agente más común de la meningitis en pacientes con SIDA?
Criptococcus neoforman.
¿Qué se usa para el tratamiento del herpes en pacientes infectados por el VIH?
Aciclovir seguido por foscarnet.
¿A qué se asocia la infección por citomegalovirus?
A la aparición de sarcoma de Kaposi (se observa como nódulos cutáneos
o faríngeos).
¿Cuál es la imagen radiológica de la neumonía por Pneumocystis carinii?
La imagen es la de un infiltrado en “alas de mariposa”.
¿Qué fármacos se usan para tratar la infección por Pneumocystis carinii?
Sulfametoxazol-trimetoprima, dapsona y pentamidina en aerosol.
¿Cuáles son los dos tipos de antivirales utilizados en casos de infección por
VIH?
Los análogos de nucleósidos y los inhibidores de proteasa.
¿Cuáles antivirales son análogos de nucleósidos?
La zidovudina (AZT), la didanosina (ddI), la zalcitabina (ddC), la es-
tavudina (d4T) y la lamivudina (3TC).
¿Cuáles son los antivirales inhibidores de la proteasa?
Los antivirales saquinavir, ritonavir, indinavir, nelfinavir y amprenavir
son inhibidores de la proteasa aprobados, que suelen usarse en conjunto
con la combinación zidovudina-lamivudina.

168
¿Cuándo está indicado iniciar la terapia con AZT?
Cuando las cuentas virales son mayores de 5,000 copias o la cuenta de
CD4 es menor a 500 o existe alguna enfermedad definitoria de SIDA.
¿Cuáles son las dosis de AZT por grupo de edad?
 Niños: 180 mg/m2/6horas.
 Adulto: 1,500 mg/24horas para SIDA y 500 mg/24horas cuando es
portados de VIH con CD4 menores de 500.
Los principales efectos secundarios son la neutropenia en 48% de los ca-
sos y la toxicidad hematológica en 26%.
¿Cuál es el segundo fármaco aprobado para el tratamiento antiviral en la
infección por HIV?
La didanosina (ddI) es el antiviral recomendado en las infecciones resis-
tentes a AZT. Su principal efecto adverso es la pancreatitis.
¿Cuál es la penetración al sistema nerviosos central de los principales anti-
virales?
El AZT tiene una penetración de 70% y la ddI de 25%.
¿Cuáles son los fármacos de primera línea para el tratamiento de la infec-
ción por el VIH?
Los análogos nucleósidos. El primero, el AZT.
¿Cuáles son dos fármacos análogos no nucleósidos?
La nevirapina y el delavirdine.
¿Qué antivirales causan neuropatía?
La vincristina, zalcitabina y didanosina.
¿Cuándo está indicado el uso de la zalcitabina?
En terapia de combinación con AZT o como monoterapia en pacientes
que no toleran el AZT.
¿Cuáles son los principales efectos tóxicos de los análogos nucleósidos?
 AZT: toxicidad medular, leucopenia y anemia.
 Didanosina y zalcitabina: pancreatitis.
 Estavudina: neuropatía periférica.
¿Cuáles son las principales bacterias encapsuladas?
H. influenzae e meningococo, neumococo y Salmonella.
¿A qué grupo de edad afectan más los encapsulados?
A los pacientes comprendidos entre los 3 meses y los 2 años.

169
¿Cuáles son las principales enfermedades causadas por cocos gram-
positivos?
Infecciones estafilocócicas, estreptocócicas y neumocócicas.
¿Qué pacientes son susceptibles de infecciones estafilocócicas?
Los usuarios de drogas IV, los diabéticos y los que presentan infecciones
de tejidos superficiales.
¿Qué enfermedades son causadas por estafilococos?
La foliculitis, los abscesos, los furúnculos y las infecciones de tejidos
blandos (ántrax estafilocócico).
¿Cuál es la principal etiología de la osteomielitis?
S. aureus es el causante de 60% de las infecciones.
¿Cuáles son los síndromes provocados por toxinas bacterianas?
El síndrome de choque tóxico, el síndrome de piel escaldada y el síndro-
me de intoxicación alimentaria.
¿Qué es el síndrome de choque tóxico?
Es un síndrome provocado por la toxina TSST-1 producida por estafilo-
cocos; es más frecuente en adultos, de aparición súbita, con fiebre alta,
vómito, diarrea, exantema, conjuntivitis y descamación de las palmas y
las plantas. Pueden ocurrir hipotensión e insuficiencia renal aguda.
¿Cuál es el diagnóstico diferencial de la intoxicación alimentaria?
La intoxicación alimentaria causada por estafilococos inicia en el transcur-
so de las primeras 6 horas (siempre en menos de 12 horas), y típicamente
se observa cuando los alimentos se han dejado enfriar al aire libre o con-
tienen productos como crema o mayonesa. La intoxicación por Bacillus
céreus tiene un inicio mas lento y generalmente se debe a la ingesta de
arroz frito contaminado. Los síntomas de intoxicación por Clostridium
aparecen entre 8 y 16 horas después de la ingesta y el dato principal es la
diarrea acuosa.
¿Qué es el gonococo?
Es un diplococo gram-negativo, reniforme, intracelular, oxidasa-positivo
como todas las Neisserias.
¿Cuáles son las principales infecciones causadas por bacilos gram-positivos?
El erisipeloide, la listeriosis la nocardiosis y el carbunco.

170
¿Qué es la nocardiosis?
La nocardiosis o micetoma es una enfermedad provocada por actinomi-
cetos aerobios, que se puede presentar con 4 variedades clínicas: celulitis,
síndrome linfocutáneo, actinomicetoma y neumonía.
¿Cuál es el tratamiento del micetoma?
Diaminodifenilsulfona y sulfametoxazol-trimetoprima (SMZ- TMP) du-
rante dos años.
¿Cuáles son las principales enfermedades causadas por bacilos gram-nega-
tivos?
Las infecciones causadas por enterobacterias, por Haemophilus, por
Pseudomonas, la salmonelosis, la shigelosis, la brucelosis, el cólera, la tu-
laremia y la peste.
¿Cuáles son los vectores de la tularemia y la peste?
La garrapata y la pulga, respectivamente.
¿Cuál es el tratamiento empírico para cualquier tipo de diarrea?
Los fluoroquinolonas.
¿Cuál es la fisiopatología de la diarrea en la infección por vibrio cholerae y
ETEC?
Ambos producen toxinas que inactivan el GMPc en la pared intestinal.
¿Cuáles son los agentes causales de gastroenteritis infecciosa con diarrea
no inflamatoria?
Infección por Clostridium, Escherichia coli enterotoxigénica, estafiloco-
cos e infestación por Giardia lamblia.
¿Cuáles son los agentes causales de gastroenteritis con diarrea de tipo in-
flamatorio?
Shigella, Salmonella, Campylobacter y E. coli enteroinvasiva.
¿Cuándo se debe sospechar que el paciente padece brucelosis?
Cuando hay fiebre de bajo grado, diaforesis, agotamiento, cefalea, estre-
ñimiento, artralgias y, en algunos casos, adenopatías y esplenomegalia. Al
interrogatorio se encuentra inicio insidioso y antecedente de haber consu-
mido carne posiblemente infectada o leche y queso no pasteurizados.
¿Cuál es el tratamiento de la brucelosis?
Se trata con un doble esquema que puede ser doxiciclina y rifampicina o
sulfametoxazol-trimetoprima (TMP-SMZ) y rifampicina.

171
¿Cuáles son los hallazgos de laboratorio en pacientes que padecen brucelosis?
Las reacciones de aglutinación se vuelven positivas después de dos a tres
semanas de ocurrida la infección. Un título mayor de 1:160 es positivo.
El análisis inmunológico de IgG o IgM específicas es ahora el método de
elección.
¿Cuándo se debe sospechar de fiebre tifoidea?
Cuando se encuentran síntomas correspondientes a una gastroenteritis
que inicialmente presenta diarrea (en sopa de chícharos), que posterior-
mente desarrolla estreñimiento. Al mismo tiempo hay ataque al estado
general y síntomas de faringitis. Puede aparecer el exantema clásico en el
tórax y abdomen. La meseta de todos estos síntomas coincide con la me-
seta de la fiebre. El hallazgo patognomónico de la enfermedad es la bra-
dicardia relativa.
¿Qué es la bradicardia relativa?
Normalmente, por cada grado que aumenta la temperatura por arriba de
38.3, el pulso se eleva 10 latidos por minuto. Cuando no ocurre una ta-
quicardia proporcional a la fiebre, se habla de bradicardia relativa.
¿Cuál es el tratamiento de la fiebre tifoidea?
El de primera elección es el (TMP-SMZ) seguido por ceftriaxona y cipro-
floxacino.
¿Cuáles son las alteraciones de las pruebas de laboratorio en la salmonelosis?
El diagnóstico se basa en un hemocultivo positivo en los primeros 10 días
de inicio de la fiebre. Existen leucopenia y anemia normocítica. La reac-
ción de Widal no es confiable porque se positiviza con la vacunación o con
la infección por cualquier especie de Salmonella y se negativiza si se usa
previamente cloramfenicol o ampicilina.
¿Cuándo se debe sospechar shigelosis?
Cuando el cuadro clínico es de fiebre, tenesmo y dolor abdominal; ade-
más se encuentra evidencia de síndrome disentérico (leucocitos y eritroci-
tos en las heces). Frecuentemente se complica con un cuadro de
deficiencia de disacaridasas transitoria y síndrome de Reiter.
¿En qué medio se cultivan la Salmonella y la Shigella?
En el medio SS.

172
¿Cuáles son las principales enfermedades causadas por bacilos anaerobios?
Las infecciones por clostridios, por bacteroides y la actinomicosis.
¿Qué es la actinomicosis?
Es una enfermedad indolente, lentamente progresiva, causada por gér-
menes gram-positivos anaerobios del género Actinomices. En la lesión
producida por la actinomicosis se observan gránulos sulfurosos que tras-
pasan todos los planos tisulares hasta drenar al exterior.
¿Cómo es la lesión del actinomicetoma?
Es una lesión supurativa, granulomatosa, filante, que forma tractos fistu-
losos, y al microscopio se observa infiltración extensa de neutrófilos.
¿Cuáles son las enfermedades provocadas por clostridios?
Los clostridios son bacilos anaerobios gram-positivos. Las principales in-
fecciones causadas por clostridios son la mionecrosis por clostridios
(gangrena gaseosa), el tétanos y el botulismo.
¿Qué es el tétanos?
Es la enfermedad causada por la toxina que produce el Clostridium Tetani
(tetanospasmina). Es secundaria a una herida profunda contaminada. Se
presenta con rigidez del cuello, hormigueo en el sitio de la inoculación y
disfagia. Posteriormente existe trismus, irritabilidad, hiperreflexia, opistó-
tono y convulsiones.
¿Cuál es el manejo de las heridas para hacer la prevención del tétanos?
En caso de una herida menor limpia debe aplicarse la primera dosis de
toxoide tetánico (esquema de tres dosis). En caso de una herida profunda
y contaminada con material que probablemente contenga anaerobios,
deben aplicarse toxoide e inmunoglobulina antitetánica.
¿Cómo actúan los antibióticos bacteriostáticos?
Inhiben la replicación y el crecimiento bacterianos y favorecen la elimi-
nación de los organismos remanentes por el sistema inmune; por tanto, si
la administración del fármaco se hace antes de que el organismo elimine
todos los gérmenes, puede ocasionarse un segundo ciclo de infección.
¿Cuáles son los mecanismos de resistencia microbiana a los antibióticos?
 Mutación del DNA.
 Transferencia de DNA por plásmidos (factor R extracromosómico).

173
 Modificación de receptores específicos del fármaco en el microorga-
nismo.
 Inactivación enzimática del fármaco.
¿Qué antibióticos inhiben la síntesis de folato?
Las sulfas: sulfadiazina de plata, sulfisoxazol, sulfametoxazol, sulfasala-
zina, etcétera.
¿Qué medicamentos inhiben la reducción de folato?
La pirimetamina y el trimetoprim.
¿Qué medicamento inhibe la síntesis y la reducción de folato?
El cotrimoxazol.
¿Por qué son bacteriostáticas las sulfas?
Las coenzimas derivadas del ácido fólico se requieren para la síntesis de
ácidos nucleicos y, por tanto, para la replicación celular. Las bacterias
son impermeables al folato y deben sintetizarlo a partir de ácido para-
aminobenzaico (PABA), pteridina y glutamato.
¿Cuáles son los efectos adversos de las sulfas?
Cristaluria, kernícterus, Stevens-Johnson, reacción leucemoide, granuloci-
topenia, poliarterisis nodosa, vasculitis y otras.
¿Qué antibióticos actúan como inhibidores de la síntesis de la pared celular
bacteriana?
Los betalactámicos, la vancomicina y la bacitracina.
¿Cómo se clasifican los betalactámicos?
En penicilinas, cefalosporinas, carbapenems y monobactams.
¿Cuál es el sitio de acción de los betalactámicos?
Actúan inhibiendo el último paso de la síntesis de la pared celular; es de-
cir, en la transpeptidación del peptidoglicano.
¿Cuáles son las penicilinas antipseudomonas?
La ticarcilina, carbenicilina, azlociclina, mezlociclina y piperacilina.
¿Cuál es el espectro antibacteriano de las cefalosporinas de primera gene-
ración?
Actúan sobre todos los gram-positivos meticilinorresistentes, Proteus, E.
Coli y Klebsiella.
¿Cuál es el espectro de las cefalosporinas de segunda generación?
Tienen la misma cobertura que las de primera generación y un espectro
extendido a gram-negativos, Haemophilus infuenzae y Enterobacter.

174
¿Cuál es el espectro de las cefalosporinas de tercera generación?
Su mayor efecto es sobre bacilos gram-negativos, H. infuenzae y entero-
bactar así como Serratia marcesens. La cefotaxima y la ceftriaxona son de
elección en casos de meningitis y la ceftazidima, en infección por Pseu-
domonas.
¿Cuál es el espectro de las cefalosporinas de cuarta generación?
Actúan sobre estafilococos y estreptococos meticilinosensibles y gram-
negativos como Enterobacter, E. Coli, Klebsiella, Proteus y Pseudomonas.
¿Cuáles son las cefalosporinas que pueden utilizarse por vía oral?
 Primera generación: cefalexina, cefadroxil y cefradina.
 Segunda generación: cefuroxima acetil y cefaclor.
 Tercera generación: ceftibuten y cefixima.
¿Cuál es el betalactámico con mayor espectro antibacteriano?
La imipenem-cilastatina. Puede ser de elección como terapia empírica en
infecciones que amenazan la vida y cuya etiología no está bien definida.
¿Cuáles son las indicaciones para el uso de vancomicina?
 Infección grave por gram-positivos betalactamasa positivos.
 Infección grave por gram-positivos en los pacientes alérgicos a los be-
talactámicos.
 Infección por Clostridium o estafilococo, que amenaza la vida.
 Profilaxis en procedimientos dentales.
 Como antibiótico en pacientes con implante de válvulas cardiacas.
¿Cuáles son los efectos adversos de los betalactámicos?
Reacciones de hipersensiblidad y disfunción plaquetaria.
¿Qué antibióticos actúan inhibiendo la síntesis de proteínas?
Las tetraciclinas, los aminoglucósidos, los macrólidos, el cloramfenicol y
la clindamicina.
¿Qué antibióticos actúan sobre la subunidad 30s?
Las tetraciclinas y los aminoglucósidos.
¿Qué antibióticos actúan sobre la subunidad 50s?
El cloramfenicol y los macrólidos.
¿Cómo actúan las fluoroquinolonas?
Actúan inhibiendo la replicación del DNA bacteriano e interfiriendo en
la acción de la DNA-girasa (topoisomerasa II).

175
¿Qué antibióticos se consideran antisépticos de las vías urinarias?
La metenamina y la nitrofurantoína.
¿Cuál es el efecto adverso más importante del cloramfenicol?
La supresión de la médula ósea.
¿Qué medicamentos causan disfunción renal?
Los aminoglucósidos y la anfotericina B.
¿Cuáles son las principales enfermedades infecciosas transmitidas por vec-
tores?
 Piojo: tifo epidémico (Rickettsia prowazecki).
 Pulga: tifo epidémico y peste.
 Ácaro: Rickettsia pustulosa y tifo de los matorrales.
 Mosquito: paludismo, fiebre amarilla, encefalitis de San Luis, encefa-
litis equina del oeste y encefalitis de California.
 Garrapata: enfermedad de Lyme, fiebre manchada de las Montañas
Rocallosas, tularemia.
 Triatoma: enfermedad de Chagas.
 Mosca: tripanosomiasis africana (enfermedad del sueño)
¿Cuál es el método de detección de tuberculosis por tinción?
Auramina-rodamina.
¿Cuál es la tinción confirmatoria para tuberculosis?
Kynyoun o Ziehl-Nielsen.
¿Qué es un complejo de Gohn?
Es un foco primario de infección tuberculosa calcificado.
¿Qué es un complejo de Ranke?
Es un foco tuberculoso primario calcificado unido a un ganglio parahiliar
calcificado.
¿Cuánto se inyecta de PPD y cuánto tiene de tuberculina para efectuar la
prueba de Mantoux?
Se inyectan 0.1 ml, que equivalen a U de tuberculina.
¿Qué es un convertidor reciente?
 Una persona menor de 35 años, con una ganancia de 10 mm en la
prueba de Mantoux en los últimos dos años.
 Una persona mayor de 35 años, con una ganancia de 15 mm en la
prueba Mantoux en los últimos dos años.

176
¿Cuáles son los cuatro grupos que deben recibir profilaxis contra la tu-
berculosis?
 Cuando se sospecha o se conoce infección por HIV.
 Todos los contactos de personas con TB.
 Todos los convertidores recientes.
 Cualquiera que padezca una enfermedad inmunosupresora como la
diabetes mellitus o tumores bajo terapia con fármacos inmunosupre-
sores.
¿Cómo se lleva a cabo la profilaxis de la tuberculosis?
Con Isoniacida durante 9 meses.
¿Cómo actúa la isoniacida?
Inactiva la enzima responsable de ensamblar los ácidos micólicos de la pa-
red externa del microorganismo. Es activa contra los microorganismos
intracelulares y contra los que se encuentran en el cáseum. Su concentra-
ción en el líquido cefalorraquídeo es prácticamente igual a la plasmática.
¿Cómo actúa la rifampicina?
Bloquea la trascripción interfiriendo con la subunidad-beta de la RNA-
polimerasa dependiente del DNA; por tanto, inhibe la síntesis de RNA y
suprime el paso inicial. Es bactericida de microorganismos intra y extra-
celulares. Frecuentemente se usa en la profilaxis de contactos intradomi-
ciliarios de pacientes con meningitis meningocócica o por H. influenzae.
Se distribuye uniformemente en todos los órganos y fluidos, incluyendo
el líquido cefalorraquideo.
¿Cuáles son los efectos adversos de la rifampicina?
Tiñe de color anaranjado la orina, las heces y otras secreciones. Puede
causar ictericia en pacientes con enfermedad hepática, alcohólicos y con
edad avanzada.
¿Cuál es el efecto adverso más importante del etambutol?
Causa neuritis óptica que produce disminución de la agudeza visual y
pérdida de la habilidad para discriminar entre los colores verde y rojo.
¿Cuáles son las principales enfermedades causadas por espiroquetas?
Las treponematosis, la fiebre recurrente, la enfermedad de Lyme y la en-
fermedad por arañazo de gato.

177
¿Cuáles son los falsos positivos para VDRL?
Paludismo, lepra, embarazo, edad avanzada, mononucleosis infecciosa,
hepatitis C, enfermedades del tejido conectivo y la fiebre.
¿Cuándo se encuentran títulos altos de VDRL?
En la sífilis secundaria; pueden ser mayores de 1:32.
¿Cuándo se hace positivo el VDRL en caso de sífilis?
A las seis semanas.
¿Cómo son los títulos de VDRL en la sífilis tardía?
Muy bajos o incluso negativos.
¿Cuál es el tratamiento antibiótico para la sífilis?
 Penicilina sódica cristalina o benzatínica.
 Doxiciclina 100 mg cada 12 horas o tetraciclina (500 mg c/6hr VO
durante 14 días).
¿Qué enfermedades infecciosas se tratan con doxiciclina?
Las provocadas por Chlamydia, Brucella y la sífilis.
¿Cuáles son las características de las diferentes etapas de la sífilis?
 Sífilis primaria: la lesión básica es el chancro; hay linfadenopatía re-
gional.
 Sífilis secundaria: hay lesiones en la piel, mucosas, huesos, hígado,
SNC y linfadenopatía generalizada.
 Sífilis terciaria: la lesión básica son los gomas en los huesos, hígado y
en las vísceras. También existe enfermedad cardiovascular caracteri-
zada por aortitis y neurosífilis.
¿Cuáles son los tipos clínicos de la neurosífilis?
 Neurosífilis sintomática (5%) y neurosífilis asintomática (15%).
 Meningovascular: puede ser de la corteza (signos meníngeos, convul-
siones) o basal (parálisis de nervios craneales).
 Parenquimatosa: causa demencia paralítica secundaria a la sífilis.
 Tabes dorsal.
¿Qué es la tabes dorsal?
Es una forma de degeneración progresiva del parénquima de las colum-
nas posteriores de los ganglios sensitivos y de las raíces nerviosas. El pri-
mer signo y más común es un dolor fulgurante en las piernas seguido por

178
inestabilidad en la marcha, ataxia, alteraciones sensoriales y pérdida de
los reflejos tendinosos.
¿Qué es la pupila de Argyl-Robertson?
Es una pupila irregular y pequeña, que reacciona de modo normal a la
acomodación pero no a la luz.
¿Qué agente produce el mal del pinto?
El Treponema carateum.
¿Qué agente causa la fiebre recurrente?
La Borrelia recurrentis.
¿Qué pruebas de laboratorio se positivizan en la fiebre recurrente?
La reacción de Weil-Felix y el VDRL.
¿Qué etapas clínicas tiene la leptospirosis?
Septicémica, inmunitaria e ictérica.
¿Cuáles son las etapas de la enfermedad de Lyme?
 I: resfriado y eritema migratorio.
 II: enfermedad generalizada y enfermedad neurológica en la que ocu-
rre parálisis de Bell (meningitis aséptica).
 III: artritis y molestias musculoesqueléticas.
¿Cuáles son las principales enfermedades parasitarias?
Las protozoosis y las helmintiasis.
¿Cuál es la lesión característica de la amibiasis intestinal?
La úlcera amibiana.
¿Cuáles son las formas clínicas de la amibiasis intestinal?
Asintomática, colitis disentérica y no disentérica.
¿Cómo se toman las muestras de heces para hacer el diagnóstico de ami-
biasis?
Se toman tres muestras, que deben examinarse dentro de los primeros
treinta minutos después de la toma.
¿Cuál es el diagnóstico diferencial del absceso hepático amibiano?
Lesiones por equinococo, cáncer y absceso piógeno.
¿Cómo se observa el ultrasonido del absceso hepático?
Como lesiones redondas no homogéneas, de transición abrupta y centro
hipoecóico.

179
¿Cómo se ve la tomografía axial computarizada del absceso hepático?
Como lesiones de baja densidad, redondas, bien definidas y de estructura
no homogénea.
¿Cuáles son los amebicidas tisulares?
La emetina y la dehidroemetina
¿Cómo inicia el ciclo de la leishmaniasis?
Con la reproducción de amastigotes en los macrófagos de algún mamífe-
ro, llamados cuerpos de Donovan.
¿Qué se observa en el frotis de un enfermo de leishmaniasis?
Amastigotes flagelados intracelulares.
¿Qué se observa en el cultivo del tejido afectado?
El parásito en fase de promastigote flagelado.
¿Cuál es el tratamiento de la leishmaniasis?
Estibogluconato de sodio o pentamidina más anfotericina B.
¿Para qué sirve la pirimetamina?
Para el tratamiento del paludismo y la toxoplasmosis.
¿Para qué sirve la pentamidina?
Para tratar la enfermedad del sueño, y para la profilaxis y el tratamiento
de la neumonía por Pneumocystis carinii.
¿Cómo se presenta la toxoplasmosis?
Clínicamente, con fiebre, linfadenopatía cervical, esplenomegalia, exan-
tema y retinocoroiditis.
¿Cómo se hace el diagnóstico de toxoplasmosis?
Obteniendo títulos cuatro veces mayores de los normales de IgM, o con
un título simple alto de 1:60.
¿Cuál es el tratamiento antimicrobiano de la toxoplasmosis?
Pirimetamina (acompañada de ácido folínico) más sulfadiazina.
¿Qué es la enfermedad de Chagas?
Es la tripanosomiasis americana, causada por el Trypanosoma cruzi. Suele
observarse más en niños pequeños y se presenta con fiebre, linfadenopa-
tías, hepatoesplenomegalia y edema facial, y puede haber meningoence-
falitis y convulsiones. La forma crónica tiene pocos síntomas y causa
cardiomiopatía, megacolon y megaesófago.

180
¿Cuál es el vector de la enfermedad de Chagas?
La chinche picuda o Triatoma reduvidae.
¿Cómo se hace el diagnóstico de enfermedad de Chagas?
 Se puede lograr la observación del parásito dentro de la capa leucoci-
taria en un centrifugado de sangre.
 Se puede observar el parásito en una tinción de Giemsa de capas
gruesas de sangre.
 Para la etapa crónica sólo sirven el cultivo (Nicolle-Novi-McNeal) y
el xenodiagnóstico.
¿Cuál es el tratamiento de la etapa crónica de la enfermedad de Chagas?
Se utiliza nifurtimox como antiparasitario y amiodarona para la insufi-
ciencia cardiaca congestivo-venosa, ya que los pacientes no toleran bien
la digoxina.
¿Cuál es la enfermedad del sueño y quién la provoca?
Es la tripanosomiasis africana, provocada por Trypanosoma brucei gam-
biense y rodeniense.
¿Cuáles son las etapas de la enfermedad del sueño?
 Etapa hemolinfática: fiebre, linfadenopatía y exantema papuloso.
 Etapa meningoencefálica: insomnio, trastornos motores y sensitivos.
¿Cuál es el tratamiento de la enfermedad del sueño?
La suramina y la eflornitina. La profilaxis se lleva a cabo con pentamidina.
¿Cuáles son las variedades clínicas de la cisticercosis?
La neurocisticercosis, la subcutánea, la muscular y la oftalmocisticerco-
sis.
¿Cuáles son las variedades de la neurocisticercosis?
Ventricular, parenquimatosa, subaracnoidea, racimososa y de la médula
espinal.
¿Cuáles son los métodos de elección para hacer el diagnóstico de laborato-
rio de esquistosomiasis?
Kato Katz y la búsqueda del parásito en orina de 24 horas.
¿Qué es la larva migrans cutánea?
Es la uncinaria del perro y del gato, Ancylostoma brasiliense y Ancylos-
toma caninum.

181
¿Cuál es el tratamiento de la esquistosomiasis y de la paragonimiasis?
El prazicuantel.
¿Cuál es el tratamiento de la fasciolosis?
El triclabendazol.
¿Cuál es el tratamiento de la uncinariasis, la ascariasis y la enterobiosis?
El albendazol.
¿Cómo se hace el diagnóstico de enterobiosis?
Con el método de Graham.
¿Cómo se hace el diagnóstico de filariasis en el laboratorio?
Observando microfilarias en sangre periférica tomada con periodicidad
nocturna. El frote de gota gruesa se utiliza para la detección y los frotes
delgados se utilizan para evaluar la morfología; se tiñen con Giemsa.
También se utiliza el método de concentración de sangre de Knott o la
filtración de membrana.
¿Cuál es el tratamiento de la filariasis?
La dietilcarbamacina.
¿Cuál puede ser la pérdida diaria de hemoglobina en la infestación por un-
cinarias?
 1 mg al día en la infestación por Necator americanus
 3 mg al día en la infestación por Ancylostoma duodenale
¿Cuál es el cuadro clínico de la oncocercosis?
Nódulos subcutáneos, deterioro visual y ceguera.
¿Cómo se hace el diagnóstico de estrongiloidosis?
Observando larvas filariformes en las heces o en el líquido duodenal (por
aspirado o cápsula de Beal).
¿Cuál es el tratamiento de la estrongiloidosis y la oncocercosis?
La ivermectina.
¿En qué se debe pensar si se observa un granuloma eosinofílico en el ojo?
En la toxocariosis.
¿Cuál es el mejor medio de cultivo para los hongos?
El de Sabouraud.
¿Cuál es el mecanismo de acción de los siguientes antimicóticos?
 Ketoconazol: inhibe la biosíntesis de ergosterol y bloquea la desmeti-
lación del sitio C14.

182
 Anfotericina B y nistatina: alteran la permeabilidad de la membrana
al formar complejos con ergosterol.
 Griseofulvina: actúa contra la formación de microtúbulos.
 Flucitocina: actúa sobre el DNA y el RNA.
¿Cuándo está indicado el uso de los siguientes antimicóticos?
 Griseofulvina: en todas las micosis, excepto la tinea capitis y la oni-
comicosis. El itraconazol y el fluconazol han reemplazado amplia-
mente a la griseofulvina para el tratamiento de las dermatofitosis
sistémicas.
 Ketoconazol: en la infección por Malassezia furfur, que causa la tiña
versicolor.
 Fluconazol: en todas las micosis mucocutáneas como la candidiosis.

183
184
INMUNOLOGÍA Y REUMATOLOGÍA

¿Qué elementos conforman la inmunidad innata-inespecífica?


 Componente celular: neutrófilos, monocitos (en la sangre) y macró-
fagos (en los tejidos).
 Componente humoral: proteínas del complemento, reactantes de fase
aguda y las diferentes citocinas.
¿Qué elementos conforman la inmunidad adquirida-específica?
 Componente celular: linfocitos.
 Componente humoral: inmunoglobulinas.
¿Cuáles son los reactantes de fase aguda y cómo se interpretan?
A excepción de la proteína Creactiva (CRP), los reactantes de fase aguda
no se usan para detectar inflamación. Sólo hay que tener en cuenta que
ésta puede ser la causa de que reactantes se eleven cuando se están estu-
diando por otra razón; por ejemplo, si se desea demostrar la presencia de
la enfermedad de Wilson con niveles de ceruloplasmina, habrá que des-
cartar que no exista un estado inflamatorio previo que provoque un falso
positivo.
 CRP: se eleva hasta 100% cuando existe daño tisular severo.
 Fibrinógeno. Se eleva 200-400%.
 Alfa-1-antitrispsina: se eleva 200-400%.
 Haptoglobina: se eleva 200-400%.
 Ferritina: se eleva 50%.
 Ceruloplasmina.
 Glicoproteína alfa-1-ácida.

185
¿Qué causan las deficiencias de complemento C5, C6, C6, C7 y C8?
Propician infecciones rápidas producidas por Neisserias.
¿Qué causa el déficit de C3?
Infecciones por gram-negativos entéricos.
¿Cómo se reconoce lo extraño de lo propio?
Gracias a los productos del complejo principal de histocompatibilidad
(MHC), que son un conjunto de genes que codifican los antígenos leuco-
citarios humanos y que se encuentran codificados en el cromosoma 6.
¿Qué son las citocinas?
Son proteínas solubles diferentes de las inmunoglobulinas, secretadas por
monocitos y linfocitos y que regulan la magnitud de la respuesta inflama-
toria e inmunitaria.
¿Qué son los eicosanoides?
Son ácidos insaturados, de 20 carbonos, precursores de la síntesis de las
prostaglandinas.
¿Qué son y como actúan las prostaglandinas?
Son mediadores locales de la inflamación. Son producidas y eliminadas
en el mismo sitio de acción, por lo que no hay concentraciones significa-
tivas en la sangre.
¿Qué citocinas median la inmunidad natural?
Interleucina (IL)-1, IL-6, IL-8 y el interferon-gamma (IFN-gamma).
¿Qué citocinas median la inflamación alérgica?
 IL4: estimula directamente a la IgE.
 IL3, 4 y 9: se encuentran en las células cebadas.
 IL3 y 5: actúan sobre el factor estimulante de colonias granulocito-
monocito (GM-CSF), que estimula la producción de eosinófilos.
¿Cómo se conforman las inmunoglobulinas o anticuerpos?
Tienen dos cadenas pesadas y dos ligeras, cada una de las cuales tiene re-
giones constantes y regiones variables.
¿Cómo reconocen los antígenos a los linfocitos B y a los linfocitos T?
Los linfocitos B tienen inmunoglobulinas de superficie y el linfocitos T
(RCT) tiene receptor de célula T.
¿Cómo se conforman los receptores de células T?
Por dos cadenas (alfa-beta o gamma-beta) y la molécula CD-3. Se conoce
entonces también como complejo RCT-CD3.

186
¿Qué respuestas median las células TH1 y TH2?
La respuesta celular patógeno-específica y la hipersensibilidad por inter-
leucina 4, respectivamente
¿Qué significa CD?
Grupo de diferenciación (Cluster of differentiation).
¿Qué es un anticuerpo completo?
Es el que puede aglutinar eritrocitos y desencadenar el complemento a
partir de C9.
¿Cuáles son anticuerpos completos e incompletos.
GI es un anticuerpo completo; la IgM es un anticuerpo incompleto.
¿Qué anticuerpo se detecta en la prueba de Coombs?
La inmunoglobulina G (IgG).
¿Qué son los antígenos leucocitarios humanos A, B y C (HLA-A,B y C)?
Los genes del MHC-I forman 3 loci que son el HLA A, B y C. Se encuen-
tran en la superficie de todas las células nucleadas y en las plaquetas.
¿Qué son las HLA D, DR y DQ?
Los genes del MHC-II se distribuyen en varias subregiones: HLA D, DR
y DQ. Sólo se encuentran en la superficie de las células B, los macrófa-
gos, las células dendríticas, las células de Langerhans y las células T acti-
vadas.
¿Cómo elimina un macrófago a un micoorganismo extra e intracelular?
Al extracelular, lo fagocita. Para eliminar al intracelular necesita ingerirlo
y después ser activado para destruirlo. (Esto explica la hipersensibilidad
retardada).
¿Cuáles son las características de las inmunoglobulinas?
 IgM: es la primera que se produce tras la inmunización primaria; se
encuentra en forma monomérica o pentamérica y sólo en la sangre.
Puede tener cadena J. Su peso molecular: 950-1150 KDa. Son anti-
cuerpos contra muchos gérmenes gram-negativos. Las grandes molé-
culas pueden activar el complemento.
 IgG: es la que aparece principalmente tras la reactivación inmunitaria
(memoria); generalmente es monomérica y se encuentra en el plasma
y extravascular. Protege principalmente frente a bacterias, virus y to-

187
xinas. Su peso molecular: 150 KDa. Es la única que atraviesa la ba-
rrera placentaria.
 IgA: la clase “secretora” se encuentra en las mucosas, el tracto gastro-
intestinal y respiratorio y en sus productos (saliva, lágrimas, calostro,
etc.). Se le identifica en forma de monómero o dímero. Puede tener
cadena J y Sc. Peso molucular: 160-400 KDa. Protege contra Bruce-
lla, Corynebacterium dipheriae y poliovirus.
 IgD: se utiliza como marcador para células B maduras. Se encuentra
en forma monomérica. Pero molucular: 175 KDa.
 IgE: es un anticuerpo reagínico, estabilizador y anafiláctico. Se en-
cuentra en los tractos respiratorio y gastrointestinal. Se encuentra
aumentada en suero en las enfermedades atópicas y parasitarias y en
el mieloma monoclonal de IgE. Pero molecular: 190 KDa.
¿Cómo se evalúan las inmunoglobulinas?
 La IgG, IgA e IgM se encuentran en grandes cantidades en el suero.
Se pueden medir por inmunoelectroforesis.
 La IgE se encuentra en cantidades mínimas y debe usarse un estudio
sensible como el RAST.
¿Cuáles son las reacciones de hipersensibilidad según la clasificación de
Gel y Coombs?
 Tipo I: un antígeno (Ag) se fija a un anticuerpo (Ac) de tipo IgE y
mediante su fracción FC éste se fija a los mastocitos y basófilos, libe-
rando factores vasoactivos e inflamatorios. Incluye las reacciones
atópicas y la anafilaxia sistémica.
 Tipo II: es la reacción citotóxica cuando el anticuerpo se fija a una
célula o a haptenos que se encuentran en su superficie. Ocurren en las
anemias hemolíticas Coombs-positivas, en la púrpura trombocitopé-
nica idiopática, el pénfigo, síndrome de Goodpasture.
 Tipo III: es la reacción que involucra complejos inmunes Ac-Ag que se
depositan en vasos y tejidos y activan así el complemento y la liberación
de enzimas proteolíticas lisosomales y factores de permeabilidad. Ocurre
en la enfermedad del suero, poliarteritis, glomerulonefirtis aguda y en
la membranoproliferativa crónica, etcétera.

188
 Tipo IV: involucra la presencia de linfocitos T sensibilizados que se
activan al contacto con el Ag específico (hipersensibilidad retardada).
Se presentan en los granulomas, la dermatitis por contacto, la neu-
momitis por hipersensibilidad, etc.
¿Cómo se clasifican las reacciones a drogas?
En esperadas e inesperadas.
¿Cómo se clasifican las reacciones inesperadas?
En reacciones de intolerancia, idiosincracia e hipersensibilidad.
¿Qué es una reacción farmacogenética?
Es una variación de la respuesta farmacológica producida por factores
hereditarios. Por tanto, se puede considerar como causa de reacciones de
idiosincracia.
¿Cuáles son las reacciones farmacogenéticas más comunes?
La reducción de la actividad de la warfarina causada por la disminución de
la afinidad de fijación de la warfarina a su receptor y la hipertermia maligna
que se produce al administrar combinadamente un relajante muscular y un
anestésico inhalado que actúan sobre los canales de rianodina y provocan
una liberación exagerada de calcio que causa contracciones extremas, una
tasa metabólica elevada y finalmente, hipertermia.
¿Cuál es la diferencia entre una reacción de Arthus y una prueba de hiper-
sensibilidad retardada?
Ambas son secundarias a la administración intradérmica de un Ag. La reac-
ción de Arthus es el resultado de la unión del Ag con un Ac que ya estaba en
el sitio, lo que provoca una respuesta a las 4-6 horas con infiltrado inflamato-
rio consistente en neutrófilos. La prueba de hipersensiblidad retardada debe
interpretarse hasta las 48 horas y es positiva cuando es menor de 5mm. El in-
filtrado que se genera consiste en células mononucleares.
¿Cuáles son las manifestaciones de la reacción de hipersensibilidad tipo III
por complejos inmunes?
Fiebre, examina, linfadenopatía y artralgias.
¿Cuáles son las manifestaciones del síndrome de vasculitis por hipersensi-
bilidad?
Artritis, urticaria, angioedema y fiebre.

189
¿Cuál es la diferencia entre la anafilaxia y la urticaria-angioedema?
Ambas se asocian a la deficiencia de inhibidor del complemento C1. La
anafilaxia es una variedad mas grave de la reacción tipo I, en la que
además de haber angioedema existe colapso circulatorio.
¿Cuáles son los tratamientos de primera y segunda línea en los casos de
anafilaxia?
La epinefrina y la reanimación con líquidos.
¿Cuál es el tratamiento en los casos de urticaria-angioedema?
Los antihistamínicos.
¿Cuáles son los tres usos principales de los antihistamínicos?
El tratamiento de reacciones alérgicas, de las náuseas y el mareo y como
somníferos.
¿Cuáles son los signos clínicos de la rinitis alérgica?
La presencia de pólipos, epistaxis, mucosa pálida y ojeras.
¿Cuáles son los síntomas de la enfermedad del suero?
Urticaria, angioedema, fiebre, artritis y eritema multiforme. Todas pue-
den aparecer entre una y dos semanas después de la exposición al antí-
geno. Las formas graves presentan vasculitis (neuropatía y nefrosis).
¿Cómo se clasifican las inmunodeficiencias primarias?
 Deficiencia de células B (anticuerpos).
 Deficiencia de células T (predominante de células T a combinada de
células T y B).
 Trastornos fagocíticos (de la actividad microbicida y del movimiento
celular).
 Trastornos del complemento (de sus componentes y de sus proteínas
reguladoras).
¿Qué tipo de inmunodeficiencia causa el alcoholismo?
Disfunción de los neutrófilos.
¿Qué es el síndrome de Chédiak-Higashi?
Es un defecto de la actividad microbicida, autosómico recesivo, con albi-
nismo parcial, tendencia a la malignidad linforreticular, infección piogénica
severa, fotofobia, linfadenopatía, hepatoesplenomegalia e inclusiones
granulares gigantes en los neutrófilos.

190
¿Qué microorganismos se consideran menos virulentos y son encontrados
como patógenos en los defectos de la fagocitosis?
S. aureus, Pseudomonas, Haemophilus y Aspergillus.
¿Cuáles son los microorganismos extracelulares o encapsulados encontra-
dos como patógenos en la deficiencia de células B?
S. pneumoniae, S. piógenes, H. influenzae.
¿Qué es la enfermedad de DiGiorge?
Es la aplasia tímica, que se acompaña con deficiencia de células T, hipo-
calcemia, faxcies característica, anomalías del arco aórtico y enfermedad
cardiaca.
¿Qué es la enfermedad de Wiskott-Aldrich?
Es la deficiencia de células T y B, que produce eczema y trombocitopenia.
¿Qué es el síndrome de ataxia-telangiectasia?
Es la deficiencia de células T y B, asociadas con dermatitis y deterioro
neurológico.
¿Cómo se saca la cuenta absoluta de neutrófilos?
Multiplicando el número de leucocitos totales por el porcentaje de neu-
trófilos.
¿Cómo se saca la cuenta de CD4?
Multiplicando el número de leucocitos totales por el porcentaje y por el
porcentaje de linfocitos por el porcentaje de linfocitos teñidos para CD4.
¿Qué infecciones provocan linfocitosis?
La infección por Bordetella pertusis, la mononucleosis infecciosa y la hepa-
titis.
¿Qué infecciones provocan neutropenia?
La hepatitis, la fiebre tifoidea y la influenza.
¿Qué fármacos provocan neutropenia?
Las fenotiacinas, la fluoxetina y los antitiroideos.
¿En qué enfermedades se encuentran granulomas eosinofílicos?
En la enfermedad de Hand-Schüller-Christian y la histiocitosis X.
¿Por qué el piroxicam y la indometacina son más dañinos a la mucosa gás-
trica?
Porque son COX-1 selectivos.

191
¿En donde se encuentran los receptores COX-2?
En las células inflamatorias.
¿Qué es la fiebre reumática?
Es una enfermedad inflamatoria multisistémica en la que los anticuerpos
producidos contra las cepas M3 y M18 del estreptococo beta-hemolítico
del grupo A (EBHGA) atacan a tejidos del propio organismo que tienen
epitopos similares. Clínicamente se presenta entre 1 y 3 semanas después
de la infección bacteriana.
¿Cuáles son los criterios de Jones para hacer el diagnóstico del ataque ini-
cial de la fiebre reumática?
Criterios mayores:
 Poliartritis: aparece hasta en 75% de los pacientes y afecta las grandes
articulaciones; es migratoria, dolorosa y se acompaña de fiebre.
 Carditis: ocurre en 50% de los casos; se evidencia por un soplo pan-
sistólico mitral.
 Corea: aparece en 10 a 30% de los pacientes y dura entre 1 y 3 sema-
nas.
 Eritema marginado.
 Nódulos subcutáneos.
Criterios menores:
 Clínicos: fiebre y artralgias.
 Laboratorio: reactantes de fase aguda (velocidad de sedimentación
globular y proteína C reactiva) elevados.
 Intervalo P-R prolongado.
El diagnóstico se hace al encontrar: 2 criterios mayores o 1 mayor y 2
menores y, además, títulos elevados de anticuerpos anti-EBHGA o un
cultivo positivo para éste.
¿Cuáles son los síntomas más frecuentes de la fiebre reumática?
Dos que aparecen principalmente en los adultos (eritema marginado y
carditis) y dos que aparecen principalmente en niños (nódulos subcutá-
neos y corea de Sydenham).
¿Qué son los nódulos de Aschoff?
Son grandes células multinucleadas que tienen cúmulos de colágeno en
su interior. Se les encuentra en el miocardio en caso de fiebre reumática.

192
¿Cuál es el tratamiento de la fiebre reumática?
 Salicilatos: para la artritis y la fiebre.
 Esteroides: si la carditis es grave.
 Clorpromazina o haloperidol: para la corea de Sydenham.
¿Qué es la artritis reumatoide?
Es una enfermedad crónica inflamatoria, con afectación articular y sis-
témica, con factor reumatoide positivo y asociada a HLA DR4.
¿Qué es el pannus?
Es la lesión inflamatoria deformante de la sinovial que ocurre en los pa-
cientes que padecen artritis reumatoide.
¿Cuál es la localización de los nódulos reumatoides?
Yuxtaarticular.
¿Cuáles son las manifestaciones clínicas de la artritis reumatoide (AR)?
 Sinovitis: afecta las articulaciones grandes de manera simétrica y que
respeta las interfalángicas distales (a diferencia de la osteoartritis),
con afectación de tejidos sinoviales periarticulares (tendones y liga-
mentos) que causa síndrome del túnel del carpo, inestabilidad C1-C2
y tendinitis del manguito rotador (hombro).
 Manifestaciones extraarticulares principales: nódulos reumatoides,
queratoconjuntivitis sicca, afección del pulmón y del corazón, vascu-
litis, etcétera.
¿Cuáles son los criterios para hacer el diagnóstico de AR?
Artritis de tipo inflamatorio, afección poliarticular afección simétrica, que
involucra las manos; factor reumatoide positivo y cambios radiológicos su-
gerentes. El diagnóstico se realiza con la presencia de 4 criterios de siete.
¿Cuál es el tratamiento de elección de la AR?
El metotrexate.
¿Qué es el etanercept?
Es un receptor artificial del factor de necrosis tumoral alfa.
¿Qué es el infliximab?
Es un anticuerpo quimérico contra los receptores del factor de necrosis
tumoral alfa.
¿Qué fármacos son antiinflamatorios de vida media larga?
La indometacina, el sulindaco y el piroxicam.

193
¿Cómo actúan el acetaminofén y la fenacetina?
Disminuye la ciclooxigenasa únicamente en el SNC; por tanto, tienen
efecto analgésico y antipirético, pero no antiinflamatorio.
¿Cuáles son los efectos adversos de las sales de oro?
La dermatitis, los cambios en las mucosas, el síndrome nefrótico y la
agranulocitosis.
¿Para qué se usa la D-penicilamina?
Se usa como antiinflamatorio, como quelante en la intoxicación por me-
tales pesados y para el tratamiento de la cistinuria.
¿Cuáles son los efectos adversos de la D-penicilamina?
Son principalmente dermatológicos; en ocasiones hay nefritis y anemia
aplástica.
¿Cuáles son los efectos adversos del metotrexate?
Úlceras orales, náusea, citopenias, cirrosis y síndrome de neumonía aguda.
¿Qué es artritis reumatoide juvenil y cuales son sus variedades?
Es la que inicia entre 1 y 4 años de edad, más común en niñas, y sus va-
riedades son: poliarticular, pauciarticular y sistémica.
¿Qué es la enfermedad de Still?
Es la artritis reumatoide juvenil de inicio sistémico, con examema morbi-
liforme, fiebre y hepatoesplenomegalia. No es lo habitual que se presente
con artrirtis de grandes articulaciones.
¿Cómo se presenta la artritis reumatoide juvenil (ARJ) pauciarticular?
Sin fiebre, con iridociclitis y anticuerpos antinúcleo y positivo; esqueleto
axial respetado.
¿Cómo es la ARJ poliarticular?
Hay cinco o más articulaciones de gran tamaño y simétricas afectadas,
anticuerpos antinúcleo y anticuerpos anti-DNA positivos.
¿Cómo está el factor reumatoide en la ARJ y cuál es el tratamiento?
El factor reumatoide es negativo y el tratamiento se lleva a cabo con áci-
do acetilsalicílico con ASA.
¿Cuáles son los once criterios para hacer el diagnóstico de lupus eritemato-
so sistémico (LES)?
 Eritema malar.
 Eritema discoide.

194
 Fotosensibilidad.
 Úlceras orales.
 Artritis.
 Serositis.
 Hematológicos: linfopenia (menor de 1,500), leucopenia (menor de
4,000), trombocitopenia (menor de 100,000).
 Renales: cilindros, menor de 5g de proteínas en orina de 24 horas
(3+).
 Sistema nervioso central: convulsiones.
 Inmunológicos: células LE, VDRL positivo, anticuerpos Anti-DNA;
anticuerpos Anti Sm, proteína P-ribosomal (marcador para demen-
cia).
 Anticuerpos antinúcleo positivo: aparece en todas las enfermedades
del tejido conectivo (95% en LES).
¿Cuáles son los usos que tiene la determinación de anticuerpos Anti-DNA?
 Es altamente específica para el diagnóstico de lupus y es un marcador
para medir su actividad.
 Sirve para diferenciar entre LES y lupus eritematoso discoide (LED).
 Es una prueba positiva junto con la determinación de anticuerpos an-
tinúcleo en caso de artritis reumatoide juvenil poliarticular.
¿Cuáles son las pruebas que sugieren la presencia de anticuerpos antifosfo-
lípido?
 El anticoagulante del lupus.
 IgG o IgM anticardiolipina.
 VDRL falso positivo.
 Tiempo de protrombina (TP) y tiempo de tromboplatina parcial
(TTP) prolongados, que no corrigen con una mezcla 1:1 de plasma.
 Pruebas de coagulación especiales como: antibeta-2 glicoproteína,
veneno viperino de Russel, TTP diluido activado y tiempo de san-
grado con caolín.
¿Cómo se diferencian por laboratorio el LES y LED?
En el último, los anticuerpos anti-DNA son casi siempre negativos.

195
¿En qué enfermedades aparecen los anticuerpos SSA (Rho) y SSB (L-alfa)?
 En el lupus y el síndrome de Sjögren.
 En el lupus cutáneo y la fotosensibilidad por lupus (SSA).
 En el lupus neonatal y el bloqueo cardiaco congénito.
¿Con qué enfermedades se relacionan los HLA DR2 y DR3?
Se asocian a LES y síndrome de Sjögren sin artritis, respectivamente.
¿Cómo se trata la nefritis por lupus?
Con altas dosis de esteroides en bolo (pulsos) y citotóxicos (azatioprina y
ciclofosfamida).
¿Qué es la artropatía de Jacoud?
Es la deformidad articular provocada por el lupus eritematoso sistémico.
¿Para qué sirven los antiinflamatorios no esteroideos en el tratamiento del
LES?
Para calmar la fiebre, el dolor articular y la serositis.
¿Para qué sirven los antimaláricos en los casos de LES?
Para tratar la enfermedad renal y el lupus refractario.
¿Cuáles son las variantes clínicas de la esclerosis sistémica progresiva (es-
cleroderma)?
Hay una variante limitada, una difusa y la enfermedad mixta del tejido
conectivo.
¿Cuáles son los anticuerpos que se relacionan con la variante difusa de la
esclerisis sistémica progresiva?
Los Scl-70 y los anticuerpos antinucleolar.
¿Cuáles anticuerpos se relacionan con la variante limitada de la esclerosis
sistémica progresiva?
Los anticuerpos anticentrómeros.
¿Cuáles anticuerpos se relacionan con la enfermedad mixta del tejido co-
nectivo?
Los Anti U1RNP.
¿Cuál es el diagnóstico diferencial de la escleroderma?
La escleroderma localizada (morfea o linear), la fascitis eosinofílica y el
síndrome de eosinofilia-mialgia.

196
¿Cuál es la lesión anatomopatológica que se encuentra en casos de esclero-
sis sistémica progresiva?
Hay un daño en el endotelio de los vasos por hiperplasia de la íntima.
¿Cuáles son los anticuerpos antifosfolípido?
 Anticoagulante lúpico (aumenta el TTP).
 Anticuerpos anticardiolipina.
 Se acompañan de pruebas positivas falsas para sífilis (VDRL).
¿Cuál es la diferencia entre polimialgia y polimiositis?
Las dos se presentan con dolor muscular, pero la primera no causa debi-
lidad muscular proximal.
¿Cuáles son los criterios para hacer el diagnóstico de polimiositis?
Fracción MM de la creatincinasa elevada, debilidad muscular proximal,
biopsia que demuestre la existencia de lesión inflamatoria del músculo y
la electromiografía anormal.
¿Cuáles son los signos clásicos de la dermatomiositis?
Edema periorbitario, sufusión heliotrópica (color violáceo de los párpa-
dos), signo de Gottron (manchas escamosas sobre las articulaciones inter-
falángicas proximales y metacarpofalángicas), signo de la “V del cuello”
y signo del chal.
¿Qué se encuentra en una biopsia de un paciente con polimiositis?
Un infiltrado dominado por linfocitos T CD8.
¿Qué se encuentra en la biopsia de un paciente con dermatomiositis?
Infiltrado muscular en el que dominan los leucocitos T CD4 y linfocitos
B; es una vasculitis de pequeños vasos.
¿Qué enzimas se miden en los casos de dermatomiositis?
Fracción MM de la creatincinasa la aldolasa y mioglobina.
¿Cuáles son los anticuerpos que se pueden encontrar en los pacientes con
dermatomiositis?
 Específicos: anticuerpos antisintetasa (jo-1), anticuerpos de recono-
cimiento de partículas (Mi-2).
 No específicos: anticuerpos antinúcleo de patrón especular (anti-
Hep2), anti-Rho, anti-la, PM-SCL, KU.

197
¿Cuáles son las seis formas clínicas de la polidermatomiositis?
Polimiositis asociada a malignidad, con cuerpos de inclusión, dermato-
miositis, juvenil y mezclada con síndrome de Reiter, síndrome de
Sjögren, LES o AR.
¿Cuáles son las características de la polimiositis con cuerpos de inclusión?
Debilidad distal y proximal, ausencia de autoanticuerpos y cambios va-
cuolares (cuerpos de inclusión).
¿Cuáles son las manifestaciones electromiográficas de la polimiositis?
Potenciales de alta frecuencia, polifásicos y fibrilación.
¿Cuáles enfermedades pueden provocar descamación periarticular?
La artritis gotosa, la artritis psoriásica y la dermatomiositis.
¿Cuáles son los cinco hallazgos radiográficos clásicos de la osteoartritis?
Formación de quistes subcondrales, formación de hueso nuevo (osteofi-
tos), esclerosis ósea, estrechamiento del espacio articular y carencia de
osteoporosis.
¿Qué son las espondiloartropatías seronegativas?
Son artritis inflamatorias diferentes de la artritis reumatoide (AR). Las
principales son: espondilitis anquilosante, artritis reactiva (síndrome de
Reiter) y artritis psoriásica. El factor reumatoide es típicamente negativo.
¿Cuáles son las manifestaciones clínicas de las espondiloartropatías?
 Afectación del esqueleto axial: sacroilitis. El esqueleto axial es respe-
tado en la artritis reumatoide (con excepción del cervical).
 Entesis: inflamación del sitio de implantación de los tendones y liga-
mentos al hueso.
 Manifestaciones extraesqueléticas: la uveítis y conjuntivitis son co-
munes (a diferencia de la escleritis y episcleritis que acompañan a la
artritis reumatoide). También aparecen uretritis, colon irritable y
examema psoriasiforme.
 Asociación a HLA-B27 y ausencia de factor reumatoide (FR) positivo.
¿Qué es la artritis reactiva?
También conocida como síndrome de Reiter, es una espondiloartropatía
que cursa con uretritis, úlceras mucocutáneas y artritis.

198
¿Qué microorganismo se asocian al síndrome de Reiter?
Chlamydia y Shigella.
¿Cuáles son las manifestaciones clínicas de la artritis psoriásica?
Las más significativas son la sacroilitis asimétrica y la onicolisis así como
la afectación de las articulaciones interfalángicas distales, sin nódulos
reumatoides. Tiende a remisiones más completas, frecuentes y rápidas
que la AR. El FR es negativo.
¿Cómo se presenta la espondilitis anquilosante?
En varones jóvenes con dolor de espalda y restricción de los movimientos
de la columna por afección axial.
¿Cuáles son los fármacos de elección para tratar las espondiloartropatías y
la artritis reumatoide juvenil?
Los antiinflamatorios no esteroideos.
¿En qué consiste el síndrome de “cola de caballo”?
Se observa frecuentemente en la espondilitis anquilosante (EA). Existe
incontinencia urinaria nocturna, arreflexia del aquíleo, impotencia e hi-
poestesia vesical. También se acompaña de ciática.
¿Cuándo es positivo el signo de Lasegue?
Cuando se pruduce dolor al levantar la pierna más de 60º.
¿Cuál es el diagnóstico diferencial de la espondilitis anquilosante (EA)?
En la EA suele haber ciática y la “columna vertebral de bambú”. Se debe
hacer con la hernia de disco intervertebral o con la hiperostosis vertebral
idiopática difusa.
¿Qué es la maniobra de Schober?
Es una prueba que se utiliza para evaluar la limitación de la flexibilidad
lumbar. En un sujeto en bipedestación, se marca la piel en un punto a ni-
vel de la V lumbar y se marca otro 10 cm arriba, y se pide al sujeto que se
incline hacia adelante. En un sujeto normal, los puntos se separan más de
5 cm. La separación menor de 3 cm indica limitación del movimiento.
¿Cuál es la diferencia entre osteofito y sindesmofito?
Los sindesmofitos son calcificaciones que aparecen en las caras ánterola-
terales de las vértebras y que con el tiempo forman un puente unidas por
osteofitos; son calcificaciones triangulares que se desarrollan a varios mi-
límetros de la unión del disco vertebral.

199
¿Cuál es la diferencia entre espondilosis y espondilolistesis?
La espondilosis es la interrupción de la pars interarticularis de la vertebra y
la espondilolistesis se refiere al desplazamiento de una vértebra sobre otra.
¿Cuándo se indica el 5-ASA (olsalazina o mesalamina) para pacientes con
CUCI?
Cuando los pacientes son alérgicos a la sulfasalazina.
¿Cuál es la diferencia entre el fenómeno y la enfermedad de Raynaud?
Ambos son provocados por un espasmo de las arteriolas de los dedos,
que causa cianosis y palidez. El fenómeno es unilateral y constante; la
enfermedad es bilateral y progresiva.
¿Cuáles enfermedades pueden presentar el fenómeno de Raynaud?
La esclerosis sistémica, el síndrome de CREST, el lupus eritematoso sis-
témico y la enfermedad mixta del tejido conectivo.
¿Cuáles son los principales fármacos inmunosupresores no selectivos?
En metotrexato, la ciclofosfamida, la azatioprina y la prednisona.
¿Cuáles son los fármacos inmunosupresores selectivos?
La ciclosporina y el tacrolimus (FK506).
¿Cuáles son los efectos inmunosupresores del metotrexato?
Disminuye la actividad celular y humoral al inhibir la proliferación de cé-
lulas T en fase S.
¿Qué es la azatioprina?
Es un fármaco análogo de la mercaptopurina y antagonista de la síntesis
de purinas.
¿Cuáles son los efectos inmunosupresores de la azatioprina?
Es citotóxico y disminuye la inmunidad celular.
¿Cómo actúa la ciclosporina sobre las células T?
Inhibe su activación.
¿Cómo actúa el metotrexato sobre las células T?
Inhibe la formación de células T activadas (en fase S).
¿Cuáles son las acciones de la ciclosporina A y de la ciclofosfamida?
La primera es inmunomoduladora y la segunda cititóxica.
¿Cuáles son los fármacos citotóxicos más utilizados?
Los antimetabolitos (azatioprina y metotrexato) y la ciclofosfamida.

200
¿Qué se debe adicionar al tratamiento, en los casos de uso prolongado de
esteroides?
Vitamina D y calcio.
¿Qué son las vasculitis?
Son enfermedades que cursan con inflamación de los vasos, a menudo
segmentaria, y que pueden involucrar venas y arterias de cualquier cali-
bre. Sin embargo, la mayoría de las vasculitis se debe a inflamación arte-
rial, lo que compromete el riego sanguíneo y lleva a la necrosis del área
que nutre el vaso afectado.
¿Cuáles son las principales vasculitis?
 Púrpura de Henoch-Schönlein: es la púrpura palpable, desencadena-
da por reacciones de hipersensibilidad, y cuyo dato anatomopatológi-
co característico son los depósitos de IgA. Los vasos más afectados
son las vénulas poscapilares que se infiltran con neutrófilos, por lo
que también se llama púrpura leucocitoclástica.
 Eritema nodoso: es una inflamación de los capilares de la dermis pro-
funda, que causa lesiones dolorosas, tumefactas, profundas y rojas en
los brazos y piernas.
 Poliarteritis nodosa: las arterias musculares de mediano tamaño son
las afectadas, y la lesión causa ruptura de la lámina elástica interna
produciendo aneurismas posinflamatorios.
 Arteritis de células gigantes: la fisiopatología es similar a la poliarteri-
tis nodosa, pero el proceso más bien es limitado a las arterias del ár-
bol carotídeo extracraneal.
 Arteritis de Takayasu: los vasos de gran calibre y sus ramas están
afectados por un infiltrado linfocitario que lleva a la estenosis posin-
flamatoria y a la disminución del pulso.
¿Cuál es el cuadro de la crioglobulinemia tipo II (mixta) y con qué se rela-
ciona?
Púrpura palpable, artritis y crioglobulinas elevadas. Se relaciona con el
VHC.
¿Qué es la polimialgia reumática?
Es una enfermedad que cursa con dolor y rigidez del hombro y la cadera,
velocidad de sedimentación globular elevada, en personas mayores de 50

201
años, con fiebre, malestar y pérdida de peso. Se asocia a arteritis de célu-
las gigantes.
¿Qué es la fibromialgia?
Es una enfermedad no inflamatoria caracterizada por dolor difuso acom-
pañada de un transtorno del sueño del tipo de movimientos oculares rá-
pidos (REM, etapa IV). Existen múltiples puntos de dolor como el
soboccipital, trapecio, glúteo, a 2 cm de los epicóndilos, en la segunda
unión costocondral, etc. La velocidad de sedimentación globular es nor-
mal. El tratamiento se encamina a mejorar la calidad del sueño mediante
amitriptilina o relajantes musculares.
¿Qué enfermedades reumatológicas se presentan con dermatitis y artritis?
 La artritis por gonococo causa el síndrome periartritisdermatitis.
 La artritis de Lyme causa artritis, dermatitis, síntomas neurológicos y
cardiacos.
 La enfermedad de Still presenta un examema morbiliforme evanes-
cente.
 La dermatomiositis presenta exantema sobre las articulaciones (signo
de Gottron).
 El síndrome de Sjögren presenta lesiones purpúricas en las pantorri-
llas y los tobillos.
¿Cuáles son los pasos para estudiar y precisar la naturaleza de una artritis?
 Determinar si el dolor proviene de la articulación o de estructuras
adyacentes.
 Determinar si es inflamatoria; es decir, si cursa con rigidez matutina
de más de una hora de duración.
 Observar si la afección articular es simétrica, en cuyo caso habrá que
sospechar lupus eritematoso sistémico, artritis reumatoide, dermato-
miositis y esclerosis sistémica progresiva.
 Si la afección es asimétrica se debe pensar en síndrome de Reiter o
espondilitis anquilosante.
¿Cuál es el componente principal del líquido articular?
El ácido hialurónico. Su presencia se puede reconocer por el signo de la he-
bra, ya que le confiere una viscosidad elevada al líquido articular. Al adi-

202
cionar ácido acético se aglutina (prueba del coágulo de mucina). Cuando
la artritis es inflamatoria estas pruebas son negativas.
¿A qué se llama deformidad en ojal?
A la flexión de la articulación interfalángica proximal por lesión de los
extensores, presente en la enfermedad inflamatoria.
¿A qué se le llama deformidad en cuello de cisne?
A la flexión de la articulación interfalángica distal y metacarpofalángica,
con extensión de la interfalángica proximal.
¿Cuáles son las principales enfermedades articulares causadas por micro-
cristales?
La gota y la pseudogota.
¿Qué es la gota?
Es una artritis aguda, recidivante, de las articulaciones periféricas, resultan-
te del depósito de cristales de urato monosódico procedente de líquidos
corporales hiperuricémicos sobresaturados en tendones, articulaciones y
cerca de ellos. Puede hacerse crónica y deformante.
¿En qué tejidos se produce el urato?
En todos aquellos en que exista la enzima xantina-oxidasa, principalmen-
te el hígado y la mucosa del intestino delgado.
¿Cuál es el nivel normal del ácido úrico sérico?
7 mg/dl.
¿Cuál es la secuencia de los cambios metabólicos que transforman el IMP
hasta urato?
IMP  hipoxantina  xantina  urato.
¿Cómo se presenta el episodio agudo de gota?
Puede ser causado por un traumatismo mínimo, por un abuso dietético o
alcohólico, por una cirugía. Se presenta con artritis, descamación y pruri-
to posinflamatorios. Cuando afecta al primer ortejo se llama podagra.
¿Cuáles son los factores que inducen la hiperuricemia secundaria?
 Diuréticos: tiacidas.
 Fármacos citotóxicos: ciclosporina.
 Estados de recambio aumentado de purinas: trastornos mieloprolife-
rativos, mieloma múltiple, hemoglobinopatías.

203
 Estados de excreción disminuida de urato: enfermedad renal crónica,
lactacidemia, cetoacidemia, probenecid a dosis subterapéuticas.
 Enfermedades crónicas: diabetes, hipotiroidismo e intoxicación por
plomo.
¿Cuáles son las causas de aumento de excreción renal de ácido úrico?
La quimioterapia, el síndrome de Lesch-Nyhan y la gota.
¿Cómo se trata la hiperexcreción de ácido úrico?
Con hidratación y alcalinización de la orina a un pH mayor de 7.5
¿Qué es la pseudogota?
Es una artritis con depósitos de cristales de monofosfato de sodio con bi-
rrefringencia positiva.
¿Qué esteroide se usa intraarticularmente y a qué dosis?
Triamcinolona, 30-40 mg.
¿Cómo actúa la colchicina?
Inhibiendo la quimiotaxis celular hacia los cristales de urato.
¿Cómo se decide usar alopurinol o un uricosúrico en los casos de hiperuri-
cemia?
Se determina el ácido úrico en la orina de 24 horas, si este es menor de
800, se usa un uricosúrico; si es mayor de 800, se usa alopurinol.
¿Cuáles son agentes uricosúricos?
El probenecid y la sulfinpirazona disminuyen la resorción tubular de áci-
do úrico si se utilizan a dosis terapéuticas. A dosis inferiores bloquean la
secreción tubular. El probenecid inhibe la secreción tubular de penicilina,
por lo que en ocasiones se usa para potenciar su efecto.
¿Cuáles son los músculos del manguito rotador del hombro?
Supraespinoso, infraespinoso, redondo menor y subescapular. La patolo-
gía del manguito rotador es causa frecuente de dolor de hombro. En estos
casos el dolor se desencadena con el movimiento hacia arriba y la rota-
ción interna del brazo.
¿Qué es el hombro de Milwaukee?
Es una lesión del hombro causada por cristales de hidroxiapatita, caracte-
rizada por cambios radiográficos como un gran derrame, sinovitis proli-
ferativa, depósito mineral e inestabilidad ligamentosa.

204
¿Cómo se identifica las lesiones ligamentarias de la rodilla?
Debemos verificar si existe deslizamiento de la tibia sobre el fémur, lo
que se conoce como signo de la gaveta. Si el deslizamiento es anterior está
afectado el ligamento cruzado anterior, si es posterior, está afectado el li-
gamento cruzado posterior.

205
206
INTOXICACIONES Y ACCIDENTES

¿Cuál es el principal sitio de obstrucción de de vías aéreas en los menores


de un año?
La laringe.
¿Cuál es el cuadro clínico cuando existe obstrucción de las vías aéreas infe-
riores?
Sibilancias y asimetría de los ruidos respiratorios.
¿Qué es la metahemoglobina?
Es la hemoglobina oxidada a su estado férrico; es incapaz de transportar
oxígeno.
¿Cuáles son las sustancias que pueden causar metahemoglobinemia?
Benzocaína, anilinas, nitritos, gases de óxido nitroso, nitrobenceno, dap-
sona, piridio y otras.
¿Cuáles son los datos clínicos de la intoxicación por monóxido de carbono?
El monóxido de carbono se une a la hemoglobina con una afinidad 250
veces mayor que el oxígeno. Las víctimas experimentan cefalea, vértigo,
dolor abdominal y náusea cuando aspiran una mezcla de 10 a 20%. Las
concentraciones mayores prodecen confusión, disnea y síncope. Cuando
las concentraciones llegan a 50 o 60% hay hipotensión, convulsiones y
coma.
¿Qué sistema evita la sobreproducción de metahemoglobina?
El sistema reductasa-NADH-metahemoglobina.
¿Cuáles son los pasos para iniciar la desintoxicación oral?
 Adsorción con carbón activado, utilizándolo en proporción de 5 a 12
veces la cantidad calculada de lo ingerido.
 Inducción del vómito con jarabe de ipecacuana o lavado gástrico.

207
 Catárticos que actúan disminuyendo el tiempo de tránsito y, por con-
secuencia, la absorción del tóxico. Se utilizan citrato o sulfato de
magnesio.
¿Cómo se trata la intoxicación por ácido acetilsalicílico (ASA)?
Se alcaniliza con HCO3 y se adjunta cimetidina en el evento agudo para
disminuir la actividad de la citocromo-P450-oxidasa, que al metabolizar
el ASA, da origen a sus metabolitos tóxicos.
¿Qué niveles séricos de ASA producen los diversos grados de intoxicación?
 Intoxicación leve: 30 mg/dl provocan al principio alcalosis respirato-
ria y luego acidosis metabólica.
 Intoxicación moderada: 100-150 mg/dl a partir de este nivel hay que
inducir el vómito y hospitalizar.
 Intoxicación grave: más de 150 mg/dl.
¿Cuándo es tóxico el acetaminofén?
Cuando, en dosis altas, su metabolito, la N-acetilbenzoquinoneimina, exce-
de al glutatión que la inactiva y produce necrosis hepática.
¿Cuáles son las fases de la intoxicación por acetaminofén?
 Fase 1: en los primeros 30 o 60 minutos, hay náusea y vómito, sin
pérdida de la conciencia.
 Fase 2: en las primeras 24 a 48 horas, hay alteración de las pruebas
de funcionamiento hepático.
 Fase 3: en los siguientes 3 a 5 días hay confusión y estupor secunda-
rios al daño hepático.
¿Por qué hay alcalosis respiratoria en los casos de la intoxicación por ace-
taminofén?
Porque aumenta la sensibilidad del centro respiratorio a la acidosis, lo
que provoca hiperventilación.
¿Por qué puede haber también acidosis metabólica en la intoxicación por
acetaminofén?
Porque se inhibe el ciclo de Krebs.
¿Por qué se debe administrar bicarbonato para tratar la intoxicación por
acetaminofén?
Porque a un pH urinario menor de 8, el acetaminofén y sus metabolitos
tóxicos se eliminan ionizados en la orina.

208
¿Cuál es el antagonista del sulfato de magnesio y del potasio?
El gluconato de calcio.
¿Cuál es el cuadro clínico causado por la sobredosificación de ansiolíticos?
Depresión respiratoria, hipotensión y coma.
¿Cuál es el cuadro clínico causado por la intoxicación grave por narcóti-
cos?
Depresión respiratoria, vasodilatación periférica, pupilas puntiformes y
muerte.
¿Cuándo aumenta la protoporfirina?
Cuando disminuye el fierro y cuando existe intoxicación por plomo.
¿Cuáles son los síntomas de cafeinismo?
Se relacionan con la ingestión de más de 500 mg al día y son de tipo car-
diovascular, gastrointestinal, nerviosismo, agitación e insomnio.
¿Cuáles son los tres grupos de síntomas producidos por la intoxicación por
órganofosforados?
Síntomas muscarínicos, nicotínicos y del sistema nervioso central.
¿Cuáles son los síntomas muscarínicos?
Diaforesis, sialorrea, pupilas puntiformes, epífora, broncoconstricción,
aumento de la secreción bronquial, espasmo abdominal, vómito, diarrea
y bradicardia.
¿Cuáles son los síntomas nicotínicos?
Taquicardia, fasciculaciones y espasmos musculares que afectan el dia-
fragma y los demás músculos respiratorios.
¿Cuáles son las manifestaciones en el sistema nervioso central (SNC)?
Cefalea, fatiga, vértigo, ansiedad, confusión, convulsiones, depresión del
centro respiratorio y coma.
¿Cómo actúan fisiopatológicamente los órganofosforados?
Inhibiendo la acetilcolinesterasa, la principal vía de exposición es la piel.
Los síntomas inician 2 a 3 horas después de la intoxicación.
¿Cuál es el primer paso en el tratamiento de la intoxicación con órganofos-
forados?
Establecer una vía aérea permeable.
¿Cuál es la dosis de Ipecac para tratar la intoxicación con órganofosfora-
dos?
15 a 30 ml.

209
¿Cuál es la dosis del sulfato de atropina para tratar la intoxicación con ór-
ganofosforados?
1 mg. IV cada 20-30 minutos hasta la desaparición de los síntomas.
¿Qué olor tienen los productos de intoxicación por arsénico y fósforo?
A ajo.
¿Cómo causan daño los álcalis concentrados?
Producen necrosis por licuefacción penetrante.
¿Cómo causan daño los ácidos concentrados?
Produce necrosis por coagulación.
¿Dónde se puede encontrar el etilenglicol?
El etilenglicol es un solvente (alcohol) incoloro, inodoro y de sabor dulce
que se puede encontrar en pinturas, plásticos, explosivos, extintores, flui-
dos hidráulicos, limpiadores de parabrisas y preparaciones anticongelan-
tes.
¿Cuál es el efecto general del etilenglicol?
Es un depresor potente del SNC; es muchas veces más potente que el
etanol.
¿Qué efectos comparten la intoxicación por carbamacepina y la causada
por anfetaminas?
Hipertensión, convulsiones y nistagmo.
¿Qué productos son susceptibles de ser eliminados por la orina?
Todos los de volumen de distribución bajo (menor de 100)
¿Qué medicamentos causan intoxicación por opiáceos?
El difenoxilato y el dextrometorfán.
¿Cuál es la tríada de la intoxicación por opiáceos?
Miosis, coma y depresión respiratoria.
¿Cuál es la dosis de la naloxona (antagonista opiáceo)?
100-200 mEq/kg/dosis
¿Cuáles son las complicaciones producidas por los opiáceos?
Íleo, arritmias y edema pulmonar no cardiógeno.
¿Qué medicamentos son anticolinérgicos?
La belladona, butilhioscina y atropina.
¿Cuáles son los síntomas anticolinérgicos?
Sequedad de ojos y mucosas, taquicardia, retención urinaria, estreñi-
miento, íleo y midriasis.

210
¿Cuál es el antídoto de los anticolinérgicos?
La fisostigmina.
¿Qué es el succímero?
También conocido como DMSA (ácido meso-2,3-dimercaptopurínico),
se utiliza para tratar la intoxicación por plomo.
¿Qué medicamentos son simpaticomiméticos?
Los anticongestivos o antigripales como la fenilefrina, efedrina, propano-
lamina, nafazolina y la oximetazolina.
¿Qué se debe sospechar ante la presencia de síntomas de neuroestimula-
ción súbita en un niño con catarro?
Intoxicación por antigripales simpaticomiméticos alfa y beta.
¿Cuándo se presenta intoxicación por talio?
Cuando hay ingestión de raticidas.
¿Cuál es la clínica de la intoxicación por talio?
Dolor abdominal, estreñimiento, neuropatía y encefalopatía.
¿Cuáles son las fuentes de intoxicación por plomo?
Los talleres de loza de barro vidriada y las fundiciones.
¿Cuál es al cuadro clínico de la intoxicación por plomo?
Encefalopatía e hipertensión endocraneal.
¿Qué intoxicaciones cursan con encefalopatía?
La de plomo y la de talio.
¿Cuáles son los quelantes del plomo?
Edatato cálcico disódico (EDTA), succímero, dimercaprol y D- penici-
lamina.
¿Cuál es el cuadro clínico de la intoxicación por hierro?
Dolor tipo cólico, vómito, diarrea y hemorragia gastrointestinal.
¿Cómo actúa el cianuro?
Es un inhibidor de la citocromo oxidasa.
¿Cómo se clasifican las quemaduras?
 Grado1: epidermis
 Grado 2A: epidermis y dermis papilar
 Grado 2B: epidermis y dermis reticular.
 Grado 3A: epidermis, dermis y tejido celular subcutáneo (TSC).
 Grado 3B: epidermis, dermis, TSC, músculo y hueso.

211
212
URONEFROLOGÍA

¿Cuáles son las cinco variedades clínicas de las glomerulopatías?


 Síndrome nefrítico agudo.
 Glomerulonefritis rápidamente progresiva.
 Síndrome nefrótico.
 Anormalidades asintomáticas del sedimento urinario (proteinuria y
hematuria).
 Glomerulonefritis crónica.
¿Cuáles son los rangos de proteinuria en orina de 24 horas?
 3 cruces en el examen general de orina (EGO), resultado significati-
vo, pero no patológico.
 300 mg: normal.
 500 mg:significativo pero no patológico.
 Menos de 1 g: tubular.
 Más de 1 g: glomerular.
 3.5 g: síndrome nefrótico.
 960 mg/m2/día: síndrome nefrótico.
¿Qué es la hematuria?
Se considera hematuria cuando se hallan más de 3 a 5 eritrocitos por
campo de gran aumento en una muestra de sedimento. Los falsos positi-
vos pueden deberse al consumo de vitamina C, betabel y mioglobina en
la dieta.
¿Para qué se utiliza la prueba de ortotoluidina?
Para demostrar hematuria detectando la presencia de hemoglobina en la
orina cuando no pueden identificarse eritrocitos en el sedimento.

213
¿Cuándo se habla de macroalbuminuria y microalbuminuria?
Macroalbuminuria es cuando la tasa de excreción de albúmina rebasa los
300 mg/día y microalbuminuria cuando va de 30 a 300 mg/día.
¿Cuándo se considera que existe oliguria?
Cuando el volumen urinario es menor de 300 ml/m2/día o menor de 0.5
ml/kg/hora.
¿Cuáles son las causas de la proteinuria?
 La proteinuria secundaria a ortostatismo.
 La nefritis túbulointersticial.
 La glomerulonefritis.
¿Qué es el síndrome nefrítico?
Es un síndrome en el que hay falla renal aguda que se desarrolla en va-
rios días, oliguria, hematuria y cilindros de eritrocitos, edema en las par-
tes declives e hipertensión arterial.
¿Qué es el síndrome nefrótico?
Es un síndrome integrado por la presencia de proteinuria (más de 3.5g
por cada 1.73 m2 en 24horas), hipoalbuminemia menor de 3, edema, co-
lesterol más de 200, lipiduria, diátesis trombótica y disminución de 10 a
30% en la velocidad de filtración glomerular (VFG).
¿Cuál es el cuadro clínico de la nefritis interstiticial?
Presencia de proteínas especiales, como la proteína de Tamm-Horsfall,
beta 2-microglobulina y albúmina. Se puede encontrar en pacientes que
sufren nefritis causada por fármacos o enfermedades inflamatorias cróni-
cas. Hay fiebre, exantema máculopapular transitorio, insuficiencia renal
aguda (IRA), hematuria y piuria.
¿Qué fármacos pueden causar nefritis intersticial?
La penicilina, la meticilina y el alopurinol, algunos diuréticos y antiin-
flamatorios no esteroideos.
¿Cuál es el hallazgo característico en el examen general de orina (EGO) de
pacientes que padecen nefritis intersticial?
Hay eosinofiluria.
¿Cuál es el cuadro clínico de la glomerulonefritis?
Se encuentra desde proteinuria glomerular (más de 1 g/24horas) hasta
proteinuria nefrótica (más de 3.5gr/24horas), IRA y síndrome nefrítico.

214
¿Cómo se clasifican las glomerulonefritis (GN) por su análisis serológico?
 GN por anticuerpos, anticitoplasmas de neutrófilo (ANCA): granu-
lomatosis de síndrome de Wegener, Churg-Strauss.
 GN antimembrana basal glomerular: síndrome de Goodpasture.
 GN de complejos inmunes: GN del lupus, GN posinfecciosa, nefro-
patía por IgA. GN crioglobulinémica y GN membranoproliferativa.
¿Cuáles son las capas de la barrera de filtración glomerular que separan a
la sangre del filtrado glomerular?
 El endotelio capilar glomerular fenestrado.
 La capa de células mesangiales.
 La embrana basal.
 El epitelio de células viscerales (podocitos).
¿Cuál es la lesión anatomopatológica que se encuentra en las siguientes en-
fermedades glomerulares?
 GN membranosa: depósitos subepiteliales electrodensos.
 GN mesangial proliferativa: proliferación de las células mesangiales.
 GN membranoproliferativa: reduplicación o engrosamiento de la
membrana basal y proliferación de la matriz mesangial.
 GN rápidamente progresiva: se encuentran lesiones “en semiluna” en
el espacio de Bowman.
 GN proliferativa difusa: existe hipercelularidad mesangial y endote-
lial, depósitos electrodensos subepiteliales, infiltrado de neutrófilos y
monocitos y depósitos de C3 en la membrana basal.
¿Cuáles son las características de la GN posinfecciosa?
La lesión anatomopatológica característica es la GN proliferativa difusa,
cuya presentación clínica típica es la del síndrome nefrítico.
¿Cuáles son las características clínicas y de laboratorio de la glomerulone-
fritis membranoproliferativa?
Puede ser idiopática o secundaria a lupus eritematoso sistémico (LES), crio-
globulinemia o infección crónica. Se presenta con síndrome nefrótico, hipo-
complementemia de C3 y evolución a la enfermedad renal terminal.
¿Cuáles glomerulopatías cursan con hipocomplementemia C3?
La glomerulonefritis membranoproliferativa y la posinfecciosa (lesión
proliferativa difusa).

215
¿Qué es el síndrome de Alport?
Es una nefritis hereditaria que puede ser autosómica dominante o ligada
a X. Cursa con piuria, hematuria, falla renal progresiva y pérdida auditi-
va neurosensorial.
¿En qué consiste la enfermedad de cambios mínimos?
Es una enfermedad glomerular también conocida como nefrosis lipoidea
o síndrome nefrótico de lesión nula. Es la causa principal de síndrome
nefrótico en niños; cursa con síndrome nefrótico, hipertensión y azoemia.
El tratamiento se lleva a cabo con esteriodes, citotóxicos (ciclofosfamida,
clorambucil) y ciclosporina.
¿Cuál es el diagnóstico diferencial del síndrome de Goodpasture?
Granulomatosis de Wegener, síndrome de Churo-Strauss, púrpura trom-
bocitopénica trombótica (PTT), crioglobulinemia, vasculitis necrosante,
enfermedad del legionario y púrpura de Henoch-Schönlein.
¿Cómo se diagnostica la nefropatía por IgA (enfermedad de Berger)?
Por una biopsia de piel que demuestre depósitos de IgA (no por el nivel
sérico de IgA).
¿Cuáles son los dos tipos de nefropatía diabética?
Nodular y difusa. La difusa es más frecuente; se presenta con un ensan-
chamiento mesangial generalizado.
¿Dónde se observa y cómo es la lesión de Kimmelstiel-Wilson?
Se observa en la nefropatía diabética nodular. Se encuentran depósitos de
material Schiff-positivo en la periferia de los glomérulos.
¿Cómo es la lesión de la glomeruloesclerosis?
Existe hialinización de la arteriola y lesión de Kimmelstiel-Wilson.
¿Cuál es la velocidad de filtración glomerular (VFG) normal?
125 ml/min.
¿Cómo se obtiene el índice de falla renal?
Sodio urinario (UNa)/ Creatinina urinaria (UCr)/Creatinina plasmática
(PCr).
¿Cómo se calcula la fracción excretada de sodio (FENa)?
(UNa/PCr) / (UCr/PCr)

216
¿Cómo se evalúan el índice de falla renal y la fracción excretada de sodio
(FENa)?
Cuando son menores que 1, la falla es prerrenal; cuando son mayores
que 1 la falla es renal (necrosis tubular aguda).
¿Cómo se interpreta el sodio urinario?
Cuando es menor que 20, la falla es prerrenal, y si es mayor que 20, es
renal o posrenal.
¿Cómo se evalúa el índice BUN/Creatinina en suero?
Mayor que 20 es prerrenal; menor que 10, es renal o necrosis tubular
aguda (NTA).
¿Cuándo se habla de NTA?
Cuando se encuentran estigmas de insuficiencia renal aguda, FENa ma-
yor a 1% y sedimento con cilindros granulosos pigmentados.
¿Cuándo se habla de insuficiencia renal aguda y crónica con base en los va-
lores de creatinina?
La insuficiencia renal aguda tiene valores de creatinina plasmática de 1.5
a 3. Cuando la creatinina es mayor que 3 se habla de insuficiencia renal
crónica.
¿Cuándo se habla de falla renal terminal?
Cuando la depuración de creatinina es menor que 5.
¿Cuál es el cuadro clínico de la insuficiencia renal crónica (IRC)?
Hiperazoemia, síndrome urémico, prurito de difícil control, hipertensión
arterial, isostenuria, osteodistrofia renal y riñón pequeño.
¿Qué es el síndrome urémico?
Ocurre después de la falla renal y cursa con aliento urémico, escarcha
urémica, malestar general, vómito, alteraciones del sensorio y derrame
pericárdico.
¿Cómo afecta la insuficiencia renal el metabolismo del calcio y del fósforo?
 Al disminuir la velocidad de filtración glomerular (VFG) por debajo
de 25% de lo normal, se deteriora la excreción de fósforo. La hiper-
fosfatemia causa hipocalcemia, ya que el fosfato es un quelante del
calcio. La hipocalcemia aumenta la secreción de hormona paratiroi-
dea, que tiene un efecto fosfatúrico y normaliza el fósforo sérico.

217
 Por la conversión de 25-hidroxicolecalciferol a 1-25-hidroxicolecal-
ciferol, lo que disminuye la absorción intestinal de calcio y da como
resultado una mineralización anormal del hueso u osteomalacia.
¿Cómo se monitoriza el fósforo cuando existe la insuficiencia renal?
Con el producto calcio-fósforo, que debe ser menor de 60-70, para evitar
calcificaciones metastásicas en vasos sanguíneos, tejidos blandos, pulmo-
nes y miocardio.
¿Qué es la osteodistrofia renal?
La hipocalcemia funcional secundaria a la hiperfosfatemia y la disminu-
ción del 1, 25-DHC provocan niveles altos de hormona paratiroidea
(PTH), estimulan un alto recambio óseo con resorción osteoclástica y le-
siones subperiósticas, lo que causa lesiones “en sal y pimienta”, princi-
palmente en clavículas y falanges.
¿Qué fármacos se usan para fijar el potasio cuando existe insuficiencia re-
nal crónica (IRC)?
Carbonato de calcio, sulfato de potasio e hidróxido de aluminio.
¿Cuáles son las restricciones dietéticas para los pacientes que padecen
IRC?
Proteínas menos de 1g/día y Na 1-2 g/día. Cuando la VFG es menos de
10-15, el potasio se restringe a menos de 1g/día, y el sodio a 60-70
mEq/día.
¿Cuáles son las indicaciones para diálisis?
Síndrome urémico, PH menor de 7.20, hiperkalemia de 7, sobrecarga de
líquidos, síntomas neurológicos y en algunos casos, creatinina de 10 y
BUN mayor de 100.
¿Qué células conforman el aparato yuxtaglomerular?
Las células yuxtaglomerulares, la mácula densa y las células “en encaje”.
¿Dónde se encuentran las células yuxtaglomerulares?
En la pars de la arteriola aferente del glomérulo.
¿Dónde se encuentra la mácula densa?
La rama ascendente gruesa del asa de Henle alcanza al glomérulo de la
nefrona de la cual se origina y pasa cerca de las arteriolas aferente y efe-
rente; en este punto el epitelio del túbulo se modifica para dar lugar a la
mácula densa.

218
¿Dónde se produce y para qué sirve la renina?
Se produce en las células yuxtaglomerulares. Su función consiste en ex-
traer la angiotensina I de su precursor, el angiotensinógeno.
¿Dónde se producen el angiotensinógeno y la enzima de conversión de la
angiotensina (ECA)?
El angiotensinógeno se produce en el hígado y la ECA en el endotelio
vascular.
¿Cuáles son los receptores de angiotensina II?
Los receptores AT1A se encuentran en las paredes de los vasos sanguí-
neos, donde ejercen su efecto presor. Los receptores AT1B se encuentran
en la corteza suprarrenal, donde promueven la secreción de aldosterona.
Un exceso de angiotensina II regula en forma descendente a los recepto-
res vasculares, y en forma ascendente, a los receptores adrenales.
¿Qué enfermedades cursan con desensibilización a la angiotensina?
Los pacientes con cirrosis no metabolizan la angiotensina II para inacti-
varla, los pacientes con un estado prolongado de depleción de sodio
siempre tienen niveles altos de A-II. Por tanto ante este exceso los recep-
tores se regulan en forma descendente provocando un estado refractario.
¿Cómo actúa la prostaciclina en el eje renina-angiotensina-aldosterona?
Es un contrarregulador de la angiotensina II.
¿A qué se llama retroalimentación túbulo-glomerular?
Al fenómeno regulado por la mácula densa, en el que una reducción del
flujo dentro del túbulo aumenta la velocidad de filtración glomerular
(VFG) y viceversa. La VFG se ajusta por constricción o dilatación de la
arteriola eferente.
¿Qué fenómeno explica la relación directa entre la resorción tubular de
agua y la de sodio?
En el fenómeno de equilibrio glomerulotubular, un aumento de la VFG
produce aumento de la resorción de solutos y, en consecuencia, de agua,
principalmente en el túbulo proximal; es particularmente prominente pa-
ra el sodio.
¿Cómo se realiza la prueba para la determinación de renina?
Se utiliza una dosis media-alta de furosemida. Lo normal es que después
de la depleción de volumen la renina aumente. El nivel no se eleva cuan-

219
do hay un estado hiperproductor de renina como en el hiperaldostero-
nismo secundario.
¿Cuál es la mejor prueba para detectar hiperaldosteronismo primario?
El índice de aldosterona/renina tomado en bipedestación.
¿Cómo se realiza la prueba para la determinación de aldosterona?
Se hace que el paciente lleve una dieta de 150 miliequivalentes de sodio
por algunos días. Luego se le administra solución salina. La respuesta
normal resulta en una elevación de la aldosterona. Ésta no se eleva cuan-
do existe hiperaldosteronismo.
¿Cuál es la osmolaridad urinaria normal?
700-1,400 m0sm/kg.
¿Qué osmolaridad urinaria se alcanza en la diabetes insípida?
200 m0sm/kg.
¿Cuál es el tratamiento de la diabetes insípida nefrogénica?
Las tiazidas, porque disminuyen el volumen urinario y aumentan la concen-
tración de sodio en el fluido tubular lo que produce orina hiperosmolar.
¿Cuál es el tratamiento en la diabetes insípida central?
Desmopresina y clorpropamida, que potencian la secreción de hormona
antidiurética hormona antidiurética (ADH).
¿Cómo se realiza la prueba para determinar hormona antidiurética (ADH)?
Se restringen los líquidos al paciente y luego se le administra un análogo
de la vasopresina (desmopresina). La respuesta normal es que la densidad
urinaria se eleve (más de 1.020), pues se evita la salida de agua. En la
diabetes insípida la densidad urinaria no se eleva tras la deprivación de
líquidos, pues no hay ADH que evite la salida de agua; sin embargo, la
densidad urinaria (que se encuentra muy baja) se eleva sólo para alcanzar
el nivel normal cuando se administra un agente que emule la función de
la ADH.
¿Qué medicamentos aumentan la secreción de ADH?
Clorpropamida, carbamacepina, vincristina y clofibrato.
¿Cómo afectan la excreción de calcio los diuréticos de asa y las tiacidas?
Los diuréticos de asa aumentan la calciuria, y las tiacidas la disminuyen.
¿Cuáles son dos indicaciones de las tiacidas?
Para producir hipocalciuria cuando hay cristales de oxalato y para pro-
ducir orina hiperosmolar en los casos de diabetes insípida.

220
¿Cómo actúa la espironolactona?
Es un competidor activo de la aldosterona en los sitios intracelulares del
túbulo colector donde estimula el recambio Na-K, por lo que el Na no se
reabsorbe y sí se reabsorben potasio e hidrógeno.
¿Cómo actúan la amilorida y el triamterene?
Bloquean los canales de Na y disminuyen el recambio Na-K en el túbulo
colector.
¿Cómo actúan los diuréticos osmóticos?
Actúan en el túbulo contorneado distal arrastrando agua por su efecto
osmótico.
¿Cuáles son las indicaciones de los diuréticos osmóticos?
Hipertensión intracraneana, falla renal por choque hipovolémico e into-
xicaciones por fármacos.
¿Cómo actúan los diuréticos de techo alto?
Inhiben la bomba Na-K-Cl en la rama ascendente gruesa del asa de Henle.
¿Cuál es el cuadro clínico de la hipertensión renovascular?
 Inicio antes de los 20 y después de los 50 años.
 Refractariedad al tratamiento.
 Hipocalcemia.
 Soplo abdominal.
 Renina elevada, en especial después de una prueba con captopril.
¿Cuáles son las dos formas clínicas de la infección de vías urinarias (IVU)?
 Infección de las vías urinarias bajas: aumento de la frecuencia urina-
ria, disuria, dolor suprapúbico, orina con mal olor y turbia y urgencia
miccional. Estos síntomas, en cualquier combinación, se observan en
la uretritis, prostatitis y cistitis.
 Pielonefritis aguda: dolor en el flanco, fiebre, malestar y sintomatolo-
gía urinaria baja.
¿Para qué sirve la prueba de los nitritos en orina?
El microorganismo patógeno habituales de vías urinarias (Proteus) rompe la
urea formando nitritos; la presencia de éstos es indicativa de infección.
¿Cuál es el tratamiento de las infecciones urinarias no complicadas?
Amoxicilina: 3g VO durante tres días, o TMP-SMZ 320 mg/1.6g durante
tres días.

221
¿Cuál es el tratamiento de la pielonefritis y de la IVU recidivante?
Debe tratarse por catorce días con trimetoprim-sulfametoxazol (TMO-
SMZ) (80 mg/400 mg) dos veces al día o con una cefalosporina de pri-
mera generación.
¿Cuál es el tratamiento de la prostatitis?
Debe tratarse durante 14 días con medicamentos que permanezcan en el
tejido prostático, como el trimetoprim o la carbenicilina.
¿Por qué se justifica el uso de eritromicina en los casos de prostatitis no
bacteriana?
Porque como agentes etiológicos pueden estar implicados ureaplasmas,
micoplasmas y chlamydias que no se evidencian por los estudios con tin-
ción o cultivo habituales.
¿Por qué los antibióticos no alcanzan fácilmente la próstata?
Por el epitelio prostático especializado y porque el líquido prostático tie-
ne un pH ácido (6.4).
¿Qué medicamentos se usan para tratar la prostatodinia (espasmo vesical)?
Bloqueadores alfa como la terazocina (1 a 10 mg) o la doxazocina (1 a 8
mg).
¿Cuál es el tratamiento de la hiperplasia prostática benigna (HPB)?
La resección transuretral (RTU).
¿Cuáles son las 4 vías que integran el reflejo de micción?
 I Corteza-puente: promueven la continencia.
 II Puente-sacro: promueven la micción.
 III Sacro-vejiga: coordina el reflejo de micción (detrusión y relaja-
ción).
 IV Corteza-sacro: promueve la continencia.
¿Cómo se gobierna el reflejo de micción?
Lo controla el cerebro, pero se coordina en el puente.
¿Cuáles desórdenes pueden alterar el mecanismo de micción?
La hiperreflexia del detrusor, la arreflexia del detrusor y la disinergia de-
trusor-esfínter.
¿Quién inhibe al músculo detrusor?
La corteza cerebral.

222
¿Cuáles son los patrones de disfunción del vaciamiento (vejiga neurogénica)?
 Hiperreflexia del detrusor: vejiga neurogénica hipertónica secundaria
a lesiones suprasacras que causan una capacidad de almacenamiento
disminuida de la vejiga y pérdida de orina en decúbito.
 Arreflexia del detrusor: secundaria a lesiones de la cauda equina que
causan aumento de la capacidad de almacenamiento y retención uri-
naria.
 Disinergia del esfínter externo y del detrusor: produce contracción del
esfínter externo mientras se contrae el detrusor, lo que causa una obs-
trucción funcional.
¿Cuál es el tratamiento en los casos de vejiga neurogénica?
Anticolinérgicos que inhiben al detrusor, o bien, agonistas alfa que actúan
disminuyendo el espasmo prostático, cervical y uretral. También debe
existir un reentrenamiento vesical.
¿Cómo funcionan el cuello vesical y la uretra?
Tienen receptores alfa que median la contracción y receptores beta que
median la relajación.
¿Cuál es la causa más común de incontinencia en la mujer?
La disfunción del ángulo uretrovesical o cistocele. También puede haber
incontinencia si hay disfunción del esfínter o uretra hipermóvil.
¿Qué factores causan incontinencia caracterizada por urgencia?
La hiperreflexia del detrusor y la inestabilidad de la uretra.
¿Cuál es el cuadro clínico de la incontinencia femenina verdadera?
Hay pérdida de orina en cualquier caso en que la presión abdominal e in-
travesical rebasen a la presión de cierre del esfínter uretral; por ejemplo al
toser.
¿Cuál es el protocolo para estudiar la incontinencia femenina?
Primero se practica un examen general de orina para descartar infección.
Después se realizan las pruebas urodinámicas y luego, la cistoscopía.
¿Cuál es el tratamiento médico del cistocele?
Los ejercicios de Kegel dan buenos resultados en 75% de los casos; tam-
bién se utilizan anticolinérgicos para distender la vejiga y simpaticomi-
méticos para dar más tono al esfínter.

223
¿Cuál es el tratamiento quirúrgico del cistocele?
La uretropexia retropúbica, que consiste en elevar la vejiga y la uretra fiján-
dolas al pubis. También se utiliza la colporrafia anterior.
¿Cuáles estudios son los indicados en caso de trauma ureteral y renal?
Primero la urografía excretora y luego la TAC.
Menciona dos causas de trauma ureteral.
En primer lugar, las heridas penetrantes, y en segundo, la yatrogenia.
¿Cuál es la causa más frecuente de trauma vesical?
La fractura pélvica.
¿Cuáles son los signos clínicos del traumatismo uretral?
Sangre en el meato y próstata elevada.
¿Cómo se evalúan radiológicamente el trauma vesical y el uretral?
Con uretrografía retrógrada y con cistografia de llenado máximo, respec-
tivamente.
¿Cómo se hace el diagnóstico de ruptura testicular?
Con el cuadro clínico y el ultrasonido.
¿Cuáles son los tumores testiculares de tipo seminomatoso?
El clásico, anaplásico y espermatocítico.
¿Cuáles son los tumores testiculares no seminomatosos?
El carcinoma embrionario, teratoma, tumor del saco de Yolk y coriocar-
cinoma.
¿Qué marcadores sirven para hacer el diagnóstico de los tumores testicula-
res no seminomatosos?
La alfa feto proteína y la gonadotropina coriónica humana.
¿Cuál es el tratamiento de :
 Tumor testicular no seminomatoso: cirugía con exploración retrope-
ritoneal.
 Tumor testicular seminomatoso: radioterapia.
 Tumor testicular seminomatoso en etapa III y IV: cisplatino, etopósi-
do y bleomicina.
¿Cuándo se debe sospechar agenesia de las vesículas seminales?
Cuando hay disminución del volumen eyaculatorio y disminución en su
contenido de fructosa.

224
¿Cuál es la inervación simpática del pene?
Le dan inervación las raíces dorsales inferiores y lumbares superiores
(nervio hipogástrico).
¿Cuál es la inervación más importante para que ocurra erección?
La parasimpática.
¿Cuáles son los estudios de laboratorio útiles en los casos de impotencia?
Los niveles de testosterona y un estudio completo que permita descartar
cualquier enfermedad sistémica.
¿En qué etapa del sueño ocurre la erección?
En la del sueño REM.
¿Cómo se descarta una causa vascular de la impotencia?
Se descarta si se obtiene una buena respuesta al instilar papaverina, fento-
lamina o Prostaglandina E.
¿Cómo actúa el sildenafil?
Es un inhibidor selectivo de la fosfodiesterasa.
¿Cuál es el cuadro clínico de la aplasia de células germinales (células de
Leydig) y la obstrucción del conducto deferente?
Ambas se presentan con volumen eyaculatorio normal, contenido normal
de fructosa en el semen y azoospermia.
¿Cuánto tarda un espermatozoide en llegar del epidídimo al conducto defe-
rente?
12 a 21 días.

225
226
NEUMOLOGÍA

¿Dónde deben auscultarse los ruidos traqueales?


En el hueco supraesternal.
¿En qué consiste la respiración de Cheyne-Stokes?
La frecuencia respiratoria sube y baja, con periodos de apnea interme-
dios. Aparece en la insuficiencia cardiaca izquierda grave o en la enfer-
medad neurológica.
¿Cómo se calcula la presión alveolar de oxígeno (PAO2)?
Se necesita calcular primero la presión inspirada de oxígeno (PIO2) mediante
la fórmula PIO2=FIO2 x (presión atmosférica (760 mmHg)- pH2O).
Entonces PAO2= PIO2 – (PaO2/RQ).
RQ= Cociente respiratorio, que tiene un valor de 0.8
¿Para qué sirve la PAO2?
Calculando la PAO2 se puede calcular la diferencia alvéolo-arterial de O2.
Utilizando la diferencia alvéolo-arterial, la PaO2, la PaCO2 y la respuesta
a la administración de O2 al 100%, se puede diferenciar entre los meca-
nismos básicos de hipoxemia.
¿Cuál es la capacidad de difusión del CO2 (DLCO2) normal?
25 ml/min/mm Hg.
¿Cuál es la PaO2 que alcanza una persona normal al recibir una fracción
inspirada de oxígeno FIO2 al 100%?
Hasta 670 mmHg.
¿Cuáles son los mecanismos básicos de la hipoxemia?
 Disminución de la PIO2: PaO2 disminuida, PaCO2 normal o dismi-
nuida
 Hipoventilación: PIO2 disminuida, PaCO2 aumentada.

227
 Anormalidad en la difusión: PaO2 disminuida, PaCO2 normal o dis-
minuida, gradiente A-a aumentada, con respuesta al O2 al 100%.
 Incompatibilidad V/Q: PaO2 disminuida, PaCO2 normal o disminuida,
gradiente alvéolo-arterial aumentada, con respuesta al O2 al 100%.
 Derivación: PaO2 disminuida, PaCO2 normal o disminuida, gradien-
te alvéolo-arterial aumentada, sin respuesta al O2 al 100%.
¿Cuáles son las indicaciones de apoyo ventilatorio?
Frecuencia respiratoria mayor de 35x’, PaCO2 mayor de 55 y PaO2 me-
nor de 70.
¿Cuál es el nivel de hemoglobina insaturada con el que clínicamente se
presta la cianosis?
Más de 5g/dl.
¿Cómo se calcula si es correcta la relación aporte consumo de O2?
Midiendo la saturación de oxígeno de la sangre venosa mezclada.
¿Qué es la capacidad vital?
Es la suma de la reserva inspiratoria más la reserva espiratoria y el volu-
men corriente.
¿Qué tipos de espacio muerto existen?
 El espacio muerto anatómico es el aire de las vías aéreas altas que no
llega a los alveolos; aproximadamente sólo los primeros 350 ml de
cada 500 ml de aire inspirado comprenden la ventilación alvéolar.
 El espacio muerto total es la suma del espacio muerto anatómico más
el fisiológico. El espacio muerto fisiológico comprende todos aquellos
estados en los que la ventilación no se encuentra equilibrada con el
flujo sanguíneo (alvéolos no perfundidos).
En condiciones normales ambos espacios muertos son equivalentes. Esta
equivalencia se pierde cuando aparecen estados patológicos como los al-
véolos sobreventilados o hipopoerfundidos.
¿Cómo se calcula el espacio muerto anatómico?
Mediante curvas de inhalación de nitrógeno, o bien, calculando la dife-
rencia entre la ventilación total minuto y la ventilación alveolar minuto.
¿Cómo puede calcularse el espacio muerto total?
Mediante la ecuación de Bohr, la cual indica que el volumen ventilatorio
(VT) multiplicado por la PCO2 del gas espirado equivale a la PCO2 arte-

228
rial multiplicada por la diferencia entre el volumen ventilatorio y el espa-
cio muerto (VD) más la PCO2 del aire inspirado, más la PCO2 del aire es-
pirado, multiplicado por el VD; es decir, PECO2 x VT= PaCO2 x (VT- VD)
+ PICO2 x VD.
En la ecuación el resultado de PICO2 + VD es despreciable, por tanto,
puede ignorarse.
¿Cuáles son los signos de hipoxemia intensa?
Taquicardia, taquipnea, confusión, ansiedad, cianosis, hipertensión,
arritmias y temblores.
¿Cuáles son los signos de hipercapnia?
Disnea, cefalea, papiledema y asterixis.
¿Cómo es la relación FEV1/FVC (volumen espiratorio forzado del primer
segundo/capacidad vital forzada) en la obstrucción y la restricción?
En la primera menos de 70% en la segunda más de 70%.
¿Cuáles son los datos de la espirometría en los casos de obstrucción de las
vías aéreas superiores?
Disminución del volumen tanto a la inspiración como a la espiración.
¿Cuál es el indicador principal de supervivencia cuando existe enfermedad
pulmonar?
El volumen espiratorio forzado del primer segundo (FEV1).
¿Para qué es útil la determinación de la capacidad pulmonar total (TLC) y
de la capacidad vital (VC)?
Para detectar patrón restrictivo.
¿Cuál es el diagnóstico diferencial del neumotórax espontáneo?
El infarto agudo del miocardio, la tromboembolia pulmonar, la neumo-
nía, la EPOC y el asma.
¿Cuáles son los cinco tipos de derrame pleural?
Empiema, hemotórax, quilotórax, trasudado y exudado.
¿Cuáles son los tres mecanismos fisiopatológicos del exudado?
El aumento de la presión hidrostática, la disminución de la presión oncó-
tica y el aumento de la presión negativa intrapleural.

229
¿Con que datos se clasifica al derrame pleural como exudado?
Variable Total empiema Relación derrame/suero Observaciones
Proteínas Más de 2.9 0.5
DHL Más de 250 0.6
Colesterol Más de 45 0.3
Glucosa Más de 60 1 Se encuentra en todos
Glucosa Menos de 60 0.5 TB, malignidad, AR, LES
Amilasa Más de 2 LES, pancreatitis
Amilasa Menos de 1 Todas
Si los valores son menores de éstos, se trata de un trasudado

¿Qué síndrome corresponde a la neumonía lobar?


El síndrome de condensación pulmonar.
¿Qué síndrome corresponde a la neumonía intersticial o bronquiolitis?
El síndrome de rarefacción pulmonar.
¿Qué síndrome corresponde a la bronconeumonía o neumonía lobulillar?
Los estertores difusos, que no forman un síndrome.
¿Cuáles son los agentes etiológicos de la neumonía en los pacientes mayo-
res de 60 años inmunocompetentes?
S. pneumoniae, virus, H. influenzae, aerobios Gram negativos y S. aureus.
¿Cuáles son las neumonías por grupo de edad, su agente etiológico y su
tratamiento?
Neumonía/Edad Etiología Tratamiento
Virus: citomegalovirus y herpes
Amplicilina + gentamicina
simple
Neonatal o vanco + gentamicina
Bacterias: EBHGA coliformes
(penicilinorresistente)
Otras: chlamydia
Chlamydia y virus sincicial respira- Eritomicina 10 mg/kg
1-3 meses
torio
Virus sincicial respiratorio, S.
1-24 meses Pneumoniae, H. influenzae, Chlamy- Cefuroxima 50 mg/kg IV
dia, Mycoplasma
3 meses - 5 años Los mismos, sin Haemophilus Eritomicina, claritromicina
5-18 años Micoplasma y virus Claritromicina, azitromi-
cina

230
Viral en adultos Influenza, parainfluenza, Virus Zanamivir 10 mg inhala-
sincicial respiratorio do, amantadina 100 mg
Sin comorbilidad: micoplasma,
chlamydia, viral raramente S.
Pneumonias
Fumador: Spneumonias, H. in-
Adultos mayores Azitromicina 500 mg
fluenza, Moraxella
de 8 años, Adquie- claritromicina 500 mg
Alcohólico: S pneumoniae, anae-
ra en la comuni- eritromicina 1 gr
robios, coliformes
dad fluoroquinolonas
Aspiración: anaerobios
Drogas IV: aureus
Pájaros: psitacosis
Conejos: tularemia
Falla orgánica: coliformes Para cualquier coliformes,
Ventilación mecánica: coliformes,S. Pneumoniae y anaero-
Hospitalaria no pseudomonas, S. Aureus bois: imipenem/cilastatina
neutropénica Obstrucción: anaerobios 500 mg o meropenem 1 g o
Esteroides crónicos: cándida piperacilina/tazobactam +
ciprofluoxacino
Historia neutropé- Cualquiera de las adquiridas en el Seguir los criterios para
nica hospital y aspergillus o cándida neutropenia y fiebre

¿Cuál es el agente etiológico más frecuente de la neumonía en los pacientes


neutropénicos?
Lysteria monocitogenes.
¿Cuáles son les tres agentes de la neumonía lobar?
S. pneumoniae, Klebsiella pneumoniae, H. influenzae.
¿Cuál es el tratamiento de la neumonía por anaerobios?
 Penicilina sódica cristalina: 1 a 2 millones de U. Cada 6 horas.
 Clindamicina: 600 miligramos IV cada 6 horas hasta mejorar, y luego
300mg.
¿Cuáles son los datos clínicos de la neumonía intersticial?
Crepitación, patrón obstructivo (FEV1/FVC menor de 70%) e imagen de
“vidrio despulido”.

231
¿Cuáles son los criterios de mal pronóstico en los pacientes que padecen
neumonía?
Edad mayor de 65 años, más de 30,000 leucocitos, hipotensión, tempera-
tura mayor de 38.3° C y que esté casado por estafilococo, neumococo
Legionella.
¿Cuáles son los agentes causales de la bronquiolitis obliterante (BOOP)?
El virus sincicial respiratorio y Mycoplasma pneumoniae.
¿Cuál es el pH del empiema?
Menor a 7.4
¿Cuál es la epidemiología del absceso pulmonar?
Ocurre principalmente sujetos propensos a la aspiración (cualquiera que
curse con estado de conciencia alterado).
¿Cuál es la imagen radiológica del absceso pulmonar?
La cavidad solitaria de pared gruesa con consolidación periférica y nivel
hidroaéreo.
¿Cuáles son los segmentos pulmonares más afectados por el absceso pul-
monar y la neumonía por aspiración?
Los segmentos posteriores de los lóbulos superiores y el superior y basal
de los inferiores.
¿Cuáles son las enfermedades cavitarias pulmonares?
La granulomatosis de Wegener, la tuberculosis, la micosis, el cáncer y el
infarto pulmonar.
¿Cuál es la imagen radiológica de la neumonía necrosante?
Múltiples cavidades dentro de un área de condensación.
¿Qué drogas causan fibrosis pulmonar?
El busulfán, la bleomicina, la nitrofurantoína, la ciclofosfamida, las sales
de oro y el metotrexate.
¿Qué otros nombres recibe la neumonitis por hipersensibilidad?
Alveolitis alérgica extrínseca o respuesta pulmonar al polvo orgánico. Es
una reacción de hipersensibilidad tipo III.
¿A qué se le llama neumonía eosinofílica (Síndrome de Löeffler)?
Es una neumonía por hipersensibilidad en la que hay un infiltrado eo-
sinofílico pulmonar y eosinofilia periférica. Los agentes casuales pueden

232
ser parásitos (filarias, nematelmintos), fármacos (hidralazina, nitrofuran-
toína) y hongos (Aspergillus), aunque la mayoría son idiopáticas.
¿Cuáles son los cambios histopatológicos producidos por el asma?
Denudación del epitelio, depósitos de colágena bajo la membrana basal,
edema de las vías respiratorias e hipertrofia del músculo y de las glándu-
las bronquiales.
¿Cuáles son dos signos de obstrucción respiratoria en los casos de asma?
La sibilancias y la fase espiratoria prolongada.
¿Cuál es el asma de los niños y de los adultos?
El asma alérgico y el intrínseco, respectivamente.
¿Cuáles son los signos de laboratorio en los casos de asma?
Alcalosis respiratoria y aumento de la A-a DO2.
¿Qué se observa en el análisis del esputo de un asmático?
Espirales de Crushman y cristales de Leyden.
¿En qué se debe pensar un paciente joven tiene signos y síntomas de asma
después de la actividad física?
Se debe sospechar que el paciente tiene asma inducida por el ejercicio. El
diagnóstico se hace con una prueba terapéutica positiva con salbutamol.
¿Cuál es el tratamiento de elección para el asma inducida por el ejercicio?
90% de los pacientes responde a los agonistas beta-2 inhalados. Una se-
gunda opción es la terapéutica combinada de cromolín sódico y bromuro
de ipratropio.
¿Cuál es el flujo espiratorio medio (FEM) en una exacerbación moderada
de asma?
50 a 80.
¿Qué fármacos aceleran la depuración de teofilina?
La rifampicina, la fenitoína, los barbitúricos y la nicotina. Todos aumen-
tan la actividad del citocromo P-450; por tanto, aceleran la depuración de
cualquier fármaco cuya vía de eliminación sea principalmente la hepática.
¿Qué fármacos disminuyen la depuración de teofilina?
La cimetidina, los macrólidos, las quinolonas y los anticonceptivos ora-
les. Todos disminuyen la actividad del citocromo P-450.

233
¿Cómo actúa la teofilina?
Inhibe la acción de la adenosina que se libera como segundo mensajero
para causar broncoconstricción. Sólo se usa en el tratamiento crónico, ya
que fácilmente causa convulsiones y arritmias.
¿En quiénes se contraindica la teofilina?
En pacientes con prostatismo, ya que puede causar un efecto alfa que
aumente los síntomas.
¿Cuál es la dosis de salbutamol para nebulizar?
2.5 mg
¿Cuál es la dosis de prednisona para tratar los casos de exacerbación asmá-
tica?
0.5-1 mg/kg/día
¿Qué fármacos son mucolíticos?
La acetilcisteína, la alfa-dornasa y el yoduro potásico.
¿Qué son los leucotrienos?
Son los productos de la vía de la 5-lipooxigenasa del metabolismo del áci-
do araquidónico. La 5-lipooxigenasa se encuentra en todas las células de la
serie mieloide y es un potente quimiotáctico de neutrófilos y eosinófilos.
¿Cuáles son los efectos de los leucotrienos en el árbol bronquial?
Promueven la constricción bronquial, aumentan la permeabilidad endo-
telial y promueven la secreción de moco.
¿Cuáles son los inhibidores de los leucotrienos?
Zafirlukast y montelukast.
¿Qué es el zileutón?
Es un bloqueador de la 5-lipooxigenasa.
¿Cuándo se usan el zileutón y el montelukast?
Sólo se usan para la profilaxis del asma; no tienen utilidad en el ataque
agudo. Con ellos logra una disminución marcada en el uso de beta ago-
nistas, disminución de los síntomas y aumento del flujo espiratorio pico
matutino.
¿Cuándo está presente el EPOC Tipo A?
Es un paciente disneico, soplador rosado, enfisematoso, menor de 60
años, con CO2 bajos.

234
¿Qué es el EPOC Tipo B?
En un paciente tosedor, azul abotagado, bronquítico crónico, menor de
60 años, con O2 muy bajo y CO2 lato.
¿Cómo se define clínicamente la bronquitis crónica?
Es una neumopatía con eventos de tos productiva durante 2 a 3 meses,
cuando menos, en dos años consecutivos.
¿Cuáles son dos signos radiológicos patognomónicos de enfisema pulmo-
nar?
Bulas parenquimatosas y vejigas subpleurales.
¿En quién se debe sospechar la presencia de fibrosis quística?
En pacientes jóvenes con EPOC, bronquitis crónica, insuficiencia pan-
creática (o pancreatitis crónica) e infertilidad.
¿Cuáles son los datos de la espirometría en los pacientes que padecen
EPOC?
Hay un aumento del volumen residual y un volumen pulmonar total
normal o elevado.
¿Con qué parámetro se diagnostica EPOC grave?
Con un FEV1 menor de 1L.
¿Qué es la fibrosis quística?
También conocida como mucoviscidosis, es un trastorno que afecta al
brazo largo del C17 (banda q31). Es la mas grave de las enfermedades
genéticas mortales que afecta a las glándulas exocrinas principalmente
del páncreas y del aparato respiratorio, y se presenta con la tríada:
EPOC, insuficiencia pancreática exocrina y cifras anormalmente altas de
electrolitos en el sudor.
¿Qué sugiere un predominio de linfocitos T en el lavado bronquioalveolar?
Sarcoidosis o neumonitis por hipersensibilidad.
¿Qué se encuentra en un lavado bronquioalveolar en los casos de fibrosis
pulmonar?
Predominio de neutrófilos, eosinófilos y macrófagos.
¿Cuál es la imagen radiológica de la enfermedad intersticial pulmonar?
Infiltrados en “vidrio molido”, retículonodulares y en “panal de abejas”.

235
¿Cuáles son los hallazgos en la espirometría de quienes padecen enferme-
dades restrictivas?
Disminución del volumen residual y disminución del volumen pulmonar
total.
¿Qué es la sarcoidosis?
Es una enfemedad granulomatosa, no caseificante, sistémica, con infil-
trado pulmonar, linfadenopatía mediastinal, lesiones cutáneas, lesiones
oculares, hipercalciuria, hipercalcemia, parálisis de Bell, eritema nodoso,
uveítis y arritmias.
¿Cómo se diagnostica la sarcoidosis?
Por biopsia de las lesiones o biopsia transbronquial.
¿Qué es la granulomatosis de Wegener?
Es una enfermedad que cursa con necrosis de pequeños vasos y con gra-
nulomas no caseificantes alrededor, que afecta más a las vías respiratorias
superiores y que cursa también con vasculitis de pequeños vasos.
¿Qué se usa para hacer el diagnóstico de granulomatosis de Wegener?
c-ANCA; tiene una especificidad de 90%.
¿Qué es el síndrome de Churg-Strauss?
Es una vasculitis idiopática de arterias de pequeño y mediano calibre,
que ocurre en pacientes con asma.
¿Cuál es el tratamiento de la granulomatosis de Wegener y del síndrome de
Churg-Strauss?
1 mg/kg de prednisona más 2 mg/kg de ciclofosfamida.
¿Qué es el síndrome de Goodpasture?
Es la enfermedad riñón-pulmón causada por la presencia de anticuerpos
antimembrana basal. Causa depósitos lineales de anticuerpos IgG en los
glomérulos.
¿Con qué otro nombre se conoce a la histiocitosis X?
Granuloma eosinofílico o granulomatosis de células de Langerhans (del
pulmón y del hueso).
¿A qué corresponden las masas del mediastino superior?
Timoma, bocio extratorácico y enfermedad de Hodgkin.
¿A qué corresponden las masas del mediastino anterior?
Quiste dermoide, quiste pleuropericárdico, quiste broncogénico y redu-
plicación esofágica.

236
¿A qué corresponden las masas del mediastino posterior?
Neurofibroma y feocromocitoma (con síndrome de Horner).
¿Qué es la osteoartropatía pulmonar hipertrófica y con qué se le relaciona?
Es un síndrome con artralgias de las mano, los tobillos, las muñecas y las
rodillas asociado a mesotelioma. Hay que diferenciarlo de la osteroartropa-
tía hipertrófica que se encuentra en la EPOC (dedos en palillo de tambor).
¿Cuáles son los factores de riesgo relacionados con la tromboembolia pul-
monar?
Estasis venosa, estados de hipercoagulabilidad, obesidad y fractura re-
ciente de la cadera.
¿Cuál es la terapéutica profiláctica para pacientes en riesgo de sufrir TEP?
5,000 U de heparina no fraccionada cada 8 horas. Se busca idealmente
lograr un tiempo de tromboplastina parcial de 31.5 a 36.
¿Cuál es el cuadro clínico de la tromboembolia pulmonar?
Tórax rígido, dolor pleurítico, hemoptisis, broncoespasmo, fricción pleu-
ral, ritmo de galope, desdoblamiento fijo del segundo ruido y cianosis.
¿Cuál es la imagen típica radiológica en los casos de TEP?
Arteria pulmonar abultada y segmento oligohémico.
¿A qué se llama joroba de Hampton?
10% de los casos de tromboembolia pulmonar evoluciona a infarto pul-
monar que cursa con hemoptisis, dolor pleurítico y fiebre de bajo grado.
La joroba de Hampton es un signo radiográfico que sugiere infarto pul-
monar, y consiste en una consolidación densa y homogénea en forma de
cuña, de base orientada hacia la pleura y punta en forma convexa orien-
tada hacia el hilio, lo que le confiere una forma de joroba.
¿Qué se observa en el electrocardiograma practicado en pacientes con TEP
(patrón de esfuerzo del ventrículo derecho)?
 Eje a la derecha.
 S anormal en D-I.
 Q anormal en D-III.
 P pulmonar.
 Bloqueo de la rama derecha del Haz de His.
 T invertida en V-1 y V-3.

237
¿Qué cambios gasométricos se observan en los pacientes con TEP?
La TEP causa hipoventilación, por lo que inicialmente la PaCO2 está
disminuida y el pH se eleva de manera compensatoria provocando alca-
losis respiratoria aguda. También existe hipoxia (PaO2 menor de 80%) y
una diferencia alvéolo-arterial aumentada.
¿Qué es el dímero-D?
Es un producto de la degradación de la fibrina que se utiliza para apoyar
el diagnóstico de TEP cuando es mayor a 500 ng/ml.
¿Qué condiciones predisponen a la tromboembolia pulmonar?
Cirugía abdominal baja, cirugía del territorio íleofemoral, y en pacientes
con infarto del miocardio, enfermedad vascular cerebral, cáncer o con fa-
lla cardiaca, postrados por tiempo prolongado.
¿Cuál es la imagen típica del infarto pulmonar?
Imagen en cuña.
¿Cuál es la presión normal de la circulación pulmonar?
25/8 mmHg
¿Cuál es el estímulo más importante para la vasoconstricción pulmonar?
La hipoxia.
¿Cuáles son los signos de hipertensión pulmonar?
Desdoblamiento del segundo ruido, reforzamiento pulmonar y chasquido
sistólico de expulsión (P2 fijo).
¿Cuál es el diagnóstico diferencial de la hipertensión pulmonar?
Hipertensión pulmonar poscapilar, cortocircuito intracardiaco, TEP y ci-
rrosis hepática (por el secuestro de líquidos).
¿Cuál es el tratamiento de la hipertensión pulmonar primaria?
Nifedipino, diltiacem y prostaciclina.
¿Qué es el SIRPA?
Es el síndrome de insuficiencia respiratoria progresiva del adulto. Presen-
ta inicio súbito de disnea, hipoxemia e infiltrado en alas de mariposa
(con PCP <18) 12 a 18 horas después del evento desencadenante.
¿Cuáles son los factores de riesgo relacionados con el SIRPA?
Sepsis, contusión pulmonar, choque, pseudoahogamiento y múltiples
transfusiones.

238
¿Cuáles son las alteraciones gasométricas de los pacientes con hipoventila-
ción crónica?
La obesidad mórbida es una causa de hipoventilación crónica. Se observa
PaCO2 aumentada con pH normal.
¿Qué es el síndrome de Pickwick?
Es el síndrome en el que hay obesidad mórbida con hipoventilación y
cambios gasométricos acordes. También hay somnolencia, apetito exce-
sivo y policitemia.
¿Cuál es la terapia médica para la fibrosis pulmonar?
D-penicilamina o ciclofosfamida más prednisona.

239
240
NEUROLOGÍA

¿Dónde se encuentra los estriosomas?


En los ganglios basales.
¿Dónde se encuentran las neuronas piramidales?
En la corteza y en el hipocampo.
¿Dónde se encuentra las células de Renshaw?
En la sustancia gris.
¿Cuáles son las capas del cerebelo?
Piamadre, molecular, de Purkinje y granulosa.
¿Cómo se diferencia una disfunción parasimpática de un estado hipersim-
pático?
En los dos hay taquipnea, taquicardia y midriasis. El estado hipersimpá-
tico se caracteriza por diaforesis marcada, a diferencia de la disfunción
parasimpática.
¿Cómo se compone la vía aferente del reflejo pupilar?
Comienza en las células ganglionares de la retina, sigue el curso del ner-
vio óptico hasta el núcleo pretectorial del mesencéfalo; se decuza en la
comisura posterior y llega al núcleo de Edinger-Westphal.
¿Cuál núcleo media la constricción pupilar?
El núcleo de Edinger-Westphal (núcleo parasimpático preganglionar del
tercer par).
¿Cuál es la vía de la dilatación pupilar?
Está compuesta por tres neuronas.
 La de primer orden sale del hipotálamo, discurre por la porción cer-
vical de la médula y hace sinapsis preganglionar ipsilateral a nivel de
T1-T2.

241
 La de segundo orden parte de T1-T2 siguiendo el tronco simpático
hasta el ganglio cervical superior.
 La neurona de tercer orden sigue el trayecto de la carótida para llegar
finalmente a la órbita e inervar el músculo liso del iris.
¿Qué es el síndrome de Horner?
Es un síndrome provocado por la interrupción de la vía simpática del ojo
en cualquiera de sus puntos, lo que produce ptosis, miosis y anhidrosis.
¿Qué signo clínico provoca el daño al núcleo de Edinger Westphal?
Pupilas grandes no reactivas. Generalmente traduce una lesión pontina
de mal pronóstico.
¿Qué signo clínico provoca el daño en la vía que va del ojo al hipotálamo?
Provoca pupilas pequeñas reactivas.
¿Qué signo provoca el daño al puente o al VI par?
Incapacidad para la abducción del ojo.
¿Qué signos provoca el daño en el fascículo longitudinal medial o en el III
par?
Incapacidad para la aducción del ojo y desviación de la mirada hacia
afuera y abajo.
¿Cuáles son las vías somatosensoriales de la cara?
 El núcleo trigéminoespinal.
 El núcleo ventral medial posterior del tálamo (ispsilateral).
 La rama posterior de la cápsula interna (contralateral).
¿Cuál es el núcleo propioceptivo de la cara?
El sensorial principal del trigémino.
¿Cuál es el núcleo de la sensación táctil de la cara?
El mesencefálico del trigémino.
¿Cuáles tipos existen de parálisis facial?
 La parálisis facial central puede ser unilateral o bilateral y respeta los
músculos inervados por la rama superior del facial (frente y orbicular
de los párpados), que tiene una inervación sinérgica por ambos he-
misferios cerebrales.
 La parálisis facial periférica siempre es unilateral y total. Esto quiere
decir que afecta los músculos inervados tanto por la rama témporofa-
cial como por la cérvicofacial.

242
¿Qué es la vía piramidal?
Es la vía córticoespinal, que parte del área cuatro de Broadman en la cor-
teza cerebral y pasa por la cápsula interna, los pedúnculos, el tallo cere-
bral y entra en la médula donde es relevada por una neurona de segundo
orden, que es la que tiene contacto con el músculo que finalmente va a
inervar. En resumen, es la vía motora voluntaria.
¿Cómo se integra el síndrome piramidal?
 Pérdida de la motilidad voluntaria.
 Anomalías de la motilidad refleja, como la exageración de los reflejos
profundos o clono; presencia de reflejos especiales, como el de Ba-
binski, y aparición de movimientos sincinéticos.
 Contractura piramidal después de algunos días; primero existe flaccidez.
 Puede haber anomalías de la sensibilidad.
¿Qué se observa en el síndrome de neurona motora inferior?
Fasciculaciones y reflejos disminuidos.
¿Cuáles son los síndromes en los que se observa hemiplejía?
 Hemiplejía cortical, faciobraquial, ipsilateral a la lesión.
 Hemiplejía capsular, contralateral total.
 Hemiplejía alterna superior, peduncular o síndrome de Weber
 Hemiplejía alterna media, protuberancial o síndrome de Millard-
Gubler
 Hemiplejía alterna inferior, bulbar lateral o síndrome de Wallemberg.
¿Qué es el síndrome de Weber?
Es un síndrome secundario a una lesión de la vía córticoespinal a nivel de
los pedúnculos, que presenta alteración del tercer par craneal ipsilateral a
la lesión (globo ocular dirigido hacia abajo y afuera, ptosis y midriasis) y
hemiplejía cruzada de los miembros torácico y pélvico.
¿Qué es el síndrome de Millard-Gubler?
Es un síndrome secundario a una lesión en la vía córticoespinal a nivel de
la protuberancia, que causa parálisis facial total de tipo periférico, ipsila-
teral a la lesión, y hemiplejía del lado opuesto.
¿Cómo se presenta el síndrome de Wallemberg izquierdo?
 Afecta el hipogloso izquierdo y provoca desviación de la lengua hacia
la izquierda y parálisis del la izquierdo del velo del paladar.

243
 Afecta la pirámide izquierda provocando hemiplejía derecha.
 Afecta al lemnisco medial izquierdo y provoca disminución de la
propiocepción y el tacto del lado derecho.
 Afecta el núcleo vestibular y causa pérdida del equilibrio, nistagmo y
vértigo y trastornos auditivos.
¿Cómo se encuentra el ojo cuando hay parálisis del tercer par craneal (mo-
tor ocular común?
Divergente y deprimido.
¿Qué se debe sospechar cuando hay parálisis del cuarto par?
Aneurisma de la comunicante posterior.
¿Cómo se comprueba la integridad del cuarto par?
Existe rotación interna al intentar la mirada hacia abajo.
¿Cuál es el cuadro clínico de la parálisis del cuarto par?
Diplopia vertical (al mirar hacia abajo) y desviación del ojo hacia arriba.
¿Cuál es el cuadro clínico de la parálisis del VI par (motor ocular externo)?
Estrabismo convergente sin abducción.
¿A qué se le llama vía extrapiramidal?
A la vía motora de los ganglios basales.
¿A qué se le llama vía piramidal?
A la vía córticoespinal o neurona motora superior
¿Cuáles son los signos para evaluar la vía piramidal?
Los signos de Babinsky, Openheimer, Scheafer y Bing.
¿Cuáles son los síntomas del tallo?
Diplopia, disartria y vértigo.
¿Cuál es el síntoma extrapiramidal más común?
La acatisia. Se caracteriza por imposibilidad para permanecer quieto; hay
movimientos y marcha constantes
¿Cuáles son las manifestaciones focales de una lesión frontal?
 Hemiplejia progresiva, y convulsiones focales o generalizadas,
 Alteraciones mentales (falta de atención y pérdida de la motivación)
y marcha atáxica si se afecta más el cuerpo calloso.
 Si la lesión se localiza en la base de los lóbulos, puede haber anosmia.
 Existe afasia de expresión si se afecta el hemisferio dominante.

244
¿Cuáles son las manifestaciones focales de una lesión parietal?
 Se alteran la estereognosia y la discriminación entre dos puntos de
manera contralateral a la lesión.
 Es característica la negación de la enfermedad (anosognosia)
 Cuando está afectado el hemisferio dominante aparecen agnosia de
los dedos, agrafia y alteraciones del habla.
¿Cuáles son las manifestaciones focales de una lesión en el lóbulo temporal?
 La lesión en el hemisferio no dominante es silente.
 La lesión temporal profunda causa hemianopsia contralateral.
 Puede haber crisis convulsivas con auras olfatorias y visuales complejas.
 Los tumores superficiales causan afasia mixta.
¿Cuáles son las manifestaciones focales de una lesión en el lóbulo occipital?
 Defecto visual en el cuadrante contralateral o hemianopsia con pre-
servación de la mácula.
 Las crisis convulsivas van precedidas de auras centelleantes.
¿Cuáles son las manifestaciones focales de las lesiones subcorticales?
 Cápsula interna: hemiplejía contralateral.
 Tálamo: deterioro sensitivo cutáneo contralateral.
 Ganglios basales: atetosis, temblores extraños o posturas distónica.
(parkinsonismo).
¿Cuál es la diferencia entre una lesión del putamen y una del territorio de
la carótida?
Ambas cursan con hemiparesia y hemianestesia contralaterales. En las
lesiones del putamen hay disminución del estado de conciencia, y en las
del territorio de la carótida hay afasia.
¿Qué es el sistema córticobulbar?
Es el sistema que relaciona las funciones voluntarias con las raíces de los
nervios craneales que se encuentran en el puente y el bulbo. La rama pos-
terior de la cápsula interna es parte de esta vía.
¿Qué ocasiona una lesión extensa de la rama posterior de la cápsula inter-
na (vía corticobulbar)?
Parálisis facial central contralateral, hemiplejía espasmódica contralate-
ral, desviación contralateral de la lengua, hipertonía e hiperreflexia con-
tralateral.

245
¿Cuál es el síndrome de la carótida primitiva?
Amaurosis, ambliopía, hemiplejía y falta de puso maxilar o soplo a ese
nivel.
¿Qué signos se encuentran en el síndrome de la cerebral anterior?
 Cuando se afecta la izquierda, hay afasia y apraxia.
 Si se afecta a nivel del origen hay hemiplejía de la pierna y pie opuestos.
 La afeción a nivel de la arteria de Heubner causa parálisis facial, de la
lengua y del miembro superior opuesto.
 Si se afecta a nivel distal de la arteria de Heubner se presenta el refle-
jo de prensión palmar.
 Existe abulia o mutismo acinético.
¿Qué signos constituyen el síndrome de la cerebral media?
 Parálisis cortical, que afecta la hemicara y el brazo contralaterales.
 Afasia central (síndrome de Gertsman), que aparece si se afecta el
hemisferio dominante y cursa con agnosia de los dedos, confusión
izquierda-derecha, alexia y acalculia.
 Hemiplejía capsular, si ocurre a nivel de la cápsula interna. La hemi-
plejía es contralateral total.
 Si se afecta la zona lentículoestriada hay hemiplejía y hemianestesia y
no hay afasia ni hemianopsia.
 Hemiplejía braquial con afasia, si se afecta la zona silviana.
¿Qué es la afasia?
Es la dificultad para hablar, escribir y leer, es decir, el defecto o la pérdi-
da de todas las funciones del lenguaje.
¿Qué es la afasia no fluida de Broca?
Es la afasia que ocurre por daño en el área frontal del cerebro, en torno a
la cisura de Silvio, con habla telegráfica y comprensión intacta.
¿Qué es la afasia fluida de Wernicke?
Es la afasia que ocurre por daño a la parte posterior del cerebro, en torno
al lóbulo temporal, con habla fluida pero sin comprensión de lo que se
dice o se escucha.
¿Qué se debe investigar en un paciente que sufre síncope (desvanecimiento)?
 Si el desvanecimiento es de origen vestibular o de origen vascular
(vahído o presíncope).

246
 Cuando es de origen hemodinámico se acompaña de una respuesta
autonómica como puede ser la palidez de tegumentos y la diaforesis
con piel fría.
 Si es vestibular, hay que diferenciar si es central (acompañado por di-
plopia, disartria, disfagia y vértigo, que son síntomas del tallo) o peri-
férico (sólo se acompaña de tinnitus y pérdida de la audición), o bien,
si es de origen cardiaco.
¿Cómo se clasifican los TCE?
 Leve (menor): no hay pérdida de la conciencia, puede haber dolor de
cabeza, desmayo, vómito en una ocasión, no se encuentra focalizado,
hay dificultad en la concentración. Se debe a una concusión y no hay
datos de hematoma en la TAC.
 Moderado (intermedio): el paciente no se encuentra comatoso pero sí en
estado confusional. Se debe a una contusión y hay hematoma en la
TAC. Puede haber pérdida de memoria, abulia, focalización, confusión,
vómito repetido, diabetes insípida. Se trata con glucocorticoides.
 Grave: el paciente se encuentra en estado comatoso, debe sospechar-
se lesión cervical asociada y hay que inmovilizar.
¿Qué es una concusión?
Es cuando un golpe causa movimiento del cerebro dentro del cráneo.
¿Qué es una contusión?
Es la desaceleración del cerebro contra el cráneo, ya sea en el punto de
impacto o en el opuesto. Puede verse como una hiperdensidad heterogé-
nea y hemorragia en la TAC.
¿Qué déficits neurológicos se producen tras una fractura del esfenoides?
Se provoca sección del nervio óptico con ceguera total del lado afectado
y pupilas no reactiva. El reflejo consensual está conservado.
¿Qué nervios se lesionan en la fractura de base de cráneo?
I, II, III y IV.
¿Cuál es el cuadro clínico del hematoma subdural y epidural?
Dolor de cabeza unilateral, pupila agrandada ipsilateral estupor o coma.
Aparece en minutos u horas. El hematoma epidural produce déficit neu-
rológico y estado comatoso de manera rápida. Los signos pupilares y el
estupor son más frecuentes en el subdural.

247
¿Cómo se presentan el hematoma subdural y el hematoma epidural en los
ancianos y en los alcohólicos?
De manera subaguda durante días o semanas con somnolencia, dolor de
cabeza, confusión y hemiparesia leve.
¿Cuál es el comportamiento típico del hematoma epidural?
Existe un intervalo lúcido de horas o minutos antes de caer en coma.
¿Cuál es la característica tomográfica de un hematoma epidural?
Que la hemorragia se observa en forma lenticular.
¿Qué es la presión de perfusión cerebral y cual es el valor normal?
Es la diferencia entre la presión sistémica y la intracerebral y debe ser <de
20 a 30.
¿A qué fractura se asocia el hematoma epidural?
A la del temporal.
¿Cuál es la causa principal de hemorragia subaracnoidea?
El aneurisma saculado que generalmente se encuentra en la comunicante
anterior, en la cerebral media o en alguna de sus ramas.
¿Cuál es la diferencia clínica entre la hemorragia subaracnoidea y la intra-
cerebral?
 Las hemorragias parenquimatosas tienen signos de focalización im-
portantes; las subaracnoideas no.
 La hemorragia intraparenquimatosa tiene como antecedente la hiper-
tensión y la subaracnoidea las malformaciones arteriovenosas.
¿Cuáles son los estímulos que provocan vasoconstricción y vasodilatación
cerebral?
El aumento de la presión artirial provoca vasoconstricción cerebral,
mientras que el aumento de CO2 y la disminución de O2 causan vasodila-
tación.
¿Cuáles son las medidas para disminuir la presión intracraneana?
No se utiliza restricción de líquidos; debe mantenerse la normocapnia, ya
que la hipercapnia (y la vasodilatación subsecuente) aumenta la presión y
habrá que evaluar la necesidad de practicar una ventriculostomía.
¿Qué es un síndrome de herniación?
Es el que se provoca por el paso de tejido cerebral a través de aberturas
intracranales fijas como la hoz del cerebelo.

248
¿Qué tipos existen de herniación transtentorial?
Los signos de ambos se deben al que el tejido que protruye comprime es-
tructuras vitales del tallo.
 Herniación central: de manera bilateral, los lóbulos temporales pro-
truyen a través de la escotadura tentorial. Hay pupilas fijas en posi-
ción media, alteraciones de la respiración, pérdida de los reflejos
oculoencefálicos (rotación de la cabeza) y oculovestibulares (estimu-
lación calórica) y parálisis motora bilateral (rigidez de descerebración
o flacidez).
 Herniación temporal: de manera asimétrica, el tejido de uno de los
lóbulos temporales protruye. Existe parálisis precoz del tercer par
(pupila dilatada y fija, unilateral, y parálisis extraocular). En algunas
ocasiones puede haber paro cardiorrespiratorio brusco.
¿Cuáles son los signos clínicos cuando el tallo cerebral está intacto?
 Respuesta pupilar normal.
 Ojos de muñeca.
 Respuesta óculovestibular normal.
 Reflejo corneal normal.
¿Cómo se evalúa clínicamente una prueba con agua fría en el oído izquierdo?
Un paciente normal presenta movimientos oculares de componente rápi-
do hacia la derecha y componente lento hacia la izquierda
¿Qué se debe pensar si al instilar agua caliente en un oído no se obtiene
respuesta?
Se debe presumir una lesión en el VII par, o en el laberinto, causada por
traumatismo o por Schwanoma del acústico.
¿Cuáles son los criterios para hacer el diagnóstico de muerte cerebral?
 Pupila midriática no reactiva.
 Ausencia de los reflejos del tallo.
 Ausencia de automatismo respiratorio.
 Electrocardiograma plano.
 Ausencia de antecedente de hipotermia o intoxicación.

249
¿En qué porcentaje se relaciona, los fenómenos trombótico cerebrales a
ataques isquémicos previos?
En 30 a 50 por ciento de los fenómenos trombóticos hay historia de
TIAS.
¿Qué es un infarto lacunar?
Son lesiones pequeñas, por lo general menores de cinco milímetros de
diámetro, que se presentan en la distribución de las arterias penetrantes
cortas en ganglios basales, puente de Varolio, cerebelo, extremo anterior
de la cápsula interna y con menor frecuencia en la sustancia blanca pro-
funda del cerebro.
¿Cuáles son las características del accidente vascular cerebral hemorrágico?
Es súbito, con déficits focales, cefalea intensa, efecto de masa y alteración
del estado mental.
¿Cuáles son las tres causas principales de muerte después de un accidente
vascular cerebral?
Son, en orden de frecuencia: neumonía, tromboembolia pulmonar y car-
diopatía isquémica.
¿En cuáles casos de enfermedad vascular cerebral se indica la anticoagula-
ción?
En el accidente isquémico que esté en evolución.
¿Cuál es la respuesta fisiológica al aumento de la presión intracraneal ele-
vada?
Hipotensión arterial y bradicardia (respuesta de Cushing).
¿Cómo se integra el síndrome de hipertensión endocraneana?
Cefalea, vómito, anomalías oculares (parálisis abducens), papiledema y
síntomas vegetativos (bradicardia, hipotensión) y bradipsiquia.
¿Cuándo son útiles los esteroides para tratar el edema cerebral?
Cuando el edema vasógeno como el de las neoplasias, pero no cuando es
citotóxico como en los casos de enfermedad vascular cerebral.
¿Cuál es el orden de aparición de los síntomas de hipertensión intracra-
neana?
Alteración del estado de conciencia, hiperventilación y signos de hernia-
ción.

250
¿Cómo es el electrocardiograma durante las crisis de ausencia?
Se caracteriza por las espigas de 3Hz.
¿Cuáles son los efectos adversos de la fenitoína?
Ataxia, mareo, náusea, hiperplasia gingival y alteraciones gastrointestinal.
¿Cuál es el mecanismo de acción de la fenitoína?
Bloquea los canales de sodio y calcio.
¿Cuál es el mecanismo de acción de la carbamazepina?
Bloquea únicamente los canales de sodio.
¿Cuáles son los efectos adversos de la carbamazepina?
Ataxia, vértigo, visión borrosa, depresión respiratoria y coma.
¿Para qué tipo de crisis convulsivas se utilizan la gabapentina y la lamotri-
gina?
Para tratar las crisis parciales, ya sean simples o complejas.
¿Cuáles son los fármacos de elección para tratar las crisis de ausencia?
El fenobarbital y la etosuccimida.
¿Cuáles son los nuevos fármacos para el tratamiento de las crisis parciales
y crisis con generalización secundaria?
 Topiramato: bloquea los canales de sodio, aumenta la acción del áci-
do gamma-aminobutírico (GABA) en sus receptores y bloquea algu-
nos receptores de glutamato.
 Vigabatrina: actúa inhibiendo la GABA-transaminasa, por lo que
aumenta los niveles de cerebrales de GABA.
¿Cuáles son las teorías sobre el origen de la migraña?
La del vasoespasmo y la de la disfunción trigéminovascular.
¿Cuál es la única migraña con aura?
La de la arteria basilar.
¿Qué es la cefalea en racimos (cluster headache)?
Es una cefalea predominante que se presenta en varones. Hay dolor pe-
riorbitario intenso todos los días durante varias semanas que puede
acompañarse de congestión nasal ipsilateral, epífora, ojo rojo y síndrome
de Horner.
¿Cuáles son las formas de terapia para la migraña?
 Para la migraña aguda: antiinflamatorios no esteroideos, ergotamina
o sumatriptán.

251
 Como profilácticos de la migraña: amitriptilina, propranolol, vera-
pamilo o nifedipino.
¿Sobre qué receptores actúan los medicamentos antimigrañosos?
 Los abortivos de la migraña actúan sobre los 5HT tipo 1.
 Los profilácticos de la migraña actúan sobre los 5HT tipo 2.
¿Cuál es el tratamiento de la cefalea por tensión?
Amitriptilina: 100 mg.
¿Qué neurotransmisores están asociados a las siguientes enfermedades?
 Parkinson, Alzheimer, esquizofrenia: dopamina.
 Alzheimer: acetilcolina.
 Depresión: serotonina.
 Ansiedad: ácido gamma-aminobutírico.
¿Qué es la demencia?
Es la disminución de la función intelectual y cognitiva sin alteración con-
comitante del estado de conciencia.
¿Cuáles son los dos tipos de demencia?
La demencia senil (Alzheimer) y la demencia multiinfarto.
¿Dónde ocurre el daño característico en quienes padecen enfermedad de
Alzheimer?
En el hipocampo.
¿Qué pacientes tienen más riesgo de padecer Alzheimer?
Los que expresan la proteína APOE 4/4.
¿Cuál es la característica anatomopatológica de la enfermedad de Alzhei-
mer?
Marañas neurofibrilares intracelulares y placas neuríticas extracelulares.
¿Qué medicamentos se utilizan en los casos de enfermedad de Alzheimer?
La tacrina es el único medicamento autorizado para el tratamiento. Para
disminuir la progresión de los síntomas se utiliza el IFN y el copolímero 1.
¿Dónde ocurre el daño característico en la enfermedad de Parkinson?
En la sustancia nigra pars compacta y en el locus coeruleus. Son característi-
cos los cuerpos de Lewy.
¿Cuáles son las vías neurológicas de los movimientos coreiformes?
 La vía indirecta o sistema paleoestriado, que está formada por el globo
pálido y la sustancia nigra. Su función consiste en inhibir el tono

252
muscular y agilizar los movimientos. Una lesión en esta vía provoca
un síndrome hipertónico-parkinsoniano.
 La vía directa o sistema neoestriado, que está compuesta por el pu-
tamen y el núcleo caudado. Su función consiste en aumentar el tono
muscular e inhibir el movimiento. Una lesión en este sistema provoca
un síndrome hipercinético atetósico.
¿Cuál es la diferencia entre el temblor de origen cerebeloso y el que se ob-
serva en quienes padecen la enfermedad de Parkinson?
El temblor cerebeloso aumenta al intentar movimientos voluntarios; el
parkinsoniano disminuye.
¿Cuál es el mecanismo de acción de la levodopa?
Es un precursor metabólico de la dopamina. Ya que la enfermedad de
Parkinson resulta de una cantidad insuficiente de dopamina en regiones
específicas del cerebro, puede ayudar a aliviar los síntomas.
¿Cómo actúa la carbidopa?
Actúa potenciando la acción de la levodopa, ya que es un inhibidor de la
enzima dopa-descarboxilasa en los tejidos periféricos, por lo que levodopa
(que cruza fácilmente la barrera hematoencefálica) alcanza el sistema
nervioso central.
¿Cómo actúa la bromocriptina?
Es un derivado de la ergotamina (alcaloide con acción vasoconstrictora);
es un agonista de los receptores de dopamina. Se usa en pacientes que no
responden a la levodopa.
¿Cómo actúa la amantadina?
Tiene acción como antiparkinsoniano. Parece actuar aumentando la sín-
tesis, liberación y recaptura de dopamina.
¿Cómo actúan el pramipexol y el ropinirol?
Son agonistas dopa no ergotamínicos, por lo que no acentúan el vasoes-
pasmo periférico y pueden usarse solos o en combinación con levodopa.
¿Qué es el tolcapone?
Es un derivado del nitrocatecol que representa una nueva clase de medi-
camento antiparkinsoniano. Su función es inhibir selectiva y reversible-

253
mente la acción de la catecol-O-metiltransferasa (COMT), lo que disminuye
la metilación de la levodopa a 3-O-metildopa, aumenta la recaptura de
levodopa en el SNC y, finalmente, incrementa las concentraciones cere-
brales de dopamina.
¿Cómo se integra el síndrome cerebeloso?
En el síndrome cerebeloso se encuentran afectadas las tres funciones ce-
rebelares principales, que son la coordinación del movimiento (causa asi-
nergia), la regulación del tono muscular y el equilibrio.
¿Qué medicamentos tienen interacción con la levodopa?
La piridoxina aumenta su metabolismo periférico y junto con los inhibidores
de la mono-aminooxidasa causa arterial por exceso de catecolaminas.
¿Cuáles son los efectos adversos de la levodopa?
Anorexia, náusea, vómito y taquicardia (por estimulación dopaminérgica).
¿Qué fármacos pueden causar parkinsonismo?
Las fenotiacinas y butirofenonas (neurolépticos).
¿Qué es el síndrome de Wernicke-Korsacoff?
Es un síndrome provocado por la deficiencia de tiamina (principalmente
por consumo de alcohol), que cursa con ataxia, confusión y oftamoplejia
(VIº par bilateral).
¿Qué se observa en la resonancia magnética de un paciente con síndrome
de Wernicke-Korsacoff?
Reforzamiento de los cuerpos mamilares.
¿Cuál es más frecuente, el EVC hemorrágico o el trombótico?
El trombótico en 85% de los casos.
¿Cuál es la lesión anatomopatológica que se encuentra en los casos de la
esclerosis múltiple?
Inflamación crónica, desmielinización y gliosis.
¿Cuál es la característica patológica de la esclerosis múltiple?
Afecta la mielina y respeta el cilindroeje.
¿Cuál es el principal agente causal de la meningitis infecciosa del adulto?
El neumococo.
¿Cuál es la presentación clínica de la meningitis bacteriana?
Fiebre, cefalea, meningismo y disfunción de los pares IV, VI y VII.

254
¿Cómo se integra el síndrome meníngeo?
Hay síndrome de hipertensión endocraneana y signos de irritación de raí-
ces como rigidez de nuca y signos de Kernig y Brundzinski.
¿Cuál es la base para el diagnóstico de meningitis bacteriana?
El análisis del líquido cafalorraquídeo (LCR).
¿Cuáles son las características del LCR en la meningitis bacteriana?
Pleocitosis neutrofílica, hipoglucorraquia, proteínas elevadas y aumento
de la presión de apertura.
¿Qué se debe pensar si el LCR es negativo a bacterias y existe monocitosis?
Se debe hacer diagnóstico diferencial entre listeria, tuberculosis, infección
viral, sífilis y enfermedad de Lyme.
¿Cuáles son las contraindicaciones de la punción lumbar?
 Efecto de masa o hipertensión endocraneana.
 Trombocitopenia (menos de 50,000).
 Tiempo de protrombina mayor de 3 segundos.
¿Cuál es la dosis de penicilina útil para el tratamiento de la meningitis
neumocócica y meningocócica?
24,000,000 U cada 6 horas.
¿Cuál es el tratamiento profiláctico de la meningitis meningocócica?
Rifampicina: 600 mg c/12 horas durante 2 días.
¿Cuál es el tratamiento de la meningitis del neonato y del mayor de 50
años?
La combinación ampicilina-cefotaxima.
¿Cuál es el tratamiento de la meningitis meningocócica en el niño de 3-6
años?
La combinación vancomicina-cefotaxima.
¿Por qué no se da dexametasona los casos de meningitis meningocócica?
Porque reduce la penetrancia de la vancomicina al SNC.
¿Cuál es el efecto de la vancomicina en la meningitis meningocócica?
Aumenta la penetrancia del antibiótico; puede usarse para contrarrestar
el efecto de la dexametasona.
¿Cuáles son las tres causas de absceso cerebral?
Un foco contiguo, un foco distante o postrauma quirúrgico.

255
¿En orden de frecuencia cuáles son las áreas afectadas por el absceso cere-
bral?
Frontal, temporal, témporoparietal, parietal, cerebelar y occipital.
¿Cuál es la presentación clínica del absceso cerebral?
 Cefalea, desorientación, convulsiones y fiebre.
 Focalización.
 Síndrome de hipertensión endocraneana.
¿Cuál es el tratamiento empírico del el absceso?
Penicilina 20-24 millones IV cada 6 horas.
¿Cómo es la sintomatología y el LCR en la meningitis viral?
Cuadro clínico: fiebre, cefalea y meningismo. LCR: normoglucemia, pro-
teínas ligeramente elevadas, pleocitosis linfocítica.
¿Qué es la leucoencefalopatía multifocal progresiva?
Es un trastorno del sistema nervioso central que se observa en los pacien-
tes inmunodeficientes; afecta a los oligodendrocitos y es provocado por el
virus JC (VJC).
¿Cuáles son los síndromes neurocutáneos?
Neurofibromatosis tipo 1 (enfermedad de Von Recklinghausen), neurofi-
bromatosis Tipo 2, esclerosis tuberosa (síndrome de Bourneville) y sín-
drome de Von Hippel Lindau.
¿Cuál es el síndrome centromedular?
Existe pérdida de las sensaciones con topografía “en capa”, debilidad
pronunciada en los brazos y debilidad leve en las piernas.
¿Cómo se clasifican las lesiones extramedulares?
En intradurales y extradurales. Las intradurales generalmente son malig-
nas y las extradurales benignas.
¿Qué causa la lesión de C-3 y C-5?
Cuadriplejía más pérdida del automatismo respiratorio.
¿Cómo se presenta la lesión a nivel de C-4 y C-5?
Cuadriplejía con automatismo respiratorio.
¿Cómo se presenta la lesión de C-5 a C-6?
El hombro se encuentra respetado; se pierden los reflejos bicipital y bra-
quiorradial.

256
¿Cuál es la inervación de las cuerdas vocales verdaderas?
El nervio laríngeo inferior inerva al músculo cricoaritenoideo posterior,
que se encarga de dar movilidad a las cuerdas vocales verdaderas.
¿Cuál es la presentación clínica de una lesión del nervio torácico largo?
Se afecta el músculo serrato anterior y se produce la llamada “escápula
alada”.
¿Qué causa la lesión del nervio radial?
Afecta los extensores del brazo y antebrazo y causa una “muñeca péndula”.
¿Cuáles son las funciones del nervio ciático poplíteo?
Su rama tibial anterior es responsable de la dorsiflexión del pie y su rama
musculocutánea inerva el compartimiento lateral de la pierna, que es res-
ponsable de la eversión del pie.
¿Con qué pruebas se evalúa la integridad de las raíces L-4 y L-5?
Caminar sobre los talones evalúa a L-4 y sobre las puntas, a L-5.
¿Cómo inicia la mielitis transversa?
Como un dolor en la espalda o el cuello seguido de déficit neurológico,
con paraplejia sensoriomotora que se detecta fácilmente como un “nivel”
con pruebas de sensibilidad o movilidad habituales. Generalmente, en la
historia clínica se encuentra que fue precedida por una infección por vi-
rus o micoplasma.
¿Qué ramas forman el plexo sacro?
El tronco lumbosacro y S-1, S-2 y S-3.
¿Cuál es el tumor más común de la médula espinal?
El ependimoma.
¿Cuáles son los tumores extramedulares más comunes?
El neurofibroma, el meningioma y los tumores metastásicos.
¿Cuál es el tratamiento de la neuropatía diabética?
Los inhibidores de la aldosa-reductasa.
¿Cómo actúan la actina y la miosina en la contracción muscular?
Cuando se proporciona energía en forma de ATP a una molécula de
miosina, ésta cambia su estructura “plegando” uno de sus extremos, que
está unido a la actina, con lo que la actina se mueve sobre la miosina.
¿Qué enfermedades se presentan con debilidad muscular proximal?
La miastenia gravis, el síndrome de Eaton-Lambert, la esclerosis lateral
amiotrófica, esclorosis múltiple y polimiositis.

257
¿Cómo se evalúa clínicamente la debilidad muscular proximal?
Si el paciente no puede realizar actividades como levantarse de la seden-
tación sin ayuda o no puede levantar los brazos para peinarse, entonces
existe debilidad muscular proximal.
¿Cómo se evalúa la debilidad muscular distal?
Pidiendo al paciente que apriete con su mano los dedos del explorador.
Subjetivamente se valorará la presencia o no de debilidad. Una historia
de “dejar caer las cosas frecuentemente de las manos” también apoya el
diagnóstico.
¿Cuál es el diagnóstico diferencial en los estados de debilidad muscular?
 Hipotiroidismo: debilidad muscular y creatincinasa elevada.
 Hipertiroidismo: debilidad muscular y creatincinasa normal.
 Cushing: debilidad muscular distal y proximal y creatincinasa normal.
 Enfermedades musculares inflamatorias: dermatomiositis y polimio-
sitis.
 Enfermedades neurológicas: miastenia gravis y esclerosis lateral
amiotrófica.
 Bacteriana: borreliosis de Lyme.
 Parasitaria: toxoplasmosis y triquinosis.
 Medicamentosa: AZT, D-penicilamina, clorpropamida, clorproma-
zina, etcétera.
¿Cuál es la diferencia entre el síndrome de Guillain-Barré y el botulismo?
 En ambos hay ausencia de reflejos sin cambios sensoriales.
 En el primero, la parálisis es ascendente y hay elevación de proteínas
en el líquido cefalorraquídeo.
 En el segundo, la parálisis es descendente y comienza afectando los
nervios craneales; el líquido cefalorraquídeo es normal.
¿Cuál es el tratamiento del botulismo?
Medidas de apoyo para mejorar la debilidad muscular.
¿Cuál es la diferencia entre polimialgia y polimiositis?
Las dos se presentan con dolor muscular, pero la primera no causa debi-
lidad muscular proximal.

258
¿Cuál es el cuadro clínico de la miastenia gravis?
Dificultad para tragar, dificultad para hablar, visión doble y debilidad de
miembros.
¿Qué diferencia hay entre el síndrome miasténico de Eaton-Lambert (EL) y
la miastenia gravis?
 En la MG hay anticuerpos antirreceptor de acetilcolina y en el EL el
anticuerpo es anticanal de calcio presináptico (cuya función consiste
en liberar calcio para que a su vez las vesículas de acetilcolina sean
liberadas).
 En el primero, la fuerza de la contracción aumenta o permanece igual
tras la neuroestimulación repetida, y en el segundo disminuye.
 El síndrome miasténico respeta los músculos oculares y extraoculares.
¿Con qué se relaciona el síndrome Eaton-Lambert?
Con el cáncer pulmonar de células pequeñas.
¿Qué es el síndrome de Miller-Fisher?
Es un cuadro clínico similar al del s´ndrome de Guillain-Barré con pará-
lisis descendente que comienza afectando los nervios craneales (hay que
hacer diagnóstico diferencial con botulismo).
¿Cuál es el tratamiento para de la miastenia gravis (MG) y el síndrome de
Eaton-Lambert (EL)?
 Para la MG se utiliza algún inhibidor de la colinesterasa, preferente-
mente la piridostigmina o mestinón.
 Para el EL se utiliza la guanidina.
¿Cuál es la diferencia de acción entre la fisostigmina y la neostigmina?
La fisostigmina sí logra el acceso al SNC. Una ventaja es que gracias a
esta acción previene el delirio producido por los muscarínicos como la
escopolamina y metoclopramida.
¿Cómo se lleva a cabo el diagnóstico de una miopatía?
Cuando coexiste la elevación de la creatincinasa con electromiografía su-
gestiva, elevada, EMG sugestiva, biopsia de músculo positiva y estudio
genético positivo.
¿Cuál es el gen afectado en la distrofia muscular de Duchenne?
El gen distrofina no detectable.

259
¿Cuál es el gen afectado en la distrofia muscular de Becker?
El gen distrofina detectable.
¿Cuál es el gen afectado en la distrofia miotónica?
Es un trastorno autosómico dominante en el que se afecta el gen mioto-
nin-proteín-kinasa en el cromosoma 19.
¿Qué son las distrofias no miotónicas?
Son canalopatías que se presentan con relajación muscular prolongada
después de actividad muscular constante o de estimulación mecánica.

260
NUTRICIÓN, METABOLISMO,
LÍQUIDOS Y ELECTROLITOS

¿A qué se le llama dieta de líquidos claros?


Es una dieta predominante en agua, con 500-1000 kcal de carbohidratos,
con electrolitos y sin fibra.
¿Cómo se calculan los requerimientos dietéticos diarios según el peso del
paciente?
En los pacientes que tienen peso adecuado se utiliza el peso real, y en los
pacientes obesos se utiliza el peso ideal.
¿Cuáles son los requerimientos diarios del paciente hospitalizado?
Kcal 2,000 y 60 g de proteínas.
¿Cuál es el marcador clínico de un aumento de proteínas en la dieta?
El nitrógeno ureico en sangre (BUN) sérico se eleva.
¿En quiénes está indicada la restricción de proteínas de la dieta?
En los hepatópatas y en los nefrópatas.
¿Cómo puede evaluarse clínicamente el peso del paciente?
Puede evaluarse como peso relativo, que se calcula como peso actual/peso
deseable x 100, o bien, como índice de masa corporal (IMC), que se calcula
como peso/estatura en m2. El IMC normal es de 20 a 25 kg/m2.
¿Cuál es la medición antropométrica más utilizada para evaluar la compo-
sición corporal?
La composición corporal (reservas de grasa y musculoesquelética) se eva-
lúa con mayor frecuencia midiendo el pliegue cutáneo del tríceps y la circun-
ferencia muscular de la parte media del brazo.
¿Cuál es el estudio de laboratorio más utilizado para evaluar la desnutrición?
La albúmina sérica.

261
¿Cuáles son las dos variantes clínicas de la desnutrición protéico-calórica?
El kwashiorkor, que es causado por una deficiencia de proteínas en pre-
sencia de energía adecuada y el marasmo, ocasionado por la deficiencia
protéico-energética combinada.
¿Cómo se define la obesidad?
Existe obesidad cuando el peso relativo es mayor a 120% y el índice de
masa corporal es mayor a 27 kg/m2.
¿Cuáles son los tipos de obesidad de acuerdo con la distribución de la grasa
corporal?
Existe la obesidad de la parte superior del cuerpo, en la que el exceso de grasa
se encuentra alrededor de la cintura y el costado, y la obesidad de la parte
inferior, en la que el tejido adiposo se acumula en los muslos y regiones
glúteas. Los individuos con obesidad de la parte superior tienen un riesgo
mayor para la salud, y los que presentan un índice cintura-cadera mayor de
1 en varones y menor de 0.85 en mujeres, tienen un riesgo incrementado
para padecer diabetes mellitus, enfermedades vascular cerebral, corona-
riopatía y muerte temprana.
¿Qué es la leptina?
La leptina es una proteína que tiene un receptor específico en el hipotá-
lamo, cuya función consiste en controlar el apetito. Un defecto en la pro-
ducción o reconocimiento de la leptina pueden ser causa de obesidad.
¿Cuáles son los fármacos aprobados para el tratamiento de la obesidad?
Los agentes anorexigénicos y los inhibidores de la lipasa.
¿Cuáles son los fármacos anorexigénicos aprobados?
La fentermina y la sibutramina son agentes similares a la anfetamina, que
actúan inhibiendo la recaptura de serotonina, norepinefrina y dopamina
(en el caso de la sibutramina), o bien, aumentando la liberación de do-
pamina (fentermina).
¿Cuáles son los efectos adversos de los anorexigénicos?
Debido a que incrementan los niveles de aminas vasoactivas puede existir
boca seca, aumento de la presión arterial y de la frecuencia respiratoria.
En el caso de la sibutramina, no debe utilizarse en pacientes que están ba-
jo tratamiento con inhibidores de la monoaminoexidasa (IMAO) (fluoxe-

262
tina), agonistas de la serotonina (sumatriptán), litio o dextrometorfán
pues sus acciones son similares y los efectos se ven potenciados.
¿Cómo actúan los inhibidores de la lipasa?
El orlistat es el único fármaco aprobado de este grupo. Actúa inhibiendo
las lipasas gástrica y pancreática, disminuye la absorción de grasas en
30%. Esta indicado solamente en pacientes con un IMC mayor de 30 o
en pacientes con IMC mayor de 27 pero con otros factores de riesgo co-
mo diabetes, dislipidemia o hipertensión.
¿Cuáles son las vitaminas hidrosolubles?
La tiamina (vitamina B-1), la rivoflavina (vitamina B-2), la biotina, la
niacina, la piridoxina (vitamina B-6), la cobalamina (vitamina B-12), el
ácido fólico y el ácido ascórbico (vitamina C).
¿Qué es el beri-beri y cuántos tipos clínicos existen?
Es la deficiencia de tiamina (B-1) y existen la variedad húmeda y la seca.
¿Qué es el beri-beri húmedo?
El beri-beri con afección cardiovascular.
¿Por qué el beri beri húmedo es una cardiopatía de alto gasto?
Porque provoca vasodilatación periférica, excesiva que obliga al incre-
mento del gasto cardiaco.
¿Qué es el beri-beri seco?
Es la presentación neurológica de la enfermedad. Tiene una presentación
central (Wernicke-Korsakoff) y una periférica (neuropatía simétrica mo-
tora y sensitiva, con dolor).
¿Cuál es la dosis de tiamina para el tratamiento del beri-beri?
50-100 mg/día hasta que haya signos de mejoría o el olor de la orina in-
dique que se ha saturado de tiamina. Después se reduce la dosis a 5-10
mg/día (que es la dosis para deficiencias leves).
¿En quiénes debe sospecharse beri-beri?
En los alcohólicos crónicos.
¿Cuáles son las reacciones que requieren cobalamina como cofactor?
La reacción de conversión de homocisteína a metionina y la de metilma-
lonil-coenzima A a succinil-coenzima A.
¿Cuáles son las acciones de la niacina en hiperlipidemias?
Aumenta las lipoproteínas de alta densidad y disminuye las lipoproteínas
de baja densidad y las de muy baja densidad.

263
¿Qué es la pelagra?
Es la deficiencia de niacina que cursa con dermatitis (mucosas secas,
obscuras y escamosas), diarrea y demencia (insomnio e irritabilidad, que
progresan hasta alucinaciones y psicosis).
¿Cuáles son los efectos adversos de la niacina?
Rubor de la piel, irritación gástrica, aumento de enzimas hepáticas, hi-
perglucemia y gota.
¿Cuál es la función de la vitamina C?
La vitamina C es necesaria para formar colágena; si se carece de ella, se
produce osteopenia y falta de cierre de las heridas, así como hemorragia.
¿Qué es el escorbuto?
Es la deficiencia de ácido ascórbico (vitamina C), que se manifiesta con
hemorragias perifoliculares y en astilla, hemartrosis y hemorragias subpe-
riósticas.
¿De dónde se obtiene la biotina?
Es una vitamina sintetizada por los microorganismos intestinales.
¿Para qué sirve la biotina?
Es un intermediario en las reacciones de transferencia de CO2 (carboxila-
ción). La ovidina, que es una proteína encontrada en el huevo, puede in-
hibirla y causar intoxicación.
¿Cómo actúa la piridoxina?
La piridoxina y sus subproductos (piridoxal, piridoxamina y sus ésteres 5
fosfato) están estrictamente implicados en el metabolismo intermedio. Es
de particular importancia el fosfato de piridoxal, que es la coenzima princi-
pal para el metabolismo de los aminoácidos y la síntesis del hem.
¿A qué se debe principalmente la deficiencia de piridoxina?
Las causas principales son el alcoholismo y la acción de fármacos anta-
gonistas (isoniacida, anticonceptivos, cicloserina y penicilamina.)
¿Cuáles son los síntomas generales de las deficiencias de las vitaminas B?
Ulceras orales, queilitis, debilidad e irritabilidad. La deficiencia grave
causa neuropatía periférica, anemia y convulsiones.
¿Qué etiología tiene la deficiencia de Vitamina A?
Ocurre en estados de malabsorción de grasas y en lactantes.

264
¿Cuáles son las manifestaciones iniciales de la deficiencia de vitamina A?
Ceguera nocturna, placas blancas en la conjuntiva y xerosis.
¿Cuáles son las manifestaciones tardías de la deficiencia de vitamina A?
Queratomalacia, endoftalmitis y ceguera.
¿Cuál es la dosis para el tratamiento de la deficiencia de vitamina A?
30,000 UI al día.
¿Cuál es la acción de la vitamina D?
Aumentar la absorción intestinal de calcio.
¿Cuáles son los tipos de raquitismo?
 Nutricional, por falta de vitamina D. Se puede encontrar calcio sérico
bajo, proteínas bajas, hormona paratiroidea aumentada y fosfatasa
alcalina elevada.
 Por metabolismo anormal de la vitamina D, como en la ausencia de
alfa1-hidroxilasa.
 Hiperfosfatemia ligada a X.
 Raquitismo de la prematurez.
¿Cuál es el cuadro clínico del raquitismo?
Existe rosario raquítico, arqueamiento de las piernas, aumento del espa-
cio articular en las muñecas y rodillas, craneotabes (cráneo blando), fon-
tanelas abiertas y metáfisis ocupadas o “ensuciadas”.
¿Cuál es el tratamiento del raquitismo?
Se utilizan 5,000 a 10,000 U de vitamina D diariamente durante 2 a 3
meses.
¿Cuál es la estructura química de las lipoproteínas?
Están compuestas por una fracción proteínica llamada apolipoproteína o
apoproteína, que puede ser desde 1 hasta 60% del total de la molécula, se-
gún el tipo de lipoproteína y por diferentes cantidades de los tres lípidos
principales (triglicéridos, colesterol y fosfolípidos).
¿A qué se le llama lipoproteínas beta, pre-beta y alfa?
Son los nombres alternativos para las lipoproteínas de baja densidad
(LDL), las lipoproteínas de muy baja densidad (VLDL) y las lipoproteí-
nas de alta densidad (HDL), respectivamente.
¿Cuál es la función de los quilomicrones?
Transportar los triglicéridos desde su lugar de absorción en el intestino
hasta el torrente circulatorio.

265
¿Cuál es la función de las VLDL?
Transportar los triglicéridos desde el hígado a la circulación.
¿Qué sucede cuando los quilomicrones y las VLDL reaccionan con la lipo-
proteínlipasa para liberar triglicéridos?
Los quilomicrones se convierten en quilomicrones remanentes y las VLDL
se convierten en lipoproteínas remanentes de densidad intermedia (IDL).
¿Cuál es la importancia de los triglicéridos plasmáticos?
Son los principales lípidos transportados en la sangre. Diariamente se
transportan entre 70 y 150 gramos de ellos mientras que se transportan
sólo 1 o 2 gramos de colesterol o fosfolípidos.
¿Cuál es la función de las HDL?
Las HDL remueven el colesterol de los quilomicrones, de las VLDL y de
las membranas celulares y lo llevan al hígado y a los macrófagos donde
lo entregan por medio de la apolipoproteínas AI.
¿Con qué valores de laboratorio se hace el diagnóstico de dislipidemia?
Colesterol total menor de 240, HDL menor de 35.
¿En que lipoproteínas se encuentran las siguientes APO?
 APO E: VLDL, LDL, HDL y IDL.
 APO D: HDL.
 APO B48: Quilomicrones.
¿Qué es la APO-CIII?
Es un inhibidor de la lipoproteín-lipasa.
¿Cuál es la hiperlipoproteinemia tipo I?
También se conoce como hiperquilomicromemia familiar. El plasma presen-
ta una capa cremosa de sobrenadante después de ser refrigerado. Clíni-
camente presenta pancreatitis o xantomas eruptivos en zonas de presión
y lipemia retinalis. Es causada por deficiencia de LPL o de APO-CII. Se
trata solo con dieta baja en grasas.
¿Cuál es la hiperlipoproteinemia tipo II-A?
También es conocida como hipercolesterolemia familiar. Existen cifras ele-
vadas de LDL y cifras normales de VLDL; por tanto, el colesterol está
elevado y los triglicéridos son normales. Las LDL no refractan la luz, así
que el plasma es traslúcido. La causa es una disminución en los recepto-

266
res de LDL. Los pacientes presentan aterosclerosis y enfermedad corona-
ria acelerada, xantomas tendinosos en el tendón de Aquiles, rotulianos y
de los extensores de los dedos. La variedad heterocigota se trata con co-
lestiramina y estatinas. Debe agregarse niacina al tratamiento de la varie-
dad homocigota.
¿Cuál es la hiperlipoproteinemia II-B?
También llamada hiperlipidemia familiar combinada o mixta. Cursa con ni-
veles altos de LDL y VLDL; por tanto, el colesterol y los triglicéridos es-
tán elevados. La presentación clínica es la misma que en la tipo II-A. La
causa es un aumento en la producción hepática de VLDL. La terapéutica
requiere triple esquema de fármacos: secuestradores de ácidos biliares,
inhibidores 3-hidroxi-3metil-glutaril-coenzima A (HMG CoA).
¿Cuál es la hiperlipoproteinemia tipo III?
También conocida como disbetalipoproteinemia familiar. Se debe al acúmu-
lo de IDL debido a la sobreproducción o a la presencia de una apolipo-
proteína (APO) E mutante. El plasma es turbio y con una ligera capa de
quilomicrones. Clínicamente presenta aterosclerosis de los vasos corona-
rios y periféricos y xantomas eruptivos, tendinosos y tuberosos. El trata-
miento incluye pérdida de peso, dieta baja en colesterol, estatinas,
fibratos y niacina.
¿Cuál es la hiperlipoproteinemia tipo IV?
Es la hipertrigliceridemia familiar en la que hay niveles altos de VLDL con
cifras normales de LDL, y se asocia con obesidad, intolerancia a la glu-
cosa o diabetes e hiperuricemia. También se observa durante la terapia
estrogénica, en el tercer trimestre del embarazo y en el alcoholismo. El
tratamiento incluye pérdida de peso, restricciones dietéticas que involu-
cran carbohidratos, grasa y alcohol. La terapia farmacológica puede no
ser necesaria si se siguen las recomendaciones; cuando es necesaria, re-
quiere niacina, fibratos y estatinas.
¿Cuál es la hiperlipoproteinemia tipo V?
Es la hipertrigliceridemia familiar mixta, en la que se encuentra elevados los
quilomicrones y las VLDL. Clínicamente hay xantomas eruptivos. Ocu-
rre en pacientes obesos y diabéticos y el tratamiento es el mismo que en
los tipos III y IV.

267
¿Cuáles son las dislipidemias que pueden debutar con pancreatitis?
Las de tipo I , IV y V.
¿Cuáles son los efectos de la niacina en las dislipidemias?
Inhibe la síntesis de ácidos grasos a partir de triglicéridos en el tejido adi-
poso; por tanto, no se forman VLDL en el hígado y no se forman LDL
en la periferia. Son útiles cuando están elevadas las VLDL y LDL con
niveles bajos de HDL baja.
¿Cuáles son los efectos adversos de la niacina?
Enrojecimiento de la piel (flushing) y prurito.
¿Cómo actúan los fibratos?
Aumentan la actividad de las LPL sobre las VLDL y los quilomicrones
(transportadores de triglicéridos y antígenos), incrementan el catabolismo
no esplácnico de las VLDL y probablemente aumentan las HDL; por tan-
to, se usan cuando están elevadas las VLDL y LDL y las HDL, bajas.
¿Cuáles son los efectos de la niacina y los fibratos sobre las HDL?
Los dos provocan un aumento de las HDL (considerablemente más la
primera).
¿Cuál es un efecto adverso de los fibratos?
Pueden causar malignidad, miositis y miopatía. También potencian la
acción de los cumarínicos.
¿Cómo actúan la colestiramina y el colestipol?
El hígado sintetiza ácidos biliares a partir del colesterol y éstos se reab-
sorben en el intestino. Estas resinas fijan el colesterol de las sales biliares
impidiendo su reabsorción.
¿En qué patología se utilizan la colestiramina y el colestipol?
Son de elección para las dislipidemias tipo II-A y II-B.
¿Cuáles son sus efectos adversos de la colestiramina y el colestipol?
Disminuyen la absorción de las vitaminas liposolubles (ADEK), de la
niacina y del ácido fólico.
¿Cómo actúan los inhibidores de la HMG-CoA reductasa (estatinas)?
Actúan inhibiendo el paso de HMG a mevalonato y de éste a colesterol.
Además de inhibir la HMG-reductasa, aumentan los receptores para
LDL e incrementan su metabolismo.

268
¿Cuáles son los efectos secundarios de las estatinas (a nivel bioquímico)?
Afectan a la coenzima Q, al hem y a las proteínas de membrana.
¿Cuál es el mecanismo común de las estatinas y la colestiramina para dis-
minuir las HDL?
Aumentan la recaptura de LDL en el hígado, ya que incrementan la acti-
vidad de las APO-B y E.
¿Cuáles son las principales enfermedades por defecto en el metabolismo li-
sosomal?
Las enfermedades por atesoramiento se deben a defectos en la función de los
lisosomas que deben degradar las macromoléculas a sus unidades básicas
(aminoácidos, monosacáridos y ácidos grasos). Las principales son las li-
pidosis, las gangliosidosis, las mucopolisacaridosis y las leucodistrofias.
¿Qué es la enfermedad de Fabry?
Es la lipidosis familiar por acúmulo de glucoesfingolípidos transmitida de
manera ligada al sexo y causada por la deficiencia de alfa-galactosidasa.
Los depósitos de lípidos producen angioqueratomas en la piel.
¿Qué es la enfermedad de Gaucher?
Es la lipidosis familiar autosómica recesiva causada por acúmulo de glu-
cocerebrósidos, y que se debe a una deficiencia de la beta-glucosidasa. Se
manifiesta por hepatoesplenomegalia, pigmentación de la piel, lesiones
esqueléticas y pingüéculas. Es la lipidosis más observada en la clínica.
¿Qué es la enfermedad de Niemann-Pick?
Es la lipidosis familiar causada por acúmulo de esfingomielina, y que se de-
be a la deficiencia de esfingomielinasa ácida. Se divide en Tipo A (es la lipi-
dosis con muerte infantil) y la lipidosis infantil hepatopulmonar (Tipo B).
¿Qué es la enfermedad de Tay-Sachs?
Es la gangliosidosis neuronal tipo GM2 debida a deficiencia de hexo-
saminidasa, caracterizada por un comienzo muy temprano, retraso en el
desarrollo, parálisis, demencia y muerte a los 3 o 4 años.
¿Qué es la enfermedad de Refsum?
Es una enfermedad debida a acúmulo de ácido fitánico, y que causa prin-
cipalmente síntomas neurológicos como neuropatía periférica, ataxia ce-
rebelosa, retinitis pigmentaria; además produce alteraciones en los huesos
y la piel.

269
¿Cuáles son las características clínicas de las mucopolisacaridosis?
Afección multiorgánica, hepatoesplenomegalia y anormalidades óseas.
¿Qué es el síndrome de Hurler?
Es un cuadro clínico compatible con mucopolisacaridosis. Los datos
principales son: facies característica, disminución de la alfa-L-
iduronidasa y acumulación de heparán-sulfato y dermatán-sulfato.
¿Cuál es la enfermedad de San Filippo?
Es un cuadro clínico de mucopolisacaridosis, degeneración grave del sis-
tema nervioso y afección multiorgánica leve.
¿Qué es el síndrome de Hunter?
Es una variedad del síndrome de Hurler, que aparece entre los 2 y los 4
años.
¿Qué es el síndrome de Morquio?
Es una enfermedad que se presenta con cuadro clínico de mucopolisaca-
ridosis, enanismo, depósitos corneales y displasia espondiloepifisiaria.
¿Qué son las leucodistrofias?
Son desórdenes debidos a atesoramiento y que afectan la sustancia blan-
ca cerebral.
¿Qué es la enfermedad de Krabbe?
Es la leucodistrofia globoide secundaria a depósitos de galactosilcerebró-
sido debidos a la deficiencia de la enzima galactosilceramidasa.
¿Qué son las glucogenosis?
Son enfermedades causadas por atesoramiento de glucógeno, que pueden
tener una variedad muscular y otra hepática.
¿Qué son las porfirias?
Son un grupo de alteraciones en la biosíntesis del hem. Pueden ser hepáticas
o eritropoyéticas, según el lugar de expresión del defecto enzimático. Las
principales características clínicas de las porfirias son la fotosensibilidad y
los síntomas neurológicos como dolor abdominal, trastornos de la moti-
lidad intestinal, disestesias y parálisis muscular o respiratoria.
¿Qué es la porfiria aguda intermitente?
Es una enfermedad autosómica dominante que aparece por deficiencia
de la uroporfobilinógeno sintetasa, cuyo síntoma inicial es un dolor ab-

270
dominal que simula ser quirúrgico, mismo que se desencadena con ma-
yor frecuencia por el uso de substancias y que aumenten la actividad del
citocromo P450.
¿Cuáles son las reglas de THUMB?
 Por cada aumento de 100 g de glucosa, el sodio disminuye 1.6mEq.
 El potasio aumenta 0.6 mEq por cada 0.1 de disminución del pH.
 El calcio disminuye 0.8 mEeq por cada gramo que disminuye la al-
búmina.
 Por cada aumento de 10 mEq de CO2, el pH disminuye 0.8 unidades.
 Por cada 10 mEq de HCO3, el pH cambia 0.15 unidades directamente.
¿Qué propiedad del epitelio tubular permite que se lleve a cabo el transpor-
te de solutos?
La diferencia en la distribución de las bombas y otras unidades a ambos
lados de la membrana es lo que hace posible el transporte de solutos en el
túbulo.
¿Qué células están capacitadas para excretar hidrogeniones a la luz de una
cavidad?
Las células del túbulo contorneado distal, del túbulo proximal y del túbu-
lo colector, así como las células parietales gástricas.
¿De dónde proviene el hidrógeno que se secreta a la luz tubular?
Las células generan CO2 como producto de desecho; la enzima anhidrasa
carbónica lo une al H2O para formar H2CO3, que fácilmente se disocia en
HCO3- y H+.
¿Cuál es el comportamiento del sodio dentro de la nefrona?
El sodio se filtra en grandes cantidades hacia la luz tubular, pero es
transportado activamente fuera de ella en todas las porciones exepto del
asa de Henle, en condiciones normales, se resorbe de 96 a 99%.
¿Cuáles son los diferentes tipos de hiponatremia?
 Isosmolar (facticia): por hiperlipemia, hiperproteinemia e hiperglu-
cemia. H.
 Hiposmolar: hipovolémica, euvolémica e hipervolémica.
 Hiperosmolar: secundaria a manitol o hiperglucemia.

271
¿Cuáles son las causas de la hiponatremia hiposmolar?
 Hipovolémica: cualquiera en la que se pierda sodio ya sea renal o ex-
trarrenal, lo que se evalúa midiendo el sodio urinario.
 Euvolémica: síndrome de secreción inadecuada de hormona antidiu-
rética (SIADH), potomanía y polidipsia psicógena.
 Hipervolémica: insuficiencia cardiaca congestivo-venosa (ICCV) y
hepatopatía.
¿Cuáles son las causas de hipernatremia?
 Puede deberse a deshidratación, en cuyo caso la osmolaridad urinaria
está aumentada (400 mOsm).
 Puede deberse a diabetes insípida central o nefrogénica, en la que el
paciente sólo pierde agua y en la que la osmolaridad urinaria está
disminuída (menos de 250 mOsm).
¿Cuáles son las causas de la hiperkalemia?
Sangrado gastrointestinal, transfusión, acidosis y falla renal.
¿Cuáles son las causas de las acidosis hiperkalémica?
 Pérdida de HCO3 por el riñón (como en la acidosis tubular renal
(ATR) tipo 2).
 Pérdida gastrointestinal de HCO3 por diarrea o por fístula del intes-
tino delgado.
¿Cuáles son las causas de la acidosis hipoclorémica?
 Pérdida de cloro por vómito repetido.
 Disminución de la excreción de HCO3 debida al uso de esteroides y
diuréticos.
¿A qué se deben la hiperkalemia y la hipocalcemia provocadas por transfu-
sión?
 Entre más viejo es un paquete globular, aumenta la posibilidad de hi-
perkalemia secundaria a la lisis celular que libera potasio del interior de
las células.
 Los paquetes contienen citrato que funciona como quelante del cal-
cio, lo que sucede a partir de que se administran cinco paquetes glo-
bulares.

272
¿Qué alteraciones plasmáticas genera la hipoproteinemia (disglobuli-
nemia)?
Aumento de colesterol y de sodio.
¿Qué alteraciones provoca la hipocolesterolemia?
Aumento de proteínas y de sodio.
¿En qué segmento del túbulo se encuentra el sistema colector de ácido?
En el contorneado proximal.
¿Cuáles son las alteraciones hidroelectrolíticas de los siguientes padeci-
mientos?
 Diabetes insípida: existe pérdida de agua, por lo que los electrolitos se
elevan uniformemente.
 SIADH: se acumula agua, por lo que los electrolitos decaen unifor-
memente.
 Diabetes descompensada: el sodio desciende 1.4 por cada 100 mg de
glucosa aumentada y el CO2 es bajo por acidosis leve (ligera compen-
sación respiratoria).
 Síndrome androgenital perdedor de sal: existe hiponatremia, hiperka-
lemia e hipocloremia.
¿Cuáles son los síndromes de Bartter y Gitelman?
Son síndromes que cursan con pérdida de potasio por la vía renal, alcalo-
sis metabólica y poliuria. El síndrome de Bartter se debe a una falla de la
bomba NaKCl en la rama ascendente del asa de Henle y el de Gitelman
se debe a falla de la bomba NaCl en el túbulo contorneado distal.
¿Cómo se calcula el anion gap (brecha aniónica)?
Na- (Cl+HCO3)
¿Cuál es el mecanismo fisiopatológico que explica la acidosis en la hiper-
cloremia?
El exceso de cloro sérico provoca entrada de HCO3 a la célula.
¿Cuáles son las causas de la acidosis hiperclorémica con anion gap normal?
La pérdida gastrointestinal de bicarbonato y la acidosis tubular renal.
¿Cuáles son los cuatro tipos de acidosis tubular renal (ATR)?
I. clásica: hipokalemia e hipercloremia secundarias a un defecto selectivo
de la secreción de hidrógeno en la nefrona distal.

273
II. proximal: existe un defecto selectivo de la capacidad del túbulo pro-
ximal para reabsorber adecuadamente el HCO3; coexiste con otros defec-
tos de absorción, por lo que puede haber aminoaciduria, fosfaturia y
glucosuria.
III. insuficiencia glomerular: por descenso de la velocidad de filtración
glomerular se produce una falla en el sistema generador de NH3; existe
hipercloremia y normokalemia.
IV. hipoaldosteronismo hiporreninémico: existe hipercloremia e hiperka-
lemia.
¿Cómo está el anion gap en la acidosis láctica?
La acidosis láctica es una acidosis de anion gap elevado que se corrige fá-
cilmente con la administración de líquidos.
¿Cuáles son los cambios gasométricos cuando existe acidosis respiratoria
aguda?
pH disminuido y CO2 aumentado.
¿Cuáles son los cambios gasométricos que se encuentran cuando existe hi-
poventilación crónica?
La hipoventilación crónica causa retención de CO2, los que conduce a
acidosis respiratoria crónica con compensación metabólica.
¿Cuáles son los cambios gasométricos que se observan en los casos de aci-
dosis metabólica aguda?
pH normal-bajo y CO2 en límites bajos por la compensación respiratoria.

274
OFTALOMOGÍA

¿Cuál es la inervación de los músculos del iris?


El sistema simpático inerva las fibras radiales que se encargan de la mi-
driasis, mientras que el parasimpático inerva el constrictor de la pupila.
¿Qué estructuras conforman la úvea?
El iris, el cuerpo ciliar y la coroides.
¿Cuál es la función del músculo constrictor del cuerpo ciliar?
Provoca la relajación de las fibras de la zónula y un aumento del diáme-
tro ánteroposterior del cristalino.
¿Cuáles son las capas de la retina?
 Epitelio.
 Conos y bastones.
 Nuclear externa.
 Plexiforme externa.
 Nuclear interna.
 Plexiforme interna.
 Células ganglionares.
 Fibras del nervio óptico.
¿Cuál es el mecanismo retiniano que permite la visión nocturna?
El ingreso de sodio a las células retinianas, gracias al canal de cationes no
selectivo, permite ver durante la noche.
¿Cuáles son los tipos de orzuelo?
 El que se encuentra en la superficie conjuntival.
 El que se encuentra en el borde ciliar.
¿Cuándo está indicado el drenaje del orzuelo?
Si en 48 horas no ha mejorado con compresas.

275
¿Qué es un chalazión?
Es una inflamación granulomatosa del absceso estafilocócico (orzuelo).
¿Qué serotipos de chlamydia causan tracoma?
El A y el C.
¿Qué serotipos de chlamydia causan la conjuntivitis de inclusión?
El D y K.
¿Cuál es la conjuntivitis viral más frecuente y cuál es su tratamiento.
Es la fiebre faringoconjuntival por adenovirus tipo 3; se administran go-
tas de sulfas y se aplican compresas para prevenir una complicación bac-
teriana.
¿Cuáles son los agentes etiológicos de la queratitis bacteriana?
Moraxella catarrhalis, neumococo y Pseudomonas.
¿Qué es una pingüécula?
Es una mancha amarilla en la conjuntiva; lo más frecuente es que se ubi-
que en el lado nasal.
¿Qué es un pterigión?
Es una carnosidad triangular con vértice hacia el iris y que se ubica del
lado nasal.
¿Cuál es el cuadro clínico de la queratitis?
Dolor, fotofobia, lagrimeo e inyección circuncorneal.
¿Cómo se observa la úlcera corneal por herpes?
Se requiere fluoresceína y es una “úlcera dendrítica”.
¿Cuál es el tratamiento de la úlcera corneal por herpes?
Trifluouridina, idoxuridina o deoxuridina más un ciclopléjico.
¿Cuáles son las características de la uveítis anterior?
Hay fenómeno de Tindall, precipitados queráticos y pupila pequeña (por
sinequias posteriores).
¿Cuál es la diferencia entre la uveítis anterior y la posterior?
La anterior es dolorosa; la posterior no lo es.
¿Cuáles son complicaciones del uso de esteroides oftálmicos?
Catarata, úlcera corneal, queratitis y glaucoma.
¿Cuáles son los factores predisponentes al desprendimiento retiniano?
Ser mayor de 50 años, antecedente de extracción de catarata y miopía.

276
¿Cuál es el signo patognomónico del desprendimiento retiniano?
El paciente describe que su visión ha sido tapada por una cortina.
¿Cuáles son causas de oclusión de la arteria central de la retina?
Embolia, trombosis de una arteria con aterosclerosis y arteritis temporal.
¿Cómo se presenta la oclusión de la arteria central de la retina (OACR)?
Ceguera unilateral súbita e indolora.
¿Cuáles son los hallazgos clínicos en los casos de OACR?
Reflejo fotomotor disminuido y consensual conservado, dilatación pupi-
lar, el fondo de ojo pálido y opaco con una fóvea como “mancha de color
rojo cereza” y arterias atenuadas y exangües “en vagón de ferrocarril”.
¿Cuál es el tratamiento de la OACR?
Disminución de la presión ocular mediante masaje sobre los párpados y
aspirar CO2 5-10% en O2 para aliviar el espasmo retiniano.
¿Cuáles son las causas de oclusión de la vena central de la retina?
Ocurre principalmente en ancianos y en pacientes jóvenes. La causa es
frecuentemente reumatológica (arteritis de células gigantes). La diabetes,
la hipertensión y la viscosidad sanguínea aumentada son factores predis-
ponentes.
¿Cuál es el cuadro clínico de la oclusión de la vena central de la retina
(OVCR)?
Disminución paulatina de la visión, unilateral e indolora; fondo de ojo
congestionado y edematoso, con venas tortuosas. Al cabo de semanas de
la obstrucción, la neovascularización causa rubeosis iridis con glaucoma
secundario.
¿Cómo se diagnostica la OVCR?
Por medio de la angiografía fluoresceínica.
¿Cuál es el tratamiento de la OVCR?
No existe uno específico. Para tratar la neovascularización secundaria, se
puede utilizar fotocoagulación.
¿Cuáles son los tipos de retinopatía diabética?
 No proliferativa: se observa aumento de la permeabilidad capilar, mi-
croaneurismas (puntos rojos en el polo posterior de la retina), hemorra-
gias (en punto y en mancha), edema (que se diagnostica mejor por la

277
angiografía fluoresceínica) y exudados. Es la más común; aparecice
en 90-95% de los casos.
 Proliferativa: se caracteriza por neoformación de vasos dentro de la re-
tina, que se extiende hasta el vítreo, y fibrosis que puede causar des-
prendimiento de la retina en etapas avanzadas.
¿Cuáles son los tipos de exudado?
 Blandos: también conocidos como algodonosos o cotonosos, son de-
bidos a anoxia (deficiencia vascular aguda).
 Duros: de coloración amarilla, causados por edema crónico (capila-
res lesionados).
¿Cuáles son las causas de la retinopatía hipertensiva?
La hipertensión esencial crónica, la hipertensión maligna y la toxemia del
embarazo.
¿Cuál es la clasificación de Keith-Wagener-Barker para la retinopatía hi-
pertensiva?
 Grado 1: constricción de las arteriolas retinianas.
 Grado 2: constricción y esclerosis de las arteriolas retinianas (hilos de
plata).
 Grado 3: exudados y hemorragias añadidos a las alteraciones vascu-
lares.
 Grado 4: hipertensión maligna (papiledema).
¿Quiénes padecen glaucoma de ángulo cerrado (glaucoma agudo)?
Los hipermétropes y los asiáticos.
¿Qué factor puede desencadenar el glaucoma agudo?
Una midriasis prolongada (por ejemplo, en el caso de la permanencia
prolongada en un lugar oscuro, como en una sala de cine).
¿Cuál es el cuadro clínico del glaucoma agudo?
Dolor extremo, visión borrosa, ojo rojo y halos alrededor de las luces.
Puede aparecer náusea y dolor abdominal.
¿Cuál es el tratamiento del glaucoma agudo?
 Pilocarpina.
 Inhibidores de la anhidrasa carbónica (acetazolamida 500 mg ).
 Iridotomía.

278
¿Cómo es el glaucoma de ángulo abierto (crónico)?
Hay una pérdida gradual de la visión periférica, un acopamiento de la
papila y no hay luces con halo.
¿Cuál es el tratamiento del glaucoma crónico?
 Timolol (o betaxolol): 1 gota cada 12 horas durante 3 a 6 semanas.
 Pilocarpina: 4 gotas al día.
 Adrenalina: 2 gotas al día.

279
280
ONCOLOGÍA

¿Cuáles son los patrones de tinción?


La eosina es un colorante ácido que se observa de color azul; se une a
componentes celulares alcalinos que contengan proteínas. La hematoxi-
lina es un colorante alcalino de color rojo.
Por orden de importancia, ¿cuáles son las neoplasias malignas más impor-
tantes en mujeres y hombres?
 Mujeres: pulmón, mama, colon.
 Hombres: pulmón, próstata y colon.
¿Cuáles son las neoplasias con patrón de herencia autosómica dominante?
Tumor de Wilms, Feocromocitoma, cáncer medular de tiroides y retino-
blastoma.
¿Cuál es la utilidad de la vacuna BCG (Bacilo de Calmette-Guérin) en los
pacientes con cáncer?
Se utiliza como modificador de la respuesta y se usa para el cáncer vesical.
¿Para qué se usa la interleucina 2 (IL2) en oncología?
Para tratar el cáncer de células renales y el melanoma.
¿Cuáles son los principales marcadores tumorales?
 Calcitonina: cáncer medular de tiroides.
 Ca 19-9: cáncer pancreático, colorrectal, gástrico y ovárico mucinoso.
 Ca 15-3: cáncer mamario.
 Ca 125: ovárico, endometrial, hepatocarcinoma, pulmonar y pan-
creático.
 Alfa fetoproteína: tumor gonadal no seminomatoso; cáncer hepatoce-
lular y gastrointestinal.

281
 Gonadotropina coriónica humana (Beta-HGC): tumor testicular se-
minomatoso.
 Antígeno carcinoembrionario (CEA): mamario, colorrectal, gástrico,
pulmonar, tiroideo y pancreático.
 Tiroglobulina: cáncer tiroideo papilar y folicular.
 Antígeno prostático específico (APE): cáncer prostático.
¿Cuál es la etiología y tratamiento del cáncer de laringe?
90% de los cánceres laríngeos son epidermoides y secundarios a alcoho-
lismo o tabaquismo. Los estudios T1 y T2 se tratan con radioterapia y los
T3 y T4, con laringectomía total.
¿Cuáles son los cuatro tipos de melanoma?
Léntigo maligno, acral lentiginoso, nodular y superficial diseminado.
¿Cuál es el tipo de melanoma más frecuente?
El superficial diseminado.
¿Qué factor aumenta 5 a 10 veces el cáncer tiroideo?
La radiación.
¿Cuáles son los cuatro tipos de cáncer tiroideo?
Papilar, folicular, medular y anaplásico.
¿Cuáles son la consistencia de la glándula tiroides según el tipo de cáncer?
 Firme: es más probablemente maligno.
 Blanda: más probablemente benigno
¿Cuál es la incidencia del cáncer tiroideo en relación con la nodularidad?
 Nódulo único: 20%.
 Nódulo múltiple: 40%.
¿Qué relación guarda el cáncer tiroideo con el tamaño del nódulo?
Cuando hay aparición rápida o aumento rápido de tamaño es probable-
mente maligno.
¿Cuál es un hallazgo clínico que se asocia a cáncer tiroideo?
Parálisis laríngea ipsilateral al nódulo.
¿Por qué los nódulos fríos son probablemente malignos?
Tanto el adenoma como el cáncer son hipofuncionantes. Aún así, 40% de
los cánceres puede captar cierta cantidad.

282
¿Cómo se interpreta el comportamiento de los nódulos tiroideos según los
hallazgos clínicos?
 Si el nódulo es francamente “caliente”, se puede descartar maligni-
dad; cuando el nódulo es “frío”, casi siempre es maligno.
 Los nódulos quísticos raramente son malignos; cuando son sólidos,
un bajo porcentaje es benigno.
¿Cuál es el tratamiento de elección del cáncer tiroideo?
El quirúrgico.
¿Cuáles son las características y la incidencia del cáncer papilar?
La incidencia es 80% en general. Las mujeres son dos veces más afecta-
das que los hombres. Si está encapsulado es de buen pronóstico.
¿Cuáles son las características y la incidencia del cáncer folicular?
La incidencia es menos de 10%. Forma parte del síndrome NEMII (neo-
plasia endocrina múltiple tipo II). Su pronóstico depende de la etapa.
Cursa con calcitonina y calciuria elevadas.
¿Cuáles son las características y la incidencia del cáncer anaplásico?
Su incidencia es menos de 10%. Afecta más a las personas de 50 a 70
años. Su pronóstico es pobre.
¿A qué se le llama nódulo pulmonar solitario?
Es una lesión pulmonar bien circunscrita, que mide hasta 5 cm de diámetro.
¿Qué características sugieren la probabilidad de nódulo pulmonar solitario
benigno?
 Sin cambios en 2 años.
 Se observan calcificaciones densas, concéntricas o en “palomitas de
maíz”.
 Es menor de 1 cm.
¿Cuál es el protocolo para el estudio de nódulo pulmonar solitario?
Primero, placa simple del tórax; después, tomografía y, por último, biopsia.
¿Cuál es el cáncer pulmonar más frecuente y su localización?
Es el adenocarcinoma de localización periférica (alejado de los bronquios
principales).
¿Cuál es el tumor pulmonar benigno más común?
El hamartoma (imagen en “palomitas de maíz”).

283
¿El adenoma bronquial es maligno o benigno?
Es maligno.
¿Cuáles son los cánceres broncogénicos centrales detectables por la citolo-
gía del esputo?
El cáncer de células escamosas y el de células pequeñas.
¿Cuál es el síndrome de la vena cava superior?
Se debe a obstrucción del vaso y cursa con edema, hiperemia y cianosis
de los miembros torácicos, el cuello y la cara. También se presenta ma-
reo, estupor, síncope, edema cerebral y laríngeo.
¿Cuándo aparecen los síndromes de Horner y de la vena cava superior?
En el cáncer intratorácico (broncogénico principalmente) cuando la le-
sión provoca un efecto de masa y aparece en 5% de los casos.
¿En el cáncer broncogénico qué significan los estadios T1 y T2?
Menor y mayor y de 3 cm, respectivamente.
¿En el cancer broncogénico que significan los estadios T3 y T4?
Extensión a la pared torácica sin y con afectación mediastínica respecti-
vamente.
¿Cuáles son los síndromes paraneoplásicos del cáncer pulmonar de células
pequeñas?
Los síndromes de Eaton-Lambert, Cushing, secreción inadecuada de
hormona antidiurética (SIADH) y la degeneración cerebelar subaguda.
¿Cuál es el régimen estándar de quimioterapia para el cáncer de células pe-
queñas?
Ciclofosfamida, adriamicina y vincristina (CAV).
¿Cuáles se consideran neoplasias pulmonares malignas de bajo grado?
Los tumores carcinoides, dentro de los que se encuentra el carcinoma de
glándulas bronquiales.
¿Qué porcentaje de los adenomas bronquiales representa el carcinoide?
80-90%.
¿De dónde procede el carcinoide bronquial y cuál es su tipo celular?
Procede de las células madre bronquiales. Se conforma de células tipo
Kultchinsky.
¿Cómo se clasifica histológicamente el hamartoma pulmonar?
Como adenocondroma.

284
¿Cuáles son los hallazgos clínicos en los casos de carcinoide bronquial?
Tos, atelectasia, infección recurrente y hemoptisis.
¿Qué es el mesotelioma y con qué se relaciona?
Es un tumor de la pleura o del peritoneo asociado a asbestosis.
¿Qué es el síndrome de Pancoast?
Es un síndrome secundario al tumor del sulcus pulmonale superior, que
cursa con ronquera (afección del laríngeo recurrente) y parálisis del he-
midiafragma del lado afectado.
¿Cuáles son los tres tumores benignos del hígado?
El hemangioma, el adenoma hepatocelular y la hiperplasia nodular focal.
¿En quiénes incide más el cáncer hepático?
En hombres (2:1) y en mayor de 50 años.
¿Qué factores se asocian al cáncer hepatocelular?
Hepatis B, 80%; hepatitis C, 50%; cirrosis macronodular, solventes, ni-
trosaminas, pesticidas órganoclorados y la aflotoxina de Aspergillus.
¿Cuál es la presentación clínica del cáncer hepatocelular?
Malestar, fiebre, ictericia, hepatomegalia (88%), pérdida de peso (85%),
masa abdominal dolorosa (50%) y hallazgos compatibles con cirrosis
(60%).
¿Cuál es la sobrevida del cáncer hepático resecable?
De 3 a 5 años (20%)
¿Cuál es la sobrevida en los casos de cáncer hepático no resecable?
De 4 meses.
¿Cuál es el cáncer hepático más frecuente en adulto?
El hepatocelular.
¿Cuál es el cáncer hepático más frecuente en niños?
El hepatoblastoma.
¿Cuál es la incidencia del colangiocarcinoma?
5-30% en personas de 60-70 años.
¿En qué órgano se asientan con más frecuencia las metástasis y cómo se de-
tectan?
En el hígado; la detección se hace con determinaciones de amino-
transferasa de aspartato (AST) y fosfata alcalina (ALP).

285
¿Cuál es el tratamiento más efectivo de las metástasis hepáticas?
El quirúrgico.
¿Cuáles son los factores predisponentes para el cáncer del colon?
Poliposis, enfermedad inflamatoria, enfermedad hereditaria y la dieta.
¿Qué medicamentos son auxiliares en la prevención primaria del cáncer del
colon?
La aspirina y los AINES (piroxicam y sulindaco). El sulindaco reduce la
incidencia de pólipos adenomatosos rectales cuando se utiliza a dosis de
150 mg/día.
¿Cuáles son los dos tipos de carcinoma colorrectal hereditario no polipode
(CCHNP)?
Lynch I y II.
¿Cuáles son los antioncogenes del cáncer del colon?
Gen APC y cromosoma 17p53.
¿Cuáles son los genes afectados en los casos de CCHNP?
hMSH1 y el hMSH2.
¿Cómo se tratan las neoplasias del tercio inferior del recto?
Con resección abdóminoperineal de Miles.
¿Qué quimioterapia se usa en los casos de cáncer colorrectal (adyuvante)?
5-fluoruracilo (5FU) más levamisol o 5FU más leucovorín.
¿Cuál es la clasificación de Dukes del cáncer colorrectal?
 A: Confinado a la pared del colon.
 B: Penetra la serosa y los tejidos perirrectales.
 C: Metástasis linfáticas.
 D: Metástasis distantes.
¿Cómo se monitorea el cáncer colorrectal?
Con la determinación de los niveles del antígeno carcinoembrionario
(CEA). Si se incrementan, están indicados los estudios de imagen como
radiografía o la tomografía.
¿Cuál es la localización más frecuente del cáncer del páncreas?
La cabeza del páncreas; se presenta como ictericia indolora.
¿Cuál es la causa más frecuente de “vesícula palpable” (signo de Courvoi-
sier)?
El cáncer del páncreas.

286
¿Cuál es el tratamiento del cáncer del páncreas resecable?
El procedimiento de Whipple.
¿Cuál es la clasificación histológica del cáncer endometrial?
Adenocarcinomas (65%) y adenoacantomas (22%). Éstos se dividen en
grados según su diferenciación: grado 1, diferenciado; grado 2, modera-
damente indiferenciado y grado 3, indiferenciado.
¿Cuál es la etapa I del cáncer endometrial?
En esta etapa el cáncer se encuentra confinado al cuerpo uterino.
 Tipo A: cuando sólo afecta al endometrio y puede ser grado 1, 2 o 3.
 Tipo B: cuando afecta menos de la mitad del miometrio con o sin in-
vasión vascular.
 Tipo C: cuando afecta más de la mitad del miometrio.
¿Cuál es la etapa II del cáncer endometrial?
En esta etapa, el tumor invade el cérvix, pero no se extiende más allá del
útero.
 Tipo A: cuando invade la mucosa cervical.
 Tipo B: cuando invade el estroma cervical.
¿Cuáles son las etapas III y IV del cáncer endometrial?
En la etapa III hay invasión a los ovarios y al peritoneo; en la etapa IV
existe invasión a vejiga, el recto y órganos distantes.
¿Cuáles son las modalidades de tratamiento para el cáncer endometrial se-
gún la etapa correspondiente, de acuerdo con la clasificación de Ginecolo-
gía y Obstetricia (FIGO)?
 IAG1: Histerectomía extrafacial (HE) + Salpingooforectomía bilate-
ral (SOB).
 IAG2: HE + SOB + Linfadenectomía pélvica y para-aórtica (LPP).
 IAG3: HE + SOB + LPP + Radioterapia (RT).
 IB1 (sin invasión vascular): HE + SOB + LPP.
 IB2 (con invasión vascular): HE + SOB + LPP + RT.
 IIA: HE + SOB + RT.
 IIB: HE + SOB + RT + Braquiterapia.
 III y IV: todas las modalidades de tratamiento: quimioterapia, radio-
terapia y hormonoterapia si el tumor es positivo a receptores.

287
¿Cuál es el cuadro clínico del cáncer cérvicouterino?
Se presenta con hemorragia uterina disfuncional (HUD), exudado fétido,
ulceración cervical y manchado poscoital.
¿Cuál es el tratamiento para la neoplasia cervical intraepitelial en etapa I
(NIC I)?
Se prefiere el uso de asa diatérmica y criocirugía, ya que con ellas se pue-
den obtener muestras de tejido para análisis.
¿Cuál es el tratamiento con radioterapia para el cáncer cérvicouterino?
La dosis de radioterapia es de 4,500 a 5,000 cGy y su complicación más
temprana es la diarrea.
¿Cuáles son los principales tumores ováricos?
 Seroso: se asemeja al tejido tubárico y presenta cuerpos de psamoma.
 Mucinoso: similar al tejido endocervical y del colon. El marcador
tumoral es el cáncer 19-9.
 Endometrial: semejante al tejido endometrial.
 De células germinales: contiene estructuras extra e intraembrionarias.
 Disgerminoma: aparecen cordones de linfocitos infiltrando el estro-
ma; es hormonalmente activo, hiperestrogénico y radiosensible (co-
mo el seminoma).
 Seno endodérmico: tiene tejido de saco vitelino con diferenciación
extraembrionaria y cuerpos de Schiller-Duval (células tumorales con
vaso central).
 Teratoma inmaduro: tejido de las tres capas embrionarias, preponde-
rantemente neuroectodérmico.
 De células granulosas: se observan cuerpos de Call-Exner (células
granulosas rodeadas de material proteínico), que secretan inhibina.
 De celulas de Sertoli-Leydig: secretores de andrógenos.
¿Cuáles son los genes asociados al cáncer ovárico?
K-RAS, HER 2 NEU, P53 y BRCA1.
¿Cuál es el cuadro clínico del cáncer del ovario?
Hay síntomas gastrointestinales como distensión, malestar general, masa
o tumor en los cuadrantes inferiores, dolor lumbar y ascitis.

288
¿Cuáles son los marcadores del tumor de células germinales del ovario?
La deshidrogenasa láctica, alfa-fetoproteína (alfa-FP) y gonadotropina
coriónica humana.
¿Cuál es el marcador para el cáncer ovárico en general?
El CA 125.
¿Cuál es el régimen de quimioterapia para el cáncer ovárico?
Carboplatino más paclitaxel.
¿Cuál es la neoplasia benigna de la mama más frecuente?
El fibroadenoma mamario se presenta en pacientes menores de 30 años,
con consistencia de caucho, relativamente móvil, separado, no hipersen-
sible y de 1 a 5 cm. En las mujeres mayores de 30 años debe pensarse en
mastopatía fibroquística y carcinoma mamario.
¿Cuál es el gen del cáncer de la mama?
C17P53 y C17BRCA1.
Por orden de frecuencia, ¿cuáles son los tipos celulares del cáncer mama-
rio?
Ductal infiltrante, medular, inflamatorio y papilar.
¿Cuál es el protocolo para el estudio de las tumoraciones mamarias?
Primero la mamografía; si la lesión es sospechosa, se realiza la biopsia
por aspiración con aguja fina (BAAF).
¿Cuándo está indicado practicar la BAAF y aguja de calibre ancho en pre-
sencia de tumores de mama?
La BAAF tiene su indicación precisa en las lesiones quísticas, y la de cen-
tro de aguja, en las lesiones sólidas.
¿Cuándo está indicada la biopsia abierta en caso de tumores de mama?
Cuando en la mamografía se observan calcificaciones o cuando hay des-
carga sanguinolenta.
¿Cuál es la conducta a seguir cuando se encuentra cáncer lobular de mama
in situ?
Está indicada la conducta espectante y no la quirúrgica.
¿Cómo se presenta un cistosarcoma filoides?
Se presenta con los mismos datos clínicos del fibroadenoma mamario; pue-
de alcanzar gran tamaño, y si no se le extirpa adecuadamente, recurre.

289
¿Qué marcador tumoral se utiliza en los casos de cáncer de mama?
CA15-3. Se utiliza sólo para investigar recaídas y nunca como prueba de de-
tección o diagnóstico pues también se eleva cuando existe cáncer hepático.
¿Cuál es la terapia de combinación para el cáncer de mama?
Ciclofosfamida, adriamicina, metotrexate y 5 fluoruracilo
¿Cuál es la estirpe más común del cáncer prostático?
El adenocarcinoma.
¿Cuál es la localización más frecuente del cáncer prostático?
Primero en la zona periférica, y luego en la periuretral.
Con base en la determinación del antígeno prostático específico (APE)
¿quiénes tienen mayor probabilidad de supervivencia?
Quienes tienen menos de 4 ng/ml.
¿Cómo se clasifica el cáncer prostático según los niveles de APE?
Se clasifica como confinado cuando el nivel de APE es menor de 3 ng/ml
y no confinado cuando el valor es mayor de 30 ng/ml.
¿Cuál es la sobrevida media de los pacientes con cáncer prostático?
De dos a dos años y medio.
¿Cuál es el tratamiento del cáncer prostático no metastásico?
Prostatectomía radical y radiación; determinar periódicamente los niveles
de APE para monitoreo.
¿Cuál es el tratamiento del cáncer prostático con metástasis?
Agonistas de la hormona liberadora de la hormona luteinizante (LHRH),
orquiectomía y estrógenos, ya sea por separado o combinados con el fin
de disminuir la producción de testosterona.
¿Cuáles son los fármacos antiandrogénicos que más se utilizan?
El ketoconazol, el finasteride para la hiperplasia prostática benigna
(HPB), la ciproterona (para el hirsutismo) y la flutamida (para el cáncer
prostático).
¿Qué factores predisponen al cáncer de la vejiga?
Edad de 60-70 años, ciclofosfamida, tabaquismo, esquistomiasis y el uso
de anilinas.
¿Cuál es la forma más común de presentación del cáncer de vejiga?
Hematuria indolora con síntomas irritativos.

290
¿Cuáles son los sitos donde se localizan más frecuentemente las metástasis
del cáncer prostático?
Hueso y ganglios retroperitoneales.
¿Cuáles son las localizaciones más frecuentes de la metástasis del cáncer
vesical?
Los ganglios pélvicos, los paraórticos y el hígado.
¿Cuál es el tratamiento del cáncer transicional superficial de la vejiga?
Resección transuretral más tiotepa, mitomicina C, doxorrubicina o vacu-
na (BCG).
¿Cuál es el tratamiento del cáncer vesical con metástasis?
Vincristina, metotrexate, cisplatino, taxol y gemcitabina.
¿Qué significan las siguientes categorías en el cáncer prostático?
T1a, T1b: hallazgo incidental con resección menor de 5% y mayor de
5%, respectivamente.
T2a, T2b: menor de 1.5 cm y mayor de 1.5 cm respectivamente.
T3: Traspasa la cápsula y llega al cuello de la vesícula o las vesículas se-
minales.
Por lo que respecta al cáncer vesical, ¿qué significan estos términos?
 T1: invade la submucosa.
 T2: invade el músculo superficial.
 T3a: invade el músculo profundo.
 T4b: invade la grasa perivesical.
En relación con el cáncer prostático y vesical ¿qué significan estos térmi-
nos?
 N1: metástasis menores de 2 cm.
 N2: metástasis de 2-5 cm o metástasis múltiples.
 N3: metástasis mayores de 5 cm.
¿Cuál es la triada clásica del cáncer renal?
Masa en un flanco, hematuria y dolor (pero es raro ver la presentación
clásica).
¿Cuáles son los datos paraneoplásicos del cáncer renal?
 Anemia: menos frecuente que la eritrocitosis.
 Eritrocitosis: el tumor puede producir eritropoyetina en exceso.

291
 Hipercalcemia: calcio sérico elevado (más de 10.5); pueden ser varios
mecanismos.
 Hipertensión: se produce renina en exceso.
 Fiebre.
 Síndrome de Stouffer: es la disfunción hepática no metastásica. Apa-
rece galactorrea, virilización o feminización, síndrome de Cushing y
disfibrinogenemia adquirida.
¿Cuáles son los estadios del cáncer renal según la clasificación de Robson?
 I/A: dentro de la cápsula.
 II/B: fuera de la cápsula y dentro de la fascia de Gerota.
 III/C: invade los ganglios regionales, la vena renal, la vena cava.
¿Cuál es el tratamiento de los estadios I y II del cáncer renal?
Nefrectomía radical con linfadenectomía.
¿De qué lado es más frecuente el varicocele?
Del izquierdo.
¿Cuál es la frecuencia por edad de los cánceres testiculares?
 Seminomatosos de los 35-60 años.
 Linfoma en mayores de 70 años.
 Teratoma del saco de Yolk en niños.
¿Cuál es el diagnóstico diferencial del cáncer testicular?
Cuando el aumento de volumen es indoloro, es más probable que se trate
de cáncer; si hay dolor puede deberse a epididimitis, hemorragia o infarto.
¿Cuál es el tratamiento del cáncer testicular seminomatoso?
Quimioterapia y radioterapia.
¿Qué significan las categorías IIA, IIB y IIC en el cáncer testicular?
Invasión retroperitoneal microscópica, invasión retroperitoneal mediana
y masa palpable, respectivamente.
¿Cuál es el tratamiento en las etapas I, IIA y IIB del cáncer testicular?
Orquiectomía radical con resección retroperitoneal.
¿Cuáles son las neoplasias que invaden los ganglios pélvicos y paraaórti-
cos?
El cáncer de la vejiga y el del cuerpo uterino.

292
¿Cuáles son las dos presentaciones de los tumores cerebrales?
 Presencia subaguda de síntomas focales
 Desórdenes no focales, como la cefalea, demencia y alteración de la
personalidad o de la marcha.
¿Cuáles son los signos de tumor cerebral en la TAC?
Efecto de masa, refuerzo del contraste y edema de patrón vasogénico,
principalmente en la sustancia blanca.
¿Cuál es la neoplasia intracraneal primaria más frecuente?
El astrocitoma, 50% (glioma, oligodendroglioma).
¿Cómo se mide la proliferación intracraneal de los tumores cerebrales?
Con el índice de captación de bromodeoxuridina (BudR), que es un aná-
logo de la timidina.
¿Cuáles son las características del astrocitoma de grado bajo?
Es más frecuente en niños; su tipo celular es el astrocitoma pilocítico, y
se trata con radioterapia.
¿Cuáles son las características del astrocitoma de grado alto?
Es más frecuente en adultos; generalmente su localización es supratento-
rial y el tratamiento es quirúrgico en los menores de 65 años.
¿Dónde se encuentran con más frecuencia los ependimomas?
Son mas frecuentes en los niños. Se ubican mayormente en los ventrícu-
los y de éstos el más afectado es el cuarto.
¿Cuáles son otros nombres del schwanoma?
Neuroma, neurinoma y neurilemoma.
¿Dónde aparece más frecuentemente el schwanoma?
En el VIII par (neurinoma del acústico). Se presenta con mareo, pérdida
de la audición y tinnitus.
¿Cuál es el tratamiento del meningioma y el schwanoma?
Cirugía y radioterapia.
¿Dónde se asientan los tumores metastásicos intracraneales?
En la unión gris-blanca en los territorios de la cerebral media y posterior.
¿Cuáles son los pasos para la conversión de purinas y pirimidinas a proteí-
nas?
Síntesis de purinas y pirimidinas  ribonucleótidos  deoxirribonucleó-
tidos  DNA  RNA  proteínas.

293
¿Cuáles son las fases del ciclo celular?
 M: mitosis.
 G0: cuando la célula está en reposo.
 G1: síntesis de enzimas que se necesitan para la síntesis de DNA.
 S: replicación del DNA.
 G2: síntesis de componentes celulares requeridos para la mitosis.
¿Cómo se clasifican los antineoplásicos?
 Antimetabolitos: metotrexato, 5- fluoruracilo (5FU), fludarabina y ci-
tarabina.
 Antibióticos: bleomicina, daunorrubicina, dactinomicina y plicami-
cina.
 Alquilantes: carmustina, lomustina, mecloretaminay estreptozocina.
 Inhibidores de microtúbulos: navelbina, paclitaxel, vinblastina y vin-
cristina.
 Hormonas esteroides y antagonistas: aminogluteimida, estrógenos,
groserelina, leuprolide, prednisona y tamioxifén.
 Otros: aspariginasa, cisplatino, carboplatino, etopósido, interferón y
procarbacina.
¿Qué antineoplásicos son ciclo-celular específicos?
Son aquellos efectivos contra neoplasias que se replican rápidamente,
como las hematológicas. En este grupo se encuentran los antimetabolitos,
la bleomicina, los alcaloides de la vinca (vincristina, vinblastina) y el eto-
pósido.
¿Cómo actúan los alquilantes?
Se unen covalentemente al DNA en las células proliferantes y no prolife-
rantes.
¿Cómo actúan los antimetabolitos?
Interfieren con la biodisponibilidad de precursores de nucleótidos (puri-
nas y pirimidinas) inhibiendo su síntesis o compitiendo con ellos en la
síntesis de DNA y RNA.
¿Qué medicamentos requieren la utilización de rescates de ácido folínico
(leucovorín)?
El metotrexate y la pirimetamina.

294
¿Qué enfermedades no neoplásicas se tratan con ciclofosfamida?
El síndrome nefrótico, la artritis reumatoide y la fibrosis pulmonar.
¿Qué medicamento se adjunta a la terapia con ciclofosfamida para prevenir
la cistitis?
El mercaptonato sulfonato de sodio (MESNA).
¿Qué fármacos quimioterápicos provocan más neutropenia?
La ciclofosfamida, el metotrexate, 5-fluoruracilo y la doxorrubicina.
¿Cómo actúa el 5-fluoruracilo?
El timidilato es el único precursor del DNA que no se forma a partir de las
purinas y pirimidinas. El 5-fluoruracilo actúa inactivándolo.

295
296
OTORRINOLARINGOLOGÍA

¿Cuál es el rango de audibilidad del oído humano?


Detecta sonidos que van de 0 a 140 dB y de 20 a 20,000 Hz.
¿Cuál es el plexo de Kiesselbach?
Es el plexo del tabique nasal anterior.
¿Con qué se asocia la rinitis alérgica?
Con pólipos nasales.
¿Cuál es el tratamiento de los pólipos nasales pequeños?
Esteroides inhalados o ingeridos.
¿Qué órganos afecta más la granulomatosis de Wegener?
Nariz y los senos paranasales.
¿Cuándo está indicada la adenoidectomía?
Cuando la obstrucción provoca apnea del sueño o corpulmonale.
¿En la prueba de Weber cómo se percibe el sonido en los casos de sordera
de conducción?
Más fuerte en el lado afectado, por razones no bien comprendidas. En el
paciente con sordera neurosensorial unilateral el sonido se oye en el oído
normal.
¿Cómo se interpreta la prueba de Rinne?
Es una prueba que compara la transmisión del sonido por las vías aérea y
ósea. Normalmente la duración de la conducción aérea es mayor que la
ósea. En las sorderas de conducción esta relación se invierte. En las sor-
deras neurosensoriales la relación permanece, pero la duración del soni-
do disminuye.
¿A qué se le llama sordera neurosensorial?
Cuando es incierto si la causa es una lesión en el oído interno o en el VIII
par.

297
¿Cuáles son las causas de sordera sensorial?
Traumatismo ótico, laberintitis, fármacos ototóxicos y enfermedad de
Ménière.
¿Cuáles son las causas de sordera neural?
Tumores del ángulo pontocerebeloso y una gran variedad de enfermeda-
des neurológicas.
¿Cómo se evalúa clínicamente el vértigo?
Con la prueba de Romberg y la presencia de nistagmo. Cuando la lesión
es periférica el nistagmo es horizontal con un componente rotatorio.
¿Cómo se tratan el mareo de la laberintitis y el secundario a movimiento?
La laberintitis se trata con difenidol, y el mareo por movimiento, con an-
tihistamínicos como el dimenhidrinato.
¿Cuáles son los agentes causales de la otitis externa?
Gérmenes gram positivos Proteus, Pseudomonas y hongos.
¿Qué es la enfermedad de Ménière?
También se le llama hidropesía endolinfática. Es una enfermedad que pro-
duce vértigo intenso y episódico, con duración de 1 hasta 8 horas acom-
pañado de sordera sensorial y causado por distensión del canal
endolinfático.
¿Cuál es el tratamiento de la enfermedad de Ménière?
Hidroclorotiazida, para disminuir la presión endolinfática.
¿Qué es la laberintitis?
Es un vértigo grave de inicio agudo, con duración hasta de una semana.
¿Cuál es el tratamiento del vértigo?
Cuando es grave, se utiliza diacepam; cuando es moderado, se usan anti-
histamínicos como el dimenhidrinato.
¿Cómo se presenta la sinusitis maxilar?
Con dolor maxilar en los caninos e incisivos.
¿Cómo se presenta la sinusitis etmoidal?
Con dolor en la pared antero-lateral de la nariz, que se irradia a la órbita.
¿Cómo se presenta la sinusitis esfenoidal?
Con cefalea justo a la mitad de la cabeza.
¿Cuál es el tratamiento de elección para la sinusitis?
Descongestionantes (pseudoefedrina: 60-120mg 3 o 4 veces al día) +
amoxicilina.

298
¿Cuáles son las causas de la parálisis unilateral de las cuerdas vocales?
El cáncer de tiroides, el tumor de Pancoast y el síndrome del agujero yu-
gular.
¿Cuál es el origen más frecuente de la epistaxis?
En 90% de los casos, la epistaxis tiene origen anterior por lo que el tapo-
namiento nasal anterior suele ser suficiente para controlarla.
¿Cuál es el tratamiento en caso de sangrado nasal activo?
Se colocan torundas impregnadas con solución de xilocaína y adrenalina
al 1 o 2% dentro de la cavidad nasal, en el sitio de la epistaxis, y se hace
compresión sobre ambas alas de la nariz durante 10 minutos. Si la hemo-
rragia cede, debe observarse al paciente durante 30 minutos más para ad-
vertir recurrencias.
¿Cuáles son las indicaciones de taponamiento nasal posterior?
El sangrado nasal posterior y el sangrado en focos múltiples no visibles,
como en los casos de discrasias sanguíneas y traumatismos.

299
300
PEDIATRÍA Y GENÉTICA

¿Cuáles son los hitos del desarrollo?


 El reflejo natatorio y el reflejo de marcha desaparecen entre los 4 a 6
meses.
 La fontanela anterior cierra entre los 17 a los 19 meses.
 La primera dentición ocurre a los 6-8 meses.
 La fontanela posterior se cierra al nacimiento o en la etapa neonatal.
 El recién nacido duerme hasta 20 horas al día.
 En el 1º y 2º día, el recién nacido sigue con los ojos un objeto y tiene
poca reacción al dolor.
 El atrapamiento del pulgar es signo de daño neurológico.
 El niño responde con una sonrisa a las 6-8 semanas.
 El niño cambia de decúbito lateral al supino y realiza pinza gruesa en
el 4º mes.
 El niño levanta la cabeza y el tronco a los 5 meses.
 El niño se sienta sin apoyo a los 6 meses.
 El niño pasa objetos de una mano a otra a los 6-7 meses.
 El niño tiene temor a los extraños a los 6 meses.
 La ansiedad de separación en el niño ocurre de los 6 a los 8 meses.
 El niño puede decir tres palabras disílabas, gatear y realizar la pinza
pulgar-índice a los 10 meses.
 El niño se mantiene parado sin apoyo a los 12 meses.
 El niño comienza a caminar sin apoyo a los 13-14 meses.
 El niño comprende el significado de “no” a los 12 meses.

301
 El niño tiene sentido de pertenencia y conducta oposicionista-
negativista a los 18-24 meses.
 El niño consigue el control de sus esfínteres a los 24 meses.
¿Cuáles son las reglas del crecimiento?
 El niño: pierde 5-10% del peso al nacimiento.
 Recupera el peso en 7-10 días.
 Pesa 500 g más y mide 3 cm más el primer mes.
 Aumenta 750 g y 3 cm mensualmente entre el segundo y cuarto mes.
 Aumenta 500 g y 2 cm por mes de los 5-8 meses.
 Aumenta 250-300 g y 1 cm por mes de los 8-12 meses.
 Ha duplicado el peso a los 4-5 meses.
 Ha triplicado el peso en el primer año.
 Ha cuadruplicado su peso a los 4-5 años.
¿Cuáles son los promedios de peso del nacimiento a los 10 años?
Nacimiento: 3.5 kg.
1 año: 10 kg.
5 años: 20 kg.
10 años: 30 kg.
¿Cuáles son los promedios de talla del nacimiento a los 4 años?
 Nacimiento: 51 cm.
 1 año: 76 cm.
 3 años: 90 cm.
 4 años: 101 cm.
¿Cuándo ocurre la hiperplasia del tejido linfoide?
De los 6 a los 10 años.
¿Cuándo se completa la primera dentición?
A los 2 años, con 20 dientes.
¿Cuándo se inicia la dentición definitiva?
A los 6 años.
¿A qué edad se diagnostica el pie plano?
A los 4 años.
¿Cuánto aumenta el perímetro cefálico durante el primer año?
1 cm por mes.

302
¿Cuál es la tasa de crecimiento de los 2 años a la pubertad?
2.25 kg y 5 cm por año.
¿En quiénes se indica el uso de plantilla correctora?
En aquellos pacientes que no tienen arco a la bipedestación, pero que sí
lo tienen al explorarlos en sedentación.
¿A qué edad se considera el diagnóstico de enuresis?
A los 5 años en los varones y a los 6 en las mujeres.
¿Qué tipo de trastorno es la enuresis nocturna?
Se le considera una disomnia no REM de las etapas 3 y 4.
¿Qué exámenes se realizan para el diagnóstico de enuresis?
EGO y examen neurológico.
¿Qué fármacos se utilizan para el tratamiento de la enuresis?
Imipramina y desmopresina.
¿Cuáles son los indicadores menores de retraso en el aprendizaje?
Sincinesias, disdiadococinesia y presencia de movimientos coreiformes.
¿Qué anomalías se asocian al transtorno de déficit de atención (TDA)?
Malformaciones en el oído, epicanto y clinodactilia.
¿Cuál es el tratamiento para el TDA en los menores de 6 años?
La dextroanfetamina. El metilfenidato sólo se utiliza en los niños mayo-
res de 6 años.
¿Cuál es el cuadro clínico del autismo?
Patrones estereotípicos de conducta, poca o nula interacción social y po-
ca o nula comunicación.
¿Cuál es el IQ normal?
Mayor de 70.
¿Qué complicaciones provoca el trauma obstétrico en el producto?
Cáput sucedáneo, céfalohematoma, hemorragia difusa del cuero cabellu-
do, moldeamiento del cráneo, fractura de huesos largos y lesión de ner-
vios periféricos (parálisis Erb-Klumpke).
¿Cómo se diferencia al céfalohematoma del cáput sucedáneo y de una he-
morragia difusa?
En el cáput sólo se observa edema, sin cambios en la coloración y puede
abarcar más de una sutura. El céfalohematoma es una colección de san-

303
gre subperióstica; por lo tanto, está contenido en la zona correspondiente
a una sola sutura y es de consistencia dura. Se alivia en 24 a 48 horas. La
hemorragia difusa abarca más de una zona de sutura, es blanda y la piel
involucrada se muestra azul; se asocia a anomalías de la coagulación y
puede ser tan extensa que haga disminuir el hematocrito, causar hipovo-
lemia y, finalmente, producir ictericia por la hemólisis del gran coágulo.
¿Qué es la parálisis de Erb-Duchenne?
Es la que resulta de un traumatismo en las raíces de C-5 y C-6 y que ge-
nera incapacidad de los movimientos del hombro e incapacidad de fle-
xionar el antebrazo.
¿Qué es la parálisis de Klumpke?
Es la que resulta de la lesión de las raíces de C-7, C-8 y T-1, con debilidad
de la mano, síndrome de Horner y parálisis del hemidiafragma corres-
pondiente.
¿Cuál es la incidencia de la gestación múltiple?
1 a 1.3%
¿Qué tipo de gemelos se asocian a la fertilización in vitro?
Los dicigóticos.
¿Cuál es la complicación más común del embarazo gemelar?
El parto pretérmino.
¿Cuál es el método de elección para el diagnóstico de gemelos idénticos?
Solo las pruebas de HLA son precisas, ya que aún siendo gemelos mono-
cigóticos, pueden ser bicoriónicos.
¿Cuáles son las complicaciones que ocurren en la madre a causa del emba-
razo múltiple?
Hipertensión en el embarazo; hiperemesis gravídica.
¿Qué complicaciones ocurren en los productos del embarazo múltiple?
Hemorragia intracraneal, síndrome de distrés respiratorio del recién na-
cido (SDRRN), y peso bajo al nacer.
¿Cómo se clasifica el grado de desarrollo intrauterino?
Como pequeño, adecuado y grande para la edad gestacional, según el pe-
so, la longitud y el perímetro cefálico.
¿Cuándo un recién nacido es pequeño para la edad gestacional?
Cuando tiene menos del percentil 10 de peso para una edad específica.

304
¿Cuándo un recién nacido tiene bajo peso y muy bajo peso al nacer?
Cuando al nacimiento pesa menos de 2500 gr y menos de 1500 gr respec-
tivamente.
¿Cuáles son los factores de riesgo relacionados con el peso bajo al nacer?
 La madre pesa menos de 45 kg.
 El aumento de peso durante el embarazo fue menor de 4.5 kg.
 La presencia de cérvicovaginitis durante el embarazo.
 El consumo de tabaco, alcohol o drogas durante el embarazo.
 La edad de la madre, si es menor de 16 o mayor de 35 años.
 Los periodos intergenésicos cortos.
¿Cuáles son las causas de retraso del crecimiento intrauterino?
Cualquiera que provoque enfermedad de pequeños vasos (como en la diabe-
tes mellitus, la hipertensión arterial y la enfermedad renal), la toxoplas-
mosis, la infección por citomegalovirus, la rubéola, el alcohol, la cocaína
y el tabaco.
¿Cuándo un recién nacido es grande para la edad gestacional?
Cuando está por arriba del percentil 90 de peso para una edad específica.
¿Cuál es la deficiencia congénita de disacaridasas?
La de lactasa-isomaltasa.
¿A qué edad funcionan la sucrasa, maltasa e isomaltasa?
Desde las 12 semanas; alcanzan el máximo a las 34 semanas.
¿A qué edad funciona la lactasa?
Desde las 37 semanas (en recién nacido a término).
¿Cuál es el contenido de la leche para el prematuro en Cal?
20-23 Cal por cada onza (30 ml).
¿Cómo se alimenta a un recién nacido de menos de 34 semanas?
Con sonda orogástrica.
¿Qué tipo de alimentación se utiliza para un niño con un peso al nacer me-
nor de 1,000 g?
Alimentación gástrica continua o transpilórica.
¿Cuáles son los requerimientos proteicos de un recién nacido a término?
2-3 gramos/kg/día.
¿Cuál es la edad más común de presentación de la deficiencia de lactasa?
De los 8 a los 15 años.

305
¿Cómo se manifiesta la deficiencia de lactasa?
Como diarrea osmótica, heces ácidas y meteorismo.
¿Cómo se presenta la intolerancia a proteínas de la leche o de soya?
Como enterocolitis o enteropatía.
¿Qué daño provoca en el intestino la intolerancia a proteínas de leche o de
soya?
Atrofia de vellosidades en el intestino delgado y colitis en el grueso.
¿Cómo evalúa el estado nutricional la clasificación de Gómez?
Evalúa el estado nutricional tomando en cuenta el peso como un por-
centaje del peso normal promedio para la edad en los menores de 5 años.
¿Cómo evalúa el estado nutricional la clasificación de Waterlow?
Lo evalúa con una relación entre el peso y la talla.
¿Cuál es la clasificación de Gómez?
 Normal: 90-110% del peso normal.
 Desnutrición de 1er. grado: 76-90%.
 Desnutición de 2º grado 61-75%.
 Desnutrición de 3º grado menos de 60%.
¿Cuáles son las indicaciones para realizar el diagnóstico prenatal?
Edad materna menor de 35 años, alfa-fetoproteínas baja, antecedente de
un hijo afectado, anomalías cromosómicas conocidas, estado cromosó-
mico incierto del hijo previo, alteración cromosómica o mendeliana de
los padres.
¿Cuál es el periodo indicado para realizar la amniocentesis?
A las 16 semanas de gestación.
¿Cuándo debe realizarse la biopsia de vellosidades coriónicas?
De las 10 a las 12 semanas de gestación.
¿Cómo se clasifican los defectos del nacimiento?
 Ambientales: los teratógenos (químicos o biológicos) son agentes que
alteran el crecimiento, la organogénesis o la fisiología celular en el
producto, y corresponden al 10%
 Genéticos: causan anomalías aisladas, anomalías del desarrollo y
síndromes reconocibles. Estas anomalías pueden ser heredadas o apa-
recer de novo.

306
 Multifactoriales: resultan de una combinación de factores genéticos o
ambientales. Cuando varios genes son los causantes, se denomina
anomalía poligénica.
 Esporádicos: debidos a accidentes que ocurren durante la gestación,
como la oclusión de un vaso, u otros que fomenten un desarrollo
inadecuado de la anatomía del producto. Tiene una incidencia de
1%.
¿Cuáles son los signos de enfermedad congénita del tejido conectivo?
Dilatación de la raíz aórtica y articulaciones elásticas.
¿Cuáles son los signos de las anomalías congénitas del desarrollo de las cé-
lulas musculares?
Se encuentran evidencias de que los tendones y ligamentos son laxos.
¿Cuáles son los signos de enfermedad ósea congénita?
Huesos largos y anchos en exceso.
¿Cuáles son los signos de enfermedad neurológica congénita?
Contracturas y falta de flexión.
¿Qué signos indican alteración de los factores de crecimiento?
Talla alta y exceso de tejido subcutáneo.
¿Qué es un dismorfismo?
Es una anomalía estructural del desarrollo que puede ser una malforma-
ción, una deformación o una disrupción. Puede ser medible objetivamen-
te como en el caso del hipertelorismo que es un aumento en la distancia
interpupilar normal, o bien, apreciable subjetivamente como en el caso de
los pliegues palmares anormales.
¿Qué es un síndrome de malformación?
Es un patrón reconocible de anormalidades que se presume se deben a
una causa común. Las anormalidades pueden ser internas o externas, es-
tructurales o funcionales.
¿Cuáles son las posibles etiologías de los síndromes de malformación?
Teratogénicas, esporádicas, de un solo gen y cromosómicas.
¿A qué se le llama tiempo de exposición al teratógeno?
Es el periodo comprendido entre las 8 y las 12 semanas durante las cuales
se lleva a cabo la organogénesis y, por tanto, pudiera verse afectada. Una

307
exposición al teratógeno antes de la implantación (7 a 10 días) sólo resul-
taría en la pérdida del embrión sin efectos posteriores.
¿Qué agente es la causa más frecuente de malformaciones congénitas de
origen infeccioso?
El citomegalovirus que causa retraso en el crecimiento, pérdida de la au-
dición y anomalías del sistema nervioso central.
¿Cuáles son las malformaciones más comúnmente asociadas a la rubéola?
El glaucoma, la catarata, el microftalmos y la uveítis.
¿Cuáles son los tres tipos de desórdenes cromosómicos?
El numérico, el debido a un defecto estructural y mosaicismo.
¿Qué es el imprinting?
Es el fenómeno por el cual la expresión de un gen se modifica sugún pro-
ceda del padre o de la madre.
¿Qué es la disomía uniparental?
Sucede cuando los cromosomas de un mismo par pertenecen al mismo
progenitor.
¿Cuál es el patrón de transmisión de los defectos autosómicos recesivos?
El defecto se expresa cuando ambos padres son portadores. Si solo uno es
portador, entonces el hijo también será portador.
¿Cuáles son las características de transmisión de los defectos autosómicos
dominantes?
 25% de la progenie es normal.
 50% son homocigotos.
 25% son heterocigotos.
¿Cuáles son los defectos cromosómicos estructurales más frecuentes?
 Cri du chat: es una deleción en el 5p.
 Tumor de Wilms-aniridia: deleción en el 11p.
 Prader Willi: deleción en el 15q.
¿Cuál es la incidencia de los principales desórdenes cromosómicos?
 Síndrome de Down 1:700.
 Síndrome de Klinefelter 1:800.
 Síndrome de Turner 1:2500.
 Trisomía 18 1:8000.

308
 Trisomía 13 1:4000-1:10,000.
 Cri du chat 1:50,000.
¿Cuáles son las enfermedades más frecuentes ligadas a X?
La ceguera al color, la distrofia muscular de Duchenne y el síndrome de
X frágil.
¿Qué enfermedades se deben a expansión de trinucleótido?
La distrofia miotónica, la ataxia de Friedrich, la enfermedad de Hunting-
ton, la ataxia espinocerebelar, el síndrome de x frágil y la atrofia muscu-
lar espinal bulbar.
¿Cuáles son las principales enfermedades por defecto en el metabolismo de
aminoácidos?
La fenilcetonuria, la homocistinuria, la alcaptonuria y la acidemia isova-
lérica, entre otras.
¿Cuál es la incidencia de la fenilcetonuria y de la homocistinuria?
1 en 1,200 y 1 en 100,000, respectivamente.
¿Qué errores del metabolismo requieren detección oportuna?
La homocistinuria y la fenilcetonuria.
¿Qué es la fenilcetonuria?
Es una deficiencia de fenilalanina hidroxilosa que cursa con aumento de
catabolitos de la fenilalanina y produce retraso mental. El daño se puede
prevenir evitando consumir alimentos que contengan fenilalanina.
¿Qué es la alcaptonuria?
Es una enfermedad causada por la deficiencia de la ácido homogentísico-
oxidasa, transmitida de manera autosómica recesiva.
¿Qué es la ocronosis?
Es la coloración opaca de los cartílagos del cuerpo principalmente en los
de la oreja por depósito de ácido homogentísico.
¿Qué es la acidemia isovalérica?
Es una enfermedad debida al aumento del ácido isovalérico (derivado de
la leucina).
¿Qué es la galactosemia?
Es la deficiencia congénita de galactosa-1-uridyl transferasa. Al consumir
galactosa se producen en el recién nacido: insuficiencia hepática (aumen-
to de las bilirrubinas y del tiempo de protrombina y disminución de la

309
glucosa), alteración de la función tubular (acidosis, glucosuria y aminoa-
ciduria) y cataratas.
¿Cuáles son algunas enfermedades peroxisómicas?
La adrenoleucodistrofia neonatal, la enfermedad de Refsum y la condro-
displasia rizomélica punteada, entre otras.
¿Cuáles son los defectos del ciclo de la urea?
La hiperamonemia, la deficiencia de ornitintrascarbamilasa y la deficien-
cia de carbamoyl fosfato sintetasa I.
¿Cuáles son los síndromes que cursan con acidosis láctica?
 MELAS: miopatía, encefalopatía, lactic acidosis, stroke (acidosis lácti-
ca, enfermedad vascular cerebral).
 MERRF: myotonic epilepsy with red ragged fibers (epilepsia mioclónica
con fibras rojas rasgadas).
 Kerns-Sayre: oftalmoplejia y degeneración retiniana.
¿Cuáles son las únicas anomalías enzimáticas que se transmiten de manera
autosómica dominante?
La enfermedad de Gilbert, la deficiencia de alfa-1-antitripsina (alfa1AT)
y porfiria intermitente aguda.
¿Qué enfermedades produce la deficiencia de alfa1AT en el adulto?
EPOC tipo enfisema en el 60%, cirrosis y carcinoma hepatocelular.
¿Qué enfermedad produce la deficiencia de alfa-1-antitripsina en el recién
nacido?
Hepatitis metabólica neonatal, que conduce a cirrosis e hipertensión portal.
¿Cuáles son las anomalías cardiacas asociadas a desórdenes cromosómi-
cos?
 Síndrome de Down: defecto de cojinetes endocárdicos y defectos ven-
triculares septales.
 Síndrome de Turner: coartación aórtica y aorta bicúspide.
¿Que se debe sospechar si la fontanela posterior es mayor de 5 cm en el re-
cién nacido?
Hipotiroidismo congénito.
¿Qué es la enfermedad de Wilson?

310
Es la toxicosis hereditaria por cobre (autosómica recesiva), en la que hay
concentraciones hepáticas de cobre elevadas 10 a 50 veces, déficit de ce-
ruloplasmina, mitocondrias hepáticas anómalas y esteatosis hepática. Las
manifestaciones pueden aparecer en cualquier momento desde los 5 a los
50 años, con anillo de Kayser-Fleischer (coloración dorada o verde ama-
rillenta de la córnea), problemas de comportamiento, alteraciones del
movimiento y anemia hemolítica.
¿Cuál es el defecto genético en la enfermedad de Wilson?
Está afectado el cromosoma 13 en el gen ATP 7B.
¿Cuál es el gen defectuoso en los casos de síndrome de Marfan?
El gen de fibrilina que se encuentra en el brazo largo del cromosoma 15
(15q).
¿Cuál es el diagnóstico diferencial del síndrome de Marfan?
Debe hacerse con la homocistinuria principalmente.
¿Cuáles son los genes defectuosos en los pacientes con osteogénesis imper-
fecta?
Los dos genes que codifican la alfa colágena: COL 1A1 y COL 1A2.
¿Cuál es el cuadro clínico de la osteogénesis imperfecta?
Escleróticas azules, osteopenia, dentinogénesis imperfecta, historia de
sordera e imagen de “palomitas de maíz” en las metáfisis.
¿Qué es la enfermedad de Hirschprung?
Es una agenesia de los plexos mientéricos en el colon distal, el cual se en-
cuentra contraído siempre e impide el paso del bolo fecal.
¿En quién se debe sospechar enfermedad de Hirschprung?
En un neonato que tiene retraso en el paso del meconio, pobre alimenta-
ción, vómito biliar y distensión abdominal. El diagnóstico debe hacerse
antes de los tres meses mediante biopsia rectal.
¿Qué defectos producen las fallas en el desarrollo mülleriano?
Defectos en la porción superior de la vagina, en el cuello uterino, el útero
o las trompas de Falopio.
¿Qué defectos producen las fallas en el desarrollo del seno urogenital?
Agenesia uterina y vaginal.
¿Cuáles son los síndromes de feminización?
 Deficiencia de colesterol-desmolasa.

311
 Deficiencia de 3beta-desoxiesteroide-deshidrogenasa.
 Deficiencia de 17-hidroxilasa.
 Deficiencia de 5 alfa-reductasa.
¿Cuáles son los síndromes de virilización?
 Deficiencia de 21-hidroxilasa.
 Deficiencia de 11-hidroxilasa.
 Deficiencia de 3-beta-hidroxiesteroide-deshidrogenasa.
¿Cuáles son las causas de pubertad retrasada en mujeres y hombres?
Constitucional, falla testicular/ovárica e hipogonadismo hipogonadotró-
pico.
¿Cuál es el cuadro clínico de la deficiencia de 17 alfa-hidroxilasa?
Son varones hipertensos, con infantilismo sexual y azoospermia. El in-
fantilismo sexual se debe a la ausencia de cortisol, andrógenos o estróge-
nos. La hipertensión se presenta porque existe mayor producción de
mineralocorticoides.
¿Qué hormona estimula la producción de testosterona?
La hormona luteinizante actúa sobre las células de Leydig para producir
testosterona.
¿Qué es el síndrome de feminización testicular?
Es un estado de refractariedad de los tejidos periféricos a la dihidrotestos-
terona, lo cual da como resultado un paciente con fenotipo femenino, pe-
ro con testículos con localización intraabdominal.
¿Cuáles son las causas de la falla ovárica/ testicular primaria?
El síndrome de Turner y el síndrome de Klinefelter, respectivamente.
¿Dónde se encuentra el defecto del síndorme de X frágil?
En el brazo largo del cromosoma X (Xq28).
¿Cómo se hace el diagnóstico de síndrome de X frágil?
Demostrando que el triplete CGG está repetido más de 200 veces en el
gen FMR1. Afecta a uno de cada 4,000 varones.
¿Cuáles son las variantes citogenéticas del síndrome de Klinefelter?
80% presenta cariotipo 47XXY; otros son el 48XXXY, el 48XXYY y el
49XXXXY.
¿Cuáles son los cariotipos de los llamados superhombre y supermujer?

312
XYY y XXX, respectivamente.
¿A qué se asocia el cariotipo XYY?
A conducta antisocial.
¿Cuál es la hernia congénita más frecuente?
La hernia diafragmática pósterolateral de Bochdalek.
¿Cuál es la pentalogía de Cantrell?
Hernia diafragmática, esternón hendido, pericardio ausente, onfalocele y
defectos intracardiacos.
¿Qué es el onfalocele?
Es una herniación de las vísceras dentro del cordón umbilical. Puede
afectarse el intestino, el hígado o ambos.
¿Qué es la gastrosquisis?
Es una unión defectuosa en la línea media que deja pasar al intestino ha-
cia el exterior.
¿Cómo se diagnostica prenatalmente el onfalocele?
Con ultrasonido.
¿Qué es el complejo VATER?
Son defectos Vertebrales, Anales, Traqueoesofágicos, Radiales y Renales.
¿Cuál es el tipo más frecuente de fístula traqueoesofágica?
La atresia esofágica proximal con fístula traqueoesofágica distal (tipo IV)
se presenta en 85% de los casos. La segunda en frecuencia es la atresia sin
fístulas; se presenta en 8%.
¿Qué signos acompañan a la atresia esofágica?
Abdomen escafoide, salivación excesiva e imposibilidad para pasar una
sonda nasogástrica. Si la atresia es pura no hay aire en el estómago.
¿Qué es la estenosis pilórica?
Es una hipertrofia de la capa muscular a nivel del píloro, que cursa clíni-
camente sin vómito al nacimiento, sigue una etapa en que inicia y au-
menta gradualmente el vómito y finalmente aparece el vómito no biliar
en proyectil. Se observa de los 2 meses a los 2 años del nacimiento.
¿Cómo se diagnostica la estenosis pilórica?
Con el hallazgo de una masa abdominal superior derecha en un niño con
historia de vómito. El ultrasonido es el procedimiento diagnóstico de

313
elección; un píloro mayor de 15 mm y una pared pilórica de grosor ma-
yor a 4 mm son altamente sugestivos.
¿Qué anomalías se asocian a la atresia intestinal?
Trisomía 21, VATER y páncreas anular.
¿Cuáles son los dos hallazgos en y la atresia duodenal?
Vómitos biliares y signo de la doble burbuja.
¿A qué se asocia el 10% de las atresias yeyuno-ileo-cólicas?
A la fibrosis quística.
¿Cuál es la diferencia radiológica entre el ileo meconial y la atresia de ye-
yuno-ileo?
En la primera sólo hay distensión e imagen en “burbujas de jabón”,
mientras que en la segunda hay distensión con niveles hidroaéreos.
¿Cómo se presenta el divertículo de Meckel?
82% como hemorragia indolora, 10% como obstrucción y 6% como di-
verticulitis (hay que hacer diagnóstico diferencial con apendicitis).
¿Cuál es la anomalía intestinal congénita más frecuente?
El divertículo de Meckel
¿Cuándo aparece la displasia de la cadera?
Es frecuente tras un parto de nalgas y su incidencia es de 1 en 100 con
predominio en las mujeres.
¿Qué es la malformación de Arnold-Chiari?
Es la protrusión del cerebelo hacia el canal espinal con distorsión de la
médula espinal.
¿Cuál es el cuadro clínico de la atresia de las coanas?
El recién nacido se presenta cianosis que sólo se alivia con el llanto; exis-
te imposibilidad para colocar una sonda nasogástrica.
¿Qué se debe sospechar al detectar estridor laríngeo en un recién nacido?
Que hay laringomalacia.
¿Cuál es el cuadro clínico del hipotiroidismo congénito?
Distrés respiratorio, cianosis, ictericia, rechazo del alimento, llanto ron-
co, hernia umbilical y retraso en el crecimiento óseo.
¿Cómo se diagnostica la hipocalcemia del neonato?

314
Con un Calcio total menor de 7 y un Calcio ionizado menor de 3-3.5,
con signos clínicos de hipocalcemia como el mioclono.
¿Cuál es una causa frecuente de la hipocalcemia del neonato?
La hiperfosfatemia que se provoca cuando se le alimenta con leche de
vaca.
¿Cuándo se considera hipoglucemia en el neonato?
Cuando la glucemia es menor de 20 mg/dl en el neonato pretérmino,
menor de 30 mg en el neonato de término y menor de 45 después del pe-
riodo neonatal. Se presenta con nerviosismo, hipotonía, apnea, taquipnea
o convulsiones.
¿Cuáles son los factores de riesgo relacionados con la hipoglucemia neonatal?
 Diabetes materna: crea un estado de hiperinsulinemia en el neonato,
que provoca episodios de hipoglucemia.
 Prematurez: no hay madurez de las enzimas gluconeogénicas y las
reservas de glucógeno son bajas.
 Retardo en el crecimiento
 Asfixia perinatal y estrés por frío,
 Sepsis y anormalidades metabólicas o endocrinas como la galactose-
mia o el panhipopituitarismo.
¿Qué anomalías se pueden presentar en los hijos de madres diabéticas?
Regresión caudal, macrosomia, muerte fetal, parto pretérmino, hipoglu-
cemia y policitemia.
¿Qué es la regresión caudal?
Es la falta de desarrollo del tejido nervioso y óseo de la columna inferior
relacionado con la presencia de diabetes en la madre.
¿Cuál es la dosis de vitamina K utilizable en los neonatos?
0.5 a 1 miligramo.
¿Qué es la enfermedad hemorrágica del recién nacido?
La secundaria a niveles bajos de factores K dependientes y un metabo-
lismo hepático acelerado (inducción microsomal por fenitoína o por fe-
nobarbital) o administración de cumarínicos a la madre. Dentro de los
primeros tres días pueden presentarse hemorragias en el cuero cabelludo,
las mucosas, el área de circuncisión o en el ombligo.

315
¿Qué es la enfermedad hemolítica del recién nacido?
También conocida como eritroblastosis fetalis, es la causa más frecuente de
anemia neonatal. Ocurre cuando los anticuerpos de la madre cruzan la
barrera placentaria para atacar a los antígenos de las células de la sangre
del producto, específicamente a los dos grupos principales de antígenos,
que son el sistema ABO (leve) y el sistema Rho (D) (grave). La anemia in
útero puede ser tan severa que conlleve hidrops fetalis.
¿Cuál es el hematocrito de un recién nacido a término?
50- 55%
¿Cuáles son las manifestaciones clínicas del hidrops fetalis?
La hipoxia prolongada por anemia en el producto provoca acidosis. Se
identifica por la triada de anemia, anasarca e hipoproteinemia.
¿Cuáles son causas de hidrops fetalis?
La anemia severa, alfa-talasemia in útero, incompatibilidad de grupo,
transfusión gemelo-gemelo y el cierre prematuro del foramen oval.
¿Cuál es el tratamiento del hidrops fetalis?
Corregir la anemia, tratar la falla cardiaca congestiva y tratar el distrés
respiratorio.
¿Cuáles son las causas del cierre del forámen oval?
Aumento de la presión pulmonar, aumento de la oxemia y disminución
de la prostaglandina E2.
¿Qué es el síndrome de hiperviscosidad del recién nacido?
También llamado policitemia (Hto mayor de 65%). Ocurre en los recién
nacido postérmino hijos de madres diabéticas y con pinzamiento tardío
del cordón. Lucen pletóricos, rubicundos y con acrocianosis. Los sínto-
mas son irritabilidad, letargo o convulsiones. Además puede observarse
ictericia y cardiomegalia.
¿Cuáles desórdenes genéticos cursan con policitemia?
El síndrome de Down y el síndrome de Beckwidth-Wiedeman.
¿Cuáles son las etapas del desarrollo pulmonar?
 Pseudoglandular (5-16 semanas): se forman desde los bronquios has-
ta los bronquíolos terminales.

316
 Canalicular (16-25 semanas): se forman los sacos alveolares.
 Alveolar (25-40 semanas): aumenta el número de sacos alveolares y
aparece el surfactante.
¿Cuál es el componente más importante del factor surfactante?
La fosfatidilcolina.
¿Cuál es el índice lecitina-esfingomielina?
Es la relación entre ambas, que se mide directamente de una muestra de
líquido amniótico. Lo normal es de 2:1, y tiene un valor predictivo sobre
la madurez pulmonar del producto al momento de tomar la muestra.
¿Cuáles son las dos causas principales de hipoplasia pulmonar?
La primera es la disminución de líquido amniótico y la segunda es una
lesión ocupante del tórax (hernia diafragmática).
¿Cuál es la diferencia clínica y terapéutica entre la taquipnea transitoria
del recién nacido (TTRN) y el síndrome de distrés respiratorio del recién
nacido (SDRRN)?
 La TTRN se debe a un drenaje linfático insuficiente del líquido pul-
monar (generalmente causado por un mecanismo anormal de parto y
realización de cesárea) que muestra ligera disnea, sin retracciones ni
cianosis. Se resuelve en pocos días con oxígeno suplementario.
 El SDRRN o síndrome de membrana hialina se debe a una produc-
ción insuficiente de factor surfactante generalmente debida a corta
edad gestacional. En las primeras tres horas de vida extrauterina hay
disnea con cianosis y retracciones. La terapia comprende oxígeno su-
plementario y reposición de factor surfactante con alguno de los
análogos sintéticos.
¿Cuál es la imagen radiológica del pulmón húmedo (TTRN)?
Hiperinsuflación, trama parahiliar densa, corazón con borde deshilacha-
do y líquido en las fisuras.
¿Cuál es la imagen en los diferentes tipos de síndrome de distrés respiratorio?
 Tipo 1: retículo granular con mayores zonas de aireación.
 Tipo 2: retículo granular con mayores zonas de atelectasia.
 Tipo 3: retículo granular con mayores zonas de ateleutasia y bronco-
grama intenso.

317
 Tipo 4: atelectasias difusas que borran la silueta cardiaca.
¿Cuáles son las complicaciones del SDRRN?
Hemorragias intraventriculares, neumotórax espontáneo, conducto arte-
rioso permeable, enterocolitis necrosante, displasia broncopulmonar y re-
tinitis de la prematurez.
¿A qué se le llama displasia broncopulmonar del recién nacido?
Es una enfermedad que resulta del uso de oxígeno y ventilación mecáni-
ca en los infantes con SDRRN. Se caracteriza por la necesidad de perpe-
tuar la terapia con oxígeno más allá de los 28 días de vida.
¿Qué es el síndrome de aspiración de meconio?
En los productos postérmino, la hipoxia es un estímulo que por vía vagal
media la liberación de meconio al líquido amniótico, que posteriormente
se deposita en las vías respiratorias y en el tubo digestivo. Los depósitos
de meconio producen signos de distrés respiratorio y signos radiográficos
que demuestran hiperinflación con áreas de infiltrado difuso.
¿Qué es la enterocolitis necrosante del recién nacido?
El tejido intestinal previamente afectado por la hipoxia es dañado ade-
más por el sobrecrecimiento bacteriano al iniciar la dieta del recién naci-
do. Se presenta con distensión abdominal, sangre en las heces, acidosis,
hipotensión o choque. Los casos leves se autolimitan al descontinuar la
vía oral e instalar el antibiótico adecuado. Los casos graves con hemorra-
gia franca requieren resección intestinal.
¿Qué es le retinopatía de la prematurez?
También se le conoce como fibroplasia retrolental y se debe a la inmadurez
de los vasos retinianos. Los factores que contribuyen a su aparición son
la hiperoxia (en el tratamiento de padecimientos respiratorios), hipercap-
nia, hipoxia intermitente y sepsis.
¿Cuáles son las principales causas de ictericia fisiológica del recién nacido?
Puede deberse a actividad retardada de la glucoronil-transferasa, aumen-
to de la carga de bilirrubina en los hepatocitos y disminución de la depu-
ración plasmática de bilirrubina.
¿Cuáles son las causas de ictericia no fisiológica del recién nacido?

318
 Hemólisis: inmune (incompatibilidad de grupo) y no inmune (esfero-
citosis, hemoglobinopatías y deficiencias enzimáticas).
 Acúmulo de sangre extravascular: céfalohematoma.
 Circulación enterohepática aumentada: obstrucción intestinal.
 Alimentación con seno materno.
 Desórdenes del metabolismo de la bilirrubina: síndromes de Lucey-
Driscoll, Crigler-Najjar y de Gilbert
 Desorden metabólico: hipotiroidismo, panhipopituitarismo y galacto-
semia.
 Sepsis bacteriana.
¿Qué enzima produce biliverdina?
La hem-oxidasa.
¿Qué enzima produce bilirrubina?
La biliverdín-transferasa.
¿Qué es el síndrome de Gilbert?
Es la deficiencia de uridín-difosfoglicerasa, que produce hiperbilirrubi-
nemia crónica congénita con niveles de 2-5 mg/dl. La deficiencia provo-
ca un defecto en la captación de bilirrubina.
¿Qué es el síndrome de Crigler-Najjar?
Es una deficiencia hereditaria de glucoronil-transferasa.
 Tipo I: hiperbilirrubinemia grave; mueren por kerníctero en el primer
año de vida.
 Tipo II: cursa con bilirrubinas menores de 20; alcanzan la edad adul-
ta.
¿Qué es el síndrome de Dubin-Johnson?
Es un síndrome con ictericia provocada por un defecto en la excreción de
bilirrubinas, en el que el hígado se encuentra pigmentado.
¿Qué es el síndrome de Rotor?
Es un defecto en la excreción de la bilirrubina, en el que el hígado no está
pigmentado.
¿Cuál es el nivel normal de bilirrubina en el recién nacido?
La total, menos de 13 y la directa, de 1.5-2; nunca aumentan más de 5 en
24 horas.
¿Cuándo se indica la fototerapia?
Cuando la bilirrubina total va de 12 a 15 mg/dl.

319
¿Cuándo se indica la exanguinotransfusión?
Cuando la bilirrubina total alcanza 20 mg/dl
¿Cuáles son los defectos cardiacos congénitos más comunes?
La comunicación interventricular (CIV) que ocurre en 26% de los casos,
seguida por la persistencia del conducto arterioso (PCA), que se presenta
en 10% de los casos y la coartación de la aorta, en 8%. La traspocisión de
los grandes vasos tiene una incidencia de 5%.
¿Cuáles son los hallazgos clínicos en los casos de coartación de la aorta?
Pulsos amplios en la mitad superior del cuerpo y soplo en el ápex o a lo
largo del borde esternal izquierdo e interescapular alto; radiografía de tó-
rax con el “signo del 3” y el signo de Roessler.
¿Cuáles son las características clínicas de la comunicación interauricular
(CIA)?
Levantamiento ventricular derecho, P2 continuo y soplo pulmonar en el
tercero y cuarto espacios.
¿Cuáles son los tres tipos de CIA?
 Defecto ostium secundum: Aparece en 80% de los casos y el defecto se
ubica a la mitad del tabique interauricular.
 Defecto ostium primum: es el siguiente en frecuencia y se ubica en la
parte inferior del tabique interauricular.
 Defecto tipo seno venoso: es el menos frecuente y se ubica en la parte
superior del tabique interauricular.
¿Qué se observa en el electrocardiograma de un paciente con defecto ostium
primum y de cojinetes cardiacos?
Desviación del eje a la extrema izquierda.
¿Qué es el síndrome de Eisenmenger?
Es un síndrome que puede ocurrir en presencia de cualquier tipo de corto
circuito intracardiaco. Normalmente provocan un corto circuito izquier-
da-derecha. Cuando la cardiopatía es avanzada desarrolla hipertensión
en el circuito pulmonar, que eventualmente se torna mayor que la pre-
sión de las cavidades izquierdas y revierte el flujo a derecha-izquierda.
¿Dónde se ausculta el soplo en los pacientes con CIA?
En el borde esternal derecho, y es una evidencia del aumento de la circu-
lación derecha.

320
¿Cuáles son causas de conducto arterioso persistente?
Hipoxia fetal o neonatal y aumento de los niveles de prostaglandinas.
¿Qué es la anomalía de Ebstein?
Es un defecto congénito de la tricúspide, que se encuentra implantada
dentro del ventrículo derecho más abajo de lo normal.
¿Cuál es el cuadro clínico de la CIV o PCA no complicada?
Hay estigmas de falla cardiaca congestiva, fallo de medro e infecciones
respiratorias bajas frecuentes.
¿Cuál es la causa principal de PCA?
La hipoxia aguda o crónica previene el cierre. Los niños con PCA son
niños de término que presentan cianosis e hipoxia progresivas.
¿Cuáles son las cardiopatías ductodependientes (dependen de la PCA o
CIV para que el paciente pueda seguir viviendo)?
La atresia pulmonar y la atresia tricuspídea.
¿Qué se encuentra en la exploración de la CIV no complicada?
Un soplo mesoesternal, mesosistólico en el borde izquierdo.
¿Qué se encuentra en la exploración de los pacientes que presentan CIV
con síndrome de Eisenmenger?
El ventrículo derecho está aumentado de tamaño, hay S2 fuerte y desdo-
blada y un soplo producido por la insuficiencia pulmonar.
¿Cuáles son los datos electrocardiográficos que se observan en la hiperten-
sión ventricular derecha?
 Patrón S-1, Q-3.
 Ondas R mayores o iguales de 20 mv.
 Ondas T positivas en las derivaciones precordiales derechas (V-1, V-
3).
¿Qué se observa en las placas radiográficas cuando la CIV es muy impor-
tante?
Aumento de tamaño del corazón y de la arteria pulmonar.
¿Cuáles son los gérmenes patógenos que producen la endocarditis de vál-
vula dañada y de los usuarios de drogas IV?
S. viridans y S. Aureus, respectivamente.
¿Cuáles son las cinco T´s de las cardiopatías cianógenas?

321
Tetralogía de Fallot, Tronco arterioso persistente, atresia Tricuspideopul-
monar, Trasposición de grandes vasos y drenaje venoso Total anómalo.
¿Qué es la tetralogía de Fallot?
Es una cardiopatía caracterizada por la presencia de estenosis pulmonar,
comunicación interventricular, hipertrofia del ventrículo derecho y ca-
balgamiento de la aorta. Radiológicamente se observa un corazón en bota y
clínicamente se presentan crisis de hipoxia.
¿Cuál es el situs normal (situs solitus)?
Cuando el ápex se encuentra a la izquierda junto con el bazo y el ven-
trículo derecho a la derecha con el hígado.
¿Cuál es la posición de la arteria pulmonar en los situs?
Anterior y a la izquierda en el situs normal, y anterior y a la dercha en el
situs inverso.
¿Cuáles son los hallazgos anatómicos en las trasposiciones de los grandes
vasos, derecha e izquierda?
La aorta se encuentra anteriormente ya sea izquierda o derecha y nace
del ventrículo derecho.
¿Cómo se usa el sistema de tres letras para describir la trasposición de los
grandes vasos?
 Primera letra: (S) solitus, (I) inversus. Indican la posición de la aurí-
cula.
 Segunda letra: (D) dextro, (L) levo. Indican la posición del ventrículo.
 Tercera letra: (D) dextro, (L) levo. Indican la posición de la trasposi-
ción.
¿Cuáles son los microorganismos más frecuentemente responsables de las
infecciones perinatales?
Estreptococos del grupo B, E. Coli, Klebsiella, Chlamydia y Neisseria.
¿Cuál es el periodo de incubación de la oftalmía por Chlamydia y por go-
nococo?
5 a 15 días y 1 a 21 días, respectivamente.
¿Cuál es el tratamiento profiláctico para evitar la oftalmía por gonococo?
Se usa una preparación con 1% de nitrato de plata, 1% de tetraciclina y
0.5% de eritromicina.

322
¿Cuáles son los principales factores de riesgo relacionados con la sepsis
neonatal?
 Favorecen la infección: rotura prematura de las membranas, co-
rioamnioítis y fiebre materna.
 Debilitan al producto: parto prematuro y bajo peso al nacer.
¿Cuál es el tratamiento empírico de la sepsis neonatal?
Ampicilina-gentamicina durante 10 días (excepto en los casos de menin-
gitis y osteomielitis)
¿Qué agentes causan frecuentemente la meningitis aséptica del recién na-
cido?
Los virus coxsackie y ECHO
¿Cuáles son los tres exantemas benignos del recién nacido?
El eritema tóxico neonatorum, la miliaria y la melanosis pustulosa.
¿Qué es la bronquiolitis?
Es la infección del tracto respiratorio de los menores de 2 años, que tiene
una incidencia máxima a los 6 meses.
¿Cuál es la causa número uno de la bronquiolitis y la neumonía en meno-
res de un año?
El virus sincicial respiratorio. El tratamiento se lleva a cabo con rivavirina.
¿Cuál es la imagen radiológica de la bronquiolitis?
Hiperinflación con áreas de atelectasia.
¿Con qué enfermedad se asocia el uso de miel en la dieta de los niños?
Con el botulismo.
¿Qué es el síndrome de Reye?
Es la hepatitis fulminante provocada por utilizar ácido acetilasalicílico
durante algunas infecciones virales.
¿Cuál es la vasculitis que aparece en los niños después de la faringitis esptrep-
tocócica?
La púrpura de Henoch-Schönlein.
¿Cuál es la diferencia entre la candidiosis y la “dermatitis del pañal”?
Las lesiones eczematosas son muy similares en ambas, pero la “dermati-
tis del pañal” respeta los pliegues cutáneos, a diferencia de la candidiosis.
¿Cómo se presenta un examema estafilocócico?

323
Con bulas y eritema superficiales.
¿Cuál es el segundo tumor sólido más frecuente en la infancia?
El neuroblastoma. Se genera en la cresta neural y tiene una incidencia de 7%.
¿Cuáles son las dos presentaciones del neuroblastoma?
Abdominal (70% de los casos) y torácica.
¿Cuándo se debe sospechar la presencia de un retinoblastoma en el recién
nacido?
Cuando el reflejo del fondo de ojo no tiene la coloración roja habitual
(leukocoria).
¿Cuáles son los tipos de hidrocefalia?
Comunicante (por disminución de la absorción del líquido cefalorraquí-
deo) y no comunicante.
¿Cuáles son las causas de hidrocefalia no comunicante en el neonato?
Malformación, infección y hemorragia intraventricular.
¿Cuáles son causas de hidrocefalía no comunicante en el niño mayor?
Las masas ocupantes o tumores son las causas más frecuentes.
¿Qué anomalía cromosómica se asocia al tumor de Wilms?
La deleción del 11p13.
¿Qué es el síndrome de WAGR?
Es la nemotecnia para tumor de Wilms, Aniridia, malformacion Geni-
tourinaria y Retraso psicomotor.
¿Con qué se asocia el síndrome de Beckwith-Wiedemann?
Con tumor de Wilms y hemihipertrofia.
¿Cuál es el tumor óseo infantil más frecuente?
El sarcoma osteogénico.
¿Dónde aparece con más frecuencia el sarcoma osteogénico?
Es proximal a la rodilla (en el fémur distal y la tibia proximal).
¿Cuál es la imagen radiológica del sarcoma osteogénico?
Se observa la imagen en rayos de sol.
¿Qué es el sarcoma de Ewing?
Es un sarcoma no diferenciado que aparece en los huesos.
¿Cuál es la imagen radiológica del sarcoma de Ewing?

324
La destrucción lítica es el signo más frecuente, pero puede haber múlti-
ples capas de neoformación reactiva de hueso subperióstico que determi-
nan la imagen de piel de cebolla.

325
PSIQUIATRÍA

¿A qué se le llama cluster A de personalidad?


Al paranoide, esquizoide y esquizotípico. Son personas raras y excéntri-
cas que guardan distancia emocional con otros.
¿A qué se le llama cluster B de personalidad?
Al antisocial, border, histriónico y narcisita. Son personas de conducta
impulsiva, excesivamente emocional y errática.
¿A qué se le llama cluster C de personalidad?
Al evasivo, obsesivo-compulsivo y dependiente. Los rasgos más consis-
tentes son de naturaleza ansiosa y temerosa.
¿Cuándo se diagnostica un trastorno de ajuste?
Cuando hay evidencias clínicas de ansiedad o depresión que son clara-
mente secundarias a un estrés identificable, generalmente cuando los su-
cesos rebasan la capacidad de adaptación del individuo.
¿Cuáles son las formas de presentación del trastorno de ajuste?
Ansiedad, depresión, un síntoma físico y el inicio de una aventura amo-
rosa o el consumo de alcohol o tabaco. Las respuestas subjetivas son el
temor (a que se repita el acontecimiento), la ira (por la frustración) y la
pena (en cuanto a la falta de capacidad para afrontar el problema).
¿Cuáles son los trastornos de ansiedad?
Los principales son el trastorno de ansiedad generalizada, el trastorno de
angustia y el trastorno obsesivo-compulsivo.
¿Qué fármacos se utilizan para tratar los trastornos de ansiedad?
 En el trastorno de ansiedad generalizada, las benzodiacepinas y la
buspirona constituyen los fármacos de elección.
 En los casos de crisis de angustia pueden utilizarse benzodiacepinas e
inhibidores selectivos de la recaptura de serotina (SSRI).

326
 En los casos de trastorno obsesivo-compulsivo, se utilizan fármacos se-
rotoninérgicos como la clorimipramina y la fluoxetina (SSRI). La buspi-
rona puede potenciar los efectos antiobsesivos de estos fármacos.
¿Cuál es el mecanismo de acción de las benzodiacepinas?
Se unen a receptores específicos adjuntos al receptor de ácido gamma-
aminobutírico (GABA) que aumentan la afinidad por este neurotransmi-
sor y hacen más frecuente la apertura de los canales de cloro activados
por el GABA, lo que da como resultado final una hiperpolarización de la
célula y la inhibición de la actividad neuronal (despolarización).
¿Cuáles son los cuatro usos clínicos de las benzodiacepinas?
Desórdenes de ansiedad, desórdenes musculares, crisis epilépticas y tras-
tornos del sueño.
¿Cuál es la benzodiacepina indicada para tratar la angustia de separación
patológica?
El clonacepam.
¿Cuál es la característica del pensamiento paranoide?
Las ideas delirantes (creencia falsa).
¿En qué padecimiento son más frecuentes las alucinaciones auditivas?
En la esquizofrenia.
¿En qué padecimiento son más frecuentes las alucinaciones visuales?
En el trastorno mental orgánico.
¿Cuáles son los síntomas positivos en la esquizofrenia?
Alucinaciones auditivas, delirio y trastornos del pensamiento formal.
¿Cuál es el tratamiento de elección de la esquizofrenia?
El haloperidol es el primer fármaco, seguido por la clozapina, en los ca-
sos de refractariedad o cuando se presentan extrapiramidalismo o disci-
nesias tardías.
¿Qué es la discinesia tardía?
Es la aparición de corea o tics de manera secundaria al uso de neurolépticos.
¿Qué es el delirio?
Es un trastorno global transitorio de la atención.
¿Qué es la demencia?
Es un trastorno caracterizado por disfunción crónica de las funciones
mentales superiores.

327
¿Cuáles son las demencias frontoparietal y frontotemporal?
La demencia de Pick y la de Alzheimer, respectivamente.
¿Cuáles son los síntomas negativos?
Inhabilidad de relaciones personales, pobreza de habla y afecto restringido.
¿Qué fármacos son inhibidores selectivos de la recaptura de serotonina
(SSRI)?
La fluoxetina, paroxetina y sertralina.
¿Cuál es el antidepresivo utilizado en los casos de trastorno de pánico?
La paroxetina.
¿Qué fármacos pertenecen al grupo de las fenotiacinas?
La clorpromacina, flufenacina, tioridacina y trifluoperacina.
¿Sobre qué receptores actúa la clorpromacina?
Sobre receptores muscarínicos, alfa-adrenérgicos e histamínicos (H-1).
¿Qué otros antipsicóticos se usan comúnmente?
El haloperidol (butirofenona) la risperidona (benzisoxazol).
¿Cómo actúan las fenotiacinas y las butirofenonas?
Son bloqueadores de receptores de dopamina tipo 2 (D2)
¿Cómo actúan la risperidona y la clozapina (dibenzotiacepina)?
Es un bloqueador de receptores de dopamina tipo 4 (D4).
¿Qué efectos produce el bloqueo central dopa?
Puede causar hipersecreción de las hormonas de la hipófisis anterior.
¿Cómo actúa la risperidona?
Es bloqueador D2 y de la serotonina (5-HT2).
¿Cómo actúa la olanzapina (tienbenzodiacepina)?
Es un bloqueador 5HT2, D1, D2, D4, y bloqueador muscarínico.
¿Cuál es el orden de potencia de los antipsicóticos?
En orden decreciente: haloperidol, tiotixeno, clorpromacina, tioridacina
y clozapina.
¿Qué es el síndrome neuroléptico maligno?
Es un estado similar a la catatonia, con aumento de la presión, signos ex-
trapiramidales e hiperpirexia.
¿Qué fármacos son útiles para tratar el síndrome neuroléptico maligno
(SNM)?
Los agonistas dopa (bromocriptina y amantadina).

328
¿Cuáles son las indicaciones de la amantadina?
 Profilaxis contra el virus de la influenza.
 Enfermedad de Parkinson (por ser agonista Dopa).
 Esquizofrenia.
¿Qué otras complicaciones aparte del SNM provoca el tratamiento antipsi-
cótico?
Distonía aguda, parkinsonismo y discinesia tardía.
¿A qué se llama parkinsonismo?
Al cuadro clínico constituido por facies de máscara, apatía, dificultad pa-
ra iniciar (o lentitud de) los movimientos.
¿Qué padecimientos se asocian con depresión?
La migraña, la epilepsia, el hipotiroidismo y la insuficiencia adrenal.
¿Qué exámenes de laboratorio se realizan en el estudio de un paciente con
depresión?
Pruebas de funcionamiento tiriodeo, prueba de supresión con dexameta-
sona y pruebas de funcionamiento hepático.
¿Cómo se interpreta la prueba con dexametasona en los pacientes que pa-
decen depresión?
La prueba es altamente sensible y utilizada para diagnosticar la depresión
mayor. En los pacientes deprimidos no hay supresión (prueba positiva), y
en estos casos se prevé que habrá mejor respuesta a los fármacos o al tra-
tamiento electro-convulsivo. Una prueba negativa no descarta la posibili-
dad de depresión.
¿Cómo se afecta la secreción de hormona estimulante del tiroides (TSH) en
los casos de depresión?
Cuando existe depresión mayor, la liberación de TSH tras la administra-
ción de factor liberador hipotalámico está disminuida o lentificada.
¿Cómo se presenta la depresión atípica?
Puede haber hipersomnia, ingestión excesiva de alimentos, hipersensibi-
lidad al rechazo y letargia.
¿Cuáles son los tres tipos de antidepresivos?
Tricíclicos, SSRI e inhibidores de la monoaminooxidasa (IMAO).
¿Cómo actúan los antidepresivos tricíclicos?
Son bloqueadores de la recaptura de serotonina y noradrenalina (a dife-
rencia de los SSRI); por tanto, tienen efectos catecolaminérgicos. Secun-

329
dariamente son bloqueadores de los receptores de acetilcolina, por lo que
causan efectos anticolinérgicos.
¿Cuál es el fármaco de elección para el tratamiento de los trastornos bipo-
lares?
El litio.
¿Qué fármacos se utilizan para tratar un episodio maníaco?
Los neurolépticos, las benzodiacepinas y el litio.
¿Cuáles son los efectos adversos del litio?
 Disminuye la sensibilidad renal a hormona antidiurética, por lo que
causa poliuria y polidipsia.
 Síntomas gastrointestinales.
 Tremor fino.
¿Cómo es el paciente obsesivo compulsivo?
Inteligente, organizado y concienzudo.
¿Cuáles son los estados disociativos?
Las fugas, el sonambulismo, la personalidad múltiple y la despersonali-
zación.
¿Qué efectos producen la sobredosis de ansiolíticos?
Depresión respiratoria, hipotensión y coma.
¿Cuál es la triada indicativa del abuso de sustancias?
Dependencia psicológica, dependencia fisiológica y tolerancia.
¿Qué parámetros de laboratorio están alterados en los pacientes alcohólicos?
Aumento del volumen corpuscular medio, elevación de las sustancias
que abarcan las pruebas de funcionamiento hepático, incremento de los
triglicéridos y aumento del ácido úrico sérico.
¿Cuándo aparece el delirium tremens después de la supresión alcohólica?
Después de 25 a 72 horas sin ingerir bebidas alcohólicas.
¿Cuál es la triada del síndrome de Wernicke-Korsakoff?
Confusión, ataxia y alteración del VI par.
¿Cuáles son los efectos de la nicotina?
Aumenta el nivel sérico de noradrenalina y serotonina.
¿Qué fármaco es un auxiliar en el tratamiento del tabaquismo?
El antidepresivo bupropiona, a dosis de 150 a 300 mg/día.

330
¿Cuándo se inicia el tratamiento del síndrome de supresión de narcóticos?
Cuando hay síntomas de grado 2 (midriasis, piloerección y alteraciones
cardiovasculares), con 10 miligramos de metadona. Puede utilizarse clo-
nidina para controlar los efectos cardiovasculares.
¿Por qué el abuso de cocaína causa destrucción del tabique nasal?
Porque provoca vasoconstricción prolongada.
¿Cuáles son las complicaciones producidas por el consumo de cocaína?
El colapso cardiovascular, el infarto, las arritmias y el ataque isquímico
transitorio (TIAS).

331
COMPENDIO DE SÍNDROMES Y ENFERMEDADES

 Alcaptonuria: defecto innato de la ácido homogentísico-oxidasa.


 Ocronosis: depósitos de ácido homogentísico en los cartílagos, que los ha-
cen verse obscuros.
 Homocistinuria: defecto autosómico recesivo de la cistationina-beta-
sintasa que se presenta entro los 10 a los 20 años, con trombosis arte-
riales y venosas sin factores de riesgo.
 Enfermedad de Gaucher: autosómica recesiva + deficiencia de beta-
glucocerebrosidasa + anemia + trombocitopenia + crisis óseas simila-
res a las de la anemia de células falciformes + hiperesplenismo + cé-
lulas de Gaucher en la médula ósea.
 Enfermedad de Hand-Schuller-Christian: granuloma eosinofílico de la hi-
pófisis posterior + diabetes insípida.
 Enfermedad de Huntington: corea + demencia + cromosoma 4 afectado.
 Enfermedad de Osler-Weber-Rendu: hemorragia-telangiectasia hereditaria
asociada a los genes HHT 1 y 2.
 Enfermedad de von Gierke: glucogenosis tipo IA que afecta la glucosa-6-
fosfatasa.
 Enfermedad de von Hippel-Lindaw: Síndrome neurocutáneo que consiste
en hemangioblastomatosis retinocerebelar + feocromocitoma + Ca
de células renales + neoplasia de islotes.
 Enfermedad de von Willebrand: anomalía cuantitativa (Tipo I) o cualitati-
va (tipo II) del factor de Von Willebrand secretado por el endotelio
vascular.
 Enfermedad de Wiscott-Aldrich: Inmunodeficiencia + trombocitopenia.

332
 Esclerosis múltiple: zonas de gliosis (cicatrización) en la sustancia blanca
encefálica y espinal + síntomas sensoriales (hormigueo, entumeci-
miento) y neuromusculares episódicas (paraparesia espástica, diplo-
pía) + etiología autoinmune.
 Macroglobulinemia de Waldenström: malignidad de células B + IgM ele-
vada + síndrome de hiperviscosidad + eritrocitos apilados (rouleaux).
 Síndrome de ataxia-telangiectasia: deficiencia de células T y B + dermati-
tis + deterioro neurológico.
 Síndrome de Beckwidth-Wiedemann: hemihipertrofia corporal + tumor de
Wilms.
 Síndrome de Churg-Strauss: es una variante de la poliarteritis nodosa con
predilección por el pulmón. El cuadro está dominado por una granu-
lomatosis alérgica pulmonar, asma y eosinofilia.
 Síndrome de CREST: síndrome que viene de las siglas en inglés: Calcinosis,
Raynaud, Esofagic disorder, Sclerodactilia y Telangiectasia.
 Síndrome de Cronkhite-Canada: poliposis adenomatosa familiar (FAP) +
alopecia + hiperpigmentación + aplasia de uñas.
 Síndrome de Diamond-Blackfan: aplasia pura de eritrocitos.
 Síndrome de DiGiorge: aplasia tímica y de células B + facies característica
+ anomalía del arco aórtico + enfermedad cardiaca + hipocalcemia.
 Síndrome de Felty: artritis reumatoide + esplenomegalia + granulocitope-
nia.
 Síndrome de Gardner: FAP + osteomas + tumores de tejidos blandos.
 Síndrome de Gilles de la Tourrete: múltiples tics motores y fónicos +
inicio antes de los 21 años + evolución progresiva.
 Síndrome de Horner: miosis, ptosis, enoftalmos y pérdida de la sudación
del lado afectado por lesión del simpático cervical.
 Síndrome de Nelson: adenoma hipofisario posadrenalectomía bilateral.
 Síndrome de Peutz-Jeghers: FAP + hipopigmentación de mucosas + póli-
pos hamartomatosos.
 Síndrome de Plummer: tirotoxicosis + bocio nodular.
 Síndrome de Plumer-Vinson: Adherencias esofágicas en el tercio proxi-
mal, con disfagia secundaria a ferropenia.

333
 Síndrome de Rokitansky-Kunster-Hauser: anomalía mülleriana caracteri-
zada por agenesia del útero y anexos.
 Síndrome de Sipple (neoplasia, endócrina múltiple tipo II-A): adenoma ti-
roideo + feocromocitoma + hiperparatiroidismo.
 Síndrome de Trousseau: tromboembolia pulmonar, trombosis venosa su-
perficial, trombosis venosa profunda y endocarditis marántica.
 Síndrome de Turcot: FAP + tumores malignos del sistema nervioso cen-
tral.
 Síndrome WAGR: Wilms + Aniridia + Genitourinarias + Retraso.
 Síndrome de Verter-Morrison: Es el VIPoma o cólera pancreática. Diarrea
+ hipokalemia + aclorhidria.
 Síndrome de Werner (noeplasia endócrina múltipletipo I): Adenoma pitui-
tario + adenoma pancreático + adenoma paratiroideo

334
335
GLOSARIO DE ABREVIATURAS

5HT: 5 hidroxitriptamina (serotonina)


1AT: alfa 1-antitripsina.
FP: alfa-fetoproteina.
A5-ASA: ácido 5-aminosalicílico.
ABVD: régimen quimioterapéutico. Son las siglas para Adriamicina (do-
xorrubicina), Bleomicina, Vincristina y Dacarbacina.
Ac: anticuerpo.
ACF: anemia de células falciformes.
ACP: fosfatasa ácida.
AF: angiografía fluoresceínica.
Ag: antígeno.
ADH: hormona antidiurética.
ALP: fosfatasa alcalina.
ALT: aminotransferasa de alanina.
ANA: anticuerpos antinúcleo.
ANCA: anticuerpos anticitoplasma de neutrófilo.
APE: antígeno prostático específico.
APO: apolipoproteína.
AR: artritis reumatoide.
ARJ: artritis reumatoide juvenil.
AS: anemia sideroblástica.
ASA: ácido acetilsalicílico.
ASCA: anticuerpos antisacaromices.
AST: aminotransferasa de aspartato.
ATLPC: angioplastía transluminal percutánea.

336
ATR: acidosis tubular renal.
BAAF: biopsia por aspiración con aguja fina.
BCG: bacilo de Calmette y Guerin.
BRQT: braquiterapia.
HGC: gonadotropina coriónica humana.
CBP: cirrosis biliar primaria.
CCHNP: carcinoma colorrectal hereditario no polipoide.
CCK: colecistocinina.
CCP: cáncer de células pequeñas.
CID: coagulación intravascular diseminada.
CMCH: concentración media de hemoglobina corpuscular.
CMV: citomegalovirus.
CNS: sistema nerviosos central.
CPS: ciclos por segundo.
CRF: factor liberador de cortisol.
CRP: proteina C Reactiva.
CPRP: colangiografía pancreática retrógrada endoscópica.
CUCI: colitis ulcerativa crónica inespecífica.
DEA: dehidroepiandrosterona.
DFH: difenilhidantoinato.
DG6PD: deficiencia de glucosa-6-fosfato deshidrogenasa.
DLCO2: capacidad de difusión del CO2.
DPKA: deficiencia de piruvatocinasa.
EA: espondilitis anquilosante.
EAP: enfermedad acidopéptica.
EBHA: estreptococo beta-hemolítico del grupo A de Lancefield.
EEG: electroencefalograma.
EEI: esfínter esofágico inferior.
EF: exploración física.
EH: enfermedad de Hodgkin.
EL: síndrome de Eaton-Lambert
ELA: esclerosis lateral amiotrófica.
EM: esclerosis múltiple.
ERCP: colangiografia pancreática retrograda endoscópica.

337
ETEC: Escherichia coli enterotoxigénica.
EVC: enfermedad vascular cerebral.
FA: fibrilación auricular.
FAL: fosfatasa alcalina leucocitaria.
FAP: poliposis adenomatosa familiar.
FC: frecuencia cardiaca.
FENa: fracción excretada de sodio.
FEV1: volumen espiratorio forzado del primer segundo.
FIO2: fracción inspirada de oxígeno.
FNT: factor de necrosis tumoral alfa.
FR: factor reumatoide.
FSH: hormona folículoestimulante.
FVC: capacidad vital forzada.
FVW: factor VonWillebrand.
GI: gastrointestinal.
GTP: gammaglutamiltranspeptidasa.
GM-CSF: factor estimulante de colonias granulocito-monocito.
GN: glomerulonefritis.
GnRH: hormona liberadora de gonadotropinas.
HAIN: hidracida del ácido nicotínico (isoniacida).
Hb: hemoglobina.
HCM: hemoglobina corpuscular media.
HE: histerectomía extrafascial.
HIV: virus de la inmunodeficiencia humana.
HLA: antígeno leucocitario humano.
HMWK: cininógeno de alto peso molecular.
HPB: hiperplasia prostática benigna.
HPTH: hiperparatiroidismo.
HTDA: hemorragia del tubo digestivo alto.
Hto: hematocrito.
HUD: hemorragia uterina disfuncional.
IAM: infarto agudo del miocardio.
IGF-1: factor de crecimiento semejante a la insulina.

338
IMAO: inhibidores de la monoamiooxidasa.
IRI: radioinmunoanálisis de insulina.
ITL: índice de tiroxina libre.
KDa: kilodaltones.
KS: cetosteroides.
LATS: estimulador tiroideo de larga acción.
LB: linfocito B.
LCR: líquido cefalorraquídeo.
LES: lupus eritematoso sistémico.
LED: lupus eritematoso discoide.
LH: hormona luteinizante.
LHRH: hormona liberadora de hormona luteinizante.
LLA: leucemia linfocítica aguda.
LLC: leucemia linfocítica crónica.
LMA: leucemia mielocítica aguda.
LMC: leucemia mielocitica crónica.
LNH: linfoma no Hodgkin.
Lpm: latidos por minuto.
LPP: linfadenectomía pélvica y paraórtica.
LPL: lipoproteínlipasa.
LT: linfocito T.
MHC: complejo mayor de histocompatibilidad.
MO: médula ósea.
MODY: mature onset diabetes of the young (diabetes juvenil de inicio tar-
dío).
MOPP: régimen quimioterapéutico. Son las siglas para Mecloretamina, On-
covin (vincristina), Procarbacina y Prednisona.
NEM: neoplasia endocrina múltiple.
NPS: nódulo pulmonar solitario.
NTA: necrosis tubular aguda.
OA: osteoartritis.
OACR: oclusión de la arteria central de la retina.
OHCS: hidroxicorticosteroides.
OHKS: hidroxicetosteroides.

339
PAF: poliposis adenomatosa familiar.
PB19: parvovirus B19.
PCA: persistencia del conducto arterioso.
PCr: creatinina plasmática.
PCR: reacción en cadena de polimerasa.
PFC: plasma fresco congelado.
PFH: pruebas de función hepática.
PGE: prostaglandina E.
PGI2: proastaglandina I2, prostaciclina.
PHS: púrpura de Henoch-Schönlein.
PIG: periodo intergenésico.
PL: punción lumbar.
PM: peso molecular.
PMN: polimorfonuclear.
PNa: sodio plasmático.
POMC: proopiomelanocortina.
PPD: derivado proteico purificado.
PTI: púrpura trombocitopénica idiopática.
PTOG: prueba de tolerancia oral a la glucosa.
RBC: célula roja de la sangre (eritrocito).
RCIU: retraso del crecimiento intrauterino.
RCT: receptor de célula T.
RMN: resonancia magnética nuclear.
RPR: prueba de reagina rápida.
RT: radioterapia.
RTU: resección transuretral.
RVR: respuesta ventricular rápida.
Sat Hb: saturación de hemoglobina.
SDR: síndrome de distrés respiratorio.
SHU: síndrome hemolítico urémico.
SIADH: síndrome de secreción inadecuada de hormona antidiurética.
SNC: sistema nervioso central.
SNM: síndrome neuroléptico maligno.
SOB: salpingooforectomía bilateral.

340
SSRI: inhibidores selectivos de la recaptura de serotonina.
TAC: tomografía axial computada.
TAM: tensión arterial media.
TCE: traumatismo craneoencefálico.
TCS: tejido celular subcutáneo.
TDA: trastorno de déficit de atención.
TEP: tromboembolia pulmonar.
TGV: trasposición de los grandes vasos.
TIAS: ataque isquémico transitorio.
TTRN: taquipnea transitoria del recién nacido.
TXA2: tromboxano A2.
T3RU: índice de recaptura de T3.
TAD: trastorno autosómico dominante.
TEP: tromboembolia pulmonar.
UCr: creatinina urinaria.
UDPG: uridindifosfoglicerasa.
UNa: sodio urinario.
VCM: volumen corpuscular medio.
VEB: virus de Epstein-Barr.
VFG: velocidad de filtración glomerular.
VIP: péptido intestinal vasoactivo.
VMA: acido vanililmandélico.
VSR: virus sincicial respiratorio.

341
BIBLIOGRAFÍA

PRIMERA PARTE

Arber, Héctor. Desarrollo de la memoria. Curso interactivo en CD. ILVEM,


México.
Arber, Héctor. Aprenda a aprender. Curso interactivo en CD. ILVEM, México.
Arber, Héctor. Lectura veloz. Curso interactivo en CD. ILVEM, México.
Bouisquie, Georges. Cómo organizar su trabajo. Segunda edición. Ediciones
Deusto, España, 1972.
Buzan, Tony. Cómo utilizar su mente con un máximo rendimiento. Segunda edi-
ción. Planeta Mexicana, México, 1991.
Goleman, Daniel. La inteligencia emocional. 37ª edición. Vergara, México,
2003.
Hernández Santiago, René Gastón. El éxito en tus estudios. Segunda edición.
Editorial Trillas, México, 1982
Ibarra, Luz María. Aprende mejor con gimnasia cerebral. Octava edición. Garnik
Ediciones, Mexico, 1999.
James, Muriel. Nacidos para triunfar: análisis transaccional con experimentos Ges-
talt. Primera edición. Fondo Educativo Interamericano S.A., México,
1976.
Segal, Jeanne. Su inteligencia emocional aprenda a incrementarla y usarla. Primera
edición. Grijalbo Mondadori, Barcelona, 2001.
Silva, José. El método Silva de control mental. Séptima edición. Editorial Diana,
México, 1994

SEGUNDA PARTE

Adler, Stephen N. A Pocket Manual of Differential Diagnosis. Cuarta edición.


Lipincott Williams & Wilkins, Philadelphia, Pennsylvania, 2000.
Barton, Thomas K. Appleton & Lange Repaso para el USMLE Paso 1. Tercera
edición. McGraw Hill, México, 2001.

342
Beck, William W. NMS Obstetrics and Gynecology. Cuarta edición. Lipincott
Williams & Wilkins, Philadelphia, Pennsylvania, 2000
Behrman, Kliegman. Nelson Compendio de Pediatría. Tercera edición. McGraw
Hill, España, 1999.
Berkow Robert. El Manual Merck. Novena edición. Harcourt Brace, España,
1997.
Chan, Carlyle H. Appleton & Lange Repaso para el USMLE Paso 2. Tercera edi-
ción, McGraw Hill, México, 2001.
Dworkin, Paul H. NMS Pediatrics. Cuarta edición. Lipincott Williams & Wil-
kins, Philadelphia, Pennsylvania, 2000
Fauci, Anthony S. Harrison´s Principles of Internal Medicine. 14ª edición.
McGraw Hill, Estados Unidos, 1998,
INNSZ. Manual de Terapéutica Médica y Procedimientos de Urgencias. Cuarta
edición. McGraw Hill, México, 2000.
Jarrell, Bruce E. NMS surgery. Cuarta edición. Lipincott Williams & Wilkins,
Philadelphia, Pennsylvania, 2000
Lederman, Robert J. Tarascon internal medicine & critical care pocketbook. Se-
gunda edición. Tarascon Publishing, Estados Unidos, 2001.
Mycek, Mary J. Lipincott´s ilustrated rewiews: pharmacology. Segunda edición.
Lipincott Williams & Wilkins, Philadelphia, Pennsylvania, 2000.
Wyers, Allen R. NMS medicine. Cuarta edición. Lipincott Williams & Wil-
kins, Philadelphia, Pennsylvania, 2000.
Rolak, Loren A. Secretos de la neurología. Segunda edición. McGraw Hill, Mé-
xico, 2000.
Surós Batlló, Juan. Semiología médica y técnica eploratoria. Séptima edición.
Masson S.A., Barcelona, 1998.
Sodi Pallares, D. Electrocardiografía clínica análisis deductivo. Primera edición,
Méndez Editores, México, 2000.
Tierney, Lawrence M. Diagnóstico clínico y tratamiento. 35ª edición. Manual
Moderno, México, 2000.
Wallach, Jacques, Interpretation of diagnostic rests. Séptima edición. Lipincott
Williams & Wilkins, Philadelphia, Pennsylvania, 2000.
Zollo, Anthony J. Secretos de la medicina. Segunda edición. McGraw Hill In-
teramericana, México, 2001.

343
ÍNDICE

Presentación 5

Parte I 9

Capítulo 1/ Introducción 11
Capítulo 2/ Aspectos técnicos 19
Visualización y lectura veloz 19
Actividad eléctrica y memoria 22
Inteligencia emocional 28
Economía del rendimiento 30
Capítulo 3/ Un programa de estudios 31
Planeación de la sesión de estudio 32
Planeación a largo plazo 35
Algunas cosas que es preciso considerar 37

Parte 2 41

Cardiología 43
Cirugía 63
Dermatología 73
Endocrinología 77
Gastroenterología 91
Ginecología y obstetricia 105
Hematología 115
Infectología 137
Inmunología y reumatología 159
Intoxicaciones y accidentes 181
Uronefrología 187
Neumología 201

344
Neurología 215
Nutrición, metabolismo, líquidos y electrolitos 235
Oftalmología 249
Oncología 255
Otorrinolaringología 271
Pediatría y genética 275
Psiquiatría 299
Compendio de síndromes y enfermedades 305
Glosario de abreviaturas 309
Bibliografía 315

345
Manual para Examen Nacional
de Residencias Médicas
de Carlos González Parra
se terminó de imprimir en agosto de 2005
en los talleres de Formación Gráfica S.A. de C.V.
con domicilio en Matamoros 112,
col. Raúl Romero de Ciudad Nezahualcóyotl,
Estado de México y con número de teléfono 57 97 60 60.
En la edición, la composición y el diseño son de José Luis Olazo García,
la revisión técnica de Orlando Escárcega Alarcón y el cuidado de la DGFE.
El tiraje consta de 1000 ejemplares.

346

S-ar putea să vă placă și